Download as pdf or txt
Download as pdf or txt
You are on page 1of 164

Downloded From : www.EasyEngineering.

net

ww
w.E
asy
En
gin
eer
ing
Cover:
.ne
F SSC CGL Tier I Exam t
F SSC CHSL Tier-I Exam
F SSC Sub Inspector Exam
F SSC Multi-Tasking Exam
F SSC Stenographer Exam
F Multiple Sitting

Downloded From : www.EasyEngineering.net


Downloded From : www.EasyEngineering.net

• Head Office : B-32, Shivalik Main Road, Malviya Nagar, New Delhi-110017

• Sales Office : B-48, Shivalik Main Road, Malviya Nagar, New Delhi-110017
Tel. : 011-26692293 / 26692294

ww
w.E
Typeset by Disha DTP Team

asy
En
gin
eer
ing
.ne
DISHA PUBLICATION
ALL RIGHTS RESERVED
t
© Copyright Publisher

No part of this publication may be reproduced in any form without prior permission of the publisher. The author and the
publisher do not take any legal responsibility for any errors or misrepresentations that might have crept in. We have tried
and made our best efforts to provide accurate up-to-date information in this book.

For further information about books from DISHA,


Log on to www.dishapublication.com or email to info@dishapublications.com

Downloded From : www.EasyEngineering.net


Downloded From : www.EasyEngineering.net

CONTENTS
1. History 1-14
2. Geography 15-28
3. Polity 29-40
4. Economics 41-54
5. ww
General Science 55-88
6. Computers w.E 89-94
7. General Knowledge asy 95-110

En
PRACTICE SETS gin
Practice Set-1 eer PS-1-3

Practice Set-2 ing PS-4-6

Practice Set-3 .ne PS-7-9

Practice Set-4 t
PS-10-12

Practice Set-5 PS-13-16

Current Affairs & GK Update GK-1-34

Downloded From : www.EasyEngineering.net


Downloded From : www.EasyEngineering.net

ww
w.E
asy
En
gin
eer
ing
.ne
t

Downloded From : www.EasyEngineering.net


Downloded From : www.EasyEngineering.net

1
CHAPTER
HISTORY

1. Arrange the following in chronological order : 12. Simon Commission was boycotted by the nationalist leaders
(SSC CGL 1st Sit. 2010) of India because: (SSC CGL 1st Sit. 2011)
1. Tughlaqs 2. Lodis (a) they felt that it was only an eyewash
3. Saiyids 4. Ilbari Turks (b) all the members of the Commission were English
5. Khiljis (c) the members of the Commission were biased against
(a) 1, 2, 3, 4, 5 (b) 5, 4, 3, 2, 1 India
(c) 2, 4, 5, 3, 1 (d) 4, 5, 1, 3, 2 (d) it did not meet the demands of the Indians
2. Who was the founder of The Servants of India Society? 13. Who among the following British persons admitted the
(SSC CGL 1st Sit. 2010) Revolt of 1857 as a national revolt?

ww
(a) G..K. Gokhale
(c) B.G. Tilak
(b) M.G. Ranade
(d) Bipin Chandra Pal (a) Lord Dalhousie
(SSC CGL 1st Sit. 2011)
(b) Lord Canning

of
(a) Bernard Shaw
(c) Lenin
w.E
3. Mahatma Gandhi was profoundly influenced by the writings
(SSC CGL 1st Sit. 2010)
(b) Karl Marx
(d) Leo Tolstoy
(c) Lord Ellenborough (d) Disraeli
14. Which of the following is called the 'shrimp capital of India'?
(SSC CGL 1st Sit. 2011)

was asy
4. The monk who influenced Ashoka to embrace Buddhism
(SSC CGL 2nd Sit. 2010)
(a) Mangalore
(c) Kochi
(b) Nagapatnam
(d) Nellore
15. Mention the place where Buddha attained enlightenment
(a) Vishnu Gupta
(c) Brahma Gupta
(b) Upa Gupta
(d) Brihadratha
5. The Lodi dynasty was founded by En (a) Sarnath
(SSC CGL 2nd Sit. 2011)
(b) Bodh Gaya

(a) Ibrahim Lodi


(SSC CGL 2nd Sit. 2010)
(b) Sikandar Lodi gin(c) Kapilavastu (d) Rajgriha
16. Coronation of Shivaji took place in
(SSC CGL 2nd Sit. 2011)
(c) Bahlol Lodi (d) Khizr Khan
6. Harshvardhana was defeated by (SSC CGL 2nd Sit. 2010)
(a) Prabhakaravardhana eer
(a) 1627 A.D.
(c) 1680 A.D.
(b) 1674 A.D.
(d) 1670 A.D.
(b) Pulakesin II
(c) Narasimhasvarma Pallava ing
17. The System of Dyarchy was introduced in India in
(SSC CGL 2nd Sit. 2011)
(d) Sasanka
7. Who among the following was an illiterate ?
(SSC CGL 2nd Sit. 2010)
(a) 1909
(c) 1919
(b) 1935
(d) 1945
.ne
18. The Editor of 'Young India' and 'Harijan' was
(a) Jahangir
(c) Akbar
(b) Shah Jahan
(d) Aurangazeb
8. Which Governor General is associated with Doctrine of Lapse ?
(SSC CGL 2nd Sit. 2010)
(a) Nehru (b) Ambedkar
(c) Mahatma Gandhi (d) Subash Chandra Bose
t
(SSC CGL 2nd Sit. 2011)

19. Who of the following attended all the Three Round Table
(a) Lord Ripon (b) Lord Dalhousie
Conferences? (SSC CGL 2nd Sit. 2011)
(c) Lord Bentinck (d) Lord Curzon
(a) B.R. Ambedkar (b) M.M. Malavia
9. The Greek ambassador sent to Chandragupta Maurya's Court
was : (SSC CGL 1st Sit. 2011) (c) Vallabhbhai Patel (d) Gandhiji
(a) Kautilya (b) Seleucus Nicator 20. Which king is referred to as Devanampiya Piyadassi (Beloved
(c) Megasthenes (d) Justin of the Gods) in the inscriptions? (SSC CGL 1st Sit. 2012)
10. Identify the European power from whom Shivaji obtained (a) Asoka (b) Harsha
cannons and ammunition: (SSC CGL 1st Sit. 2011) (c) Bindusara (d) Chandragupta Maurya
(a) The French (b) The Portuguese 21. Name the Maratha Saint who was a contemporary of Shivaji.
(c) The Dutch (d) The English (SSC CGL 1st Sit. 2012)
11. The call of "Back to the Vedas" was given by: (a) Saint Eknath (b) Saint Tukaram
(SSC CGL 1st Sit. 2011) (c) Saint Dhyaneshwar (d) Namdev
(a) Swami Vivekananda 22. Which of the following cereals was among the first to be
(b) Swami Dayanand Saraswati used by man? (SSC CGL 1st Sit. 2012)
(c) Aurobindo Ghosh (a) Rye (b) Wheat
(d) Raja Ram Mohan Roy (c) Barley (d) Oat

Downloded From : www.EasyEngineering.net


Downloded From : www.EasyEngineering.net

2 History
23. The treaty of Versailles restored Alsace-Lorraine to: 36. The original name of Nana Phadnavis was
(SSC CGL 2nd Sit. 2012) (SSC CGL 2nd Sit. 2012)
(a) Italy (b) Britain (a) Mahadaji Sindhia (b) Tukoji Holkar
(c) France (d) Belgiun (c) Narayan Rao (d) Balaji Janardan Bhanu
24. The Asokan Edicts were deciphered first by: 37. Who among the following first propounded the idea of Basic
(SSC CGL 2nd Sit. 2012) Education? (SSC CGL 2nd Sit. 2012)
(a) Sir John Marshall (b) Sir William Jones (a) Jawahar Lal Nehru
(c) Charles Wilkins (d) James Princep (b) Raja Ram Mohan Roy
25. Who favoured the Artic Home thoery of the Aryans? (c) Mahatma Gandhi
(SSC CGL 2nd Sit. 2012) (d) Dayanand Saraswati
(a) Pargiter (b) A. C. Das 38. Arrange the following in chronological order:
(c) B. G. Tilak (d) Jacobi (SSC CGL 2nd Sit. 2012)
26. Who was the teacher of Gautama Buddha? I. Dandi March
(SSC CGL 2nd Sit. 2012) II. Simon Commission
(a) Panini (b) Alara Kalama III. Poona Pact
(c) Kapila (d) Patanjali IV. Gandhi Irwin Pact
27. The prose collection of the vedic poems are: (a) II, I, III, IV (b) II, I, IV, III

ww
(a) Samhitas
(c) Aranyakas
(SSC CGL 2nd Sit. 2012)
(b) Upanishads
(d) Brahmanas
(c) IV, III, I, II

by
(d) IV, III, II, I
39. The city of Prayag was named Allahabad - the city of Allah
(SSC CGL 2nd Sit. 2012)

w.E
28. Non-violence as taught and practised by Mahatma Gandhi
is rooted in the Indian Doctrine of
(SSC CGL 1st Sit. 2012)
(a) Aurangzeb
(c) Shahjahan

Malviya ?
(b) Akbar
(d) Bahadur Shah Zafar
40. Who gave the title of "Mahamana" to Madan Mohan
(SSC CGL 1st Sit. 2013)
(a) Swaraj
(c) Satyagraha (d) Ahimsa asy
(b) Swadeshi

29. Who was the founder of the Aligarh Movement ?


(a) Bal Gandadhar Tilak
(b) Dada Bhai Naurozi

(a) Syed Amir Ali En


(SSC CGL 1st Sit. 2012)
(c) Gopal Krishna Gokhale
(d) Mahatma Gandhi

(b) Maulvi Chiragh Ali


(c) Sir Syed Ahmed Khan gin
41. Who was the Viceroy of the time of Quit India Movement?

(a) Lord Irwin


(SSC CGL 1st Sit. 2013)
(d) Abdul Halim Sharar
30. ‘Prince of Pilgrims’ was the name attributed to
(SSC CGL 1st Sit. 2012)
eer
(b) Lord Mountbatten
(c) Lord Wavell
(a) Plutarch
(c) Fa-Hien
(b) Hiuen Tsang
(d) I-Tsing
(d) Lord Lin Lithgow
ing
42. Who was the other Congress leader who joined with Motila
Nehru to start the Swaraj Party in 1923?
l
31. Where did Buddha deliver his first sermon ?
(SSC CGL 1st Sit. 2012) (a) G.K. Gokhale .ne
(SSC CGL 1st Sit. 2013)
(b) B.G. Tilak
(a) Sarnath
(c) Kapilavastu .
(b) Rajagriha
(d) Bodh-Gaya
32. Name the Mughal Prince, who translated Bhagavat Gita into
Persian ? (SSC CGL 1st Sit. 2012)
(c) Chittaranjan Das (d) M.K. Gandhi
43. What is the first sermon of Buddha called as ?

(a) Mahaparinivansutta
t
(SSC CGL 1st Sit. 2013)
(a) Dara Shukoh (b) Sulaiman Shukoh (b) Brahmajalasutta
(c) Khusru (d) Murad (c) Dhammachakkapabattanasutta
33. The surgery that was practised in ancient India is known
(d) Kachchayanagottasutta
from the works of which of the following scholars ?
44. From which of the following banks did Madan Mohan
(SSC CGL 1st Sit. 2012) Malaviya take loans for financing "The Hindustan Times"?
(a) Atreya (b) Sushruta (SSC CGL 1st Sit. 2013)
(c) Charaka (d) Vagbhata (a) State Bank of India
34. Who among the following was the First Viceroy of India? (b) Punjab National Bank
(SSC CGL 2nd Sit. 2012) (c) Bank of Maharashtra
(a) Lord Ripon (b) Lord Curzon (d) Bank of Baroda
(c) Lord Mountbatten (d) Lord Canning 45. Where are the Todas found ? (SSC CGL 1st Sit. 2013)
35. Prithvi Raj Chauhan was defeated in the Second Battle of (a) Madhya Pradesh
Tarain by (SSC CGL 2nd Sit. 2012) (b) Tamil nadu
(a) Mahmud Ghazni (b) Muhammad Ghori (c) Rajasthan
(c) Qutbuddin Aibak (d) Yalduz (d) Aruncachal Pradesh

Downloded From : www.EasyEngineering.net


Downloded From : www.EasyEngineering.net

History 3
46. Rowlatt Act 1919 was enacted during the period of 57. Which of the following movements saw the biggest peasant
(SSC CGL 2nd Sit. 2013) guerilla war on the eve of independence?
(a) Lord Chelmford (b) Lord William (SSC CGL 1st Sit. 2013)
(c) Lord Minto (d) Lord Bentinck (a) Punnapra Vayalar Movement
(b) Telangana Movement
47. Panchayati Raj System was implemented first in the pair of
(c) Noakhali Movement
states (SSC CGL 2nd Sit. 2013) (d) Tebhaga Movement
(a) Andhra Pradesh and Rajasthan 58. The founder of the Lodi Dynasty was
(b) Assam and Bihar (SSC CGL 1st Sit. 2013)
(c) Arunachal Pradesh and Uttar Pradesh (a) Bahlol Lodi
(d) Punjab and Chandigarh (b) Sikandar Shah Lodi
48. Which of the following tribes is associated with the “Tana (c) Jalal Khan Lodi
(d) Ibrahim Lodi
Bhagat” movement? (SSC CGL 2nd Sit. 2013)
59. The Gandharva School of art is also known as the
(a) Uraon (b) Munda (SSC CGL 1st Sit. 2013)
(c) Santhal (d) Kondadora (a) Buddhist-Roman art
49. Who founded the Naujawan Bharat Sabha? (b) Dravidian-Roman art
(a) B. C. Pal (c) Greco-Roman art

ww
(b) G. Subramania Iyer
(c) Sardar Bhagat Singh
(d) Rukmani Lakshmipathi
(d) Greco-Roman-Buddhist art
60. The Bolshevik Revolution of Russia symbolized following:
(SSC CGL 1st Sit. 2013)

inaugurated in 1921 byw.E


50. The Narendra Mandal or Chamber of Princes was
(SSC CGL 2nd Sit. 2013)
(a) coming of the Communist Rule
(b) establishment of Republic of Russia
(c) establishment of Democracy in Russia
(a) Lord Curzon
(b) Lord Wellesley
(c) Duke of Cannaught asy (d) overthrow of Romanov Dynasty
61. Which General, who commanded the British forces against
the Americans in their War of Independence later became
(d) Duke of Wellingdon
51. Buddha, Dhamma and Sangha together are known as En Governor-General of India ?
(a) Dalhousie
(SSC CGL 1st Sit. 2013)
(b) William Bentinck

(a) Triratna
(SSC CGL 2nd Sit. 2013)
(b) Trivarga gin(c) Wellesley (d) Cornwallis
62. Who established the Indian Civil Liberties Union in 1936?
(SSC CGL 1st Sit. 2013)
(c) Trisarga (d) Trimurti
52. Who was called Lichchavi Dauhitra?
eer
(a) Subhash Chandra Bose
(b) Bal Gangadhar Tilak

(a) Chandragupta I
(c) Kumaragupta
(SSC CGL 2nd Sit. 2013)
(b) Skandagupta
(d) Samudragupta
(c) Jawahar Lal Nehru
(d) Rajendra Prasad
ing
63. Which of the following was established first?
53. Pulakesin II was the greatest ruler of the
(SSC CGL 1st Sit. 2013)
(a) Banaras Hindu University
(b) University of Bombay .ne
(SSC CGL 1st Sit. 2013)

(a) Chalukyas of Kalyani


(b) Pallavas of Kanchi
(c) Cholas of Tamil Nadu
(c) Aligarh Muslim University
(d) University of Allahabad t
64. The first Indian Satellite Aryabhatta was launched in
(SSC CGL 1st Sit. 2013)
(d) Chalukyas of Badami
(a) 1972 (b) 1975
54. The Uttaramerur inscription provides information on the
(c) 1977 (d) 1979
administration of the (SSC CGL 1st Sit. 2013) 65. Where did Aurangzeb die? (SSC CGL 1st Sit. 2013)
(a) Pallavas (b) Cholas (a) Pune (b) Aurangabad
(c) Chalukyas (d) Satavahanas (c) Ahmad Nagar (d) Mumbai
55. Pitts India Act of 1784 was a/an (SSC CGL 1st Sit. 2013) 66. Who gave the title of "Sardar" to Ballabh Bhai Patel?
(a) Ordinance (b) Resolution (SSC CGL 1st Sit. 2013)
(c) White paper (d) Regulation Act (a) Mahatma Gandhi (b) Vinoba Bhave
56. Which one of the following statements is not true in respect (c) Women of Bardoli (d) Peasants of Gujrat
67. What Satyagraha was held at Nagpur in 1923?
of A.O. Hume? (SSC CGL 1st Sit. 2013)
(SSC CGL 1st Sit. 2013)
(a) He founded the Indian National Congress.
(a) Salt Satyagraha
(b) He presided over the Congress Annual Sessions twice. (b) Individual Satyagraha
(c) He was an ornithologist. (c) Ryots Satyagraha
(d) He was a member of the Indian Civil Service. (d) Flag Satyagraha

Downloded From : www.EasyEngineering.net


Downloded From : www.EasyEngineering.net

4 History
68. Which one of the following is not a sect of Buddhism? 80. The South East trade winds are attracted towards the Indian
(SSC CGL 1st Sit. 2013) sub continent in the rainy season due to
(a) Mahayana (b) Hinayana (SSC CGL 1st Sit. 2015)
(c) Digambar (d) Theravad (a) the effect of easterlies
69. Who was the viceroy when Delhi became the capital of British (b) the effect of Northern–East trade winds
India? (SSC CGL 1st Sit. 2013) (c) the presence of low atmospheric pressure over North–
(a) Lord Curzon (b) Lord Minto West India
(c) Lord Hardinge (d) Lord Waveli 81. What is 'Reformation'? (SSC CGL 1st Sit. 2015)
70. Multan was named by the Arabs as (SSC CGL 2014) (a) Revival of classical learning
(a) City of beauty (b) City of wealth (b) The revolt against authority of pope
(c) City of gold (d) Pink city (c) Rise of absolute monarchy
71. Which one of the following was the book written by (d) Change in attitude of man
82. Swaraj is my Birth Right and I shall have it. This was
Amoghvarsha, the Rashtrakuta King? (SSC CGL 2014)
advocated by : (SSC CGL 1st Sit. 2015)
(a) Adipurana
(a) Mahatma Gandhi (b) Lala Lajpat Rai
(b) Ganitasara Samgraha
(c) Sardar Patel (d) Lokmanya Tilak
(c) Saktayana
83. Gandhiji's Famous Quit India Movement call to the British
(d) Kavirajamarga
was given in : (SSC CGL 1st Sit. 2015)

ww
72. Who built the Kailasanatha Temple at Ellora?

(a) Rajendra I
(SSC CGL 2014)
(a) 1940
(c) 1941
(b) 1942
(d) 1943
84. In which year was the Indian National Congress formed :
(b) Mahendra Varman I
(c) Krishna I
(d) Govinda I
w.E (a) 1901 (b) 1835 (c) 1875 (d) 1885
85. Bangladesh was created in :
(SSC CGL 1st Sit. 2015)

(SSC CGL 1st Sit. 2015)


mentioned in
(a) Thalavaipuram Copper Plates asy
73. The land measures of the Second Pandyan Empire was
(SSC CGL 2014)
(a) 1973 (b) 1970 (c) 1972 (d) 1971
86. Raja Ram Mohan Roy was the founder of :
(b) Uttirameru Inscription
(c) Kudumiyammalai Inscription En (a) Brahmo Samaj
(SSC CGL 1st Sit. 2015)

(d) Kasakudi Copper Plates


74. Who was the greatest ruler of the Satavahanas? gin(b) Prathna Samaj
(c) Ram Krishna Mission
(d) Arya Samaj
(a) Satkarni I
(b) Gautamiputra Satkarni
(SSC CGL 2014)
Congress :
eer
87. Who initiated the movement to form the Indian National
(SSC CGL 1st Sit. 2015)
(c) Simuka
(d) Hala
(a) Annie Besant
(c) W.C. Banerjee
ing (b) A.O. Hume
(d) Gandhi ji
88. Kamarup is an ancient name of which region of India?
75. The greatest king of the Pratihara dynasty was

(a) Bhoj (Mihir–Bhoj) (b) Dantidurga


(SSC CGL 1st Sit. 2015) (a) Bihar .ne
(SSC CGL 1st Sit. 2016)
(b) Rajasthan
(c) Nagbhatta II (d) Vatsaraj
76. In 1939 Subhash Chandra Bose was elected as President of
the Congress Party defeating
(a) Jawaharlal Nehru
(SSC CGL 1st Sit. 2015)
(c) Karnataka (d) Assam

t
89. Which battle led to the downfall of the Vijayanagar empire?
(SSC CGL 1st Sit. 2016)
(a) Battle of Takkolam (b) Battle of Talikota
(c) Battle of Kanwah (d) Battle of Panipat
(b) Maulana Abul Kalam Azad 90. Who introduced Mansabdari system in India?
(c) V.B. Patel (SSC CGL 1st Sit. 2016)
(d) Pattabhi Sitharamayya (a) Babur (b) Humayun
77. Jallianwala incident took place at (SSC CGL 1st Sit. 2015) (c) Akbar (d) Jahangir
(a) Lucknow (b) Surat 91. The ‘Cabinet Mission’ of 1946 was led by 1946
(c) Amritsar (d) Allahabad
(SSC CGL 1st Sit. 2016)
78. Who was the founder of Lodhi dynasty ?
(a) Lord Linlithgow (b) Lord Mountbatten
(a) Sikandar Lodhi (b) Bahlol Lodhi
(c) Sir Pethic Lawrence (d) Sir Mountford
(c) Ibrahim Lodhi (d) Daulat Khan Lodhi
92. What did Gandhiji meant by ‘Sarvodaya’?
79. Which one of the following pair is not correctly matched ?
(SSC CGL 1st Sit. 2016)
(SSC CGL 1st Sit. 2015)
(a) Non-violence
(a) Akbar – Todarmal
(b) Upliftment of untouchables or dalits
(b) Chanakya – Chandragupta
(c) The birth of a new society based on ethical values
(c) Vikramaditya – Chaitanya
(d) Satyagraha
(d) Harshvardhan – Hiuen Tsang

Downloded From : www.EasyEngineering.net


Downloded From : www.EasyEngineering.net

History 5
93. Harsha moved his capital from _____ to ____. 104. Which one of the following is the most lasting contribution
(SSC CGL 1st Sit. 2016) of the Rastrakutas ? (SSC CHSL 2014)
(a) Thanesar, Kanauj (b) Delhi, Deogiri (a) Kailasha Temple
(c) Kamboj, Kanauj (d) Valabhi, Delhi (b) Pampa, Ponna, Ranna, the three writers of Kannada
Poetry and Kailasha Temple
94. A collective term used by the Jains for their sacred books is
(c) Patronage of Jainism
(SSC CGL 1st Sit. 2016) (d) Conquests
(a) Prabandhas (b) Agamas 105. Ravikirti, a Jain, who composed the Aihole Prashasti, was
(c) Nibandhas (d) Charits patronized by (SSC CHSL 2014)
95. Who propounded the "Doctrine of Passive Resistance"? (a) Pulakeshi I (b) Harsha
(SSC CGL 1st Sit. 2016) (c) Pulakeshi II (d) Kharavela
(a) Balgangadhar Tilak 106. When did the reign of Delhi Sultanate came to an end ?
(SSC CHSL 2014)
(b) Aurobindo Ghosh
(a) 1498 A.D. (b) 1526 A.D.
(c) Lala Lajpat Rai
(c) 1565 A. D. (d) 1600 A.D.
(d) Bipin Chandra Pal 107. The original founder of the Manuscripts and Editor of
96. The first Muslim to be elected President of 'Indian National Kautilya's Arthashastra was (SSC CHSL 2014)
Congress' was? (SSC CGL 1st Sit. 2016) (a) Srikanta Shastri (b) Srinivasa lyangar
(a) Maulana Azad (b) Mohammed Ali (c) R. Shamashastri (d) William Jones

ww
(c) Badruddin Tyabji (d) Shah Wali-ullah
97. The Dandi March of Gandhi-is an example of
(SSC CHSL 2013)
108. Whose army did Alexander, the Greek ruler confront on the
banks of the river Jhelum ?
(a) Ambi
(SSC CHSL 2015)
(b) Chandragupta Maurya
(a) Non-Coopefation
(c) Boycott w.E (b) Direct Action
(d) Civil Disobedience
98. Which one of the following inscriptions relate to the
(c) Porus

th
(c) 15 June 2005
(d) Dhanananda
109. When was RTI Act enacted in India ? (SSC CHSL 2015)
(a) 15th August 2005 (b) 15th March 2005
(d) 15th July 2005
Chalukya king, Pulakesin II ?
(a) Nasik (b) Maski asy(SSC CHSL 2013) 110. The famous activist Medha Patakar is associated with which
movement ?
(a) Narmada Bachao Andolan
(SSC CHSL 2015)

(c) Hathigumpha (d) Aihole


99. Who among the following introduced the Mansabdari En (b) Save the Tiger
(c) Preserve the wet lands
system ?
(a) Jahangir
(SSC CHSL 2013)
(b) Shah Jahan gin (d) Beti Padao Andolan
111. Who built the "Purana Quilla"?
(a) Babar
(SSC CHSL 2015)
(b) Shershah
(c) SherShah (d) Akbar
100. Which one of the following cities and the personalities
eer
(c) Aurangzeb (d) Akbar
112. Where did Chandragupta maurya spent his last days ?
associated with their establishment is wrongly matched ?

(a) Calcutta - Robert Clive


(SSC CHSL 2013) (a) Thaneshwar
(c) Patliputra ing (b) Kanchi
(SSC CHSL 2015)

(d) Shravanabelagola
(b) Pondicherry - Francis Martin
(c) Ahmedabad Ahmad Shah I
113. Who is the author of 'Indica'?
(a) Fa–Hien
(c) Megasthanes .ne (SSC CHSL 2015)
(b) Hiuen Tsang
(d) Seleucus
(d) Madras - Francis Day
101. Which one of the following wars decided the fate of the
French in India ? (SSC CHSL 2013)
114. Who built the famous Shiva temple at Ellora ?

(a) Mauryan Emperor Ashoka


(b) Gupta King Samudra Gupta
t
(SSC CHSL 2015)

(a) Battle of Plassey


(c) Chalukyan King Pulikeshi II
(b) Battle of Wandiwash
(d) Rashtrakuta Ruler Krishna I
(c) First Carnatic War 115. Who composed the Allahabad Pillar inscription ?
(d) Battle of Buxar (SSC CHSL 2015)
102. The Crimean War came to an end by the (a) Mahasena (b) Veerasena
(SSC CHSL 2013) (c) Vishnusena (d) Harisena
(a) Treaty of St. Germain 116. Todar Mal, the brilliant revenue officer served under :
(b) Treaty of Trianon (SSC CHSL 2015)
(c) Treaty of Versailles (a) Bhagwan Das (b) Humayun
(d) Treaty of Paris (c) Baz Bahadur (d) Sher Shah
103. Tulsidas wrote Ramcharitmanas in the reign of 117. When was the League of Nations established ?
(SSC CHSL 2014) (SSC CHSL 2015)
(a) Babar (b) Akbar (a) In 1920 (b) In 1939
(c) Aurangzeb (d) Jahangir (c) In 1914 (d) In 1918

Downloded From : www.EasyEngineering.net


Downloded From : www.EasyEngineering.net

6 History
118. In which of the Round Table Conference Mahatma Gandhi 129. Where was Christopher Columbus from?
participated ? (SSC CHSL 2015) (SSC Multitasking 2013)
(a) First Round Table Conference, 1930 (a) Portugal (b) Venice
(b) Second Round Table Conference, 1931 (c) Genoa (d) Spain
(c) Third Round Table Conference, 1932 130. To make tools and weapons, the earliest inhabitants of India
used (SSC Multitasking 2014)
(d) All of the above (a) clay (b) wood
119. The use of which of the following regional languages was (c) stones (d) bronze
popularised by the Bhakti leader, Shankaradeva ? 131. Name an Indian Saint (priest) who, attended the "World
(SSC CHSL 2012) Congress of Religions" held at Chicago (U.S.) in 1893.
(a) Bengali (b) Brijbhasha (SSC Multitasking 2014)
(c) Avadhi (d) Assamese (a) Basaveshwara (b) Swami Vivekananda
120. The Mansabdari system was introduced by: (c) Ramanuja (d) Madhavacharya
(SSC CHSL 2012) 132. Chhatrapati Shivaji was a follower of
(a) Shah Jahan (b) Jahangir (SSC Multitasking 2014)
(c) Babur (b) Akbar (a) Madhava (b) Kabir
121. When and by whom were the Asokan inscriptions (c) Basava (d) Ramadasa
deciphered for the first time ? (SSC CHSL 2012) 133. Who was the greatest Kushan ruler?

ww
(a) 1787 - John Tower
(c) 1837 - James Prinsep
122. Amuktamalyada is the work of :
(b) 1825 - Charles Metcalfe
(d) 1810 - Harry Smith
(SSC CHSL 2012)
(a) Vashiska
(c) Huvishka
(SSC Multitasking 2014)
(b) Vasudeva
(d) Kanishka
(a) Krishnadeva Raya
(c) Kharavela w.E (b) Vachcharaj
(d) Allasani Peddana
123. Name the Commander of the Arab army who conquered the
134. Who was the founder of Arya Samaj?

(a) Kabir Das


(SSC Multitasking 2014)
(b) Shankaracharya
Sindh.
(a) Muhammad bin Qasim
(b) Al Hazzaz
asy
(SSC Multitasking 2013)
(c) Ranade (d) Dayanand Saraswati
135. The French challenge to British in India came to an end with
(SSC Sub. Ins. 2012)
(c) Qutbuddin Aibak
(d) Allauddin Khilji En (a) Battle of Wandiwash
(b) Battle of Srirangapattinam
124. Who was the chairperson of the Chinese Communist Party
at the time of liberation of China? gin (c) Battle of Plassey
(d) Battle of Buxar
136. Identify the medical trio of Ancient India from the following
(a) Liu Shaoqi
(c) Deng Xiaoping
(SSC Multitasking 2013)
(b) Zhou Enlai
(d) Mao Zedong
names.
eer
(a) Charaka, Susruta and Vagbhata
(SSC Sub. Ins. 2012)

125. Who was regarded by Gandhiji as his political Guru?


(SSC Multitasking 2013) ing
(b) Charaka, Vatsyayana and Vagbhata
(c) Charaka, Susruta and Bharata
(d) Charaka, Susruta and Patanjali
(a) Gopal Krishna Gokhale
(b) Lala Lajpat Rai
(c) Bipin Chandra Pal .ne
137. Match the medieval travellers with their countries :
(SSC Sub. Ins. 2012)
(d) Bal Gangadhar Tilak
126. Which among the following movements was not led by
Mahatma Gandhi? (SSC Multitasking 2013)
A. Marco Polo
B. Ibn Battuta
C Antonio Monserrate 3. Italy
1. Spain
2. Balkh

D. Mahmud Wali Balkhi 4. Morocco


t
(a) Civil Disobedience Movement
A B C D A B C D
(b) Quit India Movement
(a) 4 3 1 2 (b) 3 1 4 2
(c) Swadeshi-Movement
(c) 3 4 1 2 (d) 1 3 2 4
(d) Non-Cooperation Movement
138. Match the following : (SSC Sub. Ins. 2012)
127. In which session of the Indian National Congress was the
A. Sarojini Naidu 1. Muslim League
“Poorna Swaraj” resolution adopted?
B. M.A. Jinnah 2. Indian National Congress
(SSC Multitasking 2013)
C. Sir Tej Bahadur Sapru 3. Hindu Mahasabha
(a) Karachi Session in 1931
D. V.D. Savarkar 4. Liberal Party
(b) Lucknow Session in 1916
A B C D A B C D
(c) Belgaum Session in 1924
(a) 2 1 4 3 (b) 2 1 3 4
(d) Lahore Session in 1929
(c) 2 4 1 3 (d) 4 1 3 2
128. The school of arts developed during the Kushan Period
139. Which Sultan received a robe of honour from the caliph?
with the mixture of Indian and Greek style is known as
(SSC Multitasking 2013) (SSC Sub. Ins. 2013)
(a) Mughal art (b) Kushan art (a) Ala-ud-din Khilji (b) lltutmish
(c) Persian art (d) Gandhara art (c) Balban (d) Qutub-ud-din Albak

Downloded From : www.EasyEngineering.net


Downloded From : www.EasyEngineering.net

History 7
140. When Margaret Thatcher was P.M. with which country 149. Which was the second capital of Akbar ?
Britain waged a war to regain Control of Falkland Islands? (SSC Sub. Ins. 2014)
(SSC Sub. Ins. 2013) (a) Delhi (b) Agra
(a) Chile (b) Argentina (c) Fatehpur-Sikri (d) Patna
(c) Brazil (d) None of the above 150. The first country which discovered sea route to India was
141. Match the following (SSC Sub. Ins. 2013) (SSC Sub. Ins. 2014)
(a) Chalukyas (b) Hoysalas (a) Portugal (b) Dutch
(c) Rashtrakutas (d) Kakatiyas (c) French (d) Britain
151. The unification of Karnataka was achieved in the year
(i) Malkhed (ii) Vatapi
(SSC Sub. Ins. 2014)
(iii) Warangal (iv) Dwarasamudra
(a) 1956 (b) 1957
(a) (a)-(ii), (b )-(iv), (c)-(i), (d)-(iii) (c) 1958 (d) 1960
(b) (a)-(iv), (b)-(iii), (c)-(i), (d)-(ii) 152. Who introduced the Indian University Act?
(c) (a)-(i), (b)-(ii), (c)-(iii), (d)-(iv) (SSC Sub. Ins. 2014)
(d) (a)-(iii), (b)-(ii), (c)-(iv),(d)-(i) (a) Lord Curzon (b) Lord Minto
142. The world's tallest statue of Mahatma Gandhi is in (c) Lord Morelay (d) Lord Rippon
(SSC Sub. Ins. 2013) 153. Chinese travellers visited India primarily because
(a) Champaran (b) Patna (SSC Sub. Ins. 2014)

ww
(c) Lucknow (d) Rajkot
143. Which one of the following is the principal source of
information on Asoka's campaign against Kalinga ?
(a) they were interested in Buddhism
(b) they were invited by the Indian kings
(c) they were interested to study Indian culture

(a) Pillar Edict VII


(c) Divyavadana
w.E (SSC Sub. Ins. 2013)
(b) Mahavamsa
(d) Rock Edict XIII
(d) they were interested to stay in India
154. Development that meets the needs of the present, without
compromising the ability of future generations to meet their

asy
144. Gandhi's concept of Trusteeship: (SSC Sub. Ins. 2013)
(a) Recognises right of private ownership of property
own needs was the focal point of Brundtland commission is
(SSC Sub. Ins. 2014)

one
En
(b) Transforms the capitalistic society into an egalitarian
(a) sustainable development
(b) mitigation
(c) disaster management
(c) Excludes legislative regulation of the ownership and
use of wealth
(d) Does not fix minimum or maximum income
gin (d) capacity building
155. Name the Kingdom which first used elephants in wars?
145. Match the following:
(a) Hunter's Commission
(SSC Sub. Ins. 2013)
eer
(a) Avanti
(SSC Sub. Ins. 2015)
(b) Champa
(b) Wardha Scheme
(c) University's Act
(c) Magadha

ing (d) Kosala


156. Tulsidas wrote Ramacharitamans during the reign of :
(SSC Sub. Ins. 2015)
(d) Radhakrishnan Commission
(i) 1948
(iii) 1937
(ii) 1904
(iv) 1882
(a) Akbar
(c) Jahangir .ne
(b) Krishnadeva Raya
(d) Rama Raya
(a) (a)-(iii), (b)-(ii), (c)-(iv), (d)-(i)
(b) (a)-(iv), (b)-(ii), (c)-(iii), (d)-(i)
(c) (a)-(iii) (b)-(iv), (c)-(i), (d)-(ii)
(a) 1945
(c) 1940
(b) 1946
(d) 1942
t
157. Cripps Mission came to India in: (SSC Sub. Ins. 2015)

158. Who among the following Sultans tried to prohibit Sati?


(d) (a)-(iv), (b)-(iii), (c)-(ii), (d)-(i)
(SSC Sub. Ins. 2015)
146. The Gandhara art nourished under:
(a) Muhammad Bin Tuglaq (b) Firoz Tuglaq
(SSC Sub. Ins. 2013) (c) Jalaluddin Khilji (d) Alauddin Khilji
(a) the Kushanas (b) the Satavahanas 159. Who constructed Humayun’s Tomb in Delhi ?
(c) the Guptas (d) the Mauryas (SSC Sub. Ins. 2015)
147. Most important safeguard of liberty is (a) Haji Begam (b) Babar
(SSC Sub. Ins. 2014) (c) Humayun (d) Akbar
(a) bold and impartial judiciary 160. Who among the following Mughal rulers banned music
(b) well-knit party system and dancing ? (SSC Sub. Ins. 2015)
(c) decentralisation of power (a) Jahangir (b) Babar
(d) declaration of rights (c) Aurangzeb (d) Humayun
148. What is the ancient school of law ? (SSC Sub. Ins. 2014) 161. Who was the governor-general during the Second Anglo-
(a) The Philosophical School Mysore War? (SSC Sub. Ins. 2015)
(b) The Historical School (a) Lord Wellesley (b) Sir John Shore
(c) The Analytical School (c) Lord Cornwallis (d) Warren Hastings
(d) The Sociological School

Downloded From : www.EasyEngineering.net


Downloded From : www.EasyEngineering.net

8 History
162. Which of the following Pacts sought to be resolved the 173. which of the following was not built by Firoz Shah Tughlaq?
Hindu - Muslim differences? (SSC Sub. Ins. 2016) (SSC Stenographer 2013)
(a) Lucknow Pact (b) Lahore Pact (a) Firozabad (b) Fatehabad
(c) Gandhi - Irwin Pact (d) Poona Pact (c) Tughlaqabad (d) Jaunpur
163. In which of the following year the Gandhi-Irwin Pact was 174. who among the follwing granted permission to the English
signed? (SSC Sub. Ins. 2016) to establish their factory in India?
(a) 1932 (b) 1935 (SSC Stenographer 2013)
(c) 1931 (d) 1929 (a) Akbar (b) Jehangir
164. The Vedic literature was composed between (c) Shah Jahan (d) Aurangzeb
(SSC Sub. Ins. 2016) 175. The permanent Land Settlement in Bengal was introduced
(a) 3500 B.C. and 1000 B.C. by (SSC Stenographer 2013)
(b) 2500 B.C. and 500 B.C. (a) Warren Hestings (b) Thomas Munro
(c) 3000 B.C. and 1000 B.C. (c) Thomas Reid (d) Lord Cornwallis
(d) 3500 B.C. and 2500 B.C. 176. Din-I-llahi was founded by (SSC Stenographer 2014)
165. Who among the following was the Prime Minister of England (a) Kabir (b) Akbar
when India was given independence? (SSC Sub. Ins. 2016) (c) Shah jahan (d) Guru Nanak Dev
(a) Mountbatten (b) Churchill 177. 'Purna Swaraj' was announced in the Indian National
(c) Attlee (d) Wavell Congress Session of (SSC Stenographer 2014)

ww
166. How many times did Babur invade India before 1526 A.D.?
(SSC Sub. Ins. 2016)
(a) Calcutta
(c) Nagpur
(b) Lahore
(d) Karachi
178. Who chaired the Boundary Commission in 1947, to demarcate
(a) None of these
(c) Five times
w.E (b) Four times
(d) Two times
167. Which of the following countries has become the first
country in the world to receive funds from United Nations
the boundary line between India and pakistan?

(a) Krishna Menon


(SSC Stenographer 2014)
(b) McMohan

asy
for its fast growing Solar Home Systems?
(SSC Sub. Ins. 2016)
(c) J. Nehru
179. Ashoka embraced Buddism
(a) before the Kalinga war
(d) Henderson Brooks
(SSC Stenographer 2014)
(a) Afghanistan
(c) Bangladesh
(b) Burma
(d) Switzerland
En (b) after the Kalinga war
(c) just before his death
168. In Indian architecture 'Surkhi' was introduced by

(a) Mughals
(SSC Sub. Ins. 2016)
(b) Kushans gin (d) when he was a Crown Prince
180. The original name of 'Mahabharata' is
(c) Guptas (d) Sultanate Sultans
169. which one of the following land reform measures can be eer
(a) Rajtarangini
(c) Kathasaritasagar
(SSC Stenographer 2016)
(b) Bharat Katha
(d) Jai Samhita
said to have been fully implemented by now in India?

(a) Tenancy reforms


(SSC Stenographer 2013)
ing
181. Sir Eyre Coote was associated with which of the following?
(SSC Stenographer 2016)
(b) Abolition of intermediaries
(c) Land ceilings
(a) Battle of Ambur
(c) Battle of Wandiwash
.ne
(b) Battle of Arcot
(d) Battle of Adyar
182. Which among the following is related to Sadr-us-Sadr?
(d) Consolidation of holdings
170. Which of the following recommended reservation for the
Other Backward Classes (OBCs)? (SSC Stenographer 2013)
(a) Mandal Commission
(a) Ecclesiastical matters
(b) Judicial administration
(c) Military administration
t
(SSC Stenographer 2016)

(b) Kothari Commission (d) Land revenue


(c) Sachar Committee 183. Which among the following inscription is known as Prayaga
(d) None of these Prashasti ? (SSC Stenographer 2016)
171. Which of the following was not an aspect of Later Vedic (a) Hathigumpha Inscription
Age? (SSC Stenographer 2013) (b) Aihole Inscription
(a) Importance of Kingship in political life (c) Mehroli Inscription
(b) Discovery of iron (d) Allahabad Pillar Inscription
(c) Polygamy 184. Which of the following pair is NOT correctly matched?
(d) Simple, non-ritualistic worship (SSC Stenographer 2016)
172. Who among the follwing presided over the fourth Buddhist (a) Ibn Batuta - Morocco
Council? (SSC Stenographer 2013) (b) Nikitin - Samarkand
(a) Ashoka (b) Kanishka (c) Marco Polo - Italy
(c) Ashvaghosha (d) Vasumitra (d) Alberuni - Uzbekistan

Downloded From : www.EasyEngineering.net


Downloded From : www.EasyEngineering.net

History 9

HINTS & SOLUTIONS


1. (d) 27. (a) As Samhita is the collection of the mantras, so
2. (d) The Servants of India Society was formed in Pune, sometimes Samhitas are referred to as Mantras. Most
Maharashtra, on June 12, 1905 by Gopal Krishna of these mantras or hymns are concerned with nature
Gokhale, who left the Deccan Education Society to and deities.
form this association. 28. (d) 29. (c) 30. (b)
3. (d) 31. (a) 32. (a) 33. (b)
4. (b) Upagupta (c. 3rd Century BC) was a Buddhist monk. 34. (d) Lord Canning was the Governor General of India from
According to some stories in the Sanskrit text 1856 - 1862 and the first Viceroy in India from 1
Ashokavadana, he was the spiritual teacher of the November 1858. Lord Mountbatten was the First
Mauryan emperor Ashoka. Governor General of Independent India.
5. (c) 6. (b) 7. (c)
35. (b) 1191 - First Battle of Tarain in which Prithviraj Chauhan
8.
ww
(b) The doctrine of lapse was an annexation policy
purportedly devised by Lord Dalhousie, who was the
Governor General for the East lndia Company in lndia
defeated Mohd. Ghori.1192 - Second Battle of Tarain
in which Mohd.Ghori defeated Prithviraj Chauhan.

9.
14.
w.E
between 1848 and 1856.
(c) 10. (b) 11. (b) 12. (b) 13. (d)
(d) 15. (b) 16. (b) 17. (c) 18. (c)
36.
37.
(d)
(c) The first major attempt in curriculum reconstruction in
India was made in 1937 when Gandhiji propounded the

19.
20.
(a)
asy
(a) King Asoka assumed the title Devanampiya Piyadasi
38.
idea of Basic Education.
(b) Simon Commission (1927) > Dandi March (1930) >
which means "Beloved-of-the-Gods, He who Looks
on with Affection". En 39.
Gandhi Irwin Pact (1931) > Poona Pact (1932)
(b) Emperor Akbar named Prayag as Allahabad - City of
21. (b) Shivaji, the great Maratha King and founder of a
nationalist tradition was contemporary of Tukaram. gin God- also called Allahabad in 1575 AD. The city of
Allahabad is situated at the confluence of three rivers
Tukaram introduced Shivaji to Ramdas for his spritual
education. eer
- Ganga, Yamuna and the invisible Saraswati. Every
12th year when the waters are felt to be especiall y
purifying, Allahabad holds a much greater festival called
22. (b) Wheat was the first cereal to be cultivated by man. In
several places in the Middle East it was sowed, tended
and reaped soon after 8000 BC. The people of Jericho
ing
Kumbh Mela. Built by Emperor Akbar in 1583 AD, the
Allahbad fort stands on the banks of the river Yamuna
are the first known to have lived mainly from the
cultivation of crops. 40. .ne
near the confluence site i.e SANGAM.
(d) Madan Mohan Malaviya was an Indian educationist
23. (c) After approximately 200 years of French rule, Alsace
and the German-speaking part of Lorraine were ceded
to Germany in 1871 under the Treaty of Frankfurt. In
t
and politician notable for his role in the Indian
independence movement and as the two time president
of Indian National Congress. He was respectfully
1919, both regions were returned to France. addressed as Pandit Madan Mohan Malaviya and also
24. (d) James Princep was an English scholar and antiquary. addressed as 'Mahamana' by Mahatama Gandhi.
From 1832 to 1838 he was assay-master in the India 41. (d) Lord Linlithgow was Viceroy of India from 1936 to 1944
Government Mint, Kolkata. He is most noted as a and this eight years period was longest reign as Viceroy
philologist for fully deciphering and translating the of India.
rock edicts of Asoka from the Brahmi script. 42. (c)
25. (c) Tilak propounded the theory of the Arctic home of the 43. (c) The Dhammacakkappavattana Sutta is considered to
Aryans, meaning that the Aryans originated in the be a record of the first teaching given by Gautama
Arctic region, and later, on the journey south, divided Buddha after he attained enlightenment. The main topic
into two branches. One branch went to Europe, while of this sutra is the Four Noble Truths, which are the
the other branch came to India.
central teachings of Buddhism that provide a unifying
26. (b) Alara Kalama was a hermit saint and a teacher of yogic theme, or conceptual framework, for all of Buddhist
meditation who lived near Rajagriha. According to the
thought.
Pali Canon scriptures, he was one of the teachers of
44. (b) 45. (b)
Gautama Buddha.

Downloded From : www.EasyEngineering.net


Downloded From : www.EasyEngineering.net

10 History
46. (a) The Rowlatt Act was passed by the Imperial Legislative Indian National Congress. Thought it was he who
Council in London on 18 March 1919, indefinitely emerged as the founder of the Congress, it had
extending "emergency measures" enacted during the somehow never happened to Hume to preside over a
First World War in order to control public unrest and plenary session even once.
root out conspiracy. Lord Chelmsford (1916 to 1921) 57. (b) The Telangana Movement was the biggest peasant
was the Viceroy of India then. guerrilla war of modern Indian history affecting 3000
47. (a) Jawaharlal Nehru inaugurated the first generation of villages and 3 million population. The uprising began
panchayat raj at Nagaur in Rajasthan on 2 October in july 1946 and was at its greatest intensity between
1959. It was also implemented in Andhra Pradesh in August 1947 and September 1948.
the same year as per the recommendations of the 58. (a) Bahlul Khan Lodi was the founder of Lodi dynasty of
Balwant Rai Mehta Committee. the Delhi Sultanate in India upon the abdication of the
48. (a) Tana Bhagat movement was apparently initiated in 1914 last claimant from the previous Sayyid rule.
by a young Oraon tribesman known as Jatra Oraon. 59. (d) Greco-Roman-Buddhist Art
49. (c) The Naujawan Bharat Sabha was founded by Bhagat 60. (a) the Bolshevik Revolution, was a seizure of state power
Singh in March 1926 and was declared illegal under instrumental in the larger Russian Revolution of 1917.
the Criminal Law Amendment Act of 1908 in September It took place with an armed insurrection in Petrograd

ww
1934. Its purpose was to forment revolution against
the British Raj by gathering together worker and 61. (b)
traditionally dated to 25 October 1917.
Lieutenant-General Lord William Henry Cavendish-

50.
peasant youths.

w.E
(c) The chamber of Princes was instituted on 8 February
1921 . The inauguration ceremony was performed by
Bentinck, GCB, GCH, PC, known as Lord William
Bentinck, was a British soldier and statesman. He
served as Governor-General of India from 1828 to 1835.

asy
His Royal Highness the Duke of Connaught in the
Diwan–i–am of Red Fort on be; half of His Majesty the
King Emperor.
62.
66.
(c)
(a)
63. (b) 64. (b) 65. (c)
67. (d) 68. (c) 69. (c)

51.
En
(a) The Three Jewels (triratna) are the three things that
70. (c) During the early period, Multan was known as the city
of gold for its large and wealthy temples. The Sun
Buddhists take refuge in, and look toward for guidance,
in the process know as taking refuge. The Three Jewels
are: Buddha, Dhamma and Sangha. gin
71. (d)
temple, Suraj Mandir, was considered one of the largest
and wealthiest temples in the entire sub-continent.
Amoghavarsha I was a follower of the Digambar a
52 (a) Chandragupta I was known as 'Lichchavidauhitra.' His
marriage with the Lichchavi Princess Kumaradevi was eer
branch of Jainism. His own writing Kavirajamarga is
landmark literary work in the Kannada language and
a

one of the significant events in the Gupta rule. The


importance of this marriage can be known further from ing
became a guide book for future poets and scholars fo r
centuries to come.
Samudragupta's Allahabad inscription in which he has
described himself as "Lichchhavis–dauhitra or
72. (c)
.ne
The Kailasa temple is a famous rock cut monument,
one of the 34 monasteries and temples known

53.
daughter's son of the Lichchavis.
(b) Pulakesin II (610 – 642 AD) was the most famous ruler
of the Chalukya dynasty. In his reign the Chalukyas of
Badami saw their kingdom extend over most of the
t
collectively as the Ellora Caves, extending over more
than 2 km.The temple was commissioned and completed
between dated 757-783 CE, when Krishna I ruled the
Rashtrakuta dynasty. It is designed to recall Mount
Deccan. Kailash, the home of Lord Shiva. It is a megalith carved
54. (b) An inscription of the 8th century AD at Uttaramerur out of one single rock. It was built in the 8th century
temple describes the constitution of the local council, by the Rashtrakuta king Krishna I.
eligibility and disqualifications for the candidates, the 73. (a) the Thalavaipuram copper plate, belonging to the period
method selection, their duties and delimits their power between 1018 and 1054 brought out by the Pandya
in Chola dynasty. kings, describes giant waves, most possibly a tsunami.
55. (b) The Pitt’s India Act of 1784 was in the nature of a 74. (b) Gautamiputra Satakarni was the greatest of the
regulating act as it was intended to address the Satavahana rulers. His reign period is noted by some
shortcomings of the Regulating Act of 1773 by bringing scholars as 80 to 104 and by others from 106 to 130; in
the East India Company’s rule in India under the control any case he is credited with a rule of 24 years.
of the British Government. 75. (a) Mihira Bhoja was a ruler of the Gurjara Pratihara
56. (b) Allan Octavian Hume was a civil servant, political dynasty 2of India. Bhoja's empire extended to Narmada
reformer and amateur ornithologist and horticulturalist River in the South, Sutlej River in the northwest, and
in British India. He was one of the founders of the up to Bengal in the east.

Downloded From : www.EasyEngineering.net


Downloded From : www.EasyEngineering.net

History 11
76. (d) Bose appeared at the 1939 Congress meeting and was Deccan sultanates, resulted in a defeat of Vijayanagara,
elected president over Gandhi's preferred candidate and ended in greatly weakening one of the greatest
Pattabhi Sitaramayya. Indian Empires originating from Southern India before
77. (c) The Jallianwala Bagh Massacre happened in Amritsar, the Maratha Empire.
in 1919. It is named after the Jallianwala Bagh (Garden) 90. (c) The mansabdari system was of Central Asian origin
atAmritsar. On April 13, 1919, British, Indian Army and it was first introduced by Babur in North India.
soldiers started shooting an unarmed gathering of men, But it was Akbar who institutionalized it in Mughal
women and children. military set up and civil administration.
78. (b) Bahlol Lodi was the founder of the Lodi dynasty.He 91. (c) Cabinet Mission was composed of three Cabinet
ruled for long thirty-nine years (1451-89). He was the Ministers of England Sir Pethick Lawrence, Secretary
governor of Lahore and Sirhind during the rule of of State for India, Sir Stafford Cripps, President of the
Muhammad Shah of Sayyid dynasty. Board of Trade;
79. (c) Sri Chaitanya Mahaprabhu arrived in the empire at the Alexander, the First Lord of the Admiralty. The mission
time of Emperor Prataparudra (Gajapatis)and stayed arrived on March 24, 1946. The objective of this mission
for 18 long years at Puri. was to Devise a machinery to draw up the constitution
80. (c) The instance heat that prevails in the Indian Sub of Independent India. Make arrangements for interim

ww
continent causes a low pressure region over the
northern plains. It is intense enough to attract the
moisture bearing winds from the Indian Ocean .Thus
Government. Thus the mission was like a declaration
of India's independence.

w.E
the south east trade winds from the southern
hemisphere are attracted towards India.
92. (c) Sarvodaya is a Sanskrit term meaning 'universal uplift'
or 'progress of all'. The term was used by Mahatma
Gandhi as the title of his 1908 translation of John
81.
asy
(b) Reformation was a 16th-century movement in Western
Europe that aimed at reforming some doctrines and
practices of the Roman Catholic Church and resulted
Ruskin's tract on political economy, Unto This Last,
and Gandhi came to use the term for the ideal of his
own political philosophy.

82.
in the establishment of the Protestant churches.
En
(d) Bal Gangadhar Tilak is considered as "Father of Indian
93. (a)

National Movement". He was a social reformer, freedom


fighter, national leader and a scholar of Indian history, gin
94.

95.
(b) The collective term given by the Jainas to their Sacred
literature is called Agamas written in Prakrt.
(b) Doctrine of Passive Resistance is authored by
sanskrit, hinduism, mathematics and astronomy. During
freedom struggle, his slogan "Swaraj is my birthright eer
Aurobindo Ghosh. It is based on a series of articles b y
Aurobindo Ghosh which were published in April 190 7

83.
and I shall have it" inspired millions of Indians.
(b) The Quit India Movement(August Kranti), was a civil ing
in the journal Bande Matram. The articles were written
when the Bengal was burning with indignation after
disobedience movement launched in India on 9 August
1942 by Mohandas Karamchand Gandhi.
.ne
its partition October 16, 1905 by Viceroy Curzon. The
main theme of the articles is the methods and ideology
84. (d) The Indian National Congress was founded on
December 28, 1885, by members of the Theosophical
Society. The founders included a prominent member
of the Theosophical Society, Allan Octavian Hume as 96.
t
which could be adopted in face of oppression and
injustice as perpetrated by the contemporary British
rule in India.
(c) Badruddin Taiyabji became the Ist Indian Barrister in
well as Dadabhai Naoroji and Dinshaw Wacha. Bombay; became the 2nd Indian Chief Justice; was the
85. (d) Modern Bangladesh emerged as an independent nation founding member of Bombay presidency association
in 1971 after achieving independence from Pakistan in and INC and also presided over the 3rd congress
the Bangladesh Liberation War. The Provisional session in Madras in 1887.
Government of Bangladesh was formed on 17 April 97. (d) The Dandi March of Gandhi was an important part of
1971. the Indian Independence Movement.It was a direct
86. (a) 'Brahmo Samaj' was founded by Raja Ram Mohan Roy action campaign of tax resistance and non-violent
in 1828. protest against British saltmonopoly and triggered the
87. (b) Allan Octavian Hume was a civil servant, political wider Civil Disobedience Movement.
reformer in British India who initiated the movement to 98. (d) Aihole inscription is found at Aihole in Karnataka state
form the Indian National Congress. India, was written by the Ravikriti,court poet of
88. (c) Chalukya king,Pulakeshin II who reigned from 610 to
89. (b) The Battle of Talikota (26 January 1565), a watershed 642 CE.The poetic verses of Ravikirti,in praise of the
battle fought between the Vijayanagara Empire and the king, can be read in the Meguti temple,dated 634CE.

Downloded From : www.EasyEngineering.net


Downloded From : www.EasyEngineering.net

12 History
99. (d) Akbar introduced the Mansabdari system. This system 107. (c) R. Shamashastri transcribed, edited and published the
came under the military reforms of Akbar. Under this Sanskrit edition in 1909. He proceeded to translate it
system each officer was assigned a rank(mansab). into English, publishing it in 1915.
Varying from 10 to 10,000, the mansab carried the Zat(the 108. (c) Porus fought Alexander the Great in the Battle of the
personal status and salary) and Sawar (the number of Hydaspes (also known as Jhelum) in 326 BC and is
cavalry men to be maintained. believed to be defeated.
100. (a) Calcutta with Robert Clive is wrongly matched. Lord 109. (c) Right to Information Act (RTI) was enacted on 15th
Curzon was associated with Calcutta. Lord Curzon felt June 2005 and came fully into force on 12th October
that the Bengal province was too big to be administered 2005.
efficiently and so he wanted to split it into two 110. (a) Indian Social activist Medha Patkar is the founder
provinces, one of which had Dacca as its capital. member of Narmada Bachao Andolan.
101. (b) Battle of Wandiwash decided the fate of French in 111. (b) Purana Qila was built by the Afghan king Sher Shah
India. Battle of Wandiwash, (Jan. 22, 1760), in the history Suri.
of India, a confrontation between the French, under the
112. (d) Chandragupta Maurya is said to have lived as an ascetic
comte de Lally, and the British, under Sir Eyre Coote. It
at Shravanabelagola for several years before starving
was the decisive battle in the Anglo-French struggle
himself to death, as per Jain Practice of Sallekhana.

102.
ww
in southern India during the Seven Years' War (1756-63).
(d) The Crimean War came to an end by the treaty of Paris.
Crimean War, (October 1853-February 1856), war fought
113. (c) Megasthenes gave an account of India in his book
‘Indica’.

w.E
mainly on the Crimean Peninsula between the Russians
and the British, French, and Ottoman Turkish, with
support from January 1855 by the army of Sardinia-
114. (d) The construction of the famous Shiva temple at Ellora
is often attributed to the Rashtrakuta king Krishna I.
115. (d) Harisena was a 4th century Sanskrit poet. His most

asy
Piedmont.The resulting Treaty of Paris, signed on
March 30, 1856, guaranteed the integrity of Ottoman
famous poem written in 345 AD is inscribed on the
Allahabad Pillar.
Turkey and obliged Russia to surrender southern
Bessarabia, at the mouth of the Danube.
En 116. (d) Todar Mal started his career as a revenue officer at the
court of Sher Shah Suri. After the Sur dynasty was
103. (b) Tulsidas wrote Shri Ram-Charit-Manas during the reign
of Mughal Emperor Akbar.Tulsidas started writing this
gin overthrown by the Mughals, Todar Mal continued his
service to the then Mughal Emperor Akbar.
117. (d) The League of Nations was an intergovernmental
greatest Hindu 'Granth' on the birth day of Lord Shree
Ram, i.e. Chaitra Navmi (9th day of Hindu month Chaitra)
in year 1574. The life span of Akbar is 1556 1605. eer
organisation founded on 10th January 1920 as a result
of the Paris Peace Conference that ended the first World
104. (a) Architecture reached a milestone in the Dravidian style
during the reign of Rashtrakutas, the finest example of
War.
ing
118. (b) A settlement between Mahatma Gandhi and Viceroy
which is seen in the Kailasanath Temple or Kailasa
Temple at Ellora. .ne
Lord Irwin known as the Gandhi-Irwin pact was reached
and Gandhi was appointed as the sole representative
105. (c) The famous Badami Chalukyas King Pulakeshi II
(610-642 A.D.) was a follower of Vaishnavism. The
inscription of Ravikirti, his court poet, is a eulogy of
the Pulakeshi II and is available at the Meguti temple.
1931.
t
of the Congress to the second Round table conference,

119. (d) Sankaradeva was an erudite scholar, a prolific writer, a


versatile saint-poet of unlimited merit, a lyricist of
It is dated 634 CE and is written in Sanskrit language universal acceptance, a musician of high calibre, a
and old Kannada script. The Aihole inscription pioneer in the field of Assamese prose, drama and
describes the achievements of Pulakeshi II and his dramatic performances, a painter and above all the
victory against King Harshavardhana. greatest religious teacher-preacher-leader of the
106. (b) The Delhi Sultanate was the name of Delhi-based medieval Vaisnava movement in Assam which is rightly
Muslim kingdoms that ruled over large parts of India known as the Sankaradeva Movement.
for 320 years (1206 – 1526). Five dynasties ruled over 120. (d) 121. (c) 122. (a)
Delhi Sultanate sequentially, the first four of which 123. (a) Muhammad bin Qasim Al-Thaqafi (c. 31 December 695-
were of Turkic origin and the last was the Afghan Lodi. 18 July 715) was an Umayyad general who conquered
The Lodi dynasty was replaced by the Mughal the Sindh and Punjab regions along the Indus River
dynasty. The five dynasties were the Mamluk dynasty (now a part of Pakistan) for the Umayyad Caliphate.
(1206–90); the Khilji dynasty (1290–1320); the Tughlaq He was born and raised in the city of Taif (in modern
dynasty (1320–1414); the Sayyid dynasty (1414–51); day Saudi Arabia). Qasim's conquest of Sindh and
and the Afghan Lodi dynasty (1451–1526). Punjab enabled further Islamic expansion into India.

Downloded From : www.EasyEngineering.net


Downloded From : www.EasyEngineering.net

History 13
124. (d) Mao Zedong, commonly referred to as Chairman Mao aspect of the fight for independence and establishing
(December 26, 1893 - September 9, 1976), was a Chinese his own kingdom. Shivaji Maharaj's assembly of eight
communist revolutionary, politician and socio-political ministers was formed based on Hindu ideals. One
theorist. The founding father of the People's Republic comes across this concept of eight ministers in the
of China from its establishment in 1949, he governed Ramayan and the Mahabharat.
the country as Chairman of the Communist Party of 133. (d) Kanishka was the greatest ruler of the Kushan Empire,
China until his death. a realm that covered much of present-day India,
125. (a) Gandhi calls Gokhale his mentor and guide. Gandhi Pakistan, Iran and other parts of central Asia and China
also recognised Gokhale as an admirable leader and during the first and second centuries.
master politician, describing him as 'pure as crystal, 134. (d) Arya Samaj is a Hindu reform movement founded by
gentle as a lamb, brave as a lion and chivalrous to a Swami Dayananda on 7 April 1875. He was a sannyasi
fault and the most perfect man in the political field'. who promoted the Vedas. Dayananda emphasised the
126. (c) The Swadeshi movement started with the partition of ideals of brahmacharya. The group found most of its
Bengal by the Viceroy of India, Lord Curzon, 1905 and support in Punjab.
continued up to 1911. It was the most successful of 135. (a) 136. (d) 137. (c) 138. (a)
the pre-Gandhian movements. Its chief architects were 139. (b) Iltutmish received contlnnation of his robe of honour

ww
Aurobindo Ghosh, Lokmanya Bal Gangadhar Tilak,
Bipin Chandra Pal and Lala Lajpat Rai.
127. (d) The Purna Swaraj declaration, or Declaration of the
and title Nasir amir al-muminin (Helper of the
Commander of the Faithful) from the 'Abbasid Caliph
al-Mustansir in 626 (1229) and remained on the throne

w.E
Independence of India was promulgated by the Indian
National Congress on January 26, 1930, resolving the
Congress and Indian nationalists to fight for Purna
for twenty-six years, This added an element of strength
to Iltutmish's authority and gave him a status in the
Muslim world .

asy
Swaraj, or complete self-rule independent of the British
Empire. The flag of India had been hoisted by Congress
140. (b) As a British Overseas Territory, the Falkland Islands
enjoy a large degree of internal self-governance with

En
President Jawaharlal Nehru on December 31, 1929, on
the banks of the Ravi river in Lahore, modern-day
the United Kingdom guaranteeing good government
and taking responsibility for their defence and foreign
Pakistan.
128. (d) Gandhra is noted for the distinctive Gandhra style of
gin affairs. In 1982, following Argentina's invasion of the
islands, the two-month-long undeclared Falklands War
Buddhist art, which developed out of a merger of Greek,
Syrian, Persian, and Indian artistic influence. This
development began during the Parthian Period (50 BC eer
between both countries resulted in the surrender of all
Argentine forces and the return of the islands to Britis
administration.
h

- AD 75). Gandhran style flourished and achieved its


peak during the Kushan period, from the 1st to the 5th
141. (a)
ing
Chalukyas : Vatapi (Badami): Hoysalas : Dwarasamudra:
Rashtrakutas : Malkhed; and Kakatiyas: Warangal.
centuries. It declined and suffered destruction after
invasion of the White Huns in the 5th century.
142. (b)
.ne
With a towering height of 70 feet, the world's tallest
statute of Mahatma Gandhi was recently unveiled in
129. (c) Christopher Columbus was an Italian explorer,
navigator, and colonizer, born in the Republic of Genoa,
in what is today northwestern Italy.
130. (c) the earliest inhabitants of India used stones to make
t
Patna in Bihar. Funded by the state government, the
Rs 10 crore statue, inclusive of a 30-feet-high pedestal,
has been built by Delhi-based sculptor Ramsutar and
Sons.
tools and weapons. 143. (d) The vivid description of Kalinga war is given in 13th
131. (b) Swami Vivekanand was the Indian saint to attend ' Rock Edict of Asoka. The edict gives description of
World Congress of Religion' held at Chicago in 1893. the devastation caused to Kalinga due to war and how
He is perhaps best known for his inspiring speech the Mauryan emperor felt remorse for it.
beginning with "Sisters and Brothers of America," 144. (b) Trusteeship principle advocated by Gandhiji provides
through which he introduced Hinduism at the opening a means of transforming the present capitalist order of
session of the Parliament on 11 September. Thereafter society into an egalitarian one. He envisaged
he conducted hundreds of public and private lectures trusteeship as a post capitalist arrangement which gives
and classes, disseminating tenets of Hindu philosophy no quarter to capitalism, but gives the present owning
in America, England and Europe. In America class a chance of reforming itself (Gandhi Nehru And
Vivekananda became India's spiritual ambassador. Globalization, p 7).
132. (d) Chhatrapati Shivaji was a follower of Ramayana and 145. (d) Hunter's Commission: 1882; Wardha Scheme: 1937;
Mahabharta from his childhood. Shivaji Maharaj did University's Act; 1904; and Radhakrishnan
everything possible to promote Hinduism in every Commission: 1948.

Downloded From : www.EasyEngineering.net


Downloded From : www.EasyEngineering.net

14 History
146. (a) Gandhara style flourished and achieved its peak during 152. (a) Lord Curzon introduced the Indian University Act. The
the Kushan period. from the 1st to the 5th centuries. It Indian Universities Act of 1904, passed on March, 21
declined and suffered destruction after invasion of the was formulated on the basis of the recommendations
White Huns in the 5th century. of the Indian University Commission of 1902. Curzon
147. (a) Bold and impartial judiciary is the most important gave importance on improving the standard and quality
safeguard of liberty and no one can restrain it. One of of higher education.
the most important safeguards is that the person has 153. (a) After the spread of the Buddhist religion, Chinese
someone appointed with legal powers to represent them travelers came to India in big numbers to collect
even in extreme case of deprivation of liberty. religious books and to visit holy places of Buddhism.
148. (a) The philosophical school of law is the ancient school 154. (a) Sustainable development is development that meets
of law. It came into existence in 3rd century in Roman the needs of the present without compromising the
empire. ability of future generations to meet their own needs.
149. (c) Akbar celebrated his conquest of Rajputana by laying 155. (c) 156. (a)
the foundation of a new capital, 23 miles (37 km) W.S.W 157. (d) The Cripps mission was an attempt in late March 1942
of Agra in 1569. It was called Fatehpur Sikri ("the city by the British government to secure full Indian
of victory"). cooperation and support for their efforts in World War

ww
150. (a) Vasco da Gama was a Portuguese explorer who
discovered the sea route to India from Europe through
the Cape of Good Hope.
158. (a)
II.

w.E
151. (a) The Unification of Karnataka refers to the formation of
the Indian state of Karnataka, then named Mysore
State, in 1956 when several Indian states were created
159. (a) Hamida Banu Begam, also known as Haji Begam,
commenced the construction of Humayun tomb in 1569.
160. (c) 161. (d) 162. (a) 163. (c) 164. (b)

asy
by redrawing borders based on linguistic
demographics
165. (c) 166. (c) 167. (c) 168. (d) 169. (d)
170. (a) 171. (b) 172. (d) 173. (d) 174. (d)

En 175. (d) 176. (b) 177. (b) 178. (b) 179. (b)
180. (d) 181. (c) 182. (b) 183. (d) 184. (b)

gin
eer
ing
.ne
t

Downloded From : www.EasyEngineering.net


Downloded From : www.EasyEngineering.net

Geography 15

2
CHAPTER
GEOGRAPHY

1 Seismic sea waves which approach the coasts at greater 12. The age of the Earth can be determined by
force are known as (SSC CGL 1st Sit. 2010) (SSC CGL 2nd Sit. 2010)
(a) Tides (b) Tsunami (a) Geological Time Scale
(c) Current (d) Cyclone (b) Radio-Metric Dating
2. The land of maximum biodiversity is(SSC CGL 1st Sit. 2010) (c) Gravity method
(a) Tropical (b) Temperate (d) Fossilization method
(c) Monsoonal (d) Equatorial 13. How much of the Earth's land surface is desert?
3. Indian Standard Time relates to (SSC CGL 1st Sit. 2010) (SSC CGL 1st Sit. 2011)

ww
(a) 75.5°E longitude (b) 82.5°E longitude
(c) 90.5°E longitude (d) 0° longitude
(a) 1/10th
(c) 1/3 rd
(b) 1/5th
(c) 1/6th
14. River Indus originates from: (SSC CGL 1st Sit. 2011)

(a) Vega
w.E
4. Which is the second nearest star to the Earth after the Sun?
(SSC CGL 1st Sit. 2010)
(b) Sirius
(a) Hindukush range (b) Himalayan range
(c) Karakoram range (d) Kailash range

asy
(c) Proxima Centauri (d) Alpha Centauri
5. The forest in Sundarban is called (SSC CGL 1st Sit. 2010)
15. The lowest layer of the atmosphere is:

(a) Stratosphere
(SSC CGL 1st Sit. 2011)
(b) Thermosphere
(a) Scrub jungle (b) Mangrove
(c) Deciduous forest (d) Tundra
En (c) Troposphere (d) Mesosphere
16. The Konkan Railway connects:
6. Which of the following Indonesian regions was a victim of
massive earthquake in 2004 ?
(a) Irian Jaya
(SSC CGL 1st Sit. 2010)
(b) Sumatra gin(a) Goa – Mangalore
(SSC CGL 1st Sit. 2011)

(c) Kalibangan (d) Java


7. The first non-stop air-conditioned 'DURANTO' train was eer
(b) Roha – Mangalore
(c) Kanyakumari – Mangalore
(d) Kanyakumari – Mumbai
flagged off between
(a) Sealdah – New Delhi
(SSC CGL 2nd Sit. 2010)
ing
17. Bark of this tree is used as a condiment–
(SSC CGL 1st Sit. 2011)
(b) Mumbai – Howrah
(c) Bangalore – Howrah
(a) Cinnamon
(c) Neem
(b) Clove
(d) Palm
.ne
(d) Chennai – New Delhi
8. Which one of the following states does not form part of
Narmada River basin ?
(a) Madhya Pradesh (b) Rajasthan
(SSC CGL 2nd Sit. 2010)
18. The atmospheric air is held to the Earth by:

(a) gravity
(b) winds
t
(SSC CGL 1st Sit. 2011)

(c) Gujarat (d) Maharashtra (c) clouds


9. Soil erosion on hill slopes can be checked by (d) rotation of the Earth
(SSC CGL 2nd Sit. 2010) 19. The common tree species in nilgiri hills is
(a) Afforestation (b) Terrace cultivation (SSC CGL 2nd Sit. 2010)
(c) Strip cropping (d) Contour ploughing (a) Sal (b) Pine
10. Who coined the word 'Geography'? (SSC CGL 2nd Sit. 2010) (c) Eucalyptus (d) Teak
20. Which is the largest living bird on Earth?
(a) Ptolemy (b) Eratosthenese
(a) Emu (b) Ostrich
(c) Hacataus (d) Herodatus
(c) Albatross (d) Siberian Crane
11. Which of the following is called the 'ecological hot spot of
21. Rihand Dam Project provides irrigation to
India'? (SSC CGL 2nd Sit. 2010)
(SSC CGL 2nd Sit. 2011)
(a) Western Ghats
(a) Gujarat and Maharashtra
(b) Eastern Ghats (b) Orissa and West Bengal
(c) Western Himalayas (c) Uttar Pradesh and Bihar
(d) Eastern Himalayas (d) Kerala and Karnataka

Downloded From : www.EasyEngineering.net


Downloded From : www.EasyEngineering.net

16 Geography
22. Which is the longest irrigation canal in India? 34. Earth is a very big magnet. In which direction does it
(SSC CGL 2nd Sit. 2011) magnetic field extend? (SSC CGL 2nd Sit. 2012)
(a) Sirhind Canal (b) Yamuna Canal (a) west to east (b) north to south
(c) Indira Gandhi Canal (d) East Kosi Canal (c) south to north (d) east to west
23. 'Loktak' is a (SSC CGL 2nd Sit. 2011) 35. The most densely populated state in India is:
(a) Valley (b) Lake (SSC CGL 2nd Sit. 2012)
(c) River (d) Mountain Range (a) Kerala (b) Uttar Pradesh
24. Which city receives the highest cosmic radiation amongst (c) West Bengal (d) Tamil nadu
the following? (SSC CGL 2nd Sit. 2011) 36. The biggest planet in the solar system is:
(a) Chennai (b) Mumbai (SSC CGL 2nd Sit. 2012)
(c) Kolkata (d) Delhi (a) Venus (b) Jupiter
25. The HYV programme in India is also called as (c) Saturn (d) Uranus
(SSC CGL 1st Sit. 2012) 37. Peninsular India has the following zonal soil types:
(a) Traditional Agriculture (SSC CGL 2nd Sit. 2012)
(b) New Agricultural Strategy (a) Red and yellow soil (b) Forest soil
(c) White Revolution (c) Saline soil (d) Alluvial soil
38. The land between two rivers is called

ww
(d) Blue Revolution
26. Railway coaches are manufactured at
(SSC CGL 1st Sit. 2012) (a) Natural Levees
(SSC CGL 1st Sit. 2012)
(b) Alluvial Cones
(a) Jamshedpur
(c) Perambur
w.E (b) Chittaranjan
(d) Varanasi
27. A series of lines connecting places having a quake at the
(c) Braided Stream (d) Doab
39. Which year is called as the ‘Demographic divide’ ?
(SSC CGL 1st Sit. 2012)
same time are called
(a) Homoseismal lines (b) Seismolines
(c) Coseismal lines
asy
(SSC CGL 1st Sit. 2012)

(d) Isoseismal lines


(a) 1941
(c) 1901
(b) 1921
(d) 1931
40. Which country has a high density of population?

forests? En
28. What would be the impact of global warming on mangrove
(SSC CGL 1st Sit. 2012)
(a) India
(SSC CGL 1st Sit. 2012)
(b) Canada
(a) They will grow more luxurious
(b) Large areas of mangroves will be submerged gin(c) Sweden (d) Greenland
41. The iron and steel plant in Chhattisgarh is at
(c) Their role as carbon sinks will become more important
(d) Both (a) and (c) above
(c) Bhilaieer
(a) Burnpur (b) Salem
(SSC CGL 1st Sit. 2012)

(d) Bokaro
29. The brightest planet is
(a) Venus
(c) Jupiter
(SSC CGL 1st Sit. 2012)
(b) Mercury
(d) Mars
ing
42. The leading sesame producing country in the world is
(SSC CGL 1st Sit. 2012)
30. Wheat, Barley, Lemon, Orange, rye and pearl millet belong
to: (SSC CGL 2nd Sit. 2012)
(a) Mexico
(c) China
(b) U.S.A.
(d) India
.ne
(a) the same plant family
(b) two plant families
(c) three plant families
(a) Darjeeling - West Bengal
(b) Mount Abu - Rajasthan
t
43. Which one of the following is not correctly matched?
(SSC CGL 2nd Sit. 2012)

(d) four plant families (c) Kodaikanal - Tamil Nadu


31. A plant known only in cultivation having arisen under (d) Simla - Uttar Pradesh
domestication is referred to as: 44. The earth is at its maximum distance from the Sun on
(SSC CGL 2nd Sit. 2012) (SSC CGL 2nd Sit. 2012)
(a) Scion (b) Cultigen (a) January 30th (b) December 22nd
(c) Cultivar (d) Clone (c) September 22nd (d) July 4th
32. The iron and steel plant in Bihar is at: 45. Consider the following pairs:
(SSC CGL 2nd Sit. 2012) (SSC CGL 2nd Sit. 2012)
(a) Visakhapatnam (b) Bokaro Tributary Main River
(c) Burnpur (d) Vijay Nagar 1. Chambal : Yamuna
33. The state which has registered the highest population 2. Sone : Narmada
growth rate according to 2001 census is: 3. Manas : Brahmputra
(SSC CGL 2nd Sit. 2012) Which one of the pairs given above is/are correctly matched?
(a) Kerala (b) Uttar Pradesh (a) 1, 2 and 3 (b) 1 and 2
(c) Nagaland (d) Sikkim (c) 2 and 3 (d) 3 only

Downloded From : www.EasyEngineering.net


Downloded From : www.EasyEngineering.net

Geography 17
46. The total population divided by available arable land area is 57. The programme of ‘Operation Flood’ was concentrated on
referred to as (SSC CGL 2nd Sit. 2012) (SSC CGL 2nd Sit. 2013)
(a) Population density (b) Nutritional density (a) increasing irrigation facilities.
(c) Agricultural density (d) Industrial density (b) flood control.
47. The eastward continuation of the Brazil current is called : (c) increasing the milk production.
(SSC CGL 1st Sit. 2013) (d) increase the flood grains production.
(a) North Atlantic drift 58. According to Ferrel’s law (Coriolis Force) winds change their
(b) South Atlantic drift direction (SSC CGL 2nd Sit. 2013)
(c) Counter Equatorial drift (a) Towards left in Northern hemisphere and towards right
(d) West Atlantic drift in Southern hemisphere.
48. Ice glacier’s melting is a common phenomenon linked to the (b) Towards right in Northern hemisphere and towards left
rise in a seawater level. The glaciers are predominantly in Southern hemisphere.
present in (SSC CGL 2nd Sit. 2013) (c) Towards right in both the hemisphere.
(a) Greenland (b) Antarctica (d) Towards left in both the hemisphere.
(c) Himalayas (d) Arctic 59. Which one of the following atmospheric layers absorb
49. Which hill station is called as the ‘Queen of the Satpuras’ ultraviolet rays of the sun? (SSC CGL 2nd Sit. 2013)

ww
(a) Pachmarhi
(SSC CGL 2nd Sit. 2013)
(b) Nilgiri
(a) Troposphere
(c) Ionosphere
(b) Stratosphere
(d) Ozonosphere
(c) Mahenderagiri

w.E (d) Cardamom


50. Which national highway connects Delhi and Kolkata via
Mathura and Varanasi? (SSC CGL 2nd Sit. 2013)
60. The drainage pattern developed on folded sedimentary rock
is termed as
(a) Trellis
(SSC CGL 2nd Sit. 2013)
(b) Dendritic
(a) NH 4
(c) NH 10
(b) NH 2
(d) NH 6
asy
51. The country where drip irrigation is more efficiently used is
(c) Radial (d) Deranged
61. Which one of the following is not a line of demarcation
between two countries ? (SSC CGL 2nd Sit. 2013)

(a) India (b) Israel En


(SSC CGL 2nd Sit. 2013) (a) Durand Line
(c) Plimsoll Line
(b) Mac Mahon Line
(d) Maginot Line
(c) Sri Lanka (d) England
52. Which river in India flows in a rift-valley? gin
62. Water potential remains lowest in (SSC CGL 2nd Sit. 2013)
(a) Water plants (b) Woody plants

(a) Narmada
(SSC CGL 1st Sit. 2013)
(b) Krishna eer
(c) Succulents (d) Halophytes
63. The free living bacterium in the soil which increases the the
(c) Cauvery (d) Tapti
53. A narrow strip of land that connects two larger land masses
yield of rice is
(a) Rhizobium ing (SSC CGL 2nd Sit. 2013)
(b) Azotobacter
is called
(a) Cape
(SSC CGL 1st Sit. 2013)
(b) Isthmus
(c) Acetobacter

.ne
(d) Anabaena
64. Frontal cyclones occur characteristically in
(SSC CGL 2nd Sit. 2013)
(c) Strait

made features are called


(a) Thematic maps
(d) Peninsula
54. Maps on large scale, representing both natural and man-
(SSC CGL 1st Sit. 2013)
(a) Equatorial region
(b) Tropical region
(c) Mid-latitudinal region
t
(b) Atlas maps (d) Polar region
(c) Wall maps 65. "Yosemite" is a (SSC CGL 1st Sit. 2013)
(d) Topographic maps (a) River (b) Peak
55. The angle between the magnetic meridian and the (c) Waterfall (d) Dam
geographical meridian at a place is (SSC CGL 1st Sit. 2013) 66. Where is the shore based steel plant located?
(a) Declination (b) Latitude (SSC CGL 2nd Sit. 2013)
(c) Azimuth (d) Dip (a) Tuticorin (b) Salem
56. An irrigation project is categorized as a major project if it (c) Vishakhapatnam (d) Mangalore
covers a cultivable command area of 67. Which two of the following are connected by the North
(SSC CGL 2nd Sit. 2013) South corridor? (SSC CGL 2nd Sit. 2013)
(a) less than 2,000 hectares (a) Srinagar and Kanyakumari
(b) 2,000 to 10,000 hectares (b) Mumbai and Chennai
(c) above 10,000 hectares (c) Amritsar and Kolkata
(d) all the above (d) Hyderabad and Bhopal

Downloded From : www.EasyEngineering.net


Downloded From : www.EasyEngineering.net

18 Geography
68. Which of the following statements is correct? 78. Which of the following river does not originate in Indian
(SSC CGL 2014) territory ? (SSC CGL 2015)
(a) Mahadeo hills are in the west of Maikala hills. (a) Mahanadi (b) Brahmaputra
(b) Mahadeo hills are the part of Karnataka Plateau. (c) Satluj (d) Ganga
(c) Mahadeo hills are in the east of Chhotanagpur Plateau. 79. Which one among the following industries in the maximum
(d) Mahadeo hills are the part of Aravalli ranges. consumer of water in India ? (SSC CGL 2015)
69. Which one of the following pairs is not correctly matched? (a) Textile (b) Engineering
(SSC CGL 2014) (c) Paper and Pulp (d) Thermal Power
(a) Hevea Tree—Brazil 80. Choose the correct option which represents the arrangement
(b) Sumatra Storm—Malaysia of atmospheric layers. (SSC CGL 2015)
(c) Kajan River—Borneo (a) Troposphere, Stratosphere, Mesosphere, Ionosphere,
(d) Dekke Toba fish—Brazil Exosphere
70. Which of the following resources is renewable one? (b) Mesosphere, Ionosphere, Exosphere, Troposphere,
(SSC CGL 2014) Stratosphere
(a) Uranium (b) Coal (c) Ionosphere, Exosphere, Mesosphere, Troposphere,
(c) Timber (d) Natural Gas Stratosphere
71.
ww
Soil erosion can be prevented by
(a) Increasing bird population
(b) Afforestation
(SSC CGL 2014)
(d) Exosphere, Troposphere, Ionosphere, Mesosphere,
Stratosphere
81. Which of the following options correctly explains the term

72.
(d) Overgrazing w.E
(c) Removal of vegetation

Natural sources of air pollution are (SSC CGL 2014)


‘heat budget’?

receives form the sun.


(SSC CGL 2015)
(a) It is the amount of heat which the surface of earth

(a) Forest fires


(b) Volcanic eruptions asy (b) It is the radiation from the earth in the form of long
waves
(c) It is a mode of transfer of heat through matter by
(c) Dust storm
(d) Smoke from burning dry leaves En molecular activity.
(d) It is the balance between incoming and outgoing
73. The 'graded profile' of a river course is a
(SSC CGL 2015) gin radiation.
82. The layer of atmosphere close to the earth's surface is called :
(a) smooth curve in the upper course
(b) smooth curve in the middle course
eer
(a) Exosphere (b) Ionosphere
(SSC CGL 2015)

74.
(c) smooth curve in the lower course
(d) smooth curve from source to mouth
Sink hole is a phenomenon of _______ topography.
(c) Stratosphere
ing
(d) Troposphere
83. Which of the following plant shows chloroplast dimorphism?
(SSC CGL 2015)

(a) Desert (b) Tundra


(SSC CGL 2015) (a) Sugarcane
(c) Rice (d) Wheat .ne
(b) Sugar beet

75.
(c) Karst

1. Coconut
3. Rubber
(d) Plain
Kerala is famous for the cultivation of (SSC CGL 2015)
2. Black pepper
4. Rice
84. Day and Night are equal at the :
(a) Prime Meridian
(c) Equator
(b) Poles
(d) Antarctic
t
(SSC CGL 2015)

85. Evergreen type forests are found in : (SSC CGL 2015)


(a) 1, 2 and 4 (b) 2, 3 and 4 (a) Mediterranean region
(c) 1 and 4 (d) 1, 2 and 3 (b) Monsoon climatic area
76. The longest continental Railway in the world is (c) Desert region
(SSC CGL 2015) (d) Equatorial region
(a) Trans Siberian Railway 86. The ash–grey soils of high latitude coniferous forests are
(b) Canadian Pacific Railway known as : (SSC CGL 2015)
(c) Canadian National Railway (a) Grey–Brown soils (b) Red and Yellow soils
(d) Trans Atlantic Railway (c) Tundra soils (d) Podsols
77. Which of the following is FALSE with respect to rain water 87. Equinox occurs when the sun is vertically above
harvesting? (SSC CGL 2015) (SSC CGL 1st Sit. 2016)
(a) It helps raising water table (a) Tropic of Capricorn
(b) It helps meet rising water demand (b) Tropic of Cancer
(c) It increases run–off losses (c) Poles
(d) It is a device of water conservation (d) Equator

Downloded From : www.EasyEngineering.net


Downloded From : www.EasyEngineering.net

Geography 19
88. Among the world oceans, which ocean is having the widest 97. With which country, India has the longest international
continental shelf ? (SSC CGL 1st Sit. 2016) boundary? (SSC CGL 1st Sit. 2016)
(a) Antarctic ocean (b) Arctic Ocean (a) Nepal (b) Pakistan
(c) Indian Ocean (d) Atlantic ocean (c) China (d) Bangladesh
89. Which is largest peninsular river in India? 98. Which State in India has the largest coastline?
(SSC CGL 1st Sit. 2016)
(SSC CGL 1st Sit. 2016)
(a) Tamil Nadu (b) Andhra Pradesh
(a) Krishna (b) Godavari
(c) Gujarat (d) West Bengal
(c) Cauvery (d) Mahanadi
99. Jog falls in Karnataka is located over which river?
90. Red soil is normally found in India in which regions?
(SSC CGL 1st Sit. 2016)
(SSC CGL 1st Sit. 2016)
(a) Kaveri (b) Godavari
(a) Eastern Region only (c) Sharavati (d) Krishna
(b) Southern Region only 100. Which one of the following areas of India is covered by
(c) Eastern & Southern part of the Deccan Plateau tropical evergreen forest? (SSC CGL 1st Sit. 2016)
(d) None of these (a) Semi-arid areas of Gujarat

ww
91. Limestone is a raw material used by which industry?

(a) Aluminium
(SSC CGL 1st Sit. 2016)
(b) Fertilizers
(b) Eastern Ghats
(c) Western Ghats
(d) Madhya Pradesh
(c) Cement
w.E (d) Petrochemicals
91. Mount Abu is a hill station located in _____ ranges.
101. Which of the following State is surrounded by Bangladesh
from three sides?
(a) Nagaland (b) Assam
(SSC CGL 1st Sit. 2016)

(a) Vindhya (b) Satpuda asy


(SSC CGL 1st Sit. 2016)
(c) Arunachal Pradesh (d) Tripura
102. Earth received heat from the sun is known as:
(c) Aravalli (d) Sahyadri

En
92. The only perennial river in Peninsular India is _____ . (a) Insolation
(SSC CHSL 2012)
(b) Infrared heat

(a) Godavari (b) Kaveri


(SSC CGL 1st Sit. 2016)
gin (c) Solar radiation (d) Thermal radiation
103. Spot the odd item in the following: (SSC CHSL 2012)
(c) Krishna (d) Bhima
93. When does solar eclipse take place ? eer
(a) Red sea
(c) Caspian sea
(b) Black sea
(d) Dead sea
(SSC CGL 1st Sit. 2016)
(a) When the sun is between the moon and earth ing
104. Match correctly the following, deserts and their location yb
choosing the correct response: (SSC CHSL 2013)
(b) When the earth is between the moon and sun
(c) When the moon is between the sun and earth
Desert
a. Kalahari
.ne
Location
1. South America
(d) When the moon does not lie on the line joining the sun
and earth
94. Strait of Gibraltar connects which of the following?
b. Atacama
c. Thar
d. Great Victoria
(a) a-3, b-1, c-4, d-2
2. Australia
3. Africa
4. Asia
t
(b) a-2, b-3, c-1, d-4
(SSC CGL 1st Sit. 2016)
(c) a-4, b-3, c-2, d-1 (d) a-3, b-2, c-1, d-4
(a) Red Sea-Mediterranean Sea
105. The longest river of peninsular India is
(b) Red Sea-Arabian Sea
(SSC CHSL 2013)
(c) Atlantic Ocean-Mediterranean Sea
(a) Godavari (b) Krishna
(d) Mediterranean Sea-Black Sea (c) Kaveri (d) Narmada
95. The largest solar power plant in India is located at 106. The Himalayan mountain range is an example of
(SSC CGL 1st Sit. 2016) (SSC CHSL 2013)
(a) Nagercoil (b) Jaisalmer (a) Fold mountain (b) Volcanic mountain
(c) Madhapur (d) Rann of Kutch (c) Residual mountain (d) Block mountain
96. On which river is the Tehri dam built? 107. Which one of the following is a warm ocean current ?
(SSC CGL 1st Sit. 2016) (SSC CHSL 2013)
(a) Alakananda (b) Bhagirathi (a) Gulf Stream (b) Kurile
(c) Ganga (d) Hooghly (c) Canary (d) Labrador

Downloded From : www.EasyEngineering.net


Downloded From : www.EasyEngineering.net

20 Geography
108. The main advantage of Rain Water Harvesting (RWH) is 119. How much of world's surface is covered by water ?
(SSC CHSL 2013) (SSC CHSL 2015)
(a) Avoid soil erosion (a) 70% (b) 80%
(b) Recharge ground water (c) 25% (d) 55%
(c) Avoid floods 120. Which of the following Scientist proved that the path of
(d) Reduce the loss of water each planet around the Sun is elliptical ?(SSC CHSL 2015)
109. India is the largest producer and exporter of (a) Galileo (b) Newton
(SSC CHSL 2014)
(c) Copernicus (d) Kepler
(a) Cotton (b) Copper
121. Which of the following rivers originates from Trans
(c) Tea (d) Mica
Himalayas ? (SSC CHSL 2015)
110. The soils which are rich in Calcium are known as
(a) Sindu (b) Saraswathi
(SSC CHSL 2014)
(c) Ganga (d) Yamuna
(a) Pedocals (b) Pedalfers
122. The most suitable soil for the production of cotton is ?
(c) Podsols (d) Laterits
111. Cultivable land is defined as (SSC CHSL 2014) (SSC CHSL 2015)
(a) Black lava soil (b) Loamy soil

ww
(a) land actually under crops
(b) Culitivable waste land + fallow land
(c) Old fallow lands + current fallow lands
(c) Well drained soil (d) Alluvial soil
123. The largest producer of Lignite in India is:

India ?
w.E
(d) Total fallow lands + net sown area
112. Which of the following is the largest Biosphere Reserves of
(SSC CHSL 2014)
(a) Kerala
(SSC CHSL 2015)
(b) Rajasthan

(a) Nilgiri
(c) Sundarbans asy
(b) Nandadevi
(d) Gulf of Mannar
(c) Tamil Nadu
124. Sex–ratio is calculated as :
(d) Gujarat
(SSC CHSL 2015)
(a) No of females per 1,000 males in a Country

En
113. With what bio-region is the term "Steppe" associated ?
(SSC CHSL 2014)
(b) No of males per 1,000 females in a Country

(a) Grasslands
(c) Savanna
(b) Tropical Forests
(d) Coniferous Forests gin (c) No. of children per 1,000 people in a Country.
(d) No of people per 1,000 children in a Country.

114. About how much of the world's land area is tropical


rainforest ? (SSC CHSL 2014) eer
125. Which state of India has made rain water harvesting
compulsory for all houses ?
(a) Tamil Nadu (b) Punjab
(SSC CHSL 2015)
(a) 2 percent
(c) 10 percent
(b) 7 percent
(d) 15 percent (c) Haryana
ing (d) Maharashtra
126. The five key indicators of global climate change of our
115. The world's growing appetite for what food product is a
leading cause of tropical deforestation ? (SSC CHSL 2014)
(a) Pork (b) Sugar
planet are :
.ne (SSC CHSL 2015)
(a) Sea–level, Rising temperatures, Rainfall, Nitrogen and
(c) Lamb
116. What is Kyoto Protocol ?
(d) Beef
(SSC CHSL 2015)
(a) It is an agreement among countries to takes steps for
Actic Sea ice
(b) None of the options t
(c) Arctic sea ice, carbon dioxide, Global temperature, Sea
planting trees to control pollution level and land ice.
(b) It is an agreement among countries to start using (d) Antartic sea ice, Oxygen, Rainfall, Drought and Sea level
nuclear energy 127. The area reserved for the welfare of wild life is called
(c) It is an agreement among countries to take steps for
(SSC CHSL 2015)
reducing global warming
(a) Sanctuary (b) Botanical garden
(d) It is an agreement among countries to takes steps for
reducing acid rain. (c) Forest (d) National pak
117. The largest reservoir of fresh water is : (SSC CHSL 2015) 128. The National Green Tribunal deals with cases relating to ?
(a) Ground Water (b) Ponds (SSC CHSL 2015)
(c) Lakes (d) Glaciers (a) Criminal offenses
118. The most abundant element by number in the living system (b) Issues relating to protection and conservation of
is : (SSC CHSL 2015) historical monuments.
(a) Hydrogen (b) Oxygen (c) Civil cases
(c) Carbon (d) Nitrogen (d) Environmental protection and conservation of forests.

Downloded From : www.EasyEngineering.net


Downloded From : www.EasyEngineering.net

Geography 21
129. The biggest oil spill in world history took place in the ? 139. Consider the following sea-ports:
(SSC CHSL 2015) (SSC Multitasking 2014)
(a) Persian Gulf (b) Caspian Sea 1. Chennai 2. Machilipatnam
(c) Mediterrarean Sea (d) South China sea 3. Nagapattinam 4. Tuticorin
130. Among the foollowing which country has the highest life The correct sequence of these ports from north to south is
expectancy ? (SSC CHSL 2015) (a) 1, 3, 2, 4 (b) 2, 1, 4, 3
(c) 1, 2, 4, 3 (d) 2, 1, 3, 4
(a) USA (b) Switzerland
140. Santa Cruz is (SSC Multitasking 2014)
(c) Japan (d) Denmark
(a) an International airport in Chennai
131. The redness in atmosphere at Sunrise and Sunset is due to:
(b) a Domestic airport in Chennai
(SSC CHSL 2015)
(c) an International airport in Mumbai
(a) Dispersion of light (b) Scattering of light (d) a Domestic airport in Mumbai
(c) Refraction of light (d) Reflection of light 141. The Almatti Dam is constructed on the river
132. Which one of the following tribes practices pastoral (SSC Multitasking 2014)
nomadism ? (SSC CHSL 2015) (a) Tungabhadra (b) Krishna

ww
(a) Boro
(c) Pygmies
(b) Masai
(d) Eskimo
(c) Kaveri (d) Sileru
142. The project similar to T.V.A. (Tennessee Valley Authority)

w.E
133. Select the High Yielding Varieties of seed-crops developed
under Green Revolution in India.
(SSC Multitasking 2013)
of U.S.A. in India is
(a) Damodar Valley Project
(b) Mahanadi Delta Project
(SSC Sub. Ins. 2012)

(a)
asy
Wheat, Rice, Sugarcane, Pulses and Maize.
(b) Rice, Wheat, Pulses, Oil seeds and Sugarcane
(c) Ramganga Multipurpose Project
(d) Idukki Project
(c) Maize, Black-gram, Jowar, Coffee and Tea.
(d) Rice, Wheat, Jowar, Bajra and Maize. En 143. Petrology is the study of
(a) rocks
(SSC Sub. Ins. 2012)
(b) soils

134. India is the largest producer and consumer of


(SSC Multitasking 2013) gin (c) earth
144. Match the following :
(d) minerals
(SSC Sub. Ins. 2012 )

(a)
(c)
Sugar
Tea
(b) Paddy
(d) Coffee
B. Bushes
eer
A. Rosewood 1. Mangrove forests
2. Alpine forests

135. The busiest rail section in respect to goods transportation is


(SSC Multitasking 2013)
C. Birch
D. Sundari tree
ing 3. Deciduous forests
4. Dry forests

(a) Mumbai – Chennai section


A B C D
(a) 4 3 2 1 (b) 3 4 2 1
.ne
A B C D

(b) Delhi – Kolkata section


(c) Kolkata – Chennai section
(d) Delhi – Mumbai section
136. The Tropic of Cancer does not pass through
(c) 1 2 3 4 (d) 2 3 4 1

remains dry because :


(a) the winds do not reach this area
t
145. During the period of South-West monsoon, Tamil Nadu
(SSC Sub. Ins. 2012)

(SSC Multitasking 2013) (b) there are no mountains in this area


(c) it lies in the rain-shadow area
(a) China (b) Myanmar
(d) the temperature is too high to let the winds cool down
(c) Nepal (d) Bangladesh
146. Which one of the following is not a line of demarcation
137. The country in East Asia which is most conspicuous for its
between two countries? (SSC Sub. Ins. 2013)
decreasing population growth since 1981 is
(a) International Date Line
(SSC Multitasking 2013) (b) MacMahon Line
(a) Japan (b) South Korea (c) Radcliffe Line
(c) Thailand (d) China (d) Durand Line
138. The recently discovered field with oil potential in Krishna- 147. Which state in India occupies the same ranking position in
Godavari Basin is called (SSC Multitasking 2014) respect of area and population? (SSC Sub. Ins. 2013)
(a) Ravva Offshore Block (b) Golkunda Block (a) Manipur (b) Meghalaya
(c) Bombay High (d) Telangana Block (c) Nagaland (d) Gujarat

Downloded From : www.EasyEngineering.net


Downloded From : www.EasyEngineering.net

22 Geography
148. Lake formed in a cut off river, meander is called: 159. The total number of planets revolving around the sun are:
(SSC Sub. Ins. 2013) (SSC Sub. Ins. 2015)
(a) Playa Lake (b) Meteoric Lake (a) Seven (b) Eight
(c) Ox-Bow Lake (d) Crater Lake (c) Five (d) Six
149. Which is the leading state in India in "Ship Breaking" 160. Which of the following is the major copper producing
industry? (SSC Sub. Ins. 2013) country? (SSC Sub. Ins. 2015)
(a) Tamil Nadu (b) Maharashtra (a) Indonesia (b) Russia
(c) Sri Lanka (d) Chile
(c) Gujarat (d) West Bengal
161. The largest forest which covers 25 percent of the world’s
150. Contour line is the imaginary line joining places of equal:
Forest land is: (SSC Sub. Ins. 2015)
(SSC Sub. Ins. 2013)
(a) Monsoon Forest
(a) Rainfall (b) Humidity
(b) Temperate forests of Europe
(c) Elevation (d) Temperature
(c) The tropical Rain forest
151. Which is the first state in India to enact the Food Security (d) Taiga forest of Siberia
Act? (SSC Sub. Ins. 2013)
162. The method of age determination called "radioactive uranium

ww
(a) Chhattisgarh
(c) Tamil Nadu
(b) Gujarat
(d) Bihar
152. The first short based integrated steel plant in the country is
dating" cannot be used to determine the age of
(SSC Sub. Ins. 2016)

(a) Vijaynagar
w.E (SSC Sub. Ins. 2014)
(b) Salem
(a) the earth
(c) minerals on earth
(b) rocks
(d) fossils and plant bodies
163. Altocumulus clouds occur at altitude of
(c) Vishakapatnam
asy
(d) Bhadravati
153. Name the condition which influences the development of (a) 10000 m - 12000 m
(SSC Sub. Ins. 2016)
(b) 6000 m - 10000 m
plants into distinctive forms.
(a) Climatic conditions
En
(SSC Sub. Ins. 2014) (c) 1000 m - 1800 m (d) 2000 m - 6000 m
164. The minor irrigation schemes have Cultural Command Are a
(b)
(c)
Soil conditions
Environmental conditions gin (CCA) upto
(a) 3000 hectares
(SSC Sub. Ins. 2016)
(b) 2500 hectares
(d) Social conditions
154. Agriculture should serve as an instrument of income, eer
(c) 1500 hectares (d) 2000 hectares
165. Which of the following Islands is under direct threat of
livelihood and opportunity to the local community - this
statement is given by (SSC Sub. Ins. 2014)
(a) Palau
ing
rising sea levels due to climate change has appealed fo r
help from European leaders? (SSC Sub. Ins. 2016)
(b) Tuvalu Islands
(a) Dr Madhavan Nair
(c) Dr Abdul Kalam
(b) Dr Manmohan Singh
(d) Dr M S Swaminathan
(c) Marshall Islands
.ne
(d) Nauru

155. The forests which act as barriers against cyclones are:

(a) Evergreen forests


(SSC Sub. Ins. 2015)
(b) Mangrove forests
world'?
(a) Temperate grasslands
(b) British type vegetation
t
166. Which of the following regions is called the 'granary of the
(SSC Steno. 2013)

(c) Monsoon forests (d) Alpine Forest (c) Laurentian type vegetation
156. The resources which are obtained from biosphere and have (d) Tropical grasslands
life are : (SSC Sub. Ins. 2015) 167. Spot the odd item in the following: (SSC Steno. 2013)
(a) Renewable resources (b) Potential resources (a) Prairies (b) Pampas
(c) Biotic resources (d) Abiotic resources (c) Selvas (d) Sundarban
157. In a solar or lunar eclipse, the region of earth’s shadow is 168. Which of the following rovers of India flows in rift-valley?
divided into. (SSC Sub. Ins. 2015) (SSC Steno. 2013)
(a) Four parts (b) Five parts (a) Kaveri (b) Tapti
(c) Two parts (d) Three parts (c) Son (d) Ken
158. What does the word ‘amphibian’ mean ? 169. The largest coral reef in the world is found near the Eastern
(SSC Sub. Ins. 2015) coast of (SSC Steno. 2013)
(a) Two lives (b) Four lives (a) Japan (b) China
(c) Three lives (d) One life (c) Cuba (d) Australia

Downloded From : www.EasyEngineering.net


Downloded From : www.EasyEngineering.net

Geography 23
170. The strong, cold, ley wind blowing in the polar region is 179. Which of the following is related to horticulture ?
known as (SSC Steno. 2013) (SSC Steno. 2016)
(a) Chinook (b) Bora (a) Green revolution (b) White revolution
(c) Blizzard (d) Harmattan (c) Operation flood (d) Golden revolution
171. The lines joining places of equal values of horizontal 180. The island of Honshu in Japan is famous for
component of Earth's magnetic field are called
(SSC Steno. 2016)
(SSC Steno. 2014)
(a) Oil (b) Diamonds
(a) isogonic lines (b) isodynamic lines
(c) Coal (d) Iron ore
(c) aclinic lines (d) isoclinic lines
172. Which one among the following states is smallest in area? 181. 40°N latitude acts as demarcation line between
(SSC Steno. 2014) (SSC Steno. 2016)
(a) Gujarat (b) Karnataka (a) North and South Korea
(c) Tamil Nadu (d) Andhra Pradesh (b) USA and Canada
173. Highest coffee growing state in India is(SSC Steno. 2014) (c) North add South Vietnam
(a) Karnataka (b) Uttar Pradesh (d) Egypt and Sudan

ww
(c) Maharashtra
174. The rootless plant is
(a) lemna
(d) Tamil Nadu
(SSC Steno. 2014)
(b) banana
182. Lakes which contain high concentration of humic acid in
water is
(a) Desert salt lakes
(SSC Steno. 2016)
(b) Volcanic lakes
(c) ginger
w.E (d) lemon
175. The highest peak of the Eastern ghats of India is
(c) Dystrophic lakes (d) Deep ancient lakes
183. The first person ever to reach the South Pole was

(a) Shevroy hills asy (SSC Steno. 2014)


(b) Mahendragiri (a) Magellan
(SSC Steno. 2016)
(b) merigo Vespucci
(c) Javadi hills (d) Annaimudi
176. Which country is the largest producer of raw jute?
En (c) Amundsen (d) Peary
184. It is reported that there is an ongoing decrease in the pH

(a) Bangladesh
(SSC Steno. 2014)
(b) Pakistan
gin value of ocean water because of global warming. It happen s
due to (SSC Steno. 2016)
(c) Myanmar (d) India
177. The largest proven oil reserve of the world lies in
(a)
(b) eer
Larger uptake of atmospheric nitrogen by ocean water.
Lesser uptake of atmospheric nitrogen by ocean water.

(a) Iran
(c) Venezuela
(b) Iraq
(SSC Steno. 2016)

(d) Saudi Arabia


(c)
ing
Larger uptake of CO2 by ocean water
(d) Lesser uptake of CO2 by ocean water.

178. Kundankulam Project is located in which state ?


(SSC Steno. 2016) .ne
185. The largest tea growing country in the World is
(SSC Steno. 2016)

(a) Telangana
(c) Karnataka
(b) Kerala
(d) Tamil Nadu
(a)
(c)
India
Sri Lanka
(b) Brazil
(d) China t

Downloded From : www.EasyEngineering.net


Downloded From : www.EasyEngineering.net

24 Geography

HINTS & SOLUTIONS


1. (b) A tsunami, also known as a seismic sea wave, is a 32. (b) Bokaro Steel Plant – the fourth integrated plant in the
series of waves in a water body caused by the Public Sector – started taking shape in 1965 in
displacement of a large volume of water, generally in collaboration with the Soviet Union. Once in Bihar, it
an ocean or a large lake. is now in Jharkhand.
2. (a) 3. (b) 4. (c) 33. (c) Nagaland had recorded the country’s highest decadal
5. (b) The Sundarbans is a natural region comprising population growth of 64.41 per cent in 2001 and 56.08
southern Bangladesh and a part in the Indian state of in 1991 respectively.
West Bengal. It is the largest single block of tidal 34. (b) The inclination of the Earth's field is 90°at the North
halophytic mangrove forest in the world. Magnetic Pole and –90° at the South Magnetic Pole.
6. (b) 7. (a) 8. (b) 35. (c) Bihar with 1,102 persons per square kilometre is the
9. (b) Terrace cultivation, method of growing crops on sides most densely populated state of India as per the 2011

ww
of hills or mountains by planting on graduated terraces
built into the slope. Though labour–intensive, the
method has been employed effectively to maximize 36.
Census. West Bengal comes second with 1,029
persons per kilometre as per the option.
(b) Jupiter is the fifth planet from the sun and the largest

10.
w.E
arable land area in variable terrains and to reduce soil
erosion and water loss.
(b) 11. (a) 12. (b) 13. (b) 14. (d)
37.
planet within the Solar System.
(a) Red and Yellow Soil in India is largely available in the
Deccan Plateau.
15.
20.
(c) 16. (b) 17. (a) 18. (a) 19. (c)
(b) 21. (c) 22. (c) 23. (b) 24. (a) asy 38.
43.
(d) 39. (b) 40. (a) 41. (c) 42. (d)
(d) 44. (d) 45. (a) 46. (a)
25. (b) The New Agricultural Strategy is based on

En
concentration of high-yielding varieties of seeds and
complementary inputs on selected water-as-sured
47. (b) South Atlantic Current is an eastward ocean current,
fed by the Brazil Current. That fraction of it which

26.
areas.
(c) Passenger coaches are manufactured at three principal gin reaches theAfrican coast feeds the Benguela Current .
It is continuous with the northern edge of the Antarcti
Circumpolar Current.
c

places : Integral Coach Factory (ICF) at Perambur,


Railway Coach Factory (RCF) at Kapurthala, And
48.
eer
(b) It ismainlycausecrbyo phenomena: global warmin g
and ozone depletion. Polar ice caps in Antarctica and

27.
Bharat Earth Movers Ltd. (BEML) at Bangalore.
(a) Homoseismal lines is the line on the Earth's surface
ing
other places are–melting fast. This has led to increase
in sea level over the years.

28.
connecting points where the seismic wave arrives,
generated by an earthquake, at the same time.
(d) Mangrove wetlands are possible sinks/sources for
49.

.ne
(a) Panchmarhi a beautiful hill station in Madhya Pradesh
is known as Queen of Satpura because of its mystic
beauty, it is a known destination for Bollywood film
carbon dioxide and other related greenhouse gases.
Global warming may promote expansion of mangrove
forests to higher latitudes. Elevated CO2 concentration
50.
shootings.
t
(b) National Highway 2, commonly referred as Delhi–
Kolkata Road, is a busy Indian National Highway that
may increase mangrove growth by stimulating runs through the states of Delhi, Haryana, Uttar
photosynthesis or improving water use efficiency. Pradesh, Bihar, Jharkhand, and West Bengal. It
29. (a) There are droplets of sulphuric acid and acidic crystals in constitutes a major portion of the historical Grand
the atmosphere of Venus. The smooth surfaces of these Trunk. Road along with NH 91 and NH 1 in India.
droplets of sulfuric acid and crystals reflect light very 51. (b) Drip irrigation is a form of irrigation that saves water
well, which is one reason why Venus is so bright. and fertilizer by allowing water to drip slowly to the
30. (b) Most cereals, including wheat, rye, rice, oats, barley, roots of many different plants, either onto the soil
corn, sorghum, millet, green bristlegrass and pearl millet surface or directly onto the root zone, through a network
belong to the botanical family Gramineae. Orange and of valves, pipes, tubing, and emitters.
lemon are citrus fruits. Drip irrigation was developed in Israel by Simcha Blass
31. (b) A cultigen is a plant that has been deliberately altered and his son Yeshayahu.
or selected by humans; it is the result of artificial 52. (a) Narmada is the only river in India that flows in a rift
selection. These "man-made" or anthropogenic plants valley, flowing west between the Satpura and Vindhya
are, for the most part, plants of commerce that are used ranges. The Tapti River and Mahi River also flow
in horticulture, agriculture and forestry. through rift valleys, but between different ranges.

Downloded From : www.EasyEngineering.net


Downloded From : www.EasyEngineering.net

Geography 25
53. (b) An Isthmus is a narrow strip of land connecting two 68. (a) The Mahadeo Hills are a range of hills in Madhya
larger land areas, usually with water on either side. Pradesh state of central India. The hills form the central
54. (d) A topographic map is a type of map characterized by part of the Satpura Range.
large scale detail and quantitative representation of 69. (d) Dekke Toba fish in found in Indonesia. Lake Toba
relief, usually using contour lines in modern mapping, (Indonesian: Danau Toba) is a lake and supervolcano.
but historically using a variety of methods. The lake is 100 kilometres long, 30 kilometres wide, and
55. (a) Magnetic declination is the angle between compass up to 505 metres (1,666 ft) deep. The fauna includes
north (the direction the north end of a compass needle several species of zooplankton and benthic animals.
points) and true north (the direction along the earth’s Since the lake is oligotrophic (nutrient-poor), the native
surface towards the geographic North Pole. fish fauna is relatively scarce, and the only endemics
56. (c) Above 10,000 hectares are Rasbora tobana.
57. (c) Operation Flood in India, a project of the National Dairy 70. (c) Timber means wood that we obtain from plants are
Development Board (NDDB) was the world's biggest called renewable in the sense that after cutting a tree if
dairy development program which made India, a milk- we plant another tree then it will grow up and again
deficient nation, the largest milk producer in the world, give us wood and timber whereas all other things given
surpassing the USA in 1998, with about 17 percent of are non-renewable, once the stock is finished we cannot

ww
global output in 2010-11, which in 30 years doubled
the milk available per person, and which made dairy
farming India's largest self-sustainable rural
71.
get more of it.
(a) Revegetation is often used to join up patches of natural
habitat that have been lost, and can be a very important

58.
w.E
employment generator. All this was achieved not merely
by mass production, but by production by the masses.
(b) the law explains that wind is deflected to the right in
tool in places where much of the natural vegetation
has been cleared. It is therefore particularly important
in urban environments, and research in Brisbane has

asy
the Northern Hemisphere and to the left in the Southern
Hemisphere, derived from the application of the Coriolis
effect to air masses.
shown that revegetation projects can significantly
improve urban bird populations. The Brisbane study
showed that connecting a revegetation patch with
59.
upper atmosphere, between roughly 15 and 35 km (9 En
(d) ozone layer, also called ozonosphere, region of the existing habitat improved bird species richness, while
simply concentrating on making large patches of habitat
and 22 miles) above Earth's surface, containing
relatively high concentrations of ozone molecules (O3). gin
72.
was the best way to increase bird abundance.
(c) A dust storm or sand storm is a meteorological
60. (b) A dendritic drainage pattern refers to the pattern formed
by the streams, rivers, and lakes in a particular drainage eer
phenomenon common in arid and semi-arid regions.
Dust storms arise when a gust front or other strong
basin. It usually looks like the branching pattern of
tree roots and it mainly develops in regions underlain
by homogeneous material.
73.
ing
wind blows loose sand and dirt from a dry surface.
(d) A river profile is a curve which shows the slop of a
river from source to mouth.A stream flowing over
61. (c) Plimsol line is not a line of demarcation between two
countries. .ne
irregular terrain may have waterfalls, rapids and lakes
along its course.Though the stream will wear away the
62. (d) A halophyte is a plant that grows in waters of high
salinity, coming into contact with saline water through
its roots or by salt spray, such as in saline semi-deserts, 74.
a graded profile. t
irregularities to leave a smoothly curving profile called

(c) Karst is a landscape formed from the dissolution of


soluble rocks including limestone, dolomite and
mangrove swamps, marshes and sloughs, and
seashores. An example of a halophyte is the salt marsh gypsum. It is characterized by sinkholes, caves, and
grass Spartina alterniflora (smooth cordgrass). underground drainage systems.
63. (b) Azotobacter is a genus of usually motile, oval or 75. (d) Kerala is famous for the cultivation of coconut, tea,
spherical bacteria that form thick-walled cysts and may coffee, cashew and spices.
produce large quantities of capsular slime. 76. (a) The Trans-Siberian Railway is the longest railway line
64. (c) Extratropical cyclones, sometimes called mid-latitude in the world. It has a length of 9,289 km which connects
cyclones or wave cyclones, are a group of cyclones Moscow to Vladivostok.
defined as synoptic scale low pressure weather 77. (c) One of the Major objectives and advantages of
systems that occur in the middle latitudes of the Earth rainwater harvesting is to reduce run off loss.
(outside the tropics) not having tropical characteristics, 78. (b) Brahmaputra River is one of the largest rivers in the
and are connected with fronts and horizontal gradients world. The origin of Brahmaputra River is in
in temperature and dew point otherwise known as southwestern Tibet as the Yarlung River.
"baroclinic zones". 79. (d) Thermal Power Plants consumes maximum industrial
65. (c) 66. (c) 67. (a) water in India.

Downloded From : www.EasyEngineering.net


Downloded From : www.EasyEngineering.net

26 Geography
80. (a) The current sequence is Troposphere-Stratosphere- 95. (c)
Mesosphere-Ionosphere-Exosphere 96. (b) Tehri Dam is a multi-purpose rock and earth-fill
81. (d) The Earth and the atmosphere are heated by energy embankment dam on the Bhagirathi River near Tehri in
from the sun. The atmospheric heat budget of the Earth Uttarakhand, India.
depends on the balance between insolation and 97. (d) India has 15,106.7 km of land border and a coastline of
outgoing terrestrial radiation. 7,516.6 km including island territories. The length of
82. (d) The troposphere is the first layer above the surface our land borders with neighbouring countries is as
and contains half of the Earth's atmosphere. Weather follows : Bangladesh : 4,096.7 China : 3,488 Pakistan :
occurs in this layer. This is closest to the Earth's surface, 3,323 Nepal : 1,751 Myanmar : 1,643 Bhutan : 699
extending up to about 10-15 km above the Earth's Afghanistan : 106.
surface. 98. (c) Gujarat is strategically located with largest share in
83. (a) Sugarcane shows chloroplast dimorphism. The India’s coastline, followed by Andhra Pradesh and
presence of bundle sheath with chloroplast in the Tamil Nadu.
leaves is granulated and lacking in starch. Such feature 99. (c) Jog Fall located at southern state of Karnataka is
is known as chloroplast dimorphism. India’s highest waterfall. Its height is 830 feet and is
84. (c) The equator receives equal day and night throughout located on Sharavati River.

ww
the year because it does not tilt in relation to the sun's
location. Because of the tilted axis of the Earth, the poles
and locations away from the equator lean towards or away
100.
104.
(c) 101. (d) 102. (a) 103. (a)
(a) Kalahari desert is present in Africa while Atacama
Desert is in South America. Thar Desert is in Australia

85.
w.E
from the sun as an orbit is completed, while the equator
stays in essentially the same location relative to the sun.
(b) The tropical rainforest is earth's most complex biome
105 .
and Great Victoria is in Australia.
(a) Godavari is the longest river of peninsular India. From

asy
in terms of both structure and species diversity. It
occurs under optimal growing conditions, abundant
its source to the Eastern Ghats, the Godavari River
flows through gentle, somewhat monotonous terrain,
along the way receiving the Darna, Purna, Manjra,
precipitation and year round warmth. Mean monthly

En
temperatures are above 64 ° F; precipitation is often in Pranhita, and Indravati rivers. Upon entering the
Eastern Ghats region, however, the river flows between
excess of 100 inches a year. There is usually a brief
season of reduced precipitation. In monsoonal areas,
there is a real dry season but that is more than gin steep and precipitous banks, its width contracting until
it flows through a deep cleft only 600 feet (180 metres)
compensated for with abundant precipitation the rest
of the year. 106. eer
wide, known as the Gorge.
(a) The Himalayan Mountain Range is an example of fold
86. (a) podsol soil is a soil that develops in temperate to cold
moist climates underconiferous or heath vegetation; ing
mountain.They are known as fold mountains becaus
the mountains extend for 2500 km in length in a series
e

87.
88.
(d)
an organic mat over a grey leached layer.

(b) The largest shelf - the Siberian Shelf in the Arctic .ne
of parallel ridges or folds and consist of three folds
namely Himadri, Himachal, Shiwalik.

Ocean - stretches to 1,500 kilometers (930 mi) in width.


The South China Sea lies over another extensive area
of continental shelf, the Sunda Shelf, which joins
107.

108.
t
(a) Gulf Stream is a warm ocean current. It flows along the
North America and drifts towards western Europe, thus
raising the temperature of western coast considerably.
(b) Recharging of ground water is the main advantage of
Borneo, Sumatra, and Java to the Asian mainland.
rain water harvesting. Rainwater harvesting provides
89. (b) In terms of length, catchment area and discharge, the
an independent water supply during regional water
Godavari river is the largest in peninsular India and
restrictions and in developed countries is often used
had been dubbed as the 'Dakshina Ganga' - the South
to supplement the main supply. It provides water when
Ganges river.
there is a drought, can help mitigate flooding of low-
90. (c) 91. (c) 91. (c) 92. (b)
lying areas, and reduces demand on wells which may
93. (c) A solar eclipse occurs when the Moon passes between
Earth and the Sun, thereby totally or partly obscuring enable ground water levels to be sustained.
the image of the Sun for a viewer on Earth. 109. (c) India is the largest producer and exporter of tea. The
94. (c) Approximately 9 miles (14 km) wide at its narrowest Indian tea industry has grown to own many global tea
point, the Strait of Gibraltar is the entry point into the brands and has evolved into one of the most
Mediterranean Sea from the Atlantic Ocean. It is technologically equipped tea industries in the world.
bordered by the continents of Africa and Europe, and Tea production, certification, exportation, and all other
the countries of Morocco, Spain, the British colony of facts of the tea trade in India is controlled by the Tea
Gibraltar, and the Spanish exclave of Ceuta. Board of India.

Downloded From : www.EasyEngineering.net


Downloded From : www.EasyEngineering.net

Geography 27
110. (a) Pedocal is a subdivision of the zonal soil order. It is a 128. (d) The National Green Tribunal deals with cases relating
class of soil which forms in semiarid and arid regions. to environmental protection and conservation of
It is rich in calcium carbonate and has low soil organic forests and other natural resources.
matter. 129. (a) Kuwait Spill in Persian Gulf, Kuwait of 19th January
111. (d) Land able to be used for farming is called "cultivable 1991 was the biggest oil spill in world history.
land". It includes total fallow land and net sown area. 130. (c) Japan has the highest life expectancy among the
112. (d) Gulf of Mannar with a area of 10500 sq.km. is the largest countries across the world.
biosphere reserve of India.
131. (b) Scattering of light causes redness in atmosphere at
113. (a) In physical geography, a steppe is an ecoregion,
Sunrise and Sunset.
characterized by grasslandplains without trees apart
132. (c) Pygmies follow practices of pastoral nomadism.
from those near rivers and lakes.
114. (a) Covering around 2 percent of the Earth's total surface 133. (d) High Yielding Variety (HYV) seeds played vital role in
area, the world's tropical rainforests are home to the progress of agriculture. They are considered as
50 percent of the Earth's plants and animals. Rainforests `miracle seeds'. The high yielding programme was
can be found all over the world from as far north as launched in the Kharif season of 1966 in selected areas
Alaska and Canada to Latin America, Asia and Africa. having assured rainfall. Seed is the basic and crucial
input for attaining sustained growth in agricultural

ww
115. (d) The world's growing appetite for beef is a leading cause
of tropical deforestation.
116. (c) Kyoto Protocol is an international treaty among
production.
134. (c) Tea is an aromatic beverage commonly prepared by

w.E
countries to reduce greenhouse gases emissions based
on the premise that (i) global warming exists and (ii)
man-made CO2 emissions.
pouring hot or boiling water over cured leaves of the
tea plant, Camellia sinensis. After water, tea is the most
widely consumed beverage in the world.

118. (a)
freshwater on Earth. asy
117. (d) Glacial ice or Glaciers are the largest reservoir of
135. (b) Delhi-Kolkata section
136. (c) The Tropic of Cancer, also referred to as the Northern
119. (a) About 70 percent of the Earth’s surface is water-
En
covered and the oceans hold about 96.5 percent of all
tropic, is the circle of latitude on the Earth that marks
the most northerly position at which the Sun may
Earth’s water.
120. (d) Johannes Kepler was a German mathematician, gin appear directly overhead at its zenith.
137. (d) The demographics of the People's Republic of Chin a
astronomer and astrologer. He is best known for his
laws of planetary motion which stated that the orbit of eer
are identified by a large population with a relatively
small youth division, which is partially a result of
a planet is an ellipse with the Sun at one of the two
foci.
ing
China's one-child policy. Chinese population reached
the billion mark in 1982.
121. (a) The Trans-Himalayan rivers originate beyond the great
Himalayas. The name of the rivers are the Indus, the
Sutlej and the Brahmaputra river. .ne
138. (a) The Ravva oil and gas field in the Krishna- Godavari
Basin was developed in partnership with Cairn India,
122. (a) Black soil or Black Lava Soil is considered most suitable
for sowing cotton crops.
123. (c) Neyveli lignite field in Tamil Nadu is the largest lignite
sharing contract (PSC) that runs until 2019. t
ONGC, Videocon and Ravva Oil, under a production

139. (d) Starting from north, Machallipatnam is located in


coal mine in India. Andhra Pradesh. Chennai, Nagapatnam and Tuticorin
124. (a) Sex ratio is used to describe the number of females per are present in Tamil Nadu.
1000 males in country. In the Population Census of 140. (d) Santa Cruz is a domestic airport in Mumbai.
2011 it was revealed that the population ratio in India 141. (b) The Almatti Dam is a dam project on the Krishna River
2011 is 940 females per 1000 of males. in North Karnataka, India which was completed in July
125. (a) In Tamil Nadu, rain water harvesting was made 2005. The Almatti Dam is the main reservoir of the Upper
compulsory for every building to avoid ground water Krishna Irrigation Project.
depletion.
142. (a) Damodar Valley Corporation, commonly known as
126. (c) NASA has developed a series of interactive a maps
DVC, was incorporated on July 7, 1948 by an Act of the
and graphs to describe the global climte and how it
Constituent Assembly of India (Act No. XIV of 1948).
change over time. The focus of 5 key climate indications
It was the oldest versatile river basin project of
include Arctic sea ice, carbon dioxide concentration,
Global surface temperature, Sea level and land ice. sovereign India. Damodar Valley Corporation has been
127. (d) The area reserved for the welfare of wildlife is called planned like the Tennessee Valley Authority of the
National Park. USA.

Downloded From : www.EasyEngineering.net


Downloded From : www.EasyEngineering.net

28 Geography
143. (a) Petrology is the branch of geology that studies the 151. (a) The Chhattisgarh Assembly, in December 2012,
origin, composition, distribution and structure of rocks. enacted a landmark legislation by unanimously passing
144. (b) the Food Security Act, becoming the first state in the
145. (c) The eastern coast of India, particularly in Tamilnadu, country to introduce such a law. The Act makes food
remains relatively dry during the south-west monsoon entitlements a right and its non-compliance has been
period. This is because the Tamilnadu coast lies in the made an offence.
rain-shadow area of the Arabian Sea current and is 152. (c) Visakhapatnam Steel Plant (VSP) is the first shore based
parallel to the Bay of Bengal current. integrated steel plant Public sector in the country
146. (a) The International Date Line (IDL) is an imaginary line located at Visakhapatnam in Andhra Pradesh. The plant
on the surface of the Earth from the north to the south was commissioned in August 1992 with a capacity to
pole and demarcates one calendar day from the next. It produce 3 million tonnes per annum (MTPA) of liquid
passes through the middle of the Pacific Ocean, roughly steel. The plant has been built to match international
following the 180° longitude but it deviates to pass standards with state-of-the-art technology,
around some territories and island groups. incorporating extensive energy saving and pollution
control measures. VSP has an excellent layout capable
147. (b) The position of Meghalaya is 22nd in terms of both
of expanding up to 16 MTPA.
area as well as population. The position of rest of the

ww
states in terms of area and population respectively are:
Nagaland: 25, 24; Manipur: 23, 23 (excludes Mao-
153. (c) Environmental conditions play a key role in defining
the function and distribution of plants, in combination
with other factors which influence the development of

w.E
Maram. Paomata. and Purul sub-divisions of Senapati
district of Manipur); and Gujarat: 7, 10.
148. (c) An oxbow lake is a U-shaped body of water formed
plants into distinctive forms.
154. (d) Dr. M S Swaminanthan said the given lines.
Swaminathan is known as "Indian Father of Green

asy
when a wide meander from the main stem of a river is
cut off to create a lake. This landform is called an oxbow
lake for the distinctive curved shape, named after part
Revolution" for his leadership and success in
introducing and further developing high-yielding

of a yoke for oxen.


En varieties of wheat in India.
155. (b) Mangrove trees act as a natural barrier against
149. (c) Gujarat is the leading state in ship breaking industry
because of its long coastline and a well developed
infrastructure for the industry-specific works. Alang gin cyclones and storms.
156. (c) 157. (d) 158. (a) 159. (b)
160. (d) Chile is the world's largest copper producer, producin g
in Gujarat, considered to be the world's largest, is a
famous ship breaking centre. eer
1.79 million tons in 2013.
161. (d) 162. (d) 163. (d) 164. (d) 165. (b)
150. (c) Contour line is a line on a map or chart joining points
of equal height (elevation) or depth. Closely spaced ing
166. (a) 167. (d) 168. (b) 169. (d) 170. (c)
171. (b) 172. (c) 173. (a) 174. (a) 175. (b)
contour lines indicate a steeper grade than the more
loosely spaced lines. .ne
176. (a) 177. (d) 178. (d) 179. (a) 180. (a)
181. (c) 182. (c) 183. (c) 184. (c) 185. (d)

Downloded From : www.EasyEngineering.net


Downloded From : www.EasyEngineering.net

3
CHAPTER
POLITY
1. The term 'Caste' was derived from 12. Which one of the following is the guardian of Fundamental
(SSC CGL 1st Sit. 2010) Rights ? (SSC CGL 2nd Sit. 2010)
(a) Portuguese (b) Dutch (a) Legislature (b) Executive
(c) German (d) English (c) Political parties (d) Judiciary
2. The term 'Greater India' denotes 13. Sarkaria Commission was concerned with
(SSC CGL 1st Sit. 2010) (SSC CGL 2nd Sit. 2010)
(a) Political unity (b) Cultural unity (a) Administrative Reforms
(c) Religious unity (d) Social unity (b) Electoral Reforms
3. Who is rightly called the "Father of Local Self Government" (c) Financial Reforms

ww
in India ?
(a) Lord Mayo
(SSC CGL 1st Sit. 2010)
(b) Lord Ripon 14.
(d) Centre-State relations
The speaker of the Lok-Sabha has to address his/her letter

4.
(c) Lord Curzon
w.E (d) Lord Clive
The two forms of democracy are (SSC CGL 1st Sit. 2010)
(a) Parliamentary and Presidential
of resignation to
(a) Prime Minister of India
(b) President of India
(SSC CGL 2nd Sit. 2010)

(b) Direct and Indirect


(c) Monarchical and Republican asy 15.
(c) Deputy Speaker of Lok Sabha
(d) Minister of Parliamentary Affairs
A Presidential Ordinance can remain in force
5.
(d) Parliamentary and King
Which is an extra-constitutional body ?
En (a) For Three months
(SSC CGL 2nd Sit. 2010)
(b) For six months
(a) Language Commission
(b) Planning Commission
(c) Election Commission
(SSC CGL 1st Sit. 2010)

gin
16.
(c) For nine months (d) Indefinitely
Judicial review in the Indian Constitution is based on:

6.
(d) Finance Commission
The Prime Minister of India is (SSC CGL 1st Sit. 2010) eer
(a) Rule of Law
(b) Due process of Law
(SSC CGL 1st Sit. 2011)

(a) Elected
(c) Nominated
(b) Appointed
(d) Selected
17.
ing
(c) Procedure established by Law
(d) Precedents and Conventions
The Drafting of the Constitution was completed on:
7. Which is not an All India Service ? (SSC CGL 1st Sit. 2010)
(a) Indian Administration Service th
(a) 26 January, 1950 .ne
(SSC CGL 1st Sit. 2011)
(b) 26th December, 1949

8.
(b) Indian Police Service
(c) Indian Foreign Service
(d) Indian Forest Service
The declaration that Democracy is a Government 'of the
18.
(c) 26th November, 1949 (d) 30th November, 1949
t
Who was the President of the Constituent Assembly?
(SSC CGL 1st Sit. 2011)
(a) Pt. Jawahar Lal Nehru (b) Sardar Patel
people' by the people; for the people' was made by
(c) Dr. Rajendra Prasad (d) Dr. B.R. Ambedkar
(SSC CGL 2nd Sit. 2010)
19. Which innovative discussion process is introduced by the
(a) George Washington (b) Winston Churchill Indian parliament to the World Parliamentary systems?
(c) Abraham Lincoln (d) Theodore Roosevelt (SSC CGL 1st Sit. 2011)
9. India attained 'Dominion Status' on (a) Question hour (b) Zero hour
(SSC CGL 2nd Sit. 2010) (c) Resolutions (d) Presidential Speech
(a) 15th January, 1947 (b) 15th August, 1947 20. The judges of the Supreme Court retire at the age of :
(c) 15th August, 1950 (d) 15th October, 1947 (SSC CGL 1st Sit. 2011)
10. Despotism is possible in a (SSC CGL 2nd Sit. 2010) (a) 60 years (b) 65 years
(a) One party state (b) Two party state (c) 62 years (d) 58 years
(c) Multi party state (d) Two and multi party state 21. Who was the architect of North and South Blocks of the
11. Marx belonged to (SSC CGL 2nd Sit. 2010) Central Secretariate in Delhi? (SSC CGL 1st Sit. 2011)
(a) Germany (b) Holland (a) Sir Edward Lutyens (b) Herbert Baker
(c) France (d) Britain (c) Robert Tor Russell (d) Antonin Raymond

Downloded From : www.EasyEngineering.net


Downloded From : www.EasyEngineering.net

30 Polity
22. If the Anglo–Indian community does not get adequate (a) Commissioner for Scheduled Castes and Tribes
representation in the Lok Sabha, two members of the (b) Prime Minister
community can be nominated by the (c) President
(a) Prime Minister (SSC CGL 2nd Sit. 2011) (d) Governor
(b) President 33. The seat of Kerala High Court is located at:
(c) Speaker (SSC CGL 2nd Sit. 2012)
(d) President in consultation with the Parliament (a) Kottayam (b) Thiruvananthapuram
23. For the election of President of India, a citizen should have (c) Kollam (d) Ernakulam
completed the age of (SSC CGL 2nd Sit. 2011) 34. The first speaker of Lok Sabha was:
(a) 25 years (b) 30 years (a) S. Radhakrishnan (SSC CGL 2nd Sit. 2012)
(c) 35 years (d) 18 years (b) M. Ananthasayanam Ayyangar
24. A member of Parliament will lose his membership of (c) Sardar Hukum Singh
Parliament if he is continuously absent from Sessions for
(d) G. V. Mavlankar
(SSC CGL 2nd Sit. 2011)
35. The Social Contract theory deals with
(a) 45 days (b) 60 days
(SSC CGL 1st Sit. 2012)
(c) 90 days (d) 365 days
(a) The Purpose of State
25.
ww
In India, the Residuary Powers are vested with
(a) Union Government
(b) State Government
(SSC CGL 2nd Sit. 2011)
(b) The Origin of the State
(c) The Nature of State

26.
(d) Local Government w.E
(c) Both the Union Government and the State Government

The National Commission for Minorities was constituted in


36.
(d) The Functions of State
The Supreme Court of India offers advice to the President
on matters of Legal, Public or Constitutional importance
the year
(a) 1990 (b) 1992asy
(SSC CGL 1st Sit. 2012)
based on
(a) Article-148
(SSC CGL 1st Sit. 2012)
(b) Article-129

27.
(c) 1980 (d) 1989
En
In which of the following systems of government is bi–
37.
(c) Article-147 (d) Article-143
Which is the most effective means of executive control of
administration ? (SSC CGL 1st Sit. 2012)
cameralism an essential feature? (SSC CGL 1st Sit. 2012)
(a) Federal system (b) Unitary system
gin (a) Financial administration
(b) Political direction
28.
(c) parliamentary system (d) Presidential system
Socialism succeeds in achieveing (SSC CGL 1st Sit. 2012)
(a) higher standard of living of the people eer
(c) Appointment and removal of top officials
(d) Subordinate legislation
(b) equal distribution of income in the society
(c) higher individual welfare in the society
38.
ing
Violation of‘Rule of Law’ arises mostly from
(SSC CGL 1st Sit. 2012)

29.
(d) maximum social welfare in the society
The seat of Madhya Pradesh High Court is located at
(a) Lack of Checks and Balances
.ne
(b) Executive’s Quasi-Judicial Powers

30.
(a) Gwalior
(c) Bhopal
(SSC CGL 1st Sit. 2012)
(b) Indore
(d) Jabalpur
Who said that the Directive Principles of State Policy are
39.
(c) Limited Franchise
(d) Delegated Legislation
t
The Constitutional Amendment Act that has introduced
safeguards against the misuse of proclamation of national
just like “a cheque on bank payable at the convenience of emergency is the (SSC CGL 2nd Sit. 2012)
the bank”. (SSC CGL 2nd Sit. 2012) (a) 42nd Amendment Act (b) 43rd Amendment Act
(a) Pandit Nehru (b) K. T. Shah (c) 44th Amendment Act (d) 45th Amendment Act
(c) B. R. Ambedkar (d) N. G. Ranga 40. The Fundamental Rights can be suspended by the
31. The proposal for the creation of new All-India Services can (SSC CGL 2nd Sit. 2012)
be considered only: (SSC CGL 2nd Sit. 2012) (a) Governor (b) President
(a) if majority of State Legislatures make such demand (c) Law Minister (d) Prime Minister
(b) if Lok Sabha passes a resolution by two-thirds majority 41. The main reason for the growth of communalism in India is
(c) if the Rajya Sabha passes a resolution by two-thirds (SSC CGL 2nd Sit. 2012)
majority (a) Educational and economic backwardness of minority
(d) None of the above groups
32. The authority to specify which castes shall be deemed to (b) Political consciousness
be scheduled castes rests with the: (c) Social inequalities
(SSC CGL 2nd Sit. 2012) (d) Imposing ban on communal organisations

Downloded From : www.EasyEngineering.net


Downloded From : www.EasyEngineering.net

Polity 31
42. A Retired Judge of a High Court is not permitted to practice 53. Who presides over the Joint Session of Indian Parliament?
as a lawyer in (SSC CGL 2nd Sit. 2012) (SSC CGL 1st Sit. 2013)
(a) Supreme Court (a) Chairperson of Rajya Sabha
(b) Any Court in India (b) Seniormost Member of Parliament
(c) High Courts (c) Speaker of Lok Sabha
(d) Except the High Court where he retired (d) President of India
43. Which one of the following does not match? 54. Under which Article of the Constitution of India, can the
(SSC CGL 2nd Sit. 2012) fundamental rights of the members of the Armed Forces be
(a) Hindu Marriage Act : 1955 specifically restricted? (SSC CGL 1st Sit. 2013)
(b) Medical Termination of Pregnancy Act : 1971 (a) Article 21 (b) Article 25
(c) Domestic Violence on women Act : 1990 (c) Article 33 (d) Article 19
(d) Cruelty against Women : 1995 55. Which Article of the Indian Constitution did Dr. B. R.
44. The vacancy of the office of the President must be filled Ambedkar term as the “Heart and Soul of the Indian
within : (SSC CGL 1st Sit. 2013) Constitution”? (SSC CGL 1st Sit. 2013)
(a) 3 months (b) 6 months (a) Article 356 (b) Article 32

45. ww
(c) 12 months (d) 1 month
In which part of the Indian Constitution, the fundamental
duties are enshrined ? (SSC CGL 1st Sit. 2013)
56.
(c) Article 14 (d) Article 19
Who was the first to use the term ‘State’?
(SSC CGL 1st Sit. 2013)
(a) IV
(c) IV B w.E (b) IV A
(d) V
(a) Aristotle
(c) Hobbes
(b) Machiavelli
(d) Plato
46.
charter of bondage ?
(a) B.R. Ambedkar
asy
Who described the Government of India Act, 1935 as a new
(SSC CGL 1st Sit. 2013)
(b) Mahatma Gandhi
57. Who is the Chairman of the 20th Law Commission?

(a) Justice K.G. Balakrishnan


(SSC CGL 1st Sit. 2013)

(c) Rajendra Prasad


En
(d) Pt. Jawaharlal Nehru (b) Justice D. K. Jain
(c) Justice Usha Mehra
47. Which of the following is not a fundamental right as per
the Indian Constitution?
(a) Right to Education
(SSC CGL 2nd Sit. 2013)
(b) Right to Information gin
58.
(d) Justice J. S. Verma
Which one of the following is not a function of Electio n

48.
(c) Right to Speech (d) Right to Life
Who is custodian of the Indian Constitution? eer
Commission ?
(a) Allotment of symbols
(SSC CGL 1st Sit. 2013)

(a) President of India


(SSC CGL 2nd Sit. 2013)
(b) Chief Justice of India ing
(b) Fixation of election dates
(c) Maintaining fairness of election

49.
(c) Prime Minister of India (d) Chairman of Rajya Sabha
Constituent Assembly of India was formulated on the 59. .ne
(d) Selecting the candidates for election
‘State is a necessary evil’ is associated with
recommendation of
(a) Wavel Plan
(c) August Offer
(SSC CGL 2nd Sit. 2013)
(b) Cripps Mission
(d) Cabinet Mission
(a) Individualism
(c) Marxism
(b) Idealism t
(SSC CGL 1st Sit. 2013)

(d) Constructivism
50. Which of the following is an essential element of the state? 60. Who was the first President of the All India Trade Union
(SSC CGL 2nd Sit. 2013) Congress (AITUC) ? (SSC CGL 1st Sit. 2013)
(a) Sovereignty (b) Government (a) C.R. Das (b) V.V. Giri
(c) Territory (d) All these (c) Lala Lajpat Rai (d) Sarojini Naidu
51. Which has become a legal right under 44th Amendment? 61. If there is a deadlock between Rajya Sabha and Lok Sabha
(a) Right to Education (SSC CGL 2nd Sit. 2013) over an ordinary bill, it will be resolved by
(b) Right to Property (a) The President (SSC CGL 1st Sit. 2013)
(c) Right to Judicial Remedies (b) The Council of Ministers
(d) Right to work (c) The Joint Session of Parliament
52. By which Constitution Amendment Act, Right to Property (d) The Supreme Court
ceased to remain a fundamental right? 62. Provisions of citizenship in Indian Constitution, became
(SSC CGL 2nd Sit. 2013) applicable in (SSC CGL 1st Sit. 2013)
(a) 44th (d) 42nd (a) 1950 (b) 1949
(c) 43rd (d) 45th (c) 1951 (d) 1952

Downloded From : www.EasyEngineering.net


Downloded From : www.EasyEngineering.net

32 Polity
63. The National Emergency in India declared by the President 75. The South East trade winds are attracted towards the Indian
of India due to the external aggression or armed revolt sub continent in the rainy season due to
through (SSC CGL 1st Sit. 2013) (SSC CGL 1st Sit. 2015)
(a) Article–352 (b) Article–356 (a) the effect of easterlies
(c) Article–360 (d) Article–368 (b) the effect of Northern–East trade winds
64. The most important feature of Cabinet system of (c) the presence of low atmospheric pressure over North–
Government is (SSC CGL 2014) West India
(a) Individual responsibility (d) the development of cyclone over the equator
(b) Collective responsibility 76. The 'graded profile' of a river course is a
(c) Responsibility to none (SSC CGL 1st Sit. 2015)
(d) Non-responsibility (a) smooth curve in the upper course
65. Direct legislation in Switzerland has (SSC CGL 2014) (b) smooth curve in the middle course
(a) a natural growth (b) a haphazard growth (c) smooth curve in the lower course
(c) an artificial growth (d) None of the above (d) smooth curve from source to mouth
66. Who gave the idea of "Cabinet Dietatorship"? 77. Sink hole is a phenomenon of _______ topography.
(SSC CGL 2014) (SSC CGL 1st Sit. 2015)
(a) Muir (b) Lowell

67. ww
(c) Marriot (d) Laski
In which of the following countries are the judges of the 78.
(a) Desert
(c) Karst
(b) Tundra
(d) Plain
Kerala is famous for the cultivation of
Legislature?
(a) Switzerland
w.E
federal court elected by the two Houses of the Federal

(b) Germany
(SSC CGL 2014)
1. Coconut
3. Rubber
(SSC CGL 1st Sit. 2015)
2. Black pepper
4. Rice
68.
(c) Canada
asy
(d) Both (a) and (b)
The President of the USA appoints Supreme Court Judges
(a) with Senate's consent (SSC CGL 2014)
(a) 1, 2 and 4
(c) 1 and 4
(b) 2, 3 and 4
(d) 1, 2 and 3
(b) at his discretion
(c) with consent of the House of Representatives En 79. The longest continental Railway in the world is
(a) Trans Siberian Railway (SSC CGL 1st Sit. 2015)

69.
(d) None of these
Which of the following is not a Union Territory ? gin (b) Canadian Pacific Railway
(c) Canadian National Railway

(a) Lakshadweep
(SSC CGL 1st Sit. 2015)
(b) Puducherry 80.
eer
(d) Trans Atlantic Railway
Who was the first Speaker of the Lok Sabha :
(SSC CGL 1st Sit. 2015)
70.
(c) Nagaland (d) Dadra and Nagar Haveli
The greatest king of the Pratihara dynasty was
(SSC CGL 1st Sit. 2015)
(a) B.R. Ambedkar
(c) N. Sanjeev Reddy ing (b) G.V. Mavalankar
(d) Dr S.P. Mukherjee
(a) Bhoj (Mihir–Bhoj)
(c) Nagbhatta II
(b) Dantidurga
(d) Vatsaraj
81. What is the plural volting system?
.ne
(SSC CGL 1st Sit. 2015)
71. In 1939 Subhash Chandra Bose was elected as President of
the Congress Party defeating
(a) Jawaharlal Nehru
(SSC CGL 1st Sit. 2015)
(a) All the citizens caste three votes each
t
(b) Eligible voter exercises one vote and some voters with
specific qualifications cast more than one vote.
(c) Only the higher officials caste more than one votes
(b) Maulana Abul Kalam Azad
(c) V.B. Patel (d) Candidates themselves caste more than one vote.
(d) Pattabhi Sitharamayya 82. Voting is : (SSC CGL 1st Sit. 2015)
72. Jallianwala incident took place at (SSC CGL 1st Sit. 2015) (a) The unit of area who constitute a unit for electing
(a) Lucknow (b) Surat representative
(c) Amritsar (d) Allahabad (b) The process by which voters exercise their right to vote
73. Who was the founder of Lodhi dynasty ? (c) The process of selecting representatives
(SSC CGL 1st Sit. 2015) (d) Universal adult franchise.
(a) Sikandar Lodhi (b) Bahlol Lodhi 83. An amendment of the constitution may be initiated.
(c) Ibrahim Lodhi (d) Daulat Khan Lodhi (SSC CGL 1st Sit. 2016)
74. Which one of the following pair is not correctly matched ? (a) by introduction by the President of India.
(a) Akbar – Todarmal (SSC CGL 1st Sit. 2015) (b) by introduction of a Bill in Rajya Sabha.
(b) Chanakya – Chandragupta (c) by the Governors of States.
(c) Vikramaditya – Chaitanya (d) by the introduction of a bill in either House of
(d) Harshvardhan – Hiuen Tsang Parliament.

Downloded From : www.EasyEngineering.net


Downloded From : www.EasyEngineering.net

Polity 33
84. The Directive Principles of State Policy has been adopted 95. Which one of the following items comes under the
from which Constitution? (SSC CGL 1st Sit. 2016) Concurrent List ? (SSC CHSL 2012)
(a) U.S. Constitution (b) British Constitution (a) Trade Unions (b) Citizenship
(c) Irish Constitution (d) French Constitution (c) Local Government (d) Inter-State rivers
85. Which type of democracy do we follow in India?
96. Which Amendment of the Constitution deals with Political
(SSC CGL 1st Sit. 2016)
defections ? (SSC CHSL 2012)
(a) Direct (b) Presidential
(a) 50th (b) 52th
(c) Representative (d) Dictatorship
86. The Chairman of the Drafting Committee of the Constituent (c) 60th (d) 44th
Assembly of India was (SSC CGL 1st Sit. 2016) 97. Which schedule of the Indian Constitution prescribes
(a) K.M. Munshi (b) D.P. Khaitan distribution of seats in Rajya Sabha ? (SSC CHSL 2012)
(c) Dr. B.R. Ambedkar (d) T.T. Krishnamacharia (a) 4th schedule (b) 5th schedule
87. Who has the right to decide whether a Bill is a money bill or (c) 6th schedule (d) 3rd schedule
not? (SSC CGL 1st Sit. 2016) 98. Who said in the Constituent Assembly that the Directive
(a) Speaker of Lok Sabha (b) Prime Minister Principles of the State Policy are like a 'Cheque on a bank
(c) President (d) Finance Minister payable at the convenience of the bank' ?
88. The discretionary powers of a Governor is limited in

ww
(a) Appointment of Chief Minister
(SSC CGL 1st Sit. 2016) (a) K.M. Munshi
(c) Austin
(SSC CHSL 2012)
(b) B.R. Ambedkar
(d) K. T. Shah

w.E
(b) Dismissal of the Ministry
(c) Dissolution of the Legislative Assembly
(d) Assent to Bills
99. How many members can be nominated to both the Houses
of the Parliament by the President ? (SSC CHSL 2013)
(a) 14 (b) 16
89.
asy
Who is the first law officer of the country?
(SSC CGL 1st Sit. 2016)
(a) Chief Justice of India (b) Attorney General 100.
(c) 10 (d) 12
Under which Article of the Constitution can an individual

90.
(c) Law Minister (d) Solicitor General
En
Which one of the following was established with a definite
move to the Supreme Court directly in case of any violation
of Fundamental Rights ? (SSC CHSL 2013)
provision under an Article of the Constitution of India?
(SSC CGL 1st Sit. 2016) gin (a) Article 31
(c) Article 28
(b) Article 32
(d) Article 29
(a) Union Public Service Commission
(b) National Human Rights Commission
101.
eer
Presidential form of government consists of the following:
(SSC CHSL 2013)

91.
(c) Election Commission
(d) Central Vigilance Commission
The President can advance money to meet unforeseen ing
(a) Popular election of the President
(b) No overlap in membership between the executive an d
expenses from the
(a) Consolidated Fund of India
(SSC CGL 1st Sit. 2016) the legislature
(c) Fixed term of office .ne
92.
(b) Grants of the Central Government
(c) Aid from the Union Government
(d) Contingency Fund
How many cricketers, who have represented India in test
102.
(d) All the above

borrowed from the Constitution of


(a) U.S.A. (b) Japan
t
The concept of Concurrent List in Indian Constitution is
(SSC CHSL 2013)

matches, are presently Lok Sabha members ? (c) Canada (d) Australia
(SSC CHSL 2012) 103. Who admits a new State to the Union of India ?
(a) Two (b) One (SSC CHSL 2014)
(c) Nil (d) Three
(a) President (b) Supreme Court
93. Which one of the following was created by the 'Pitt's India
Act' ? (SSC CHSL 2012) (c) Prime Minister (d) Parliament
(a) Board of Control (b) Board of Revenue 104. Who has got the power to create All India Services ?
(c) Standing Council (d) Court of Directors (SSC CHSL 2014)
94. Term 'Federal' has been used in the Indian Constitution in: (a) Supreme Court (b) The Parliament
(SSC CHSL 2012) (c) Council of Ministers (d) Prime Minister
(a) Part-III 105. In the provisional Parliament of India, how many members
(b) Article-368 were there ? (SSC CHSL 2014)
(c) Nowhere in the Constitution (a) 296 (b) 313
(d) Preamble (c) 318 (d) 316

Downloded From : www.EasyEngineering.net


Downloded From : www.EasyEngineering.net

34 Polity
106. The 73rd Constitutional amendment act is related to ? 117. According to Karl Marx, the change economic system results
(SSC CHSL 2015) in inevitable changes in (SSC Sub. Ins. 2012)
(a) Panchayat Raj (b) Foreign Exchange (a) political system only (b) social system only
(c) Finance Commission (d) RBI (c) the entire systems (d) religious system only
107. Impeachment Proceedings against the President for 118. The model code of conduct for political parties and
Violation of the Constitution can be initiated in : candidates to be followed during the elections is
(SSC CHSL 2015) (SSC Sub. Ins. 2012)
(a) The Supreme Court (a) laid down in the Consitution of India
(b) The Rajya Sabha
(b) specified in The Representation of the People Act, 1951
(c) Either House of Parliament
(c) enjoined by the Supreme Court
(d) The Lok Sabha
(d) agreed through a voluntary agreement among the
108. Who was the First Speaker of the Lok Sabha ?
recognised political parties
(SSC CHSL 2015)
119. A citizen can directly move the Supreme Court for any
(a) K.S. Hegde (b) Hukum Singh
violation of Fundamental Rights under
(c) Ganesh Vasudev (d) Neelam Sanjeeva Reddy
109. Which Article of the Indian Constitution deals with Election (SSC Sub. Ins. 2012)

ww
Commission ?
(a) Article 356
(c) Article 324
(SSC CHSL 2015)
(b) Article 360
(d) Article 352
(a) Article 31
(c) Article 33
(b) Article 32
(d) Article 34
120. Which one of the following sitting Vice-Presidents of India

w.E
110. If the President wants to resign, he shall address his letter
of resignation to : (SSC CHSL 2015)
(a) Vice-President of India (b) Speaker of Lok Sabha
contested for the post of President and lost the election?
(a) S. Radhakrishnan (SSC Sub. Ins. 2012)

asy
(c) Chief Justice of India (d) Prime Minister of India
111. Constitution of India came into force in
(b) V.V. Giri
(c) Bhairon Singh Shekhawat
(d) Both (B) and (C)

(a) 1949 (b) 1951 En


(SSC Multitasking 2013) 121. Which article of the Indian Constitution provides for equal
opportunities for all citizen in Public employment?
(c) 1956
112. Cold War refers to
(d) 1950
(SSC Multitasking 2013) gin (a) Article-22
(SSC Sub. Ins. 2013)
(b) Article-16
(a) tension between East and West
(b) ideological rivalry between Capitalist and Communist
eer
(c) Article-20 (d) Article-25
122. Which one of the following is opposite to democratic
world
(c) tension between Superpowers
(d) All of the above
state?
(a) Despotism ing (SSC Sub. Ins. 2013)
(b) Republic
113. The concept of Judicial Review has been borrowed from
the Consitution of (SSC Multitasking 2014)
(c) Socialism
.ne
(d) Monarchy
123. Who said: "Where there is no law, there is no freedom"?
(a) France
(c) U.S.A.
(b) Great Britain
(d) U.S.S.R.
114. In the Constitution of India, the budget is known as
(a) Bentham
(c) Marx
(b) Lenin
(d) Locke
t
(SSC Sub. Ins. 2013)

(SSC Multitasking 2014) 124. Which one of the following statements about the Chief
(a) Annual Financial Statement Justice of India (CJI) is not correct ?
(b) Annual Revenue Statement (SSC Sub. Ins. 2013)
(c) Annual Budget Statement (a) He appoints the Chief Justice of all High Courts.
(d) Annual Expenditure Statement (b) The CJI administers the oath or-office to the President
115. Which of the following countries has a Parliamentary form (c) When both the offices of the President and Vice-
of Government? (SSC Multitasking 2014) President fall vacant simultaneously, the CJI discharges
(a) New Zealand the duties of the President.
(b) Cuba
(d) The CJI can hold his office till he attains the age of 65
(c) United States of America
years.
(d) France
125. A philosophy that the worker should share in industrial
116. Which of the following systems in independent India goes
decisions is termed as (SSC Sub. Ins. 2014)
against the very basis of democracy? (SSC Sub. Ins. 2012)
(a) industrial democracy (b) worker sovereignty
(a) Caste system (b) Economic system
(c) industrial socialism (d) worker dictatorship
(c) Party system (d) Parliamentary system

Downloded From : www.EasyEngineering.net


Downloded From : www.EasyEngineering.net

Polity 35
126. Liberty stands for (SSC Sub. Ins. 2014) 135. To whom the President has to submit his resignation ?
(a) absence of restraint (SSC Sub. Ins. 2015)
(b) consists in the presence of restraint (a) Speaker (b) Chief Justic
(c) feeling enjoyed in a cogenial atmosphere (c) Prime Minister (d) Vice President
(d) the eager maintenance of that atmosphere in which 136. The total number of planets revolving around the sun are:
men have the opportunity to be their best selves - Laski
(SSC Sub. Ins. 2015)
127. When the Vice-President is acting as President he
(a) Seven (b) Eight
(SSC Sub. Ins. 2014)
(c) Five (d) Six
I. will have all powers and functions of both President
and Vice-President 137. Immunization technique was developed by:
II. gets all the allowances and privileges of the President (SSC Sub. Ins. 2015)
III. should continue to work as the Chairman of the Rajya (a) Louis Pasteur (b) Robert Koch
Sabha (c) Joseph Lister (d) Edward jenner
(a) I, II and III (b) I and III 138. Which one of the following chapters in the Indian
(c) I and II (d) II only Constitution guarantees fundamental rights to the people?
128. In Presidential Government, the President is (SSC Sub. Ins. 2015)
(SSC Sub. Ins. 2014)

ww
(a) independent of the Legislature
(b) dependent on the Legislature
(a) Part III
(c) Part I
(b) Part IV
(d) Part II
139. Mac Iver says ‘Kinship creates society and society at length

w.E
(c) dependent on the Judiciary
(d) bound by the advice of the Council of Ministers
129. The method of amending the Constitution by popular veto
creates ________’.
(a) Association
(c) Nation
(SSC Sub. Ins. 2015)
(b) City State
(d) State
is found in
(a) Britain
(c) Russia
asy
(SSC Sub. Ins. 2014)
(b) Switzerland
(d) India
140. If the Union Parliament is to assume legislative power over
and subject included in the State List, the resolution to the

En
130. Which of the following is the inalienable attribute of the
parliamentary system of government ? (SSC Sub. Ins. 2014)
effect has to be passed by which of the following ?
(SSC Sub. Ins. 2015)
(a) Flexibility of the Constitution
(b) Fusion of Executive and Legislature gin (a) Lok, Sabha, Rajya Sabha and legislatures of th e
Concerned States.
(c) Judicial Supremacy
(d) Parliamentary Sovereignty eer
(b) Both Lok Sabha and Rajya Sabha
(c) Lok Sabha
131. Which one of the following kinds of equality is not
compatible with the liberal notion of equality ?
(SSC Sub. Ins. 2014)
(d) Rajya Sabha
ing
141. Which Article of the Indian constitution defines the duties

(a) Legal Equality


(c) Social Equality
(b) Political Equality
(d) Economic Equality
of the Chief Minister?
(a) Article 164
(c) Article 167 .ne
(SSC Sub. Ins. 2015)
(b) Article 166
(d) Article 163
132. The states recorganisation in 1956 created :
(SSC Sub. Ins. 2015)
(a) 17 States and 6 Union Territories.
142. Political Science is a science of
(a) Economic Engineering (b) Cultural Engineering
(c) Social Engineering
t
(SSC Sub. Ins. 2016)

(d) Political Engineering


(b) 17 States and 9 Union Territories.
143. Which of the following is not the essential element of the
(c) 14 States and 6 Union Territories.
State? (SSC Sub. Ins. 2016)
(d) 15 States and 9 Union Territories.
(a) Government (b) Sovereignty
133. Reservation for the Scheduled castes and Scheduled tribes
in the service has been provided in the Indian Constitution (c) Population (d) Institutions
under : (SSC Sub. Ins. 2015) 144. Parliamentary Government is a form of Constitutional
(a) Article 365 (b) Article 375 democracy in which (SSC Sub. Ins. 2016)
(c) Article 315 (d) Article 335 (a) the legislature emerge from and is responsible to the
134. The Members of the Constituent Assembly which enacted executive.
the constitution of India were: (b) the executives emerge from and is responsible to the
(a) Nominated by Governor-General. judiciary.
(b) Nominated by the Political Parties. (c) the executive emerge from and is responsible to the
(c) Elected by the Legislative Assemblies of various legislature.
Provinces. (d) the legislatures emerge from and is responsible to the
(d) Directly elected by the People. judiciary.

Downloded From : www.EasyEngineering.net


Downloded From : www.EasyEngineering.net

36 Polity
145. The concept of Liberty, Equality and Fraternity enshrined 152. Fundamental Rights are borrowed from the Constitution of
in the Indian Constitution is inspired from (SSC Stenographer 2014)
(SSC Sub. Ins. 2016) (a) America (b) Ireland
(a) US Constitution (b) Irish Constitution (c) Australia (d) Russia
(c) French Constitution (d) UK Constitution 153. The term "politics" was first used by
146. The authority to alter the boundaries of states in India rests (SSC Stenographer 2016)
with the (SSC Stenographer 2013) (a) Plato (b) John Locke
(a) President (b) Prime minister (c) Aristotle (d) Socrates
(c) Parliament (d) State Government 154. Arrange the names of the President in the order they served.
147. Schedule VII of Indian Constitution contains (SSC Stenographer 2016)
(a) Presidential election (SSC Stenographer 2013) (a) N.S. Reddy, Dr. Shankar Dayal Sharma, R.
(b) Acts beyond judicial review Venkataraman, Gyani Zail Singh
(c) States and Union territories (b) R. Venkataraman, Dr. Shankar Dayal Sharma, Gyani Zail
(d) Division of Powers into 3 lists Singh, N.S. Reddy
148. Which Article of the Indian Constitution mentions about (c) N.S. Reddy, Gyani Zail Singh, R. Venkataraman, Dr
financial emergency? (SSC Stenographer 2013) Shankar Dayal Sharma

ww
(a) 360
(c) 340
(b) 350
(d) 330
(d) N.S. Reddy, R. Venkataraman, Gyani Zail Singh, Dr
Shankar Dayal Sharma

(a) 62 years w.E


149. Maximum age of superannuation for the judges of the
Supreme Court of India is (SSC Stenographer 2013)
(b) 65 years
155. Who first gave the concept of "Distributive Justice"?

(a) Machiavelli
(SSC Stenographer 2016)
(b) Locke
(c) 60 years

asy
(d) 70 years
150. Name the portfolio held by Sri Pranab Mukherjee before he
was elevated as President of India (SSC Stenographer 2014)
(c) Plato (d) Aristotle
156. The reorganization of states on linguistic basis was done in

(a) Railway Minister


(b) Finance Minister En (a) 1952
(SSC Stenographer 2016)
(b) 1956

(c) External Affairs Minister


(d) Home Minister gin (c) 1950 (d) 1951
157. Bodo and Dogri were added in the 8th Schedule by th e
151. Who addresses the joint sessions of the parliament?
(a) The President (SSC Stenographer 2014) eer
following amendment :
st
(a) 91 Amendment
(b) 92nd Amendment
(SSC Stenographer 2016)

(b) The Prime Minister


(c) The Chairman of Rajya Sabha
(c) 81st Amendment
(d) 85th Amendment ing
(d) The Lok Sabha Speaker

.ne
t

Downloded From : www.EasyEngineering.net


Downloded From : www.EasyEngineering.net

Polity 37

HINTS & SOLUTIONS


1. (a) 2. (b) 40. (b) The Fundamental Rights can be suspended during the
3. (b) The real benchmarking of the government policy on Emergency under Article 359 of the Constitution by
decentralisation can, however, be attributed to Lord the President of India.
Ripon who, in his famous resolution on local self- 41. (a) 42. (d)
government on May 18, 1882, recognised the twin 43. (c) Domestic Violence on Women Act 2005 is the first
considerations of local government: (i) administrative significant attempt in India to recognise domestic abuse
efficiency and (ii) political education. as a punishable offence, to extend its provisions to
4. (a) 5. (b) 6. (b) 7. (c) 8. (c) those in live-in relationships, and to provide for
emergency relief for the victims, in addition to legal
9. (b) 10. (a) 11. (a) 12. (d) 13. (d)
recourse.
14. (c) 15. (b) 16. (c) 17. (c) 18. (c) 44. (b) 45. (b) 46. (d)
19. (b) 20. (b) 21. (b) 22. (a) 23. (b) 47. (*) Right to Information is a part of fundamental rights
24.
26.
ww
(b) 25. (a)
(b) The Union Government set up the National
Commission for Minorities (NCM) under the National
under Article 19 (1) of the Constitution which states
that every citizen has freedom of speech and expression.
The 86th Constitutional amendment making education

27. w.E
Commission for Minorities Act, 1992.
(a) Some countries, Such as Argentina, Austria, Australia,
Belgium, Brazil, Canada, Germany, India, Malaysia,
a fundamental right was passed by Parliament in 2002.
Right to speech comes under freedom of speech and
expression which is a fundamental right. The

asy
Mexico, Pakistan, Russia, Switzerland and the United
States, link their bicameral systems to their federal
48.
constitution guarantees the right to life and personal
liberty as a fundamental right under article 21.
(b) The Constitution has made the Supreme Court as the

28.
political structure.
(b) Socialism is an economic system characterised by
En custodian and protector of the Constitution. The
Supreme Court decides disputes between the Centr e
social ownership and/or cannot of the means of
production and cooperative management of the
economy. gin
49.
and the Units as well as protects the Fundamental
Rights of the citizens of India.
(d) May preceded by the Shimla Conference of 1945.
29. (d) The Court was established in Nagpur, but after the
reorganisation of states on 1 November 1956, it was
50.
eer
(d) The state has four essential elements: population,
territory, government and sovereignty. Absence of any

30.
moved to Jabalpur.
(b) K.T. Shah said that Dr. Rajendra Prasad won his first 51.
ing
of these elements denies to it the status of statehood.
(b) The 44th amendment eliminated the right to acquire,
hold and dispose of property as a fundamental right.

31.
election with 507,400 votes over his nearest rival K.T.
Shah who got 92,827 votes.
(c) Article 312 provides that an All India Service can be .ne
However, in another part of the Constitution. Article
300 (A) was inserted to affirm that no person shall be
created only if the Council of State declares by a
resolution supported by not less than a two-thirds
majority that it is necessary in the national interest to
52.
t
deprived of his property save by authority of law.
(a) The 44th amendment of 1978 eliminated the right to
acquire, hold and dispose of property as a fundamental
right. The Constitution originally provided for the right
create one or more such All india Services. to property under Articles 19 and 31.
32. (a) In the original Constitution, Article 338 provided for a 53. (c) The Speaker of Lok Sabha presides over the joint
Special officer, called the Commissioner for SCs and sittings of both Houses of Parliament. The joint session
STs, to have the responsibility of monitoring the of parliament is convened by the President of India.
effective implementation of various safeguards for SCs/ 54. (c) Parliament may restrict the application of the
STs in the Constitution as well as other related Fundamental Rights to members of the Indian Armed
legislations and to report to the President. Forces and the Police, in order to ensure paper
33. (d) The High Court of Kerala is headquartered at Kochi. It discharge of their duties and the maintenance of
is located in Ernakulam. discipline, by a law made under Article 33.
55. (b) Dr. B R Ambedkar, the chairman of the Drafting
34. (d) Ganesh Vasudev Mavalankar, popularly known as
committee called the fundamental right to
Dadasaheb, was an independence activist, the President
constitutional remedies as the heart and soul of the
(from 1946 to 1947) of the Central Legislative Assembly,
Indian constitution. According to this right, a person
then Speaker of the Constituent Assembly of India, an can move the Supreme Court in case of violation of
later the first Speaker of the Lok Sabha. their fundamental rights. In the Constitution, this right
35. (b) 36. (d) 37. (c) 38. (a) 39. (c) is enshrined in Article 32.

Downloded From : www.EasyEngineering.net


Downloded From : www.EasyEngineering.net

38 Polity
56. (b) Machiavelli has been credited with formulating for the 68. (a) The Court consists of the Chief Justice of the United
first time the “modern concept of the state”. The term States and eight associate justices who are nominated
‘lo state’ appears widely in Machiavelli’s writings, by the President and confirmed by the Senate. Once
especially in The Prince, in connection with the appointed, justices have life tenure unless they resign,
acquisition and application of power in a coercive retire, take senior status, or are removed after
sense. impeachment (though no justice has ever been
57. (b) Justice D.K Jain, Judge, Supreme Court of India, is the removed).
Chairman of the Twentieth Law Commission of India. 69. (c) Nagaland is a state in Northeast India. The state capital
The Twentieth Law Commission was constituted is Kohima. Nagaland became the 16th state of India
through a Government Order with effect from 1st from 1 December 1963.
September, 2012. 70. (a) Mihira Bhoja was a ruler of the Gurjara Pratihara
58. (d) Selecting the candidate of election is the function of dynasty of India. Bhoja's empire extended to Narmada
the political party River in the South, Sutlej River in the northwest, and
59. (a) This quote is given by the theory of Individualism up to Bengal in the east.
60. (c) The All India Trade Union Congress is the oldest trade 71. (d) Bose appeared at the 1939 Congress meeting and was
union of India, established in 1920, History of AITUC elected president over Gandhi's preferred candidate
is coterminous with the history of organised labour Pattabhi Sitaramayya.

ww
movement in India. Since its birth, AITUC has had a
major role to play in mass movement phase in India's
72. (c) The Jallianwala Bagh Massacre happened in Amritsar,
in 1919. It is named after the Jallianwala Bagh (Garden)

61.
freedom struggle.

w.E
(c) The Joint Session of Parliament resolves the deadlock
between Lok Sabha and Rajya Sabha over an ordinary
atAmritsar. On April 13, 1919, British, Indian Army
soldiers started shooting an unarmed gathering of men,
women and children.

62.
64.
(a)
bill.
63. (a)
asy
(b) Cabinet collective responsibility is constitutional
73. (b) Bahlol Lodi was the founder of the Lodi dynasty.He
ruled for long thirty-nine years (1451-89). He was the
governor of Lahore and Sirhind during the rule of
convention in governments is that members of the
Cabinet must publicly support all governmental En 74.
Muhammad Shah of Sayyid dynasty.
(c) Sri Chaitanya Mahaprabhu arrived in the empire at the
decisions made in Cabinet, even if they do not privately
agree with them. This support includes voting for the gin time of Emperor Prataparudra (Gajapatis)and stayed
for 18 long years at Puri.
government in the legislature. Cabinet collective
responsibility is related to the fact that, if a vote of no
confidence is passed in parliament, the government is
75.
eer
(c) The instance heat that prevails in the Indian Sub
continent causes a low pressure region over the

responsible collectively, and thus the entire


government resigns. ing
northern plains. It is intense enough to attract the
moisture bearing winds from the Indian Ocean .Thus
the south east trade winds from the southern
65. (a) Switzerland is a small country located in the heart of
western Europe, at the intersection of German, French
76. .ne
hemisphere are attracted towards India.
(d) A river profile is a curve which shows the slop of a
and Italian language and culture. Switzerland has been
multicultural in its own way for centuries. Direct
Democracy in particular, has a long, but not undisputed
tradition in this country. Switzerland's unique political
t
river from source to mouth.A stream flowing over
irregular terrain may have waterfalls, rapids and lakes
along its course.Though the stream will wear away the
irregularities to leave a smoothly curving profile called
system is today world's most stable democratic system,
a graded profile.
offering a maximum of participation to citizens.
77. (c) Karst is a landscape formed from the dissolution of
66. (a) John Muir was a Scottish-American naturalist, author,
soluble rocks including limestone, dolomite and
and early advocate of preservation of wilderness in
gypsum. It is characterized by sinkholes, caves, and
the United States.
underground drainage systems.
67. (a) The Federal Assembly is bicameral, being composed
78. (d) Kerala is famous for the cultivation of coconut, tea,
of the 200-seat National Council and the 46-seat Council
of States. The houses have identical powers. Members coffee, cashew and spices.
of both houses represent the cantons, but, whereas 79. (a) The Trans-Siberian Railway is the longest railway line
seats in the National Council are distributed in in the world. It has a length of 9,289 km which connects
proportion to population, each canton has two seats Moscow to Vladivostok.
in the Council of States, except the six 'half-cantons' 80. (b) G. V Mavalankar was the first speaker of the Lok Sabha.
which have one seat each. Both are elected in full once 81. (d) Plural voting is the practice whereby one person might
every four years, with the last election being held in be able to vote multiple times in an election.
2011. 82. (c) Voting is the process of selecting representatives.

Downloded From : www.EasyEngineering.net


Downloded From : www.EasyEngineering.net

Polity 39
83. (d) As per the procedure laid out by article 368 for Congress and Muslim League.The first official meeting
amendment of the Constitution, an amendment can be of this Constituent Assembly was held on 9 Dec, 1946
initiated only by the introduction of a Bill in either while the last meeting was held on 24 Jan, 1950. On
House of Parliament. The Bill, passed by the required 26th Jan, 1950, the day when Constitution of India
majority, is then presented to the President who shall finally took in effect, Constitutional Assembly was
give his assent to the Bill. renamed as Provisional Parliament of India.This
84. (c) The concept of Directive Principles of State Policy was Provisional Parliament was dissolved after the first
borrowed from the Irish Constitution. The makers of general election of India in 1952.
the Constitution of India were influenced by the Irish 106. (a) 73rd Constitutional amendment act is related to provide
nationalist movement. Hence, the Directive Principles 3-tier system of Panchayati Raj for all states having
of the Indian constitution have been greatly influenced population of over 20 lakh.
by the Directive Principles of State Policy. 107. (c) Impeachment of the president for violation of
85. (c) 86. (c) 87. (a) 88. (d) 89. (b) consititution of India may start in either of the two
90. (c) 91. (d) 92. (d) 93. (a) 94. (c) houses of the Parliament.
95. (a) 96. (b) 97. (a) 98. (d) 108. (c) Ganesh Vasudev Mavalankar was the first speaker of
99. (a) According to the Indian Constitution, 14 members can Lok Sabha.
be nominated to both the houses of parliament by the 109. (c) The Article 324 of the Indian constitution basically

ww
President. This is the legislative power of the President
where he nominates 12 members to the Rajya Sabha
and if not adequately represented 2 Anglo-Indian
empowers the Election Commission to exercise its
power to prevent criminals and corrupt persons from
entering politics.

100. w.E
members to the Lok Sabha.
(b) Under Article 32 of the Constitution, an individual can
directly move to the Supreme Court in Case of any
110. (a) If the President wants to resign he/she shall address
his letter of resignation to the Vice-President.

asy
violation of fundamental rights. Fundamental Rights
are those rights which are essential for the growth of
111. (d)

112. (d)
The constitution of India came into effect from 26th
January 1950
The Cold War, often dated from 1947 to 1991, was a

En
an individual's personality and are enjoyed by every
citizen irrespective of caste, color, creed, race and sex.
sustained state of political and military tension between
powers in the Western Bloc, dominated by the United
101. (d) A presidential system is a republican system of
government where a head of government is also head
of state and leads an executive branch that is separate gin States with NATO among its allies, and powers in the
Eastern Bloc, dominated by the Soviet Union along
with the Warsaw Pact. This began after the success of
from the legislative branch. The United States, for
instance, has a presidential system. Popular election eer
their temporary wartime alliance against Nazi Germany,
leaving the USSR and the US as two superpowers with
of President, no overlap in membership and fixed term
of office are the main criteria of Presidential form of 113. (c) ing
profound economic and political differences.
The concept of Judicial Review in the Constitution of

102.
Government.
(d) The concept of Concurrent List in Indian Constitution
is borrowed from the Constitution of Australia. The .ne
India is borrowed from USA. In the Indian constitution,
Judicial review is dealt with under Article 13. Judicial
Review refers that the Constitution is the supreme
Concurrent List or List-III is a list of 52 items(though
the last item is numbered 47) given in Part XI of the
Constitution of India, concerned with relations between 114. (a)
supremacy. t
power of the nation and all laws are under its

In the Constitution of India, the budget is known as


the Union and States. This part is divided between Annual Financial Statement. The Union Budget of
legislative and administrative powers. The legislative India, referred to as the Annual Financial Statement[1]
section is divided into three lists: Union List, State List in Article 112 of the Constitution of India, is the annual
and Concurrent List. budget of the Republic of India, presented each year
103. (d) The parliament of India has power to add a new state on the last working day of February by the Finance
to the Union of India. This is done by collecting votes Minister of India in Parliament. The budget, which is
of Members of Parliament in the favour of new state. presented by means of the Financial Bill and the
104. (b) the Parliament has the power to create all India Services. Appropriation bill has to be passed by the House
105. (b) The Provisional Parliament of India was consisted of before it can come into effect on April 1, the start of
313 members. The Constitutional Assembly of India India's financial year.
was introduced in 1934. This will become the major 115. (a) New Zealand has parliamentary form of government. A
assembly to draft constitution for India (Including parliamentary system of government means that the
present day Pakistan and Bangladesh). Members of executive branch of government must have the direct
this assembly was indirectly elected representatives or indirect support of the parliament. This support is
from across the India. It consists of the members of usually shown by a vote of confidence. The

Downloded From : www.EasyEngineering.net


Downloded From : www.EasyEngineering.net

40 Polity
relationship between the executive and the legislature determined by Parliament by law and, until provision
in a parliamentary system is called responsible in that behalf is so made, such emoluments, allowances
government. Parliamentary systems usually have a and privileges as are specified in the Second Schedule.
head of government and a head of state. The head of When the President is unable to discharge his functions
government is the prime minister, who has the real power. owing to absence, illness or any other cause, the Vice-
The head of state often is an elected (either popularly President shall discharge his functions until the date
or through parliament) president or, in the case of a on which the President resumes his duties. The Vice-
constitutional monarchy, hereditary. President shall not be a member of either House of
116. (a) 117. (c) Parliament or of a House of the Legislature of any State,
118. (b) The Representation of The People Act , 1951 : An Act and if a member of either House of Parliament or of a
to provide for the conduct of elections to the Houses House of the Legislature of any State be elected Vice-
of Parliament and to the House or Houses of the President, he shall be deemed to have vacated his seat
Legislature of each State. in that House on the date on which he enters upon his
119. (b) A citizen has the right to 'move the supreme court' office as Vice-President.
(under article 32) directly in case s/he faces any violation 128. (a) a system of government in which the powers of the
of his/her fundamental rights. president are constitutionally separate from those of
120. (c) Bhairon Singh Shekhawat was the 11th Vice-President the legislature.

ww
of India. He served in that position from August 2002,
when he was elected to a five-year term, until he
resigned on July 21, 2007, after losing the presidential
129. (b)

130. (b)
The method of amending the Constitution by popular
veto is found in Switzerland.
A parliamentary form of government is a democratic

121. (b) w.E


election to Pratibha Patil.
Article 16 of the Indian Constitution deals with equality
of opportunity in matters of public employment. It states
one where the head of state and head of government
are held by different people and the executive and
legislature branches of government are linked. The

asy
that no citizen shall, on grounds only of religion, race,
caste, sex, descent. place of birth, residence or any of
executive and legislature branch are linked because
the executive branch gets its power from the legislative

En
them, be ineligible for, or discriminated against in
respect or, any employment or office under the State.
branch and is held accountable for their actions by
them as well. The leaders of the majority party or alliance
in the legislature become the members of the Cabinet
122. (a) Democracy is a political system whereby people elect
their representatives who then administer the region/
nation. Despotism is the opposite of democracy as the gin or Ministry. Naturally, the ministers can easily exten
their influence on the legislature. Consequently, the
d

principle of representation is absent in it. It is mostly


associated with autocratic/authoritarian set ups. eer
programs and policies of the Cabinet are backed by a
majority inside the legislature. This feature is inalienable
123. (d) John Locke in his Second Treatise of Civil Government,
Chapter 6, said that the end of law is not to abolish or
restrain, but to preserve and enlarge freedom.
131. (d)
ing
in Parliamentary form of government.
Economic equality is not compatible with the liberal
notion of equality. Liberal means free of restraints an d
According to him. in all the states of created beings
capable of laws, where there is no law, there is no .ne
economic equality is about a level playing field where
everyone has the same access to the same wealth;

124. (a)
freedom.
Judges in a High Court are appointed by the President
of India in consultation with the Chief Justice of India
and the governor of the state. The Constitution confers
132. (c)
137. (d)
which is not possible.
133. (d) 134. (c) 135. (d) 136. (b)
Edward Jenner is often called "the father of
immunology".
t
the power of appointment of judges on the President 138. (a) The Fundamental Rights are defined in Part III of the
of India. Constitution which apply to every Indian citizen
125. (a) Industrial democracy is an arrangement which involves irrespective of race, place of birth, religion, caste, creed
workers making decisions, sharing responsibility and or gender.
authority in the workplace. 139. (d) "Kinship" says MacIver, creates society and society
126. (a) Liberty means absence of restraint and assures freedom at length creates the state."
of thought, expression, belief, faith and worship. 140. (d)
127. (d) The Vice-President shall, during, and in respect of, the 141. (c) Article 167 defines the duties of the chief minister.
period while he is so acting as, or discharging the 142. (a) 143. (d) 144. (c) 145. (c) 146. (c)
functions of, President have all the powers and 147. (c) 148. (a) 149. (b) 150. (b) 151. (a)
immunities of the President and be entitled to such 152. (a) 153. (c) 154. (d) 155. (c) 156. (b)
emoluments, allowances and privileges as may be 157. (a)

Downloded From : www.EasyEngineering.net


Downloded From : www.EasyEngineering.net

Economics 41

4
CHAPTER
ECONOMICS

1. Formalised system of trading agreements with groups of 11. Which among the following agencies released the report,
countries is known as (SSC CGL 1st Sit. 2010) Economic Outlook for 2009–10 ? (SSC CGL 2nd Sit. 2010)
(a) Trading blocks (b) Trade ventures (a) Planning Commission
(c) Trade partners (d) Trade organisations (b) PM's Economic Advisory Council
2. Depression formed due to deflating action of winds are called (c) Finance Commission
(SSC CGL 1st Sit. 2010) (d) Reserve Bank of India
(a) Playas (b) Yardang 12. India and U.S. have decided to finalise agreements related
(c) Ventifacts (d) Sand dunes to which of the following ? (SSC CGL 2nd Sit. 2010)
3.
ww
Which one of the following is not a method of estimating
National Income ? (SSC CGL 1st Sit. 2010)
(a) Trade and Investment (b) Intellectual Property
(c) Traditional Knowledge (d) All of the above

4.
(c) Matrix method
w.E
(a) Expenditure method (b) Product method
(d) Income method
The monetary policy is India is formulated by
13. The exchange of commodities between two countries is
referred as
(a) Balance of trade
(SSC CGL 2nd Sit. 2010)
(b) Bilateral trade

(a) Central Government asy


(SSC CGL 1st Sit. 2010)
14.
(c) Volume of trade (d) Multilateral trade
A want becomes a demand only when it is backed by the
(SSC CGL 2nd Sit. 2010)
(b) Industrial Financial Corporation of India
(c) Reserve Bank of India
En (a) Ability to purchase (b) Necessity to buy

5.
(d) Industrial Development Bank of India
A short-term government security paper is called
(SSC CGL 1st Sit. 2010) gin
15.
(c) Desire to buy (d) Utility of the product
The terms 'Micro Economics' and "Macro Economics" wer e
coined by (SSC CGL 2nd Sit. 2010)
(a) Share
(c) Mutual fund
(b) Debenture
(d) Treasury bill eer
(a) Alfred Marshall
(c) Ragner Frisch
(b) Ragner Nurkse
(d) J.M. Keynes
6. WTO basically promotes
(a) Financial support
(SSC CGL 1st Sit. 2010)
(b) Global peace
16.
ing
During periods of inflation, tax rates should
(SSC CGL 2nd Sit. 2010)

7.
(c) Unilateral trade (d) Multilateral trade
Under which market condition do firms have excess
(a) increase
(c) remain constant .ne
(b) decrease
(d) fluctuate
capacity?
(a) Perfect competition
(b) Monopolistic competition
(c) Duopoly
(SSC CGL 1st Sit. 2010) 17. Which is the biggest tax paying sector in India ?

(a) Agriculture sector


(c) Transport sector
t
(SSC CGL 2nd Sit. 2010)
(b) Industrial sector
(d) Banking sector
(d) Oligopoly 18. "Economics is what it ought to be" – This statement refers
8. Price theory is also known as (SSC CGL 1st Sit. 2010) to (SSC CGL 2nd Sit. 2010)
(a) Macro Economics (b) Development Economics (a) Normative economics (b) Positive economics
(c) Public Economics (d) Micro Economics (c) Monetary economics (d) Fiscal economics
9. At present, India is following (SSC CGL 1st Sit. 2010) 19. The excess of price a person is to pay rather than forego the
(a) Fixed exchange rate consumption of the commodity is called
(b) Floating exchange rate (SSC CGL 2nd Sit. 2010)
(c) Pegged up exchange rate (a) Price (b) Profit
(d) Pegged down exchange rate (c) Producer's surplus (d) Consumer's surplus
10. National Income is the (SSC CGL 1st Sit. 2010) 20. When there is an official change in the exchange rate of
(a) Net National Product at market price domestic currency, then it is called:
(b) Net National Product at factor cost (SSC CGL 1st Sit. 2011)
(c) Net Domestic Product at market price (a) Appreciation (b) Depreciation
(d) Net Domestic Product at factor cost (c) Revaluation (d) Deflation

Downloded From : www.EasyEngineering.net


Downloded From : www.EasyEngineering.net

42 Economics
21. Inflation redistributes income and wealth in favour of: 32. The study of population is known as.
(SSC CGL 1st Sit. 2011) (SSC CGL 2nd Sit. 2012)
(a) Pensioners (b) Poor (a) Demography (b) Climatology
(c) Middle class (d) Rich (c) Petrology (d) Hydrology
22. The fringe benefit tax was introduced in the budget of 33. The BSE Sensex is based on how many stocks ?
(SSC CGL 1st Sit. 2011) (SSC CGL 1st Sit. 2012)
(a) 2003-04 (b) 2004-05 (a) 80 (b) 100
(c) 2005-06 (d) 2006-07 (c) 30 (d) 50
23. The purchase of shares and bonds of Indian companies by 34. The funds raised by the Government within the country is
Foreign Institutional Investors is called known as (SSC CGL 1st Sit. 2012)
(SSC CGL 2nd Sit. 2011) (a) Domestic budgetary source
(a) Foreign Direct Investment (b) Domestic banking source
(b) Portfolio Investment (c) Domestic saving source
(c) NRI Investment (d) Domestic monetary source
(d) Foreign Indirect Investment 35. Sick units are referred to (SSC CGL 1st Sit. 2012)
24. Monopolist resorts to price discrimination depending upon (a) SIA (b) DGTD
the
ww
(a) Elasticity of supply
(c) Law of demand
(SSC CGL 1st Sit. 2012)
(b) Elasticity of demand
(d) Law of supply
36.
(c) FIPB (d) BIFR
The concept ‘Duopoly’ was introduced by
(SSC CGL 1st Sit. 2012)
25.
w.E
Economic planning is an essential feature of

(a) Socialist economy


(SSC CGL 1st Sit. 2012)
(b) Capitalist economy 37.
(a) Sweezy
(c) Ricardo
(b) Malthus
(d) Cournot .
Which of the following is the feature of monopolistic

26.
(c) Mixed economy
asy
(d) Dual economy
The Rashtriya Barh Ayog (RBA) is related with
competition ?
(a) Single firm
(SSC CGL 1st Sit. 2012)
(b) Large number of firms

(a) Droughts and Floods (b) Poverty Alleviation En


(SSC CGL 1st Sit. 2012)
38.
(c) Group of firms (d) None of the above
Which one of the following is an example for a non-economi c

27.
(c) Floods (d) Disaster Management
Low cost housing is an example for: gin good?
(a) Doctor’s service
(SSC CGL 2nd Sit. 2012)
(b) Teacher’s service

(a) Mixed wants


(c) Private wants
(SSC CGL 2nd Sit. 2012)
(b) Social wants
(d) Merit wants
39.
eer
(c) Mother’s service (d) Banker’s service
Which one of the following committee is associated wit
banking sector reforms in India? (SSC CGL 2nd Sit. 2012)
h

28. Consumption for the sake of enjoying social


acknowledgement is called: (SSC CGL 2nd Sit. 2012)
(a) L. C. Gupta
(c) Chakravarty ing (b) Narsimhan
(d) Kelkar
(a) Rational consumption
(b) Social consumption
40.
measure of the RBI? .ne
Which one of the following is not a qualitative credit control
(SSC CGL 2nd Sit. 2012)

29.
(c) Conspicuous consumption
(d) Demonstration consumption
Of the following economists, whom do you consider to be
the Master of “Partial Analysis”? (SSC CGL 2nd Sit. 2012)
(a) Fixing margin requirements
(b) Variable interest rates
(c) Open market operations
(d) Credit rationing
t
(a) Leon Walras (b) Alfred Marshall 41. The 13th Five Year Plan will be operative for the period.
(c) J. M. Keynes (d) Lionel Robbins (SSC CGL 2nd Sit. 2012)
30. India making ‘Double Taxation Avoidance Agreements’ (a) 2010 - 2015 (b) 2011- 2016
(DTAA) with other countries for the promotion of: (c) 2012 - 2017 (d) 2013 - 2018
(SSC CGL 2nd Sit. 2012) 42. The national income of a nation is the
(a) Bilateral trade (SSC CGL 2nd Sit. 2012)
(b) External commercial borrowings (a) Government’s annual revenue
(c) Foreign direct investments (b) Sum total of factor incomes
(d) Foreign institutional investment (c) Surplus of public sector enterprises
31. Brain drain has been caused by: (SSC CGL 2nd Sit. 2012) (d) Exports minus imports
(a) failure to recognise talent in the originating country. 43. Externality theory is the basic theory of the following branch
(b) the lure of high living standards of Economics : (SSC CGL 1st Sit. 2013)
(c) lack of employment opportunities (a) Macro Economics (b) Environomics
(d) socio-economic instability (c) Fiscal Economics (d) International Economics

Downloded From : www.EasyEngineering.net


Downloded From : www.EasyEngineering.net

Economics 43
44. Diamonds are priced higher than water because : 53. Forced Savings refer to (SSC CGL 1st Sit. 2013)
(SSC CGL 1st Sit. 2013) (a) Compulsory deposits imposed on income tax payers
(a) consumers do not buy them at lower prices. (b) Provident fund contribution of private sector
(b) they are sold by selected firms with monopolistic employees
powers. (c) Reduction of consumption consequent to a rise in
(c) their marginal utility to buyers is higher than that of prices
water (d) Taxes on individual income and wealth
(d) their total utility to buyers is higher than that of water 54. High powered money is (SSC CGL 1st Sit. 2013)
45. "Functional Finance" is associated with : (a) Banks reserves at Central Bank
(SSC CGL 1st Sit. 2013) (b) All loans and advances of banks
(a) Abba 'P' Lerner (b) Adolph Wogner (c) Money held by banks
(c) Adam Smith (d) Adams (d) Currency held by public and reserves with the Central
46. Of the following land uses, which is restricted to Special Bank
Economic Zones ? (SSC CGL 1st Sit. 2013) 55. Hardening interest rate means (SSC CGL 1st Sit. 2013)
(a) Information Technology Companies (a) interest rate is remaining sticky

ww
(b) Educational Institutions
(c) Free trade Centres
(d) Marketing Centres
(b) interest rate is very low
(c) interest rate is increasing
(d) interest rate is falling
47.
w.E
As per the TRIPS Agreement-1994, a good originating from
a region with specific character/quality/reputation is
covered/to a protected under the IPR as
56. “Dumping” is a situation when the seller
(SSC CGL 1st Sit. 2013)

(a) Patent asy


(SSC CGL 2nd Sit. 2013)
(a) supplies mote than the demand for products in the
World Market.
(b) supplies more in the Domestic Market.
(b) Trademark
(c) Trade secret En (c) sells a commodity at a lower price in the World Market
and charges a higher price in the Domestic Market.

48.
(d) GI (Geographical Indicator)
‘Gold’ is mainly related to (SSC CGL 2nd Sit. 2011) gin (d) sells a commodity at a higher price in the World Marke
and charges a lower price in the Domestic Market.
t

(a) Local market (b) National market


(c) International market (d) Regional market
57.
eer
Tick the correct option with regards to the contribution
towards GDP (Gross Domestic Product) from Agriculture
49. The demand for labour is called (SSC CGL 1st Sit. 2013)
(a) Derived demand (b) Factory demand ing (SSC CGL 1st Sit. 2013)
(a) Durin g 1950–51 (GDP 51-88%) and 2011–12

50.
(c) Market demand (d) Direct demand
Which of the following is not an investment expenditure in
(GDP 14-01%)
.ne
(b) During 1950–51 (GDP 11-00%) and 2011–12 (GDP 25%)
goods and services?
(a) Purchase of machinery
(SSC CGL 1st Sit. 2013)

(b) An increase in business inventories


(c) Expansion of the main plant of a company
(c) Durin g 1990–91 (GDP 29-53%) and 2011–12
(GDP 66-77%)
(d) Durin g 1980–81 (GDP 35-69%) and 2011–12
(GDP 20-69%)
t
(d) Purchase of a house 58. The principle of maximum social advantage is the basic
51. Which one of the following represents the Savings of the principle of (SSC CGL 1st Sit. 2013)
Private Corporate Sector? (SSC CGL 1st Sit. 2013) (a) Micro Economics
(a) Undistributed profits (b) Macro Economics
(b) Excess of income over expenditure (c) Fiscal Economics
(c) Dividends paid to shareholders (d) Environmental Economics
(d) Total profits of a company 59. Which Five Year Plan is not correct among the following?
52. Social accounting system in India is classified into (SSC CGL 1st Sit. 2013)
(SSC CGL 1st Sit. 2013) (a) First 1951–56 (b) Second 1956–61
(a) Assets, liabilities and debt position (c) Third 1961–66 (d) Fourth 1966–71
(b) Public sector, Private sector and Joint sector 60. An economic theory is a/an (SSC CGL 1st Sit. 2013)
(c) Income, product and expenditure (a) Axion (b) Proposition
(d) Enterprise, households and government (c) Hypothesis (d) Tested hypothesis

Downloded From : www.EasyEngineering.net


Downloded From : www.EasyEngineering.net

44 Economics
61. Indian Special Economic Rules amendment came in the year 73. Which of the following method is not used in determining
(SSC CGL 1st Sit. 2013) National Income of a country? (SSC CGL 1st Sit. 2014)
(a) 2000 (b) 2002 (a) Income Method (b) Output Method
(c) 2004 (d) 2006 (c) Input Method (d) Investment Method
62. According to Marx, the source of value is 74. What does the letter 'e' denotes in the term 'e-banking'?
(SSC CGL 1st Sit. 2013) (SSC CGL 1st Sit. 2014)
(a) Capital (b) Land (a) Essential Banking (b) Economic Banking
(c) Labour (d) None of the above (c) Electronic Banking (d) Expansion Banking
63. The Community Development Programme was launched in 75. Which among the following is not a Bretton Woods
the year (SSC CGL 1st Sit. 2013) Institution ? (SSC CGL 1st Sit. 2015)
(a) 1950 (b) 1952 (a) International Monetary Fund (IMF)
(c) 1951 (d) 1953 (b) World Bank
64. Prof. Milton Friedman was the leader of
(c) Organisaiton of Economic Cooperation an d
(SSC CGL 1st Sit. 2014) Development (O.E.C.D.)
(a) Ohio school (b) Chicago school
(d) None of these
(c) Cambridge school (d) London school
65.
ww
Which one of the following is not a qualitative control of
credit by the Central Bank of a country?
76. Equilibrium price in the market is determined by the
(SSC CGL 1st Sit. 2015)
(a) equality between total cost and total revenue

w.E
(a) Rationing of credit
(b) Regulation of consumer credit
(SSC CGL 1st Sit. 2014)
(b) equality between average cost and average revenue.
(c) equality between marginal cost and marginal revenue
(c) Variation of the reserve ratio
(d) Regulation of margin requirements
asy 77.
(d) equality between marginal cost and average cost.
In the national context which of the following indicates
Macro Approach ? (SSC CGL 1st Sit. 2015)
66.
called
(a) Reserve market En
The market in whcih loans of money can be obtained is
(SSC CGL 1st Sit. 2014)
(b) Institutional market
(a) Sales of Bata Shoe Company
(b) Exports of Mangoes to U.K.

67.
(c) Money market (d) Exchange market
If the marginal return increases at a diminishing rate, the gin (c) Income from Railways
(d) Inflation in India
total return
(a) increases
(SSC CGL 1st Sit. 2014)
(b) decreases
78.
eer
Internal economies (SSC CGL 1st Sit. 2015)
(a) arise in an economy as it makes progress

68.
(c) remains constant (d) becomes zero
The law of Increasing Returns means ing
(b) accrue to a firm when it expands its output
(c) arise when there is expansion in internal trade

(a) increasing cost


(SSC CGL 1st Sit. 2014)
(b) decreasing cost 79. .ne
(d) arise when there is expansion in an industry
One of the features of a free market economy is

69.
(c) increasing production (d) increasing income
Which of the following is the Regulator of the credit rating
agencies in India?
(a) RBI
(SSC CGL 1st Sit. 2014)
(b) SBI
(a) public ownership of factors of production
(b) rationing and price control
t
(SSC CGL 1st Sit. 2015)

(c) consumer's sovereignty


(c) SIDBI (d) SEBI
(d) active state intervention
70. Which is the first Indian Company to be listed in NASDAQ?
80. Gross National Product – Depreciation Allowance = ?
(SSC CGL 1st Sit. 2014)
(SSC CGL 1st Sit. 2015)
(a) Reliance (b) TCS
(a) Gross Domestic Product
(c) HCL (d) Infosys
(b) Personal Income
71. RRBs are owned by (SSC CGL 1st Sit. 2014)
(c) Net National Product
(a) Central Government (b) State Government
(c) Sponsor Bank (d) Jointly by all of the above (d) Per Capita Income
72. The Monetary and Credit Policy is announced by which of 81. The Panchayat Samiti remains accountable for its functions
the following? (SSC CGL 1st Sit. 2014) to (SSC CGL 1st Sit. 2015)
(a) Ministry of Finance of Centre (a) The Gram Panchayats and Gram Sabhas
(b) Reserve Bank of India (b) Zilla Parishads
(c) State Bank of India (c) Anchal Panchayats
(d) Planning Commission of India (d) Janpad Panchayats

Downloded From : www.EasyEngineering.net


Downloded From : www.EasyEngineering.net

Economics 45
82. The one rupee note bears the signature of : (c) NRI deposits
(SSC CGL 1st Sit. 2015) (d) External commercial borrowing
(a) Governor, Reserver Bank of India 93. Which one of the following is not an instrument of Fiscal
(b) Finance Minister policy? (SSC CGL 1st Sit. 2016)
(c) Secretary, Ministry of Finance (a) Open Market Operations
(d) None of these (b) Taxation
83. NABARD stands for (SSC CGL 1st Sit. 2015) (c) Public borrowing
(a) National business for Accounting and Reviewing (d) Public expenditure
(b) National Bank for Agriculture and Rural Development 94. In which of the following market forms, a firm does not
(c) National Bank for Aeronautics and Radar Development exercise control over price? (SSC CGL 1st Sit. 2016)
(d) National Bureau for Air and Road Transport (a) Monopoly
84. Surplus budget is recommended during : (b) Perfect competition
(SSC CGL 1st Sit. 2015) (c) Oligopoly
(a) Depression (b) Boom (d) Monopolistic competition
(c) War (d) Famines 95. What is the name of portal launched by RBI recently to
85. Economic profit or normal profit is the same as : check illegal money collection? (SSC CGL 1st Sit. 2016)

ww
(a) accounting profit
(SSC CGL 1st Sit. 2015)
(b) optimum profit
(a) Sahyog
(c) Sampark
(b) Sahayata
(d) Sachet

86.
(c) net profit

w.E (d) maximum profit


CENVAT is related to which of the following ?
(SSC CGL 1st Sit. 2016)
96. Fiscal policy in India is formulated by?

(a) Reserve Bank of India


(SSC CGL 1st Sit. 2016)

87.
(a) Sales Tax
(c) Custom Duty
asy
(b) Excise Duty
(d) Service Tax
Malthusian theory is associated with which of the following ?
(b) Planning Commission
(c) Finance Ministry

En
(SSC CGL 1st Sit. 2016)
(d) SEBI
97. Open market operation refers to : (SSC CHSL 2012)

88.
(a) Poverty
(c) Diseases
(b) Employment
(d) Population
Which is the parameter for the economic development ? gin (a) borrowing by scheduled banks to industry and trade
(b) purchase and sale of government securities bythe rb i

(a) Per capita monetary income


(SSC CGL 1st Sit. 2016)
eer
(c) deposit mobilisation
(d) borrowing by scheduled banks from the RBI
(b) National income
(c) Per capita rural income ing
98. Who are the price-takers under Perfect Competition ?
(SSC CHSL 2012)

89.
(d) Population
A ‘Market Economy’ is one which (SSC CGL 1st Sit. 2016)
(a) industry
(c) firms
.ne
(b) government
(d) buyers
99. An employment situation where the marginal productivity
(a) is controlled by the Government
(b) is free from the Government control
(c) in influenced by international market forces
(d) All of these
of agricultural labour is zero is known as :

(a) Seasonal unemployment


t
(SSC CHSL 2012)

90. Regional Rural Banks are sponsored by (b) Cyclical unemployment


(SSC CGL 1st Sit. 2016) (c) Disguised unemployment
(a) Nationalised Commercial Bank (d) Disguised unemployment
(b) Reserve Bank of India 100. Which one of the following is a raw material oriented
(c) State Bank of India industry ? (SSC CHSL 2012)
(d) Government of India (a) Sugar industry
91. Who is the Chief Economic Advisor to the Government of (b) Petroleum refinery
India? (SSC CGL 1st Sit. 2016) (c) Light engineering industry
(a) Raghu Ram Rajan (b) Arvind Subramanian (d) Ship building
(c) Rajiv Mahirishi (d) Arvind Pangariya 101. The basic object of all production is to (SSC CHSL 2013)
92. Which type of foreign investment is considered as unsafe? (a) increase physical output
(SSC CGL 1st Sit. 2016) (b) satisfy human wants
(a) Foreign Direct Investment (FDI) (c) provide employment
(b) Portfolio investment (d) make profits

Downloded From : www.EasyEngineering.net


Downloded From : www.EasyEngineering.net

46 Economics
102. Inflation is caused by (SSC CHSL 2013) 110. Other things being equal, a decrease in quantity demanded
(a) decrease in production of a commodity can be caused by (SSC CHSL 2014)
(b) increase in money supply and decrease in production (a) a rise in the price of the commodity
(c) increase in money supply (b) a rise in the income of the consumer
(d) increase in production (c) a fall in the price of a commodity
103. Dumping is a form of price discrimination at (d) a fall in the income of the consumer
(SSC CHSL 2013) 111. Lender of the Last Resort is : (SSC CHSL 2015)
(a) local level (b) within industry (a) IDBI (b) NABARD
(c) national level (d) international level (c) SBI (d) RBI
104. The equilibrium of a firm under perfect competition will be
112. Perfectly inelastic demand is equal to : (SSC CHSL 2015)
determined when (SSC CHSL 2013)
(a) One (b) Zero
(a) Marginal Cost > Average Cost
(c) Infinite (d) Greater than one
(b) Marginal Revenue > Average Cost
113. When price of a substitute of commodity 'x' falls, the demand
(c) Marginal Revenue > Average Revenue
for 'x' (SSC CHSL 2015)
(d) Marginal Revenue = Marginal Cost
(a) remains unchanged
105. Expenditure on advertisement and public relations by an

ww
enterprise is a part of its
(a) fixed capital
(SSC CHSL 2013) (b) Increases at increasing rate
(c) rises
(d) falls

w.E
(b) consumption of fixed capital
(c) final consumption expenditure
(d) intermediate consumption
114. Mixed Economy means : (SSC CHSL 2015)
(a) Co-existence of public and private sectors

asy
106. Which of the following is not an economic problem ?
(SSC CHSL 2014)
(b) Co-existence of rich and poor
(c) Co-existence of small and large Industries
(d) Promoting both agriculture and Industries in the
(a) Deciding between paid work and leisure.

En
(b) Deciding between expenditure on one good and the
other.
economy

(c) Deciding between alternative methods of personal


saving. gin
115. By whom was the autonomous investment separated from
induced investment ?
(a) Malthus
(SSC CHSL 2015)
(b) Joan Robinson
(d) Deciding between different ways of spending leisure
time. eer
(c) Adam Smith
116. A demand curve will not shift:
(d) Schumpeter
(SSC CHSL 2015)
107. Which of the following occurs when labour productivity
rises ? (SSC CHSL 2014) ing
(a) When only prices of substitute products change
(b) When there is a change in advertisement expenditure
(a) The equilibrium nominal wage falls
(b) The equilibrium quantity of labour falls
.ne
(c) When only price of the commodity changes
(d) When only income changes.
(c) Competitive firms will be induced to use more capital
(d) The labour demand curve shifts to the right
108. Which of the following are consumer semi-surable goods ?
(SSC CHSL 2014)
(a) Alfred Marshall
t
117. The time element in price analysis was introduced by
(SSC CHSL 2015)
(b) J.S. Mill
(a) Cars and television sets (c) J.R. Hicks (d) J.M. Keynes
(b) Milk and milk products 118. A camera in the hands of a professional photographer is a
(c) Foodgrains and other food products ___________ good. (SSC Multitasking 2013)
(d) Electrical appliance like fans and electic irons (a) Capital (b) Free
109. Which of the following statements is correct ? (c) Intermediary (d) Consumer
(SSC CHSL 2014) 119. A portion of an individual's total income is spent on
(a) Most workers will work for less then their reservation consumption. The remaining part is called
wage. (SSC Multitasking 2014)
(b) The reservation wage is the maximum amount any firm (a) Savings (b) Deposits
will pay for a worker. (c) Surplus (d) Excess
(c) Economic rent is the difference between the market 120. Phase of increasing returns is otherwise called
wage and the reservation wage. (SSC Multitasking 2014)
(d) Economic rent is the amount one must pay to enter a (a) increasing cost (b) decreasing cost
desirable labour market. (c) increasing revenue (d) decreasing revenue

Downloded From : www.EasyEngineering.net


Downloded From : www.EasyEngineering.net

Economics 47
121. Monopoly refers to (SSC Multitasking 2014) 128. Which of the following is not a function of the Exim Bank of
(a) competition among monopolists India? (SSC Sub. Ins. 2012)
(b) absence of competition (a) Financing of export and import of goods and services
(c) a firm charging different prices to different customers (b) Inspection of exported goods for quality assurance
(d) a market situation in which there is only one buyer of a (c) Financing of joint ventures in foreign countries
commodity and one seller (d) Loans to Indian parties for contribution to share capital
122. Primary sector of an economy includes of joint ventures abroad
(SSC Multitasking 2014) 129. The Social Forestry Scheme was introduced during
(a) Service sector rendering services like banking, (SSC Sub. Ins. 2012)
transport, etc. which one of the primary importance (a) Fourth Five Year Plan (b) Eighth Five Year Plan
(b) Important enterprises of the manufacturinig sector (c) Second Five Year Plan (d) Sixth Five Year Plan
(c) Indsutrial sector which is of primary importance for 130. Which one of the following iron and steel works in India is
the economy not under public sector? (SSC Sub. Ins. 2012)
(d) Agriculture and allied activities (a) Bokaro (b) Jamshedpur
123. Liquidity preference theory of interest is propounded by (c) Bhilai (d) Durgapur
(SSC Multitasking 2014) 131. If a firm is operating at loss in the short-period in perfect

ww
(a) J.S. Mill
(c) I. Fisher
(b) A. Marshall
(d) J.M. Keynes
combination. it should; (SSC Sub. Ins. 2013)
(a) decrease the production and the price.
(b) increase the production and the price

w.E
124. Which one of the following statements is correct?
(SSC Sub. Ins. 2012)
(a) A commodity will value if it is wanted by somebody.
(c) continue to operate as long as it covers even the
variable costs.

to demand. asy
(b) A commodity will have value only if it is scarce relative
(d) shut-down and leave the industry
132. Which of the following is an indirect tax ?
(SSC Sub. Ins. 2013)
(c) The value of a commodity depends upon its price.
En
(d) The value of a commodity is entirely dependent upon
(a) Capital Gains Tax
(c) Wealth Tax
(b) Excise Duty
(d) Estate Duty
the substitutes.
125. Which of the following formulae could be used for gin
133. Taxes on professions can be levied by:
(SSC Sub. Ins. 2013)
calculating the per capita income of a country?
(SSC Sub. Ins. 2012)
eer
(a) State government only
(b) both by state and union government

(a)
Total familyincome
Number of family members
(c) by panchayats only
ing
(d) Union government only

(b)
Nationalincome
Total population .ne
134. Which one is not a function of money?
(SSC Sub. Ins. 2013)

(c)
Totalincomeof various industries
Total number of industrial wor kers
(a) Transfer of value
(c) Price stabilisation
135. "Closed Economy" means:
(b) Store of value

(a) no provision for public sector


t
(d) Value measurement
(SSC Sub. Ins. 2013)

(b) no provision for private sector


TotalGovernment Revenue
(d) (c) economy policy not well defined
Totalpopulation
(d) a country having no imports and exports
126. For controlling inflation, the central bank should 136. At "Break-even point", (SSC Sub. Ins. 2013)
(SSC Sub. Ins. 2012) (a) the industry is in equilibrium in the long-run.
(a) sell Government securities in the open market (b) the producers suffers the minimum losses
(b) lower the bank rate (c) the seller earns maximum profit
(c) purchase Government securities in the open market (d) the firm is at zero-profit point
(d) lower the reserve ratio of the banks 137. In the balance of payments account, unrequited receipts
127. There is no selling cost under (SSC Sub. Ins. 2012) and payments are also regarded as (SSC Sub. Ins. 2014)
(a) Perfect competition (a) bilateral transfers
(b) Monopolistic competition (b) unilateral transfers
(c) Oligopoly (c) capital account transfers
(d) Duopoly (d) invisible transfers

Downloded From : www.EasyEngineering.net


Downloded From : www.EasyEngineering.net

48 Economics
138. Price and output are determinates in market structure other 146. Basic problems of an economy is/are
than (SSC Sub. Ins. 2014) (SSC Sub. Ins. 2016)
(a) monopoly (b) perfect competition (a) Providing Social Security and employment to all.
(c) oligopoly (d) monopsony (b) Elimination of poverty and reduction of inequalities of
139. If average cost falls, marginal cost (SSC Sub. Ins. 2014) income and wealth.
(a) increases at a higher rate (c) Providing basic requirements of life to all.
(b) falls at the same rate (d) Decide as to what, how and for whom to produce.
(c) increases at a lower rate 147. Demand curve is indeterminate under
(d) falls at a higher rate (SSC Sub. Ins. 2016)
140. Which one of the following disburses long term loans to (a) duopoly (b) monopoly
private industry in India ? (SSC Sub. Ins. 2014) (c) pure competition (d) oligopoly
(a) Food Corporation of India 148. Marginal Revenue is (SSC Sub. Ins. 2016)
(b) Life Insurance Corporation of India (a) Revenue realized on the sale of an extra units.
(c) Primary Credit Society (b) Revenue realized from the sale of all units.
(d) Land Development Banks (c) The average revenue of a firm.
(d) Revenue realized on every unit sold.

ww
141. A low interest policy is also known as :

(a) investment policy


(SSC Sub. Ins. 2015) 149. All economic goods are considered are ..................in
economics. (SSC Steno. 2013)

(c) dear money policyw.E


(b) income generating policy
(a) Wealth
(c) Capital
(b) Money
(d) Materials
150. Total demand for goods and services at various levels of
(d) cheap money policy

asy
142. The market equilibrium for a commodity is determined by:
(SSC Sub. Ins. 2015)
employment is called as
(a) Effective demand
(SSC Steno. 2013)
(b) National demand

En
(a) the balancing of the forces of demand and supply for
the commodity.
(c) Market Demand (d) Employment demand
151. Relationship between price of related goods and quantity

(b) the intervention of the Government.


(c) the market demand of the commodity. gin of a particular commodity is called as (SSC Steno. 2013 )
(a) Income - demand (b) Market - demand

(d) the market supply of the commodity.


143. “Economies” of a firm are: (SSC Sub. Ins. 2015) eer
(c) Cross - demand
152. Devaluation means
(d) Price - demand
(SSC Steno. 2014 )
(a) reduction in the external value of currency
(a) an increase in its profits
(b) its dominance of the market ing
(b) fall in valuation of the essentials in an economy
(c) depreciation of the fixed assets
(c) saving in its production costs
(d) a reduction in its selling expenses .ne
(d) increase in the currency value in terms of foreign
currency
144. Regarding money supply situation in India it can be said
that the: (SSC Sub. Ins. 2015)
(a) Currency with the public is almost equal to the deposits
with the banks.
153. Disguised unemployment means
(a) willing to work and not getting the work
(b) not getting work on all days
t
(SSC Steno. 2014)

(c) insufficiency of capital structure to absorb the large


(b) Currency with the public is inconvertible only. force
(c) Currency with the public is more than the deposits (d) more people working, than required
with the banks. 154. The '3 B 's' - brokers, bankers and businessmen were
(d) Currency with the public is less than the deposits with responsible for (SSC Steno. 2014)
the banks. (a) The Great Depression
145. The equilibrium price of a commodity will definitely rise of (b) Russian Revolution
there is a/an: (SSC Sub. Ins. 2015) (c) French Revolution
(a) increase in supply combined with a decrease in (d) First world war
demand. 155. ATM means (SSC Steno. 2014)
(b) increase in demand accompanied by a decrease in (a) All Time Money
supply. (b) Automatic Teller Machine
(c) decrease in both demand and supply. (c) Automated Teller Machine
(d) increase in both demand and supply. (d) Any Time Money

Downloded From : www.EasyEngineering.net


Downloded From : www.EasyEngineering.net

Economics 49
156. Which market forms allow free entry and exit of firms? 160. An indirect instrument of monetary policy is
(SSC Steno. 2014) (SSC Steno. 2016)
(a) Perfect and Monopolistic (a) Open market operations
(b) Perfect and Oligopoly (b) Statutory liquidity ratio
(c) Oligopoly and Monopoly (c) Bank rate
(d) Monopoly and Monopolistic (d) Cash reserve ratio
157. GNP differs from NNP due to (SSC Steno. 2014) 161. One of the following is NOT a component of foreign
(a) net indirect taxes (b) Direct taxes exchange reserves in India (SSC Steno. 2016)
(c) interest on public debt (d) Depreciation (a) Gold stock of RBI
158. Which one of the following is an example of joint supply? (b) SDR holdings of government
(SSC Steno. 2014) (c) Foreign exchange assets of RBI
(a) Petrol and Bus (d) Foreign exchange assets of government
(b) Ink and Fountain pen 162. The market system in which there are only two buyers facing
(c) Sugar and Coffee a large number of sellers is called (SSC Steno. 2016)
(d) Wool and Cotton (a) monopsony (b) duopsony
159. Who fixes the REPO rate in India ? (SSC Steno. 2016) (c) duopoly (d) oligopoly

ww
(a) WTO - World Trade Organization
(b) SEBI - Securities and Exchange Board of India
163. 'Investing opportunities model' was proposed by
(SSC Steno. 2016)

w.E
(c) RBI - Reserve Bank of India
(d) IMF - International Monetary Fund
(a) Revenstein
(c) E.S. Lee
(b) Davis
(d) S.A. Stouffer

asy
En
gin
eer
ing
.ne
t

Downloded From : www.EasyEngineering.net


Downloded From : www.EasyEngineering.net

50 Economics

HINTS & SOLUTIONS


1. (a) A trade bloc is a type of intergovernmental agreement, 25. (a) Economic planning is an essential feature of socialism.
often part of a regional intergovermental organization, The most prominent example of a planned economy
where regional barriers to trade, (tariffs and non-tariff was the economic system of the Soviet Union.
barriers) are reduced or eliminated among the 26. (c) The Government of India decide to set up the Rashtriya
participating states. Barh Ayog (National Flood Commission) in 1976 to
2. (b) 3. (c) evolve a coordinated, integrated and scientific
4. (c) The Reserve Bank of India is India’s central banking approach to the flood control problems in the country.
institution, which controls the monetary policy of the 27. (d) The concept of a merit good introduced in economics
indian rupee. It commenced its operations on 1 April by Richard Musgrave (1957, 1959) is a commodity
1935 during the British Rule in accordance with the which is judged that an individual or society should
provisions of the Reserve Bank of India Act, 1934. have on the basis of some concept of need, rather
5. (d) T- bills are issued to meet short-term mismatches in than ability and willingness to pay. Examples include

6.
ww
receipts and expenditure. Bonds of longer maturity
are called dated securities.
(d) 7. (b) 8. (d) 9. (b)
the provision of food stamps to support nutrition, the
delivery of health services to improve quality of life

10.
w.E
(b) National income is the total value a country’s final
output of all new goods and services produced in one
year.
28.
and reduce morbidity, subsidized housing and
arguably education.
(c) Conspicuous consumption is the purchase of goods

11.
asy
(b) Economic Advisory Council to the Prime Minister
(PMEAC) is a non–constitutional, non–permanent and
29.
or services for the specific purpose of displaying one's
wealth. It is a means to show ones social status.
(b) Alfred Marshall was committed to partial equilibrium
to the Government of India, specifically the Prime
En
independent body constituted to give economic advice
models over general equilibrium on the grounds that
the inherently dynamical nature of economics made
Minister. The council serves to highlight key economic
issues facing the country to the government of India
from a neutral viewpoint. gin
30.
the former more practically useful.
(a) These agreements give the right of taxation in respec t
12.
13.
(d)
(c) A bilateral trade is the exchange of goods between two eer
of the income of the nature of interest, dividend, royalty
and fees for technical services to the country o f
countries that facilitates trade and investment by
reducing or eliminating tariffs, import quotas, export 31.
ing
residence. It promotes bilateral trade and investment.
(c) Brain drain can be described as the process in which a
country loses its most educated and talented workers
14.
15.
(d)
restraints and other trade barriers.

(d) Ragnar Anton Kittil Frisch was a Norwegian economist .ne


to other countries through migration. The main causes
include seeking employment or higher paying jobs,
and the co-recipient of the first Nobel Memorial Prize
in Economic Sciences in 1969 (with Jan Tinbergen). He
is known for having founded the discipline of
32. (a) Demography is the statistical study of human t
political instability and to seek a better quality of life.

populations and sub-populations. It encompasses the


study of the size, structure, and distribution of these
econometrics, and for coining the widely used term
pair macroeconomics /microeconomics in 1933. populations, and spatial and/or temporal changes in
16. (a) 17. (b) them in response to birth, migration, aging and death.
18. (a) Normative economics (as opposed to positive 33. (c) 34. (a) 35. (d) 36. (d) 37. (c)
economics) is a part of economics that expresses value 38. (c)
or normative judgments about economic fairness or 39. (b) In order to raise the standards of the banks
what the outcome of the economy or goals of public internationally, a number of committees were appointed
policy ought to be. by RBI. Among them Narasimham committee I (1991),
19. (c) 20. (c) 21. (d) 22. (c) 23. (d) Narasimham Committee II (1998) and Verma Committee
24. (b) The act of selling the same article, product under a (1999) were influential in improving international
single control, at different prices to different buyers is standards, and led to banking sector reforms, globally
known as price discrimination. Information on the price flexible to its deregulation, norms and conditions etc.
elasticity of demand can be used by business as part The above said committees have basically identified
of a policy of price discrimination (also known as yield the causes for the weak banks and guidelines have
management). been given to improve their efficiency.

Downloded From : www.EasyEngineering.net


Downloded From : www.EasyEngineering.net

Economics 51
40. (c) 52. (c) Social accounting is a method by which a firm seeks
41. (c) Here, statement of the question is wrong. The tenure to place a value on the impact on society of its
from 2012 to 2017 is designated as12th Five Year Plan operations. One social accounting system primarily
not 13th Five Year Plan. The tenure of 13th Five-Year attempts to measure National Income, final product,
Plan would be 2018 - 2022. consumption and accumulation of capital.
42. (b) 53. (c) Forced saving is an economic situation in which
43. (d) In economics, an externality is the cost or benefit that consumers spend less than their disposable income,
affects a party who did not choose to incur that cost or not because they want to save but because the goods
benefit. Economists often urge governments to adopt they seek are not available or because goods are too
policies that "internalize" an externality, so that costs expensive.
and benefits will affect mainly parties who choose to 54. (a) Bank's reserves at Central Bank
incur them. 55. (a) Interest rate is remaining sticky
56. (c) Sells a commodity at lower price in the world market
44. (c)
and charges a higher price in the domestic market
45. (a) Functional finance is an economic theory proposed
57. (a) 58. (c) 59. (d) 60. (b) 61. (d)
by Abba P. Lerner, based on effective demand principles
62. (c) 63. (b)
and chartalism. It states that government should
64. (b) Milton Friedman (July 31, 1912 - November 16, 2006)

ww
finance itself to meet explicit goals, such as taming the
business cycle, achieving full employment, ensuring
growth, and low inflation.
was an American economist, statistician, and writer
who taught at the University of Chicago for more than
three decades. He was a recipient of the 1976 Nobel
46.
47.
(c)
w.E
(d) Geographical Indication (GI) means the name of a
region or a locality, a specific place, or, in exceptional
Prize in Economic Sciences, and is known for his
research on consumption analysis, monetary history
and theory, and the complexity of stabilization policy.

asy
cases, a country, used to describe a product originating
in that region, locality, specific place or country, which
possesses a specific quality, reputation or other
65. (c) The qualitative or selective methods of credit control
are adopted by the Reserve Bank in its pursuit of

En
characteristics attributable to that geographical origin,
an d the production and/or processin g and or
economic stabilization an d as part of credit
management. The four important methods are Margin

preparation of which take place in the defined


geographical area. gin
66.
Requirements, Credit Rationing, Regulation of
Consumer Credit, Moral Suasion.
(c) Asegment ofthe financial market in which financia l
48. (c) Gold is mainly related to the international market as of
all the precious metals, it is the most popular as an eer
instruments with high liquidity and very short
maturities are traded. The money market is used by
investment. Gold has been used throughout history
as money and has been a relative standard for currency
equivalents specifc to economic regions or countries, 67.
ing
participants as a means for borrowing and lending in
the short term, from several days to just under a year.
(a) In economics, diminishing returns (also called law of
until recent times. Gold price has shown a long term
correlation with the price of crude oil. .ne
diminishing returns, law of variable proportions,
principle of diminishing marginal productivity, or
49. (a) The demand for labour is “derived” from the
production and demand for the product being
demanded. If the demand for the product increases,
t
diminishing marginal returns is the decrease in the
marginal (incremental) output of a production process
as the amount of a single factor of production is
incrementally increased, while the amounts of all other
either the price will increase or the demand for
production labour will increase until the equilibrium factors of production stay constant.
price and production numbers are met. 68. (b) The law of increasing returns is the opposite of the law
50. (d) Investment expenditure refers to the expenditure of decreasing returns. Where the law of diminishing
incurred either by an individual or a firm or the returns operates, every additional investment of capital
and labour yields less than proportionate returns. But,
government for the creation of new capital assets like
in the case of the law of increasing returns, the return
machinery, building etc. The purchase of house cannot
is more than proportionate.
be considered as investment exienditure as it may be for
69. (d) The Securities and Exchange Board of India (SEBI) is
personal use.
the regulator for the securities market in India. It was
51. (a) For private corporate sector, retained profits adjusted
established in the year 1988 and given statutory powers
for non operating surplus/ deficit is considered as its
on 12 April 1992 through the SEBI Act, 1992.
Net Saving. Retained profits are those which are 70. (d) Infosys Ltd is an Indian multinational corporation that
ploughed back into business after making provides business consulting, information technology,
commitments to depreciation provision for various software engineering and outsourcing services. It is
fixed assets, debts, government and to share-holders. headquartered in Bangalore, Karnataka.

Downloded From : www.EasyEngineering.net


Downloded From : www.EasyEngineering.net

52 Economics
71. (d) Regional Rural Banks are local level banking 82. (d) The one rupee note bears the signature of secretary
organizations operating in different States of India. ministry of Finance.
They have been created with a view to serve primarily 83. (b) NABARD is set up as an apex Development Bank with
the rural areas of India with basic banking and financial a mandate for facilitating credit flow for promotion and
services. The main purpose of RRB's is to mobilize development of agriculture, small-scale industries,
financial resources from rural / semi-urban areas and cottage and village industries, handicrafts and other
grant loans and advances mostly to small and marginal rural crafts. It was established on 12 July 1982 by a
farmers, agricultural labourers and rural artisans. special act by the parliament
72. (b) The Reserve Bank of India is India's Central Banking 84. (b) Surplus budget is the order of the economies in boom
Institution, which controls the Monetary Policy of the time.
Indian Rupee. It commenced its operations on 1 April 85. (b) Economic profit is the difference between total revenue
1935 during the British Rule in accordance with the and total opportunity cost. If a firm's total opportunity
provisions of the Reserve Bank of India Act, 1934. cost is less than the total revenues then the firm is
73. (d) The national income of a country can be measured by
making economic profit. If a firm's total opportunity
three alternative methods: (i) Product Method (ii)
cost is greater than the total revenues then the firm is
Income Method, and (iii) Expenditure Method.
making economic losses. It is similar to optimum profit.
74. (c) For many people, electronic banking means 24-hour

ww
access to cash through an automated teller machine
(ATM) or Direct Deposit of paychecks into checking
or savings accounts. But electronic banking involves
86. (c) Concept relating to Central Excise Duty and Service
Tax:- For. a proper understanding what is CENVAT,
some basic knowledge about Central Excise duty,

75.
w.E
many different types of transactions, rights,
responsibilities - and sometimes, fees.
(c) The Bretton Woods Institutions are the World Bank,
87.
92. (b)
Service Tax etc, is necessary.
(d) 88. (b) 89. (b) 90. (a) 91. (b)

asy
and theInternational Monetary Fund (IMF). They were
set up at a meeting of 43 countries in Bretton Woods,
93. (a) The major instruments of fiscal policy are as follows:
A. Budget B. Taxation C. Public Expenditure D. Public
Works E. Public Debt.
76.
New Hampshire, USA in July 1944.
(c) The price and output under monopoly are determined
En 94.
95.
(b)
(d) The Reserve Bank of India (RBI) Governor,
by equality between marginal cost and marginal
revenue and not by the intersection of demand and
supply curves. gin RaghuramRajan, on 4 August 2016 launched Sachet
portal, sachet.rbi.org.in, tocheck illegal money
77. (d) Macroeconomics is a branch of economics dealing with
the performance, structure, behavior, and decision- 96. eer
collection.
(c) 97. (b) 98. (c) 99. (c) 100. (a)
making of an economy as a whole, rather than individual
markets. Macroeconomists develop models that explain
the relationship between such factors as national income,
101.
ing
(b) The basic object of all production is to satisfy huma n
wants. In terms of Economics, satisfaction ofhuman
wants comes first, second comes the provision of

78.
output, consumption, unemployment, inflation etc.
(d) Internal economies arise within the firm because of the .ne
employment, third comes the profit motive and at last
comes the increase of physical outputs.

79.
expansion of the size of a particular firm. They are called
the economies of scale.
(c) In a free market economy there is a freedom of choice
for the consumers to buy goods and services which
102.
t
(b) Inflation is caused by increase in money supply and
decrease in production. In economics, inflation is a
sustained increase in the general price level of goods
and services in an economy over a period of time. It
suit their tastes and preferences. This is generally called can be defined as too much money chasing too few
the principle of consumer sovereignty. This means in goods. When the general price level rises, each unit
a market economy the consumers are just like a king or
of currency buys fewer goods and services.
sovereign who dictate what goods and services and
103. (d) Dumping is a form of price discrimination at the
what quantities of them are produced.
international level.In economics, "dumping" is a kind
80. (c) Net national product (NNP) refers to gross national
of predatory pricing, especially in the context of
product (GNP), i.e. the total market value of all final
international trade. It occurs when manufacturers
goods and servicesproduced by the factors of
export a product to another country at a price either
production of a country or other polity during a given
time period, minus depreciation. below the price charged in its home market or below
NNP = GNP – Depreciation its cost of production.
81. (b) Zilla Parishad is the apex body of the PR(Panchayati 104. (d) The equilibrium of a firm under perfect competition
Raj) system located at the district level. Chairpersons/ will be determined when marginal revenue equals
Presidents of Panchayat Samitis come within its marginal cost. In the short run, perfectly-competitive
jurisdiction. markets are not productively efficient as output will

Downloded From : www.EasyEngineering.net


Downloded From : www.EasyEngineering.net

Economics 53
not occur where marginal cost is equal to average cost 120. (d) Phase of increasing returns is otherwise called
(MC=AC). They are allocatively efficient, as output decreasing revenue. If output increases by more than
will always occur where marginal cost is equal to that proportional change in inputs, there are increasing
marginal revenue (MC=MR). returns to scale (IRS).
105. (d) Expenditure on advertisement and public relation by 121. (b) A monopoly exists when a specific person or enterprise
an enterprise is a part of its intermediate consumption. is the only supplier of a particular commodity.
Visibility through advertising and promotion builds Monopolies are thus characterized by a lack of
an enterprise' reputation with the customer. Although economic competition to produce the good or service
most advertising is through word of mouth, that word and a lack of viable substitute goods.
of mouth starts with awareness that customers have 122. (d) The primary sector of the economy is the sector of an
gained about the enterprise through their advertising economy making direct use of natural resources. This
and promotions. includes agriculture, forestry, fishing and mining. This
106. (d) Deciding between different ways of spending leisure is contrasted with the secondary sector, producing
time is not an economic problem. manufactured goods, and the tertiary sector, producing
107. (d) When labour productivity rises, the labour demand services.
curve shifts to the right. As the productivity increases, 123. (d) J.M. Keynes propounded what has come to be known
as the liquidity preference theory of interest. According

ww
the production function shifts up and simultaneously
the labour demand curve shifts out and right. At a
given real wage, more workers are hired and output
to this theory, the rate of interest is determined by the
demand for and supply of money.

108. (c)
increases.
w.E
Semi durable consumer goods are products with
durability of about one year. E.g., food grains and food
124. (b)
125. (b) Per capita income, more simply known as income per
person, is the mean income within an economic

109. (c)
products.
asy
Economic rent is the difference between the market
wage and the reservation wage.
aggregate such as a country or city. It is calculated by
taking a measure of all sources of income in the
aggregate (such as GDP or Gross national income) and
110. (a) Other things being equal, a decrease in quantity
En
demanded of a commodity can be caused by a rise in
dividing it by the total population.
126. (a) The Central Bank can use various different method s

111. (d)
the price of the commodity.
As a Banker to Banks, the Reserve Bank of India acts gin for reducing inflation. To control inflation, central ban
sells the government securities to the public through
k

112. (b)
as the “lender of the last resort”.
When the price elasticity of demand for a good is eer
the banks. This results in transfer of a part of bank
deposits to central bank account and reduces credi t
creation capacity of the commercial banks.
113. (d)
perfectly inelastic i.e. Ed = 0.
It is because demand for goods which have substitutes
is more elastic because when price of a substitute falls
ing
127. (a) In perfect competition, there must be large number o f
buyers and sellers. According to Scitovsky buyers and
in relation to its commodity, the demand for the
commodity also falls. .ne
sellers are price takers in the purely competitive market.
Each seller (or firm) sells its products at the price
114. (a) Mixed economy means an economic system in which
both the private enterprise and a degree of state
monopoly (usually in public services, defense,
128. (b) t
determined by the market. Similarly, each buyer buys
the commodity at the price determined by the market.

129. (d) Social forestry means the management and protection


infrastructure, and basic industries) coexist. of forests and afforestation on barren lands with the
115. (d) 116. (b) purpose of helping in the environmental, social and
117. (a) Alfred Marshall Propounded the theory that price is rural development. This scheme was introduced in Sixth
determined by both demand and supply and also gave Five Year Plan.
great importance to the time element in the 130. (b) At present all important steel plants except TISCO (Tata
determination of price. Iron and Steel co. Ltd) which is located in Jamshedpur
118. (a) Camera is a capital for a photographer because he earns are under public sector.
his livelihood as it is his occupation 131. (c) The situation when a firm is operating at loss in the
119. (a) If a portion of individuals income is spent on short period in perfect competition arises when the
consumption, the remaining portion will be saving. price is so low that total revenue is not even enough to
Since whatever is not consumed must be saved, as cover the variable cost of production. Shut down point
soon as we specify a consumption function we have is that point at which the price is equal to average
necessarily specified a savings function. "Function" variable costs or the firm covers its variable costs. So
just means that one thing depends on another thing or it should operate as long as it covers even the variable
things. costs.

Downloded From : www.EasyEngineering.net


Downloded From : www.EasyEngineering.net

54 Economics
132. (b) Some examples of indirect taxes include value added 137. (b) In the balance of payment accounts, unrequited
tax, excise duty, sales tax, stamp duty and custom duty payments or receipts (gifts) are regarded s unilateral
levied on imports. These are taxes levied by the state transfers. In the balance-of-payments statement of a
on expenditure and consumption, but not on property country, the accounts that show the amount of money
or income. sent or received as gifts.
133. (a) In India, the professional tax is imposed at the state 138. (b) Pr ice, output and perfect competition are the
level. However, not all the states impose this tax. determinants of market structure. The demand curve
Business owners, working individuals, merchants and of an individual firm under a purely competitive
people carrying out various occupations comes under industry is perfectly elastic. This is the increase or
the purview of this tax. Professional tax is levied by decrease of the output of a single seller has no effect
on the total supply and market place.
particular Municipal Corporations.
139. (b) Marginal cost and average cost curves are related. If
134. (c) Generally, economists have defined four types of
average costs falls, marginal cost falls at the same rate.
functions of money which are as follows: (i) Medium
140. (d) Land development bank (LDB) disburses long term
of exchange (transfer of value) (ii) Measurement of
loans to private industry in India.The main objective
value. (iii) Standard of deferred payments. and (iv) Store
of the LDB is to promote the development of land,
of value. Price stabilization is a function of monetary agriculture and increase the agricultural production.

ww
policy.
135. (d) Closed economy is an economy in which no activity is
conducted with outside economies. A closed economy
The LDB provides long-term finance to members
directly through its branches

w.E
is self-sufficient, meaning that no imports are brought
in and no exports are sent out. The goal is to provide
consumers with everything that they need from within
141. (d) Cheap Money policy is a monetary policy through
which a bank sets low interest rates so that credit is
easily achievable.

the economy's borders.


asy
136. (d) The break-even point (BEP) is the point at which cost
142. (a) Market equilibrium is a market state where the supply
in the market is equal to the demand in the market.
143. (d) 144. (d) 145. (b) 146. (d) 147. (d)

En
or expenses and revenue are equal: there is no net loss
or gain, and one has "broken even." For businesses,
148. (a) 149. (d) 150. (c) 151. (c) 152. (a)
153. (b) 154. (a) 155. (c) 156. (a) 157. (d)
reaching the break-even point is the first major step
towards profitability. gin
158. (d) 159. (c) 160. (a) 161. (c) 162. (c)
163. (d)

eer
ing
.ne
t

Downloded From : www.EasyEngineering.net


Downloded From : www.EasyEngineering.net

5
CHAPTER
GENERAL SCIENCE
1. A concave lens always forms an image which is 11. In which of the following processes is energy released ?
(SSC CGL 1st Sit. 2010) (SSC CGL 1st Sit. 2010)
(a) real and erect (b) virtual and erect (a) Respiration (b) Photosynthesis
(c) real and inverted (d) virtual and inverted (c) Ingestion (d) Absorption
2. A vitamin requires cobalt for its activity. The vitamin is 12. Animals living in the three trunks are known as
(SSC CGL 1st Sit. 2010) (SSC CGL 1st Sit. 2010)
(a) Vitamin B12 (b) Vitamin D (a) Arboreal (b) Volant
(c) Vitamin B2 (d) Vitamin A (c) Amphibious (d) Aquatic
3. One of the constituents of tear gas is 13. In input frequency of a full wave rectifier be n, then output

ww
(a) Ethane
(c) Ether
(b) Ethanol
(d) Chloropicrin
4. The modulus of rigidity is the ration of
frequency would be

(a)
n
(b) n
(SSC CGL 1st Sit. 2010)

w.E
(b) Volume stress to volume strain
(SSC CGL 1st Sit. 2010)
(a) longitudinal stress to longitudinal strain
(c)
2

3n
(d) 2n
(c) shearing stress to shearing strain
(d) tensile stress to tensile strain asy 14.
2
Heat transfer horizontally within the atmosphere is called
(SSC CGL 1st Sit. 2010)
5. The propagation of sound waves in a gas involves
En
(SSC CGL 1st Sit. 2010) (a) Conduction
(c) Absorption
(b) Convection
(d) Advection
(a) adiabatic compression and refraction
(b) isothermal compression and rarefaction
(c) isochoric compression and rarefaction gin
15. Noise is measured in
(a) Watt (b) REM
(SSC CGL 1st Sit. 2010)

(d) isobaric compression and rarefaction


6. An stomic clock is based on transitions in 16. eer
(c) Centigrade (d) Decibel
The bats can fly in the dark because

(a) Sodium
(c) Magnesium
(SSC CGL 1st Sit. 2010)
(b) Caesium
(d) Aluminium
ing (SSC CGL 1st Sit. 2010)
(a) they can see the objects in darkness

7. Plasma membrane in eukaryotic celle is made up of


(SSC CGL 1st Sit. 2010)
predators
.ne
(b) they have weak legs and are likely to be attacked by

(a) Phospholipid (b) Lipoprotein


(c) Phospholpo-protein (d) Phospho-protein
8. Which one of the following is also called the 'power plants'
of the cell ? (SSC CGL 1st Sit. 2010)
17.
(c) they generate flashes of light
(d) they generate ultrasonic sound waves
t
What changes will happen to a bow of ice and water kept at
exactly zero degree Celsius? (SSC CGL 1st Sit. 2010)
(a) Golgi body (b) Mitochondrion (a) All ice will melt
(c) Ribosome (d) Lysosome (b) All water will become ice
9. What is the chemical name of vinegar ? (c) No change will happen
(SSC CGL 1st Sit. 2010) (d) Only some ice will melt.
(a) Citric acid (b) Acetic acid 18. The art and science of map making is called
(c) Pyruvic acid (d) Malic acid (SSC CGL 2nd Sit. 2010)
10. Which of the following is not a property of heavy water ? (a) Remote Sensing (b) Cartography
(SSC CGL 1st Sit. 2010) (c) Photogrammetry (d) Mapping
(a) Boiling point of heavy water is lower than that of 19. Silver halides are used in photographic plates because they
ordinary water
are (SSC CGL 2nd Sit. 2010)
(b) Density of heavy water is higher than that of ordinary
(a) oxidised in air
water
(c) Freezing point of heavy water is higher than that of (b) soluble in hyposolution
ordinary water (c) reduced by light
(d) It produces corrosion (d) totally colourless

Downloded From : www.EasyEngineering.net


Downloded From : www.EasyEngineering.net

56 General Science
20. Tetra ethyle lead (TEL) is (SSC CGL 2nd Sit. 2010) 32. The longest bone in the human body is:
(a) a catalyst in burning fossil fuel (SSC CGL 1st Sit. 2011)
(b) an antioxidant (a) Ulna (b) Humerus
(c) a reductant (c) Femur (d) Tibia
(d) an antiknock compound 33. The time period of a pendulum when taken to the Moon
21. Curie point is the temperature at which would: (SSC CGL 1st Sit. 2011)
(SSC CGL 2nd Sit. 2010) (a) remain the same (b) decrease
(a) Matter becomes radioactive (c) become zero (d) increase
(b) A metal loses magnetic properties 34. The function of ball hearings in a wheel is:
(c) A metal loses conductivity (SSC CGL 1st Sit. 2011)
(d) Transmutation of metal occurs.
(a) to increase friction
22. The isotope used for the production of atomic energy is
(b) to convert kinetic friction into rolling friction
(SSC CGL 2nd Sit. 2010)
(c) to convert static friction into kinetic friction
(a) U-235 (b) U-238
(d) just for convenience
(c) U-234 (d) U-236
35. 'Shock–absorbers' are usually made of steel as it:
23. The acceleration due to gravity at the equator
(SSC CGL 2nd Sit. 2010) (SSC CGL 1st Sit. 2011)

ww
(a) is less than that at the poles
(b) is greater than that at the poles
(c) is equal to that at the poles
(a) is not brittle
(b) has lower elasticity
(c) has higher elasticity

w.E
(d) does not depend on the earth's centripetal acceleration
24. Which of the following is not a nucleon ?
(SSC CGL 2nd Sit. 2010)
(d) has no ductile property
36. Which of the following could be used as fuel in propellant or
rockets? (SSC CGL 1st Sit. 2011)
(a) Proton
(c) Electron asy
(b) Neutron
(d) Positron
(a) Liquid Hydrogen + Liquid Nitrogen
(b) Liquid Oxygen + Liquid Argon
(c) Liquid Nitrogen + Liquid Oxygen
25. The material used in the manufacture of lead pencil is

(a) Graphite (b) Lead En


(SSC CGL 2nd Sit. 2010) (d) Liquid Hydrogen + Liquid Oxygen
37. The addition of gypsum to portland cement helps in:
(c) Carbon (d) Mica
26. Angle of friction and angle of repose are gin (SSC CGL 1st Sit. 2011)
(a) increasing the strength of cement

(a) equal to each other


(SSC CGL 2nd Sit. 2010)
eer
(b) rapid setting of cement
(c) preventing rapid setting of cement
(b) not equal to each other
(c) proportional to each other
(d) None of the above ing
(d) reduction in the cost of cement
38. White lung disease is prevalent among the workers of:
(SSC CGL 1st Sit. 2011)
27. What happens to a person who receives the wrong type of
blood? (SSC CGL 2nd Sit. 2010)
(a) Paper industry
.ne
(b) Cement industry
(c) Cotton industry (d) Pesticide industry
(a) All the arteries constrict
(b) All the arteries dialates
(c) The RBCs agglutinate
(d) The spleen and lymphnodes deteriorate
39. lodoform is used as an:
(a) antipyretic
(c) antiseptic
(b) analgesic
(d) anaesthetic
t
(SSC CGL 1st Sit. 2011)

40. An artificial ecosystem is represented by:


28. If all bullets could not be removed from gun shot injury of a
man, it may cause poisoning by (SSC CGL 2nd Sit. 2010) (SSC CGL 1st Sit. 2011)
(a) Mercury (b) Lead (a) pisciculture tank (b) agricultural land
(c) Iron (d) Arsenic (c) zoo (d) aquarium
29. Ringworm is a . .............. disease. (SSC CGL 2nd Sit. 2010) 41. The constituents of automobile exhaust that can cause
(a) Bacterial (b) Protozoan cancer is are: (SSC CGL 1st Sit. 2011)
(c) Viral (d) Fungal (a) Oxides of nitrogen
30. Pituitary gland is situated in (SSC CGL 2nd Sit. 2010) (b) Carbon monoxide
(a) the base of the heart (c) Polycyclic hydrocarbons
(b) the base of the brain (d) Lead
(c) the neck 42. The optimum dissolved oxygen level (in mg/litre) required
(d) the abdomen for survival of aquatic organisms is:
31. Saliva helps in the digestion of: (SSC CGL 1st Sit. 2011) (SSC CGL 1st Sit. 2011)
(a) Fats (b) Starch (a) 4 – 6 (b) 2 – 4
(c) Proteins (d) Vitamins (c) 8 – 10 (d) 12 – 16

Downloded From : www.EasyEngineering.net


Downloded From : www.EasyEngineering.net

General Science 57
43. The world's only floating national park is situated in: (c) refraction and total internal reflection of light
(SSC CGL 1st Sit. 2011) (d) dispersion of light only
(a) Manipur (b) Kuala Lumpur 56. The phenomenon of light associated with the apprearance
(c) Bilaspur (d) Dispur of blue colour of the sky is (SSC CGL 2nd Sit. 2011)
44. Who invented vaccination for 'Small Pox'? (a) Interference (b) Reflection
(SSC CGL 2nd Sit. 2011) (c) Refraction (d) Scattering
(a) Sir Fredrick Grant Banting 57. Lens is made up of (SSC CGL 2nd Sit. 2011)
(b) Sir Alexander Fleming (a) Pyrex glass (b) Flint glass
(c) Edward Jenner (c) Ordinary glass (d) Cobalt glass
(d) Louis Pasteur 58. The element which is used for vulcanizing rubber
45. BT seed is associated with (SSC CGL 2nd Sit. 2011) (SSC CGL 2nd Sit. 2011)
(a) Rice (b) Wheat (a) Sulphur (b) Bromine
(c) Cotton (d) Oil seeds (c) Silicon (d) Phosphorus
46. Which one of the following minerals is found in Monazite 59. Which of the following is responsible for the extra strength
sand? (SSC CGL 2nd Sit. 2011) of pyrex glass? (SSC CGL 2nd Sit. 2011)
(a) Potassium (b) Uranium (a) Potassium Carbonate (b) Lead Oxide
(c) Thorium (d) Sodium (c) Borax (d) Ferric Oxide

ww
47. In coriander, the useful parts are (SSC CGL 2nd Sit. 2011)
(a) roots & leaves
(b) leaves & flowers
60. The noble gas used for the treatment of cancer is

(a) Helium (b) Argon


(SSC CGL 2nd Sit. 2011)

w.E
(c) leaves & dried fruits
(d) flowers & dried fruits
48. Which plant is called Herbal Indian Doctor?
61.
(c) Krypton (d) Radon
Fertilization occurs normally in the
(SSC CGL 1st Sit. 2012)

(a) Amla (b) Mango asy


(SSC CGL 2nd Sit. 2011)
62.
(a) Cervix
(c) Fallopian tube
(b) Vagina
(d) Uterus
People consuming alcohol in heavy quantities generally die of
(c) Neem
49. The pH of human blood is
(a) 7.2
(d) Tulsi

(b) 7.8 En
(SSC CGL 2nd Sit. 2011)
(a) liver or stomach cancer
(SSC CGL 1st Sit. 2012)

(c) 6.6 (d) 7.4


50. Which amongst the following is largest endocrine gland in gin (b) weakening of heart muscles leading to cardiac arrest
(c) blood cancer
(d) Cirrhosis
the body?
(a) Thyroid
(SSC CGL 2nd Sit. 2011)
(b) Parathyroid
63.
eer
The organisms at the base of the grazing food-chain are
(SSC CGL 1st Sit. 2012)
(c) Adrenal (d) Pituitary
51. Which amongst the following is the largest mammal?
(SSC CGL 2nd Sit. 2011)
(a) Carnivores
(c) Producers ing(b) Decomposers
(d) Herbivores
The mass of 10 moles of water is (SSC CGL 1st Sit. 2012)
(a) Elephant
(c) Dinosaur
(b) Whale
(d) Rhinoceros
64.
(a) 90 g
(c) 18 g
(b) 45 g
(d) 180 g .ne
52. Which part becomes modified as the tusk of elephant?

(a) Canine
(c) Second incisor
(SSC CGL 2nd Sit. 2011)
(b) Premolar
(d) Molar
65.

66.
Vitamin A is rich in
(a) Carrot
(c) Beans
(b) Lime
(d) Rice
t
(SSC CGL 1st Sit. 2012)

The high boiling point of water compared to hydrogen


53. Optical fibres are based on the phenomenon of sulphide or hydrogen chloride is due to
(SSC CGL 2nd Sit. 2011) (SSC CGL 1st Sit. 2012)
(a) Interference (a) Dipole insulation
(b) Dispersion (b) Van der Waal's attraction
(c) Diffraction (c) Polar covalent bonding
(d) Total Internal Reflection (d) Hydrogen bonding
54. Now–a–days yellow lamps are frequently used as street 67. Which of the following determines the chemical properties
lights. Which of the following gases is used in these lamps? of an element? (SSC CGL 1st Sit. 2012)
(SSC CGL 2nd Sit. 2011) (a) Number of electrons
(a) Sodium (b) Neon (b) Number of neutrons
(c) Hydrogen (d) Nitrogen (c) Number of protons
55. 'Mirage' is an example of (d) All of the above
(SSC CGL 2nd Sit. 2011) 68. 'Lumen' is the unit of (SSC CGL 1st Sit. 2012)
(a) refraction of light only (a) Illuminance (b) Brightness
(b) total internal reflection of light only (c) Luminous flux (d) Luminous intensity

Downloded From : www.EasyEngineering.net


Downloded From : www.EasyEngineering.net

58 General Science
69. Which one of the following forces is a 'dissipative force'? 83. We receive sunlight on earth surface. What type of light
(SSC CGL 1st Sit. 2012) beams are these? (SSC CGL 2nd Sit. 2012)
(a) Electrostatic force (b) Magnetic force (a) Random (b) Parallel
(c) Gravitational force (d) Frictional force (c) Converging (d) Diverging
70. If a resistive wire is elongated, its resistance 84. Polar-bears hold cures for: (SSC CGL 2nd Sit. 2012)
(SSC CGL 1st Sit. 2012) (a) Type II diabetes (b) Osteoporosis
(a) decreases (b) remains constant (c) Breast-cancer (d) Kidney failure
(c) increases (d) All of the above 85. Which colour/colours of light has the highest velocity
71. If a magnet has a third pole, then the third pole is called through vacuum? (SSC CGL 2nd Sit. 2012)
(SSC CGL 1st Sit. 2012) (a) Blue (b) Red
(a) defective pole (b) consequent pole (c) Green (d) All of the above
(c) extra pole (d) arbirary pole 86. The ultimate source of energy in a hydroelectric power
72. The sweet taste of fruits is due to (SSC CGL 1st Sit. 2012) station is: (SSC CGL 2nd Sit. 2012)
(a) Lactose (b) Fructose (a) solar energy
(c) Maltose (d) Ribose (b) the potential energy of water
73. Which is NOT a correct statement? (c) the kinetic energy of water
(SSC CGL 2nd Sit. 2012) (d) the electro-chemical energy of water

ww
(a) Phenols are acidic
(b) In benzene all the atoms lie in one plane
(c) Methylated spirit contains only methanol
87. The disease that kills more people than lung cancer as a
consequence of air pollution is: (SSC CGL 2nd Sit. 2012)
(a) chronic bronchitis (b) asthma

w.E
(d) Dilute solutions contain less amount of solute
74. The infective stage of Malaria is:
(SSC CGL 2nd Sit. 2012)
(c) emphesema (d) heart attack
88. Which of the following pairs is correctly matched?
(SSC CGL 2nd Sit. 2012)
(a) Gametocyte
(c) Sporozoite
asy
(b) Ring stage
(d) Merozoite
75. Which of the following is meant for the ex-situ conservation
(a) Milk of lime–sodium sulphate
(b) Glauber’s salt–calcium sulphate
(c) Salt petre–potassium nitrate
of various species?
(a) Sperm bank (b) Blood bank En
(SSC CGL 2nd Sit. 2012) (d) Gypsum–calcium hydroxide
89. ‘Eutrophication’ is associated with
(c) Germplasm bank (d) Herbarium
76. An algae type ocean deposit is: (SSC CGL 2nd Sit. 2012)
(a) Neritic remains (b) Diatom Ooze
gin(a) Nitrates and Phosphates
(SSC CGL 1st Sit. 2012)

(c) Pteropod Ooze (d) Pelagic deposits


77. Photosynthetic vesicle found in bacteria is called a:
(b) Sewage

eer
(c) Silt load
(d) Vegetation

(a) Mesosome
(SSC CGL 2nd Sit. 2012)
(b) Chromatophore ing
90. The best way to maintain a natural equilibrium between the
pest and predator is by using (SSC CGL 1st Sit. 2012)
(c) Genophore (d) Pneumatophore
78. What type of mirror is used in a view finding mirror of a
vehicle? (SSC CGL 2nd Sit. 2012)
(a) insecticides
(c) pesticides
.ne
(b) biological control
(d) herbicides

(a) Convex mirror


(c) Concave mirror
(b) Plane mirror
(d) Paraboloidal mirror
79. Pyroligneous acid obtained from wood contains:
(SSC CGL 2nd Sit. 2012)
91. ‘Dry ice’ is the condensed form of

(a) sulphur tri-oxide


(b) carbon dioxide
t
(SSC CGL 1st Sit. 2012)

(c) highly cooled water


(a) 10% Formaldehyde (b) 10% Acetic acid (d) oxygen
(c) 10% Formic acid (d) 10% ethanol 92. The materials which are strongly attracted by magnet are
80. Union Carbide India Ltd. manufactured essentially: called (SSC CGL 1st Sit. 2012)
(SSC CGL 2nd Sit. 2012) (a) ferro-magnetic substances
(a) Heavy water (b) Petrochemicals (b) universal substances
(c) Fertilizers (d) Leather goods (c) para-magnetic substances
81. Drying oils contain a fairly large proportion of: (d) dia-magnetic substances
(a) Unsaturaled fatty acids (SSC CGL 2nd Sit. 2012) 93. In a pin-hole camera, we usually get
(b) Fats
(SSC CGL 1st Sit. 2012)
(c) Proteins (a) erect impression (b) inverted impression
(d) Saturated fatty acids
(c) erect image (d) inverted image
82. The red, orange and yellow colours of leaves are due to:
94. What happens to the kinetic energy of gas molecules with
(SSC CGL 2nd Sit. 2012)
rise of temperature ? (SSC CGL 1st Sit. 2012)
(a) Carotenoids (b) Aldehydes
(a) Remains same (b) Fluctuates
(c) Tannins (d) Lignins
(c) Increases (d) Decreases

Downloded From : www.EasyEngineering.net


Downloded From : www.EasyEngineering.net

General Science 59
95. The sex of a child is determined (SSC CGL 1st Sit. 2012) 108. During respiration, the gases enter into the blood and leave
(a) six to seven weeks after conception the same by the process of (SSC CGL 2nd Sit. 2012)
(b) in the third month of pregnancy (a) Active transport
(c) at the time of sperm’s entry (b) Diffusion
(d) at the time of fertilisation of ovum (c) Diffusion and active transport
96. The food chain of the ecosystem is maintained by the (d) Osmosis
activities of (SSC CGL 1st Sit. 2012) 109. Heart is devoid of (SSC CGL 2nd Sit. 2012)
(a) Decomposers (b) Predators (a) Cardiac muscle (b) Involuntary muscle
(c) Producers (d) Consumers (c) Voluntary muscle (d) Smooth muscle
97. Which one of these primates is closest to the modern man? 110. The soil salinity is measured by (SSC CGL 2nd Sit. 2012)
(SSC CGL 1st Sit. 2012) (a) Conductivity meter (b) Hygrometer
(a) Orang-utan (b) Chimpanzee (c) Psychrometer (d) Auxanometer
(c) Gorilla (d) Gibbon 111. Which of the following is a fungal disease?
98. A gram of fertile agricultural soil may contain bacteria upto (SSC CGL 2nd Sit. 2012)
(SSC CGL 1st Sit. 2012) (a) Leucoderma (b) Eczema
(a) five million (c) Ringworm (d) Elephantiasis
(b) one billion and above 112. Chickenpox is caused by (SSC CGL 2nd Sit. 2012)

ww
(c) fifty thousand
(d) five hundred thousand
99. A reduction reaction involves (SSC CGL 1st Sit. 2012)
(a) DNA virus
(c) Streptococcus
(b) Variola virus
(d) Vibrio cholerae

(a) addition of oxygen


w.E
(b) addition of nitrogen
113. Instruments can be shielded from outside magnetic effects
by surrounding them with
(a) Iron shield
(SSC CGL 2nd Sit. 2012)
(b) Rubber shield
(c) addition of hydrogen
(d) None of the above
100. An antiknock for petrol is asy
(SSC CGL 1st Sit. 2012)
(c) Brass shield
114. Find the odd one.
(a) Marble
(d) Glass shield

(b) Chalk
(SSC CGL 2nd Sit. 2012)

(a) Sodium hydroxide (b) Ethanol


(c) Sodium benzoate (d) Lead tetraethyl En (c) Limestone (d) Slaked lime
115. The following is a pseudo-force: (SSC CGL 2nd Sit. 2012)
101. Which one of the following pairs is not correctly matched ?
(SSC CGL 1st Sit. 2012) gin (a) Centrepetal force
(b) Centrifugal reaction force
(a) Vitamin C - Scurvy
(b) Vitamin K - Clotting of blood
(c) Vitamin A - Night blindness eer
(c) Centrifugal force
(d) Strong nuclear force
116. The hydraulic brake used in automobiles is a direct
(d) Vitamin E - Rickets
102. The fibre that will yield ammonia when destructively distilled
application of
ing
(a) Archimedes’ principle
(SSC CGL 2nd Sit. 2012)

is
(a) Wool
(SSC CGL 1st Sit. 2012)
(b) Cellulose acetate
(b) Torricellean law
(c) Bernoulli’s Theorem .ne
(c) Cotton

(a) ATP
(d) Silk
103. During respiration, energy is derived from
(SSC CGL 1st Sit. 2012)
(b) Chlorophyll
(d) Pascal’s law

(a) Perkin
t
117. Amides can be converted to amines by the reaction named
(SSC CGL 2nd Sit. 2012)
(b) Claisen
(c) RNA (d) DNA (c) Hoffman (d) Clemmesen
104. When was a global network of daily temperature records 118. The base used as an antacid is (SSC CGL 2nd Sit. 2012)
created ? (SSC CGL 1st Sit. 2012) (a) Calcium hydroxide
(a) Around 1890 (b) Around 1920 (b) Barium hydroxide
(c) Around 1800 (d) Around 1850 (c) Magnesium hydroxide
105. Sex hormones are (SSC CGL 1st Sit. 2012) (d) Silver hydroxide
(a) Vitamins (b) Alkanes 119. A process which is not helpful in the prevention of rusting
(c) Carbohydrates (d) Steroids of iron is (SSC CGL 2nd Sit. 2012)
106. A form of condensation that reduces visibility and causes (a) annealing (b) applying grease
breathing problems is (SSC CGL 2nd Sit. 2012) (c) galvanising (d) painting
(a) Dew (b) Frost 120. Denatured alcohol (SSC CGL 2nd Sit. 2012)
(c) Smog (d) Mist (a) is a form of alcohol
107. Green glands are associated with (SSC CGL 2nd Sit. 2012) (b) is unfit for drinking as it contains poisonous substances
(a) Reproduction (b) Excretion (c) contains coloured impurities
(c) Respiration (d) Digestion (d) is sweet to taste

Downloded From : www.EasyEngineering.net


Downloded From : www.EasyEngineering.net

60 General Science
121. Phenolics as pollutants can be removed from waste water 132. The disease that has been eradicated from the world, is :
by use of (SSC CGL 2nd Sit. 2012) (SSC CGL 1st Sit. 2013)
(a) Ion exchange resin technique (a) Small pox (b) Leprosy
(b) Electrolyte decomposition technique (c) Poliomyelitis (d) Chicken pox
(c) Reverse osmosis method 133. In dicots the pollen grains possess :
(d) Polymeric adsorbents (SSC CGL 1st Sit. 2013)
122. The stability of a pond ecosystem depends on (a) one germ pore (b) two germ pores
(SSC CGL 2nd Sit. 2012) (c) three germ pores (d) four germ pores
(a) micro-organisms and fishes 134. 'Farad' is the unit of : (SSC CGL 1st Sit. 2013)
(b) micro-organisms and zoo planktons (a) Conductnce (b) Capacitance
(c) fishes and reptiles (c) Inductance (d) Resistance
(d) producers and consumers 135. Steel is more elasitc than rubber because it :
123. Supersonic air planes create a shock wave called (SSC CGL 1st Sit. 2013)
(SSC CGL 2nd Sit. 2012) (a) is harder than rubber
(a) Transition wave (b) Ultrasound (b) requires larger deforming force
(c) Transverse wave (d) Sonic boom (c) is never deformed
124. The danger signals are red while the eye is more sensitive to (d) is deformed very easily

ww
yellow because (SSC CGL 2nd Sit. 2012)
(a) absorption in red is less than yellow and hence red is
136. Identify the old term amongst the following group :

(a) Coaxial cable


(SSC CGL 1st Sit. 2013)
(b) Optical fibre

w.E
visible from a distance
(b) scattering in yellow light is less than red
(c) the wavelength of red light is more than yellow light
(c) Twisted pair wire (d) Microwaves
137. Stains of rust on clothes can be removed by :
(SSC CGL 1st Sit. 2013)
(d) none of the above reasons

asy
125. Transboundary pollution (or) Acid rain is caused by :
(SSC CGL 1st Sit. 2013)
(a) H2O2
(c) Petrol
(b) Oxalic acid
(d) Alcohol
(a) Nitrogen oxide and sulphur dioxide
(b) Carbon monoxide En 138. The percentage of nitrogen present in ammonium sulphate
is : (SSC CGL 1st Sit. 2013)
(c) Carbon dioxide
(d) Hydrocarbon gin (a) 18%
(c) 25%
(b) 21%
(d) 30.5%
139. Ethanol containing 5% water is known as :
126. Which of the following is an endemic species?

(a) Nicobar pigeon


(SSC CGL 1st Sit. 2013)
(b) Horn bill eer (SSC CGL 1st Sit. 2013)
(a) Absolute alcohol (b) Dilute alcohol
(c) Indian Rhino (d) Pink head duck
127. The natural disaster in which carbon-di-oxide suddenly
(c) Power alcohol
ing (d) Rectified spirit
140. The hormone used as an oral contraceptive is :
erupts from a deep lake water is known as __________.
(SSC CGL 1st Sit. 2013) (a) Aldesterone
.ne
(SSC CGL 1st Sit. 2013)
(b) Cortisone
(a) Liminic
(c) Fluvial
(b) Lacaustrine
(d) Glacial
128. Blood group was discovered by :
(a) Alexander Fleming (b) William Harvey
(c) Progesterone
141. Expand the term IPCC:
(d) Testosterone

(a) International Pollution Control Council


(b) International Panel of Climate Control
t
(SSC CGL 1st Sit. 2013)

(c) Landsteiner (d) Pavlov (c) Interim Panel of Climate Change


129. Pick out the correct match : (SSC CGL 1st Sit. 2013) (d) Intergovernmental Panel on Climate Change
(a) Egg yolk Protein and Fat 142. Acceptable "Noise Pollution Level" in India range between:
(b) Fleshy foods Calcium and Protein (SSC CGL 1st Sit. 2013)
(c) Fish Starch and Vitamin (a) 10-15 dec (b) 16-35 dec
(d) Milk Fibre and Minerals (c) 40-45 dec (d) 70-100 dec
130. Blood is a : (SSC CGL 1st Sit. 2013) 143. Endosulfan spray on cashew crop resulted in the pollution
(a) reproductive tissue (b) connective tissue to the tune of tragedy in : (SSC CGL 1st Sit. 2013)
(c) epithelial tissue (d) muscular tissue (a) Tamil Nadu (b) Kerala
131. DPT vaccine is administered to prevent diseases like : (c) Andhra Pradesh (d) Karnataka
(SSC CGL 1st Sit. 2013) 144. Hypothensmia occurs due to loss of excessive heat from
(a) Diphtheria, Pertussis and Typhoid body due to sudden low body temperature in :
(b) Diphtheria, Pertussis and Tetanus (SSC CGL 1st Sit. 2013)
(c) Dengue, Pertussis and Typhoid (a) Snakes (b) Frogs
(d) Dengue, Polio and Tetanus (c) Human beings (d) Lizards

Downloded From : www.EasyEngineering.net


Downloded From : www.EasyEngineering.net

General Science 61
145. Solids which conduct electricity at higher temperature but 157. Of the following man-made disasters, which is socially
not at lower temperature are called induced? (SSC CGL 2nd Sit. 2013)
(SSC CGL 2nd Sit. 2013) (a) Debris Avalanche (b) Salt Water Intrusion
(a) super-conductor (d) metallic-conductor (c) Arson (d) Ozone depletion
(c) semi-conductor (d) insulator 158. Which one of the following endocrine gland is situated in
146. Which one of the following has greatest mass? the neck? (SSC CGL 2nd Sit. 2013)
(SSC CGL 2nd Sit. 2013) (a) Pancreas (b) Thyroid
(a) electron (d) proton (c) Pituitary (d) Adrenals
(c) neutron (d) hydrogen nucleus 159. The seat of intelligence is situated in the
147. A television channel is characterised by (SSC CGL 2nd Sit. 2013)
(SSC CGL 2nd Sit. 2013) (a) cerebrum (b) cerebellum
(a) frequency of transmitted signal (c) medulla (d) thalamus
(b) velocity of transmitted signal 160. What is the Normal Blood Volume in human adult?
(c) physical dimension of television screen (SSC CGL 2nd Sit. 2013)
(d) size of picture tube (a) One litre (b) Three litres
148. The density of water is 1 g/cc. This is strictly valid at (c) Five litres (d) Seven litres
(SSC CGL 2nd Sit. 2013) 161. The fasting blood glucose level in adults in mg/100 ml is

ww
(a) 0°C
(c) 25°C
(b) 4°C
(d) 100°C (a) 200 (b) 160
(SSC CGL 2nd Sit. 2013)

w.E
149. The process of photosynthesis involves conversion of
(SSC CGL 2nd Sit. 2013)
(a) chemical energy into radiant energy
(c) 100
162. Entomology is the study of
(a) Birds
(d) 60
(SSC CGL 2nd Sit. 2013)
(b) Insects

(c) solar energy into chemical energy


(d) mechanical energy into solar energy
asy
(b) chemical energy into mechanical energy (c) Fossils (d) Fungi
163. Exobiology is a science that deals with
(SSC CGL 2nd Sit. 2013)

is called. En
150. A colloidal system in which a liquid is dispersed in a liquid
(SSC CGL 2nd Sit. 2013)
(a) extinct forms
(b) life in other planets
(a) gel
(c) sol
(b) emulsion
(d) precipitate gin (c) life in the outer space
(d) life in marine habitat
151. The antiseptic compound present in dettol is
(SSC CGL 2nd Sit. 2013) eer
164. In radio-communication, the signals emitted by transmittin g
antenna are reflected on
(a) stratosphere
(SSC CGL 2nd Sit. 2013)
(b) ozonosphere
(a) Iodine
(c) Biothional
(b) Enloroxylenol
(d) Cresol
152. Genomic (DNA) studies in camel have been completed
(c) ionosphere
ing (d) troposphere
165. Tiny marine animals which constitute limestone skeletons
recently by the scientists of
(a) South Africa (b) India
(SSC CGL 2nd Sit. 2013)
are called
(a) Coral reefs
.ne
(SSC CGL 1st Sit. 2013)
(b) Diatoms
(c) China (d) Pakistan
153. Air quality depicting PM 2.5 is more hazardous to
(SSC CGL 2nd Sit. 2013)
(c) Clamitomonous

far-red region of light?


(a) Phytochrome
(d) Foraminifera

t
166. Which of the following plant pigments absorbs in red and
(SSC CGL 1st Sit. 2013)
(b) Cryptochrome
(a) Archaeological Monuments (c) Carotenoide (d) Chlorophyll
(b) National Parks 167. The process through which excess of light energy is
(c) Botanical Gardens dissipated in photosynthesis is known as
(d) Old Men and Women (SSC CGL 1st Sit. 2013)
154. Piped Natural Gas (PNG) is used for (a) Photolysis (b) Photophosphorylation
(SSC CGL 2nd Sit. 2013) (c) Quenching (d) Scavenging
(a) Mining (b) Welding 168. AIDS virus destroys (SSC CGL 1st Sit. 2013)
(c) Anaesthesia (d) Cooking (a) Neutrophils (b) Basophils
155. Greenpark Stadium is in (SSC CGL 2nd Sit. 2013) (c) Lymphocytes (d) Monocytes
(a) Bengaluru (b) Dehradun 169. The device used for measuring the wavelength of X-rays is
(c) Chandigarh (d) Kanpur (SSC CGL 1st Sit. 2013)
156. Which of the following is an endangered species? (a) Bragg Spectrometer
(SSC CGL 2nd Sit. 2013) (b) Mass Spectrometer
(a) Black buck (b) Blue sheep (c) G. M. Counter
(c) Gangetic dolphin (d) Mithun (d) Cyclotron

Downloded From : www.EasyEngineering.net


Downloded From : www.EasyEngineering.net

62 General Science
170. Alpha particle is the nucleus of an atom of 183. Reduction of nitrates to ammonia can be achieved through
(SSC CGL 1st Sit. 2013) one of the following methods : (SSC CGL 1st Sit. 2013)
(a) Helium (b) Oxygen (a) in alkaline medium using Devarda’s alloy.
(c) Lithium (d) Hydrogen (b) in neutral medium using Devarda’s alloy.
171. Teeth and Bones acquire strength and rigidity from (c) in acidic medium using Devarda’s alloy.
(SSC CGL 1st Sit. 2013) (d) in neutral medium using Cupric oxide.
(a) Chlorine (b) Sodium 184. Grits of sewage are removed in (SSC CGL 1st Sit. 2013)
(c) Calcium (d) Fluorine (a) Grit chamber (b) Detritus tank
172. The type of tail found in Shark is (c) Skimming tank (d) Trickling filter
(SSC CGL 1st Sit. 2013) 185. The most affected sulphur containing amino acid by PAN is
(SSC CGL 1st Sit. 2013)
(a) Heterocercal (b) Diphycercal
(a) Cysteine (b) Methonine
(c) Protocercal (d) Homocercal
(c) Proline (d) Globuline
173. The Sigmoid Colon is part of (SSC CGL 1st Sit. 2013)
186. Which of the following has zero electron affinity ?
(a) Large Intestine (b) Ileum
(SSC CGL 1st Sit. 2013)
(c) Small Intestine (d) Anal Canal (a) Oxygen (b) Fluorine
174. A good conductor while carrying current is (c) Nitrogen (d) Neon
(SSC CGL 1st Sit. 2013)

ww
(a) positively charged
(b) electrically neutral
187. The human body’s largest blood vessel is

(a) Pulmonary artery (b) Aorta


(SSC CGL 1st Sit. 2013)

w.E
(c) alternately charged positive and negative
(d) negatively charged
175. Coating of solid waste with imperviouos material is known
(c) Renal artery (d) Coronary artery
188. In human body, which one of the following hormones regulates
blood calcium and phosphate ? (SSC CGL 1st Sit. 2013)
as
(a) Landfill
(c) Encapsulation
asy
(SSC CGL 1st Sit. 2013)
(b) Capping
(d) Chemical fixation
(a) Glucagon
(b) Growth hormone
(c) Parathyroid hormone
176. Ultraviolet rays can be used in water treatment as
En
(SSC CGL 1st Sit. 2013)
(d) Thyroxine
189. How do most insects respire ? (SSC CGL 1st Sit. 2013)
(a) Hydrolyser
(c) Flocculator
(b) Disinfectant
(d) Precipitator gin (a) Through skin (b) Through gills
(c) By tracheal system (d) By lungs
177. Thiamidine dimer formation in DNA is caused by
(SSC CGL 1st Sit. 2013) eer
190. In nuclear reactions, there is conservation of

(a) mass only


(SSC CGL 1st Sit. 2013)
(b) momentum only
(a) b and g-rays
(c) IR-rays
178. Silicone is a polymer of
(b) UV-rays
(d) X-rays
(SSC CGL 1st Sit. 2013)
(c) energy only
ing (d) mass, energy and momentum
191. When a particle and an antiparticle come in contact with

(a) Dialkyl dichloro silane


(b) Silane
each other, they

.ne
(SSC CGL 1st Sit. 2013)
(a) repell each other (b) annihilate each other
(c) go undisturbed (d) spin about a common axis
(c) Tetraalkyl silane
(d) Silicon tetrachloride
179. Which is a natural colloid? (SSC CGL 1st Sit. 2013)
192. Photoelectric effect is
(a) an instantaneous process
(b) delayed process
t
(SSC CGL 1st Sit. 2013)

(a) Cane-sugar (b) Blood (c) emission of protons


(c) Sodium chloride (d) Urea (d) emission of neutrons
180. Which one of the following does not contain Silver? 193. For a particle moving with a constant speed along a straight
(SSC CGL 1st Sit. 2013) line PQ, the hodograph is (SSC CGL 1st Sit. 2013)
(a) German Silver (b) Horn Silver (a) a straight line parallel to PQ
(c) Ruby Silver (d) Lunar Caustic (b) a straight line perpendicular to PQ
181. The presence of Cobalt in Vitamin B12 was established for (c) a point
(d) a circle
the first time by (SSC CGL 1st Sit. 2013)
194. Aluminium is obtained by the electrolysis of pure Al2O3
(a) Borax-Bead test (b) Sodium Nitroprusside test
dissolved in (SSC CGL 1st Sit. 2013)
(c) Hydrolysis test (d) Spectroscopy
(a) Bauxite (b) Cryolite
182. Which bacterial strain developed from natural isolates by (c) Feldspar (d) Alumina
genetic manipulations can be used for treating oil spills? 195. Complete hydrolysis of cellulose gives
(SSC CGL 1st Sit. 2013) (SSC CGL 1st Sit. 2013)
(a) Clostridium (b) Nitrosomonas (a) D-fructose (b) L-glucose
(c) Pseudomonas (d) Agrobacterium (c) D-glucose (d) L-fructose

Downloded From : www.EasyEngineering.net


Downloded From : www.EasyEngineering.net

General Science 63
196. Each body segment of Earthworm is called 209. Rutherford's scattering experiment proved the presence of
(SSC CGL 1st Sit. 2013) (SSC CGL 1st Sit. 2013)
(a) Proglottid (b) Metamere (a) atoms in all matter
(c) Scolex (d) Rostellum (b) electrons in atoms
197. The pollutants which move downward with percolating (c) neutrons in atoms
ground water are called (SSC CGL 1st Sit. 2013) (d) nucleus in atoms
(a) Leachates (b) Pollutates 210. When a metal is heated in a flame, the electrons absorb energy
(c) Earthites (d) Percolates and jump to higher energy state. On coming back to the
198. Lungs are located in the (SSC CGL 1st Sit. 2013) lower energy state, they emit light, which we can observe in
(a) abdominal cavity (b) pericardial cavity (SSC CGL 1st Sit. 2013)
(c) peritoneal cavity (d) pleural cavity (a) Raman spectra (b) Absorption spectra
199. Which one of the following is the ideal food for newborn (c) Emission spectra (d) Fluorescence
babies? (SSC CGL 1st Sit. 2013) 211. Blood pressure may be increased by the excessive secretion
(a) Water (b) Sugar of (SSC CGL 1st Sit. 2013)
(c) Honey (d) Milk (a) Thyroxine (b) Testosterone
200. Transcription means the synthesis of (c) Estradiol (d) Estrol
(SSC CGL 1st Sit. 2013) 212. The Concept of 'Green House Gases" was postulated by

ww
(a) Lipids
(c) DNA
(b) Protein
(d) RNA
201. Hydrochloric acid is secreted by the cells lining the
(a) Joseph Furier
(SSC CGL 1st Sit. 2013)
(b) Abdul Kalam
(c) M. S. Swaminathan (d) Richael Carlson

(a) Oral cavity


(c) Ileum w.E (SSC CGL 1st Sit. 2013)
(b) Stomach
(d) Colon
213. "Bhopal gas tragedy" 1984 is related to

(a) Aluminium Phosphide


(SSC CGL 1st Sit. 2013)

202. Emulsification is
(a) breaking fats into small globules
(b) digestion of fats asy
(SSC CGL 1st Sit. 2013) (b) Methyl bromide
(c) Methyl isocyanate
(c) absorption of fats
(d) storage of fats En (d) Carbon dioxide
214. The Particulate Matter (PM–10) exhaled from the polluted
atmosphere is often filtered out during the process of
203. Taxonomy is a science that deals with

(a) Morphology
(SSC CGL 1st Sit. 2013)
(b) Anatomy
gin (a) Coughing
(SSC CGL 1st Sit. 2013)
(b) Sneezing
(c) Classification (d) Economic uses
204. Which of the following is responsible for the working of eer
(c) A and B (d) Urination
215. The animal who can consume more salt among the followin
(SSC CGL 1st Sit. 2013)
g
Newton's colour disc experiment? (SSC CGL 1st Sit. 2013)
(a) Formation of pure spectra
is
(a) Sheep
(c) Donkey ing(b) Camel
(d) Dog
(b) Formation of impure spectra
(c) Persistence of vision
(d) Principle of complementary colour
216. Cactus is referred to as
(a) Hydrophyte .ne
(b) Mesophyte
(SSC CGL 2014)

205. The dimension MLT–2 corresponds to

(a) force
(c) acceleration
(SSC CGL 1st Sit. 2013)
(b) work done
(d) velocity
(c) Xerophyte

(a) Thorium
(d) Epiphyte

(b) Geothermal heat


t
217. Which of the following is not a renewable resource?
(SSC CGL 2014)

206. Who is the founder of quantum theory of radiation? (c) Tidal power (d) Radiant energy
(SSC CGL 1st Sit. 2013) 218. How many neck canal cells are found in the archegonium of
(a) Einstein (b) Bohr a fern? (SSC CGL 2014)
(c) Plank (d) S.N. Bose (a) One (b) Two
207. Fiber optics cable used in communication, works on the (c) Three (d) Four
principle of (SSC CGL 1st Sit. 2013) 219. Which angiosperm is vesselless? (SSC CGL 2014)
(a) regular reflection of light (a) Hydrilla (b) Trochodendron
(b) diffuse reflection of light (c) Maize (d) Wheat
(c) refraction of light 220. Who was the first child born after operative procedure?
(d) total internal reflection of light (SSC CGL 2014)
208. The outer skin most of the crustaceans are made up of a (a) Caesar (b) Huxley
carbohydrate. This carbohydrate is (c) William (d) Pasteur
(SSC CGL 1st Sit. 2013) 221. Myrmecology is study of (SSC CGL 2014)
(a) cellulose (b) galactose (a) Insects (b) Ants
(c) chitin (d) starch (c) Crustaceans (d) Arthropods

Downloded From : www.EasyEngineering.net


Downloded From : www.EasyEngineering.net

64 General Science
222. HIV often changes is shapes due to the presence of an 234. Which of the following Genetically Modified vegetable is
enzyme called (SSC CGL 2014) recently being made available in Indian market?
(a) Reverse Transcriptase (SSC CGL 2014)
(b) Enterokinase (a) Carrot (b) Radish
(c) Nucleotidase (c) Brinjal (d) Potato
(d) Nucleoditase 235. Electric current is measured using which of the following
223. Fleming's right hand rule is used to find the direction of the instrument ? (SSC CGL 1st Sit. 2015)
(SSC CGL 2014) (a) Voltmeter (b) Anemometer
(a) Alternate current (b) Direct current (c) Wattmeter (d) Ammeter
(c) Induced current (d) Actual current 236. Photoperiodisrn affects (SSC CGL 1st Sit. 2015)
(a) Flowering (b) Vegetative growth
224. The unit of electrical power is (SSC CGL 2014)
(c) Fruiting (d) All of these
(a) Volt (b) Watt
237. Match the following : (SSC CGL 1st Sit. 2015)
(c) Kilowatt hour (d) Ampere
I II
225. The resistance of the human body (dry condition) is of the
A. Ascorbic acid 1. Photosynthetic
order of (SSC CGL 2014)
pigment
(a) 101 Ohm (b) 102 Ohm B. Chlorophyll 2. Quencher
(c) 103 Ohm (d) 104 Ohm

ww
226. Certain substances loose their electrical resistance
completely at super low temperature. Such substances are
C. Carotenoid
D. Superoxide
dismutase
3. Enzyme
4. Vitamin–C

called

(c) dielectrics
w.E
(a) super conductors(b) semi conductors
(SSC CGL 2014)

(d) perfect conductors


(a) 4
(b) 2
A B
2
4
C
1
1
D
3
3

(a) Memory
asy
227. The section of the CPU that selects, interprets and monitors
the execution of program instructions is (SSC CGL 2014)
(b) Register unit
(c) 4
(d) 4
1
1
3
2
2
3
(SSC CGL 1st Sit. 2015)
(c) Control unit
228. Brass contains
(d) ALU
En
(SSC CGL 2014)
238. Allantois of Embryo helps in
(a) respiration
(c) protection
(b) excretion
(d) digestion
(a) Copper and Zinc
(b) Copper and Tin gin
239. Which one of the following animals belongs to mollusca ?
(SSC CGL 1st Sit. 2015)
(c) Copper and Silver
(d) Copper and Nickel
(a) Hare
(c) Hyla
eer (b) Hydra
(d) Haliotis
229. Which is the purest commercial form of iron?
(a) Pig iron (b) Steel
(c) Stainless steel (d) Wrought iron
(SSC CGL 2014)
(a) Mitochondria ing
240. Outside the nucleus DNA is found in
(SSC CGL 1st Sit. 2015)

230. In galvanization, iron is coated with


(a) Copper (b) Zinc
(SSC CGL 2014) (b) Ribosome
(c) Endoplasmic reticulum .ne
(c) Tin (d) Nicked
231. Which one of the following is also known as solution?
(a) A compound (SSC CGL 2014)
(d) Golgi bodies

(a) delicious in taste


t
241. Animal protein is called first class protein because it is
(SSC CGL 1st Sit. 2015)
(b) A homogeneous mixture
(b) cheaper in the market
(c) A heterogeneous mixture (c) rich in essential amino acids
(d) A suspension (d) easily digestible
232. The cells which are closely associated and interacting with 242. It is easy to burst a gas filled balloon with a needle than with
guard cells are (SSC CGL 2014) a nail. It is because (SSC CGL 1st Sit. 2015)
(a) Transfusion tissue (a) nail exerts more pressure than needle on the balloon
(b) Complementary cells (b) needle exerts more pressure than nail on the balloon
(c) Subsidiary cells (c) gas is reactive with the needle
(d) Hypodermal cells (d) nail is more longer than needle
233. Conversion of starch to sugar is essential for 243. The velocity of sound in moist air is more than in dry air
(SSC CGL 2014) because the moist air has (SSC CGL 1st Sit. 2015)
(a) Stomatal opening (a) less pressure than dry air
(b) Stomatal closing (b) more pressure than dry air
(c) Stomatal formation (c) more density than dry air
(d) Stomatal growth (d) less density than dry air

Downloded From : www.EasyEngineering.net


Downloded From : www.EasyEngineering.net

General Science 65
244. X–rays can be used (SSC CGL 1st Sit. 2015) 256. Reverse transcription was discovered by :
(a) to detect heart diseases. (SSC CGL 1st Sit. 2015)
(b) to detect defects in precious stones and diamonds. (a) Beadle and Tatum
(c) to detect gold under the earth. (b) Watson and Crick
(d) for cutting and welding of metals. (c) Temin and Baltimore
245. Ice is packed in saw dust because (SSC CGL 1st Sit. 2015) (d) Har Govind Khorana
(a) saw dust is poor conductor of heat.
257. Burns caused by steam are much more severe than those
(b) saw dust is a good conductor of heat.
caused by boiling water because: (SSC CGL 1st Sit. 2015)
(c) saw dust does not stick to the ice.
(d) saw dust will not get melted easily. (a) Steam pierces through the pores of body quickly
246. What happens when a drop of glycerol is added to crushed (b) Temperature of steam is higher
KMnO4 spread of a paper ? (SSC CGL 1st Sit. 2015) (c) Steam is gas and engulfs the body quickly
(a) There is a violent explosion (d) Steam has latent heat
(b) There is no reaction 258. Which among the following is the sweetest sugar?
(c) The paper ignites (SSC CGL 1st Sit. 2015)
(d) There is a crackling sound. (a) lactose (b) maltose
247. Most commonly used bleaching agent is (c) glucose (d) fructose
(SSC CGL 1st Sit. 2015)

ww
(a) Alcohol
(c) Chlorine
(b) Carbon dioxide
(d) Sodium chloride
259. Ultra purification of a metal is done by :

(a) smelting
(SSC CGL 1st Sit. 2015)
(b) leaching

(a) a–Ray w.E


248. The least penetrating power ray is
(SSC CGL 1st Sit. 2015)
(b) b–Ray
(c) zone melting (d) slagging
260. Microbial degradation of nitrates into atmospheric nitrogen
is known as : (SSC CGL 1st Sit. 2015)
(c) g–Ray (d) X–Ray

asy
249. Hydrogen peroxide is an effective sterilizing agent. Which
one of the following product results when it readily loses
(a) Ammonification
(c) Putrefacation
(b) Denitrification
(d) Nitrifcation
active oxygen ?
(a) Water (b) Hydrogen En
(SSC CGL 1st Sit. 2015) 261. The best milch breed in the world is :
(SSC CGL 1st Sit. 2015)
(c) Ozone (d) Nasant Hydrogen
250. The maximum fixation of solar energy is done by
(SSC CGL 1st Sit. 2015)
gin (a) Deoni
(c) Sindhi
(b) Holstein – Friesian
(d) Chittagong

(a) Bacteria
(c) Green plants
(b) Fungi
(d) Protozoa eer
262. Muddy water is treated with alum in purification process, ti
is termed as :
(a) absorption
(SSC CGL 1st Sit. 2015)
(b) adsorption
251. The term 'brown air' is used for (SSC CGL 1st Sit. 2015)
(a) Photochemical smog
(c) coagulation
ing(d) emulsification
263. An enzyme produced by HIV that allows the integration of
(b) Sulfurous smog
(c) Industrial smog
(d) Acid fumes
HIV DNA into the host cell's DNA is:
.ne
(SSC CGL 1st Sit. 2015)
252. Peroxyacetyl nitrate is a
(a) Plant hormone
(b) Vitamin
(c) Secondary pollutant
(SSC CGL 1st Sit. 2015) (a) DNA gyrase
(c) Integrase
(b) Ligase
(d) Helicase
264. The two specific heats of gases are related by :
t
(SSC CGL 1st Sit. 2015)
(d) Acidic dye (a) Cp / Cv = R (b) Cp – Cv = RJ
253. Which of the following particles has the dual nature of (c) Cp – Cv = R/J (d) Cp + Cv = RJ
particle–wave ? (SSC CGL 1st Sit. 2015) 265. The antibiotic penicillin is obtained from which of the
(a) Neutron (b) Electron following ? (SSC CGL 1st Sit. 2016)
(c) Meson (d) Proton (a) synthetic process (b) a bacterium
254. The metal ion present in vitamin B12 is : (c) fungus (d) virus infected cells
(SSC CGL 1st Sit. 2015) 266. Which of the following is indicated by the colour of a star ?
(a) nickel (b) cobalt (SSC CGL 1st Sit. 2016)
(c) iron (d) zinc
(a) weight (b) distance
255. Who of the following has given the term rhizosphere :
(c) temperature (d) size
(SSC CGL 1st Sit. 2015)
(a) Alexopolus 267. Atomic number of an atom gives the number of which of the
(b) Garret following ? (SSC CGL 1st Sit. 2016)
(c) Hiltner (a) electrons (b) protons
(d) None of the given options (c) neutrons (d) neutrons and protons

Downloded From : www.EasyEngineering.net


Downloded From : www.EasyEngineering.net

66 General Science
268. DPT vaccine is categorized as which of the following ? 279. Which of the following acts as best adsorbent?
(SSC CGL 1st Sit. 2016) (SSC CGL 1st Sit. 2016)
(a) Anti viral vaccine (a) Charcoal
(b) Anti protozoan vaccine (b) Activated Charcoal
(c) Anti rickettsial vaccine (c) Activated Coconut Charcoal
(d) Carbon black
(d) A combined vaccine
280. The maximum fixation of solar energy is done by
269. Which is the highest quality of hard coal?
(SSC CGL 1st Sit. 2016)
(SSC CGL 1st Sit. 2016) (a) Bacteria (b) Protozoa
(a) Anthracite (b) Bituminous (c) Fungi (d) Green plants
(c) Lignite (d) Peat 281. 'Parsec' is the unit measurement of
270. For which one of the following, ‘Diodes’ are generally used (SSC CGL 1st Sit. 2016)
for? (SSC CGL 1st Sit. 2016) (a) Density of stars
(a) Rectification (b) Amplification (b) Astronomical distance
(c) Modulation (d) Filtration (c) Brightness of heavenly bodies
271. An anemometer measures which of the following ? (d) Orbital velocity of giant stars
(SSC CGL 1st Sit. 2016) 282. At room temperature, the metal that remains liquid is:
(SSC CGL 1st Sit. 2016)

ww
(a) Speed of light
(b) Speed of wind
(c) Speed of water current
(a) Mercury
(c) Lead
(b) Platinum
(d) Zinc

w.E
(d) Speed of satellites
272. Which cell disorder in our body is responsible for colour
blindness? (SSC CGL 1st Sit. 2016)
283. Which is known as carbolic acid? (SSC CGL 1st Sit. 2016)
(a) Phenol
(c) Acetic acid
(b) Ethanol
(d) Oxalic acid

(a) WBC
(c) Rod Cell (d) Neuron asy
(b) Cone cell
284. Which one of the following substances is normally found in
urine?
(a) Blood proteins
(SSC CGL 1st Sit. 2016)
(b) Creatinine

En
273. Which one of the following forms the base of vegetable
fibres? (SSC CGL 1st Sit. 2016)
(c) Red blood cells (d) White blood cells
285. The thymus gland produces a hormone called
(a) Cellulose
(c) Fats
(b) Proteins
(d) Oils gin (a) thyroxine
(c) thyronine
(SSC CGL 1st Sit. 2016)
(b) thymosin
(d) calcitonin
274. A particle is thrown vertically upward. When it reaches the
highest point, it has __________.
(SSC CGL 1st Sit. 2016)
eer
286. Blood group AB has
(a) No antigen
(SSC CGL 1st Sit. 2016)

(a) a downward acceleration


(b) an upward acceleration
(b) No antibody
ing
(c) Neither antigen nor antibody

(c) a downward velocity


(d) a horizontal velocity
(d) Both antigen and antibody

.ne
287. Which vitamins are those, if taken in excess can be dangerous
as they are stored in the body? (SSC CGL 1st Sit. 2016)
275. During fermentation of sugar, the compound which is always
formed is
(a) Methyl Alcohol
(c) Acetic Acid
(SSC CGL 1st Sit. 2016)
(b) Ethyl Alcohol
(d) Ethylene
(a) B Complex
(c) B and C
(b) E and C
(d) A and D t
288. Atomic explosion is triggered by (SSC CGL 1st Sit. 2016)
(a) thermo nuclear reaction
276. The quality or tone of a musical sound produced by a (b) chemical reaction
stringed instrument depends on (SSC CGL 1st Sit. 2016) (c) controlled chain reaction
(a) frequency of vibration (d) uncontrolled chain reaction
289. The phenomenon of change in direction of light when it
(b) length of the strings in the instrument
passes from one medium to another is called
(c) Amplitude of vibration
(SSC CGL 1st Sit. 2016)
(d) waveform of the sound (a) Propagation (b) Reflection
277. Removal of carbon particles from air involves the principle (c) Refraction (d) Dispersion
of (SSC CGL 1st Sit. 2016) 290. When ice cubes are made, the entropy of water
(a) Precipitation (b) Filteration (SSC CGL 1st Sit. 2016)
(c) Electrophoresis (d) Sedimentation (a) does not change
278. Which light is least effective in photosynthesis? (b) decreases
(SSC CGL 1st Sit. 2016) (c) increases
(a) Blue light (b) Green light (d) may either increase or decrease depending on the
(c) Red light (d) Sunlight process used

Downloded From : www.EasyEngineering.net


Downloded From : www.EasyEngineering.net

General Science 67
291. Two vectors are said to be equal if 304. Which enzyme is obtained from Red gram ?
(SSC CGL 1st Sit. 2016) (SSC CHSL 2012)
(a) only their magnitudes are same (a) Zymase (b) Maltase
(b) only their directions are same (c) Diastase (d) Urease
(c) both magnitude and direction are same 305. A radioactive substance has a half-life of four months.
(d) magnitudes are same but directions are opposite Three-fourth of the substance will decay in:
292. The chemical component that is invariably found in all (SSC CHSL 2012)
viruses is (SSC CGL 1st Sit. 2016) (a) 8 months (b ) 12 months
(a) proteins (b) lipids (c) 4 months (d) 6 months
(c) DNA (d) RNA 306. Chiropterophily means: (SSC CHSL 2012)
293. Which is used as an Air pollution indicator? (a) production of flowers (b) pollination by wind
(SSC CGL 1st Sit. 2016) (c) pollination by bat (d) production of leaves
(a) Algae (b) Fungi 307. Which one among the following is a solid lubricant ?
(c) Bacteria (d) Lichens (SSC CHSL 2012)
294. Salination of soil is caused by (SSC CGL 1st Sit. 2016) (a) Germanium (b) Sulphur
(a) Pesticides (b) soil erosion (c) Graphite (d) Indium
(c) excess irrigation (d) crop rotation 308. Longest cell in human body is: (SSC CHSL 2012)

ww
295. Which enzyme digests proteins in the stomach?

(a) Trypsin (b) Pepsin


(SSC CGL 1st Sit. 2016)
(a) Blood cell
(c) Nerve cell
(b) Bone cell
(d) Muscle cell
309. The acid rain destroys the vegetation because it contains:

w.E
(c) Salivary amylase (d) Pancreatic canal
296. Fermentation is a type of _____ _______ process.
(SSC CGL 1st Sit. 2016)
(a) Ozone
(c) Sulphuric acid
(SSC CHSL 2012)
(b) Carbon monoxide
(d) Nitrates
(a) Aerobic Respiration
(b) Anaerobic Respiration
(c) Exothermic Reaction asy 310. Activated sludge treatment is called: (SSC CHSL 2012)
(a) Preliminary treatment
(c) Pre treatment
(b) Biological treatment
(d) Chemical treatment
(d) Transpiration
297. Which part of the plant is used as 'saffron'? En 311. Caustic soda is :
(a) deliquescent
(SSC CHSL 2012)
(b) oxidant

(a) Petals
(c) Style and Stigma
(SSC CHSL 2012)
(b) Stamens
(d) Sepals
gin (c) reductant

because:
(d) efflorescent
312. Red light is used in traffic signal for stopping the traffic
(SSC CHSL 2012)
298. Suspended colloidal particles in the water can be removed
by the process of : (SSC CHSL 2012) eer
(a) eye is more sensitive to red light.
(b) it is least scattered and hence can be easily notice d
(a) Filtration
(c) Absorption
(b) Adsorption
(d) Coagulation ing
from long distance.
(c) it is very pleasant to the eye.
299. Grave's disease is caused due to:
(a) hyperactivity of thyroid
(b) hypoactivity of thymus
(SSC CHSL 2012)
313. What is a neuron ?
(a) Basic unit of nervous system .ne
(d) it is visible even to longsighted people.
(SSC CHSL 2013)

(c) hypoactivity of thyroid


(d) hyperactivity of thymus
300. A white solid ‘A’ on heating gives off a gas which turns lime
water milky. The residue is yellow when hot but turns white
(b) Basic unit of energy
(c) Particle released during radioactivity
(d) The anti-particle of neutron
t
314. A cellulosic wall is found in the cells of (SSC CHSL 2013)
on cooling. The solid A is: (SSC CHSL 2012) (a) plants (b) animals
(a) Zinc Carbonate (b) Lead Sulphate (c) bacteria (d) fungi
(c) Lead Carbonate (d) Zinc Sulphate 315. The filtration unit of kidney is (SSC CHSL 2013)
301. Which of the following is the strongest coagulant ? (a) yellow fiber (b) axon
(SSC CHSL 2012) (c) nephron (d) neuron
(a) Zinc Chloride (b) Aluminium Chloride 316. The nutritive tissue in the seeds of higher plants is known
(c) Barium Chloride (d) Magnesium Sulphate as (SSC CHSL 2013)
302. The property which is seen in light wave but not in sound (a) nucellus (b) hypocotyl
wave is : (SSC CHSL 2012) (c) embryo (d) endosperm
(a) Diffraction (b) Refraction 317. Yeast is an important source of (SSC CHSL 2013)
(c) Polarization (d) Interference (a) protein (b) vitamin B
303. Iron and manganese are removed in water by the process (c) invertase (d) vitamin C
of : (SSC CHSL 2012) 318. Enzymes are (SSC CHSL 2013)
(a) Chlorination (b) Filtration (a) Lipids (b) Steroids
(c) Lime-soda treatment (d) Aeration (c) Carbohydrates (d) Proteins

Downloded From : www.EasyEngineering.net


Downloded From : www.EasyEngineering.net

68 General Science
319. Heating of a ore below its melting point in the absence of air 329. Which of the following is a Biological method of soil
is known as (SSC CHSL 2013) conservation ? (SSC CHSL 2014)
(a) Smelting (b) Refining (a) Contour farming (b) Contour terracing
(c) Calcination (d) Roasting (c) Gully control (d) Basin listing
320. The most electronegative element among the following is 330. Glucose is a type of (SSC CHSL 2014)
(SSC CHSL 2013) (a) Pentose sugar (b) Hexose sugar
(a) Oxygen (b) Fluorine (c) Tetrose sugar (d) Diose sugar
(c) Sodium (d) Chlorine 331. Number of mitochondria in bacterial cell is
321. Mark the compound which possesses ionic, covalent and (SSC CHSL 2014)
co-ordinate bonds. (SSC CHSL 2013) (a) one (b) two
(a) H2O (b) NH4C1 (c) many (d) zero
(c) SO3 (d) S02 332. Rainbow is formed due to (SSC CHSL 2014)
322. The depletion of Ozone layer is mainly due to (a) refraction and dispersion
(b) scattering and refraction
(SSC CHSL 2013)
(c) diffraction and refraction
(a) Chlorofluorocarbons
(d) refraction and reflection
(b) Volcanic eruptions
333. Golden view of sea shell is due to (SSC CHSL 2014)
(c) Aviation fuels

ww
(d) Radioactive rays
323. Match correctly the infectious agents given in List I with
(a) Diffraction
(c) Polarization
(b) Dispersion
(d) Reflection
334. An object covers distance which is directly proportional to

List I w.E
the diseases caused by them given in List II :

List II
(SSC CHSL 2013)
the square of the time. Its acceleration is (SSC CHSL 2014)
(a) increasing
(c) zero
(b) decreasing
(d) constant
a. Bacterium
b. Fungus
c. Protozoan
asy
1. Kala-azar
2. Tuberculosis
3 Influenza
335. If the horizontal range of a projectile is four times its maximum
height, the angle of projection is (SSC CHSL 2014)

d. Virus
(a) a-4, b-2, c-3, d-1
4. Ringworm
(b) a-1, b-2, c-4, d-3 En (a) 30°
æ1ö
sin -1 ç ÷
(b) 45°

-1 æ 1 ö
(d) tan ç ÷
(c) a-2, b-4, c-1, d-3
324. Matter waves are
(d) a-3, b-1, c-2, d-4
(SSC CHSL 2013) gin
336.
(c)
è4ø è 4ø
Which of the following metals has least melting point ?
(a) Longitudinal waves
(b) de Broglie waves (a) Gold
eer
(c) Mercury
(SSC CHSL 2014)
(b) Silver
(d) Copper
(c) Electromagnetic waves
(d) Transverse waves
325. When the milk is churned vigorously the cream from it is
337.
of vegetation is ing
The gas produced in marshy places due to decomposition
(SSC CHSL 2014)
separated out due to
(a) Centrifugal force
(SSC CHSL 2013)
(b) Centripetal force
(a) Carbon monoxide
(c) Sulphur dioxide
.ne
(b) Carbon dioxide
(d) Methane
(c) Gravitational force (d) Frictional force
326. Gas thermometers are more sensitive than the liquid
thermometers because the gases
(a) have high specific heat
(SSC CHSL 2013)
338.

339.
(a) stem
(c) leaves
(b) sripules
(d) buds
The smallest known prokaryotic organism is
t
In cactue, the spines are the modified (SSC CHSL 2014)

(SSC CHSL 2014)


(b) have large coefficient of expansion (a) Microcystis (b) Mycoplasma
(c) are lighter (c) Bacteria (d) Chlorella
(d) have low specific heat 340. According to your text, what can "be thought of as the
327. Which of the following is not caused by atmospheric genetic library that keep life going on Earth" ?
refraction of light ? (SSC CHSL 2013) (SSC CHSL 2014)
(a) Sun becoming visible two or three minutes before actual (a) A bio-engineering lab
sunrise (b) Human genes
(b) Sun appearing red at sunset (c) The human genome project
(c) Twinkling of stars at night (d) Biodiversity
(d) Sun appearing higher in the sky than it actually is 341. The boiling point of water decreases at higher altiudes is
328. From which part of Opium Plant we get morphine ? due to (SSC CHSL 2014)
(SSC CHSL 2014) (a) low tenperature
(a) Leaves (b) Stem (b) low atmospheric pressure
(c) high temperature
(c) Bark (d) Fruit coat
(d) high atmospheric pressure

Downloded From : www.EasyEngineering.net


Downloded From : www.EasyEngineering.net

General Science 69
342. The chemical name of "Hypo"commonly used in 354. Maximum oxygen is available from : (SSC CHSL 2015)
photography is (SSC CHSL 2014) (a) Green forests (b) Deserts
(a) Sodium thiosulphate (b) Silver nitrate (c) Grass lands (d) Phytoplanktons
(c) Sodium nitrate (d) Silver iodide 355. In a reaction of the type A + B ® C + D one could ensure it
343. Soldering of two metals is possible because of the property to be a first order reaction by : (SSC CHSL 2015)
of : (SSC CHSL 2015) (a) Increasing the concentration of a reactant
(a) Osmosis (b) Viscosity (b) Adding a catalyst
(c) Surface tension (d) Cohesion (c) Increasing the temperature
344. Stalactites & Stalagmites form due to the precipitation of : (d) Increasing the concentration of a product
(SSC CHSL 2015) 356. The amount of matter in a ball of steel is its :
(a) CaCl2 (b) MgCO3 (SSC CHSL 2015)
(c) MgCl2 (d) CaCO3 (a) Mass (b) Density
345. Which of the following is a form of sexual reproduction : (c) Volume (d) Weight
357. Transpiration increases in : (SSC CHSL 2015)
(SSC CHSL 2015)
(a) Hot, damp and windy condition
(a) Fission (b) Fragmentation
(b) Cool, damp and windy condition
(c) Budding (d) Harmaphroditism
(c) Cool, dry and still condition
346. In which region of electromagnetic spectrum does the Lyman
(d) Hot, dry and windy condition

ww
series of hydrogen atom lie ?

(a) x–ray
(SSC CHSL 2015)
(b) Ultraviolet
358. If xylem and phloem are arranged in the same radius, such a
vascular bundle is called :
(a) bicollateral
(SSC CHSL 2015)
(b) concentric
(c) Visible
w.E (d) Infrared
347. An electrochemcial cell which is used as a source of direct
electrical current at constant voltage under standard
(c) radial (d) collateral
359. Plank's constant has the dimensions of :
(SSC CHSL 2015)
conditions is called a :
(a) Power transistor asy
(SSC CHSL 2015) (a) linear momentum
(c) force
(b) angular momentum
(d) energy
(b) Battery
(c) Generator
(d) Uninterrupted power supply (UPS) En 360. The most abundant element by number in the living system
is :
(a) Hydrogen
(SSC CHSL 2015)
(b) Oxygen
348. A light wave is incident over a plane surface with velocity
X. After reflection the velocity becomes : gin (c) Carbon (d) Nitrogen
361. Which of the following phenomenon helps to conclude ttha

(a) x (b) 2x
(SSC CHSL 2015)
eer
light is a transverse wave ?
(a) diffraction
(SSC CHSL 2015)
(b) polarisation

(c)
x
4
(d)
x
2
(c) refraction

ing (d) interference


362. Among the following districts of Tamil Nadu, which district
is unfit for cultivation due to increased salinity :
349. The area reserved for the welfare of wild life is called
(SSC CHSL 2015) (a) Tiruchirapalli
.ne (SSC CHSL 2015)
(b) Negapattinam

350.
(a) Sanctuary
(c) Forest
(b) Botanical garden
(d) National pak
The gas dissolved in water that makes it basic is ?
(SSC CHSL 2015)
(c) Ramanathapuram

botanists.
(a) German
t
(d) Coimbatore
363. Natural system of classification was proposed by ____
(SSC CHSL 2015)
(b) Swedish
(a) ammonia (b) hydrogen (c) British (d) Indian
(c) sulphur dioxide (d) carbon dioxide 364. Minamata disease is caused by pollution of water by :
351. Yellow complexion, Medium stature, Oblique eye with an (SSC CHSL 2015)
epicanthic fold is the characteristic feature of : (a) tin (b) methyl isocyanate
(SSC CHSL 2015) (c) mercury (d) lead
365. The "King of Metals" is : (SSC CHSL 2015)
(a) Australoids (b) Negroid
(a) Silver (b) Iron
(c) Mengoloid (d) Cancosoid
(c) Aluminium (d) Gold
352. Chromosome designation of Turner syndrome is :
366. Amino acids are required for the synthesis of :
(SSC CHSL 2015)
(SSC CHSL 2015)
(a) 44A + XO (b) 44A + XXY (a) Lipids (b) Proteins
(c) 44A + XXX (d) 44A + XYY (c) Carbohydrates (d) Alkaloids
353. Distant objects are visible as a little out of focus in this 367. The most suitable unit for expressing nuclear radius is:
condition : (SSC CHSL 2015) (SSC CHSL 2015)
(a) hypermetropia (b) presbiopia (a) fermi (b) angstrom
(c) astigmatism (d) myopia (c) micron (d) nanometre

Downloded From : www.EasyEngineering.net


Downloded From : www.EasyEngineering.net

70 General Science
368. Blowing Air with open pipe is an example of : 381. The type of mirror used in automobiles to see the traffic on
(SSC CHSL 2015) the rear side is (SSC Multitasking 2013)
(a) Isochoric Process (b) Isobaric process (a) Convex (b) Concave
(c) Adiabatic process (d) Isothermal process (c) Plano-Convex (d) Plane
369. Christmas factor is involved in : (SSC CHSL 2015) 382. The hottest part of the gas flame is known as
(a) Excretion (b) Digestion (SSC Multitasking 2013)
(c) Respiration (d) Blood Coagulation (a) dark zone (b) blue zone
370. Who is the author of the book “Romancing with Life ”. (c) non-luminous zone (d) luminous zone
(SSC CHSL 2015) 383. Which of the following radiations has the least wavelength?
(a) Dev Anand (b) Shashi Tharoor (SSC Multitasking 2013)
(c) Bill Clinton (d) Kapil Dev (a) b-rays (b) X-rays
371. Which one out of the following helps in burning (c) a-rays (b) g-rays
(SSC Multitasking 2013) 384. The earth is a (SSC Multitasking 2013)
(a) Carbon dioxide (b) Oxygen (a) bad absorber and bad radiator of heat
(c) Carbon monoxide (b) Nitrogen (b) good reflector of heat
372. In organic compounds, nitrogen is estimated by (c) non-absorber of heat
(SSC Multitasking 2013) (d) good absorber and good radiator of heat

ww
(a) Dumas’ method
(b) Carius method
(c) Victor-Meyer’s method
385. BCG vaccination is given at the age of

(a) Within 15 days


(SSC Multitasking 2013)
(b) 2 – 3 years
(d) Liebig’s method
w.E
373. Master copy of genetic information is
(SSC Multitasking 2013)
(c) 10 years (d) Newborn
386. Which of the following atmospheric gases constitute
greenhouse gases? (SSC Multitasking 2013)
(a) DNA
(c) r-RNA asy
(b) Nucleus
(d) m-RNA
1. Carbon dioxide
3. Nitrous oxide
2. Nitrogen
4. Water vapour
Select the correct answer using the codes given below.
374. Contraceptive pills in the market contain

(a) Steroid-hormones En
(SSC Multitasking 2013)
(a) 1, 2 and 4
(c) 1 and 4
(b) 1, 3 and 4
(d) 1 and 3
(b) Inorganic compounds
(c) Herbicides gin
387. Hydrogen bomb is based on the principle of

(a) Double decomposition


(SSC Multitasking 2014)
(d) Antibiotics
375. Nematocysts are present in (SSC Multitasking 2013)
eer
(b) Artificial radioactivity
(c) Nuclear fission
(a) Sea anemone
(c) Ascaris
(b) Starfish
(d) Centipede
376. Which of the following micro-organisms is used in milk
(d) Nuclear fusion
ing
388. The commonly used safety fuse-wire is made of
curdling?
(a) Lactobacillus
(SSC Multitasking 2013)
(b) Acctobacter
(a) an alloy of Nickel and Lead
(b) an alloy of Tin and Lead .ne
(SSC Multitasking 2014)

(c) Leuconostoc

(a) K
(d) Bacillus
377. Which of the following is present in Chlorophyll molecule?
(SSC Multitasking 2013)
(b) Mn
(c) an alloy of Tin and Nickel
(d) an alloy of Lead and Iron t
389. At what temperature is the density of water the maximum?
(SSC Multitasking 2014)
(c) Mn (d) Fe (a) 2°C (b) 4°C
378. When a body falls from an aeroplane, there is increase in its (c) 0°C (d) 1°C
(SSC Multitasking 2013) 390. The linear expansion of a solid rod is independent of its
(a) Potential energy (b) Kinetic energy (SSC Multitasking 2014)
(c) Mass (d) Acceleration (a) increase in temperature
379. What does ‘Ozone Layer’ absorb? (b) time of heat flow
(SSC Multitasking 2013) (c) initial length
(a) g-rays (b) Infrared rays (d) material
(c) Ultraviolet rays (d) X-rays 391. Cathode rays when obstructed by metal cause emission of
380. In a water lifting electric pump, we convert (SSC Multitasking 2014)
(SSC Multitasking 2013) (a) g- rays (b) X-rays
(a) Electrical energy into Kinetic energy (c) a-rays (d) b-rays
(b) Electrical energy into Potential energy 392. Who is the father of biology? (SSC Multitasking 2014)
(c) Kinetic energy into Electrical energy (a) Lamarck (b) Robert Hooke
(d) Kinetic energy into Potential energy (c) Aristotle (d) Pasteur

Downloded From : www.EasyEngineering.net


Downloded From : www.EasyEngineering.net

General Science 71
393. The smallest unit of classification is 408. Anticoagulants are not present in
(SSC Multitasking 2014) (SSC Sub. Ins. 2012)
(a) Species (b) Genus (a) Mosquito (b) Bed bug
(c) Family (d) Order (c) Leech (d) Wasp
394. Arenchyma is present in (SSC Multitasking 2014) 409. Vegetation is effective in absorbing (SSC Sub. Ins. 2012)
(a) Banana stem (b) Palm stem (a) Pollutant gases (b) Polluted water
(c) Aquatic plants (d) Xerophytic plants (c) High frequency sound (d) Pollutant metals
395. The deficiency of vitamin A causes 410. When a person cries, there is a watery discharge from the
(SSC Multitasking 2014) nose due to activation of (SSC Sub. Ins. 2012)
(a) Scurvy (b) Night blindness (a) Salivary gland (b) Lachrymal gland
(c) Beri-Beri (d) Dermatitis (c) Thyroid gland (d) Endocrine gland
396. Clove is a (SSC Multitasking 2014) 411. The fundamental role of root hairs in plants is
(a) Dried flower bud (b) Flower (SSC Sub. Ins. 2012)
(c) Fruit (d) Seed (a) to protect the young root from damage by coarse soild
397. On heating, Gypsum loses certain percentage of its water particles
content and becomes (SSC Multitasking 2014) (b) to protect the root from soil microbes
(a) Chalk (b) Calcium sulphate (c) to absorb water and mineral salts from the soil
(d) to bind soil particles to the root for firm fixation of the

ww
(c) Plaster of Paris (d) a pearl
398. The name of the scientist who discovered neutron is
(SSC Multitasking 2014) 412.
plant
Catch crops are (SSC Sub. Ins. 2012)
(a) crops palnted to attract certain insect pests to be
(a) Fermi
(c) Chadwick
w.E (b) Rutherford
(d) Bohr
399. The bubbles in Champagne and Soda are
destroyed
(b) crops planted to attract certain useful insects to be
used for biological control of pests
(a) Nitrogen
(c) Carbon dioxide asy
(SSC Multitasking 2014)
(b) Oxygen
(d) Hydrogen
(c) crops to be cut and fed green to the cattle
(d) substitute crops planted after the regular crop has failed.
400. Gobar gas contains mainly
(a) Butane En
(SSC Multitasking 2014)
(b) Carbon monoxide
413. The pigment that protects plants form UV damage is
(SSC Sub. Ins. 2012)
(c) Methane (d) Carbon dioxide
401. "Carbon Credit" is a term associated with the
(SSC Multitasking 2014)
gin
414.
(a) Chlorophyll
(c) Phycocyanin
(b) Xanthophyll
(d) Carotenoids
A bioenergy source obtained by fermentation to supplemen t
(a) Global deforestation
(b) Offshore banking eer
fossil fuel petrol is
(a) Kerosene
(SSC Sub. Ins. 2012)
(b) Ethanol
(c) Protection of environment
(d) Deforestation in India 415.
(c) Diesel

ing (d) Methane


The substance that causes the worst air pollution is
(SSC Sub. Ins. 2012)
402. The green colour of plant leaves is due to

(a) Protein
(SSC Multitasking 2014)
(b) Chlorophyll
(a) Smoke
(c) Carbon dioxide .ne
(b) Sulphur dioxide
(d) Carbon monoxide
(c) Cellulose (d) Starch
403. Temperature is measured by the instrument called

(a) Voltmeter
(SSC Multitasking 2014)
(b) Calorimeter
416. A liquid is said to boil when its
(a) vapour pressure equals the surrounding pressure
(b) vapour pressure vanishes to zero
(c) vapour pressure is greater than the surrounding
t
(SSC Sub. Ins. 2012)

(c) Thermometer (d) Ammeter pressure


404. The pH of pure water is (SSC Multitasking 2014) (d) vapour pressure is less than the surrounding pressure
(a) Seven (b) Foruteen 417. Which is not correct regarding covalent compounds?
(c) Zero (d) One (SSC Sub. Ins. 2012)
405. The process of separation of pure water from impurities is (a) The reaction is slow
called (SSC Multitasking 2014) (b) The reaction is fast
(a) Fractional crystallisation (b) Decantation (c) Compounds are usually liquids and gases
(c) Distillation (d) Sublimation (d) Boiling points and melting points are low
406. H2SO4 cannot be used as (SSC Multitasking 2014) 418. Choose the correct statement
(a) Disinfectant (b) Food preservative (SSC Sub. Ins. 2012)
(c) Drying agent (d) Dehydrating agent (a) The components of a mixture cannot be separated
407. The red colour of ripe tomatoes is due to the presence of (b) The properties of a mixture are the same as those of its
(SSC Sub. Ins. 2012) components
(a) Hormones (b) Vitamins (c) Mixtures are homogeneous
(c) Chlorophyll (d) Carotenoids (d) In a mixture the components are present in a fixed ratio

Downloded From : www.EasyEngineering.net


Downloded From : www.EasyEngineering.net

72 General Science
419. PVC is obtained by the polymerisation of 430. Which of the following elements is not radio-active?
(SSC Sub. Ins. 2012) (SSC Sub. Ins. 2013)
(a) Propene (b) Vinyl chloride (a) Radium (b) Plutonium
(c) Styrene (d) Acetylene (c) Zirconium (d) Uranium
420. To eliminate the glare of headlights in motor cars, 431. Wilting of plants occurs due to excessive :
(SSC Sub. Ins. 2012) (SSC Sub. Ins. 2013)
(a) polaroids are used (b) glass prisms are used (a) Respiration (b) Guttation
(c) thin films are used (d) filters are used (c) Absorption (d) Transpiration
421. The substances which have infinite electrical resistance are 432. Gypsum is used for improvement of:
(SSC Sub. Ins. 2013)
called (SSC Sub. Ins. 2012)
(a) Alkaline soils (b) Saline soils
(a) insulators (b) condensers
(c) Podsols (d) Acidic soils
(c) conductors (d) resistors
433. Indicate the correct arrangement for electromagnetic
422. Stainless steel is an alloy of (SSC Sub. Ins. 2012) radiation in order of their increasing wavelength.
(a) iron, chromium and nickel (SSC Sub. Ins. 2013)
(b) iron, chromium and carbon (a) Microwave. infrared, visible. X-rays
(c) iron, carbon and zinc (b) X -rays. visible, infrared, microwave
(d) iron, zinc and manganese (c) Visible. infrared. microwave. X-rays

ww
423. Enriched uranium used in a nuclear reactor is

(a) uranium free of all impurities


(SSC Sub. Ins. 2012)
(d) X - rays, infrared, visible, microwave.
434. The rapidly growing mass of phytoplankton covering the
surface water of a lake or pond is known as:

w.E
(b) uranium treated with radiation
(c) uranium alloyed with aluminium
(d) uranium with a high percentage of a particular isotope
(a) Eutrophication
(c) Water pollution
(SSC Sub. Ins. 2013)
(b) Water bloom
(d) Water hyacinth

asy
424. In a refrigerator, cooling is produced by
(SSC Sub. Ins. 2012)
435. Wings of birds are:
(a) Modified hind limbs
(SSC Sub. Ins. 2013)

(a) the ice which deposits in the freezer


(b) the sudden expansion of a compressed gas
En (b) New structure
(c) Integumentary outgrowth
(d) Modified fore limbs
(c) the evaporation of a volatile liquid
(d) None of these
425. Which one of the following statements about PYROLYSIS.
gin
436. Spontaneous change is one in there is:
(SSC Sub. Ins. 2013)
which is a process for solid waste treatment is incorrect?
(SSC Sub. Ins. 2013) eer
(a) A lowering of entropy
(b) A lowering of free energy
(a) It converts the waste into solid, liquid and gas of which
the resultant liquid and gas can be used to produce ing
(c) Increase in free energy
(d) An increase in Internal energy
437. Sandstone is metamorphosed to : (SSC Sub. Ins. 2013)
energy.
(b) The process occurs at a temperature above 430°C at
atmospheric pressure.
(a) Shale
(c) Quartzite
.ne
(b) Slate
(d) Marble

(c) The process occurs under high pressure at temperature


above 430°C.
(d) It is a thermochemical decomposition of organic waste
is :
(a) Barometer
(c) Hygrometer
(b) Hydrometer
(d) Sonometer
t
438. Instrument used to study the behaviour of a vibrating string
(SSC Sub. Ins. 2013)

426. Which of the following supports particle nature of photons? 439. The casual organism of Polio is : (SSC Sub. Ins. 2013)
(SSC Sub. Ins. 2013) (a) A fungi (b) A virus
(a) Diffraction (b) Polarization (c) A worm (d) A bacteria
(c) Photoelectric effect (d) Interference 440. Panda belongs to the same family as that
427. The heaviest naturally occurring element is: (SSC Sub. Ins. 2013)
(SSC Sub. Ins. 2013) (a) Kangaroo (b) Porcupine
(a) Mercury (b) Polonium (c) Whale (d) Bear
(c) Thorium (d) Uranium 441. The pancreas secretes (SSC Sub. Ins. 2014)
428. Haptens are: (SSC Sub. Ins. 2013) (a) Insulin (b) Bile juice
(a) Pseudoantigens (b) Incomplete antigens (c) Peptic juice (d) None of these
442. When we touch leaves of "Touch me not plant", they close,
(c) Antibodies (d) Isoantigens
these movements are called (SSC Sub. Ins. 2014)
429. Sulphur dioxide bleaches colouring matter by :
(a) photonastic movements
(SSC Sub. Ins. 2013)
(b) nyctinastic movements
(a) Reduction (b) Dehydration
(c) seismonastic movements
(c) Decomposition (d) Oxidation
(d) chemonastic movements

Downloded From : www.EasyEngineering.net


Downloded From : www.EasyEngineering.net

General Science 73
443. The concept of tissue culture was introduced by 455. The process of imbibition involves (SSC Sub. Ins. 2014)
(SSC Sub. Ins. 2014) (a) Diffusion (b) Capillary action
(a) Halfmeister (b) Hanstein (c) Absorption (d) Both (A) & (B)
(c) Haberlandt (d) Hanning 456. A cell increases in volume when it is placed in
444. Beak is formed by (SSC Sub. Ins. 2014) (SSC Sub. Ins. 2014)
(a) cheeks (b) jaws (a) Hypertonic solution (b) Hypotonic solution
(c) teeth (d) none (c) Isotonic solution (d) None of these
445. Pinna (external ear) is present in (SSC Sub. Ins. 2014) 457. A reversible and an irreversible engine are working between
(a) Amphibian (b) Fish the same limits of temperature. The efficiency of.
(c) Mammal (d) Reptile (SSC Sub. Ins. 2015)
446. Purity of a metal can be determined with the help of (a) The reversible engine is greater than the irreversible
(a) Pascal's law (SSC Sub. Ins. 2014) engine.
(b) Boyle's law (b) Each engine is 100%.
(c) Archimedes principle (c) The two engines are equal
(d) Conservation of mass principle (d) The irreversible engine is greater than the reversible
447. If both the mass and the velocity of a body is increased to engine.
twice of their magnitude, the kinetic energy will increase by 458. Which of the following is used in the treatment of cancer?

ww
(a) 2 times
(c) 8 times
(SSC Sub. Ins. 2014)
(b) 4 times
(d) 16 times
(a) Electrotherapy
(SSC Sub. Ins. 2015)
(b) Psychotherapy

w.E
448. Two bodies kept at a certain distance feel a gravitational
force F to each other. If the distance between them is made
double the former distance, the force will be
(c) Chemotherapy
459. The most abundant element is :
(a) Silicon
(d) Physiotherapy
(SSC Sub. Ins. 2015)
(b) Calcium

(a) 2F (b) asy


1
(SSC Sub. Ins. 2014)
F
(c) Nitrogen

an optical microscope because :


(d) Oxygen
460. An electron microscope gives higher magnifications than
(SSC Sub. Ins. 2015)

(c) 4F (d)
1
2

F En (a) The electrons have more energy than the light


particles.

449. Stationary wave is formed by


4
(SSC Sub. Ins. 2014) gin (b) The electron microscope uses more powerful lenses.
(c) The wavelength of electrons is smaller as compare d
(a) a transverse wave superposing a longitudinal wave
(b) two waves of the same speed superposing
(c) two waves of same frequency travelling in the same eer
to the wavelength of visible light.
(d) The velocity of electrons is smaller than that of light.
461. What does the word ‘amphibian’ mean ?
direction
(d) two waves of same frequency travelling in the (a) Two lives ing (SSC Sub. Ins. 2015)
(b) Four lives

450.
opposite direction
In an oxygen molecule, two atoms are united by
(c) Three lives
.ne
(d) One life
462. Nitrogen in water is commonly found in the form of:

451.
(a) the bond
(c) three bonds
(SSC Sub. Ins. 2014)
(b) two bonds
(d) four bonds
The inert gas which is substituted for nitrogen in the air
(a) Nitric oxide
(c) Nitrate
(b) Nitrous oxide
(d) Nitrite
463. Immunization technique was developed by:
t
(SSC Sub. Ins. 2015)

used by deep sea divers for breathing is (SSC Sub. Ins. 2015)
(SSC Sub. Ins. 2014) (a) Louis Pasteur (b) Robert Koch
(a) Neon (b) Krypton (c) Joseph Lister (d) Edward jenner
(c) Argon (d) Helium 464. The hydrophilic nature of DNA is due to the presence of.
452. How many neutrons are there in 92U238 atom ? (SSC Sub. Ins. 2015)
(SSC Sub. Ins. 2014) (a) a number of hydrogen bonds
(b) phosphate group
(a) 92 (b) 238
(c) deoxyribose sugar
(c) 146 (d) 330
(d) thymine base
453. Root pressure is measured by (SSC Sub. Ins. 2014)
465. The sense of balance is achieved by:
(a) Barometer (b) Atmometer
(SSC Sub. Ins. 2015)
(c) Manometer (d) Auxanometer
(a) Cerebellum equilibrium
454. Cell becomes turgid because of (SSC Sub. Ins. 2014)
(b) Thalamus equilibrium
(a) Plasmolysis (b) Exosmosis
(c) Cerebrum equilibrium
(c) Endosmosis (d) Diffusion
(d) Spinal cord equilibrium

Downloded From : www.EasyEngineering.net


Downloded From : www.EasyEngineering.net

74 General Science
466. Aluminium salt commonly used to stop bleeding is: 475. Antigen presenting cells are specialized cells present in all
(SSC Sub. Ins. 2015) of the following, except (SSC Sub. Ins. 2016)
(a) Aluminium chloride (b) Aluminium nitrate (a) Skin (b) Lymph node
(c) Aluminium sulphate (d) Potash alum (c) Kidney (d) Spleen
467. Total internal reflection cannot take place when light goes 476. Which of the following determines whether a group of
from: (SSC Sub. Ins. 2015) organisms that is from the same genus and species arise
(a) water to glass (b) water to air from a common source or from different sources?
(c) glass to air (d) glass to water (SSC Sub. Ins. 2016)
468. Interferons are synthesized in response to: (a) Biotyping
(SSC Sub. Ins. 2015) (b) DNA hybridization / DNA
(a) mycoplasma (b) fungi (c) Serotyping
(c) virus (d) bacteria (d) Phage typing
469. The first law of thermodynamics is simply the case of: 477. Starch is insoluble in water but still it is stored in large
(SSC Sub. Ins. 2015) quantity in potato because (SSC Sub. Ins. 2016)
(a) Charle’s law. (a) soil microorganisms deposit it in the tuber.
(b) the law of conservation of energy. (b) it is synthesized in potato root.
(c) the law of heat exchange. (c) it is useful for human.

ww
(d) Newton’s law of cooling.
470. Red rot of sugarcane is caused by: (SSC Sub. Ins. 2015)
(a) Colletotriehum falcatum
(d) it is translocated in the form of sugar from leaves.
478. Viruses that infect bacteria are called (SSC Sub. Ins. 2016)

w.E
(b) Cercospora personata
(c) Alternaria alternata
(d) Phylophthora Infestans
(a) Basal body
(c) Bacteriophages
479. A dispersion indicates
(b) Basidiospores
(d) Basophils
(SSC Sub. Ins. 2016)

asy
471. A clone is a group of individuals obtained through :
(SSC Sub. Ins. 2015)
(a) the value of standard deviation.
(b) spread of data around central measure.
(c) the value of mean
(a) self pollination
(c) hybridisation
(b) micropropagation
(d) cross pollination
472. Chemical name of Gammaxane is: (SSC Sub. Ins. 2015)En (d) the value of mode
480. The principle involved in the absorption of water by soilsi
(a) Aniline (b) Toluene
(c) Benzene hexachloride (d) Chloro benzene gin (a) Suction action
(SSC Sub. Ins. 2016)
(b) Condensation
473. The fleshy thalamus is edible in:
(a) Mango
(SSC Sub. Ins. 2015)
(b) Orange
eer
(c) Capillary action (d) Principal of absorptio n
481. Which of the following is present in maximum amount ni
acid rain? (SSC Sub. Ins. 2016)
(c) Tomato (d) Apple
474. A new molecule Heat Shock Protein 90 (HSP90) was
discovered in 2014 by the Didier Picard. The new discovery
(a) HNO3
(c) H2CO3 ing (b) H2SO4
(d) HCL
could help in effective treatment of ____
(SSC Sub. Ins. 2016) under standard is referred as .ne
482. The lethal dose required to kill 50% of the lab animals tested
(SSC Sub. Ins. 2016)
(a) TB
(c) Malaria
(b) AIDS
(d) None of these
(a) MLD
(c) LD50
(b) ID50
(d) ID
t

Downloded From : www.EasyEngineering.net


Downloded From : www.EasyEngineering.net

General Science 75

HINTS & SOLUTIONS


1. (b) 25. (a) 26. (a) 27. (c) 28. (d) 29. (d)
2. (a) Vitamin B-12, also called cobalamin, is a water-soluble 30. (b) 31. (b) 32. (c) 33. (d) 34. (b)
vitamin that has a key role in the normal functioning 35. (c) 36. (d) 37. (c) 38. (d) 39. (c)
of the brain and nervous system, and the formation of 40. (d) 41. (d) 42. (a) 43. (a) 44. (c)
red blood cells. 45. (c) 46. (c) 47. (c) 48. (a) 49. (d)
3. (d) 4. (b) 5. (a) 6. (b) 7. (a) 50. (a) 51. (b) 52. (c) 53. (d) 54. (a)
8. (b) Acetic acid, also known as ethanoic acid, is an organic 55. (c) 56. (d) 57. (b) 58. (a) 59. (c)
chemical compound best recognized for giving vinegar 60. (d)
its sour taste and pungent smell. It is one of the 61. (c) This process usually occurs in the upper third of the
simplest carboxylic acids and has the chemical formula fallopian tube of the woman.
CH3COOH. 62. (d) Cirrhosis is a condition in which the liver does not
9. (b) function properly due to long-term damage. Cirrhosis
is most commonly caused by alcohol, hepatitis B,
10.

11. (a) ww
(a) Boiling point of heavy water is lower than that or
ordinary water hepatitis C, and non-alcoholic fatty liver disease.
Typically, more than two or three drinks per day over a
number of years is required for alcoholic cirrhosis to
12.
w.E
(a) Arboreal animals are creature who spend the majority
of their lives in trees. They eat, sleep and play in the
tree canopy. There are thousands of species that live
63. (c)
occur.
The food webs we see are grazing food chains since at
their base are producers which the herbivores then

snakes and a variety of insects. asy


in trees, including monkeys, koalas, possums, sloths,
various rodents, parrots, chameleons, geckos, tree
64. (c)
graze on.
A single mole is set to the number of particles found in
13.
14.
(a)
En
(d) Advection is the transfer of heat or matter by the flow
12.000 grams of carbon-12. A mole of water has 6.022 x
1023 water molecules. One mole of water weighs 18.0152
grams.
of a fluid, especially horizontally in the atmosphere or
the sea. gin
65.
66.
(a)
(d)
Carrot is rich in Vitamin A and it improves eyesight.
The reason for this is the hydrogen bonding betwee n
15.
16.
(d)
(d) Bats are good at flying at night because they use
sound rather than sight to navigate. Bats send pulses eer
neighboring water molecules. Because hydrogen
bonding is a relatively strong intermolecular force, high

of sound through their mouths or noses, and these


pulses echo back outlining the objects in the bats
67. (a)
ing
heat energy is required to break up the force.
Valence electrons are important in determining how a n
elements reacts chemically with other elements. Sinc e
flight path. The ears of a bat are large and oddly
constructed but they help it to determine where the
echoes are coming from. .ne
the valence electrons are the electrons in the highest
energy level, they are the most exposed of all the

17.
18.
(c) No change will happen.
(b) Cartography, the art and science of graphically
representing a geographical: area, usually on a flat
68. (c)
involved in chemical reactions.
t
electrons, so they are the electrons that get most

The lumen is the SI derived unit of luminous flux, a


measure of the total "amount" of visible light emitted
surface such as a map or chart. It may involve the by a source.
superimposition of political, cultural, or other non 69. (d) A dissipative force counteracts motion. Its direction is
geographical divisions onto the representation of a opposite to the direction of the velocity vector.
geographical area. Dynamic friction is a dissipative (non-conservative)
19. (c) force : it dissipates energy (mainly through heat and
20. (d) An antiknock agent is a gasoline additive used to sound), and energy lost by moving in one direction.
reduce engine knocking and increase the fuel's octane 70. (c) Poisson strain is defined as the negative ratio of the
rating by raising the temperature and pressure at which strain in the traverse direction (caused by the
auto ignition occurs. contraction of the bar's diameter) to the strain in the
The typical antiknock agents in use are: Tetraelhyllead longitudinal direction. As the length increases and the
(Still in use as a high octane cross sectional area decreases, the electrical resistance
21. (b) 22. (a) 23. (a) of the wire also rises.
24. (d) A positron is a particle of matter with the same mass as 71. (b) Consequent Poles are magnetic poles that exist where
an electron but an opposite charge. It is a form of the specimen has been successively magnetized in
antimatter because, when a positron encounters an different sections to create more than two poles ; e.g.,
electron, the two completely annihilate to yield energy. two north poles with one south pole between them.

Downloded From : www.EasyEngineering.net


Downloded From : www.EasyEngineering.net

76 General Science
72. (b) Fructose, or fruit sugar, a is one of the three dietary 86. (b) In hydroelectric power plants the potential energy of
monosaccharide, along with glucose and galactose, water is utilized to produce electricity. The height of
which is absorbed directly into the bloodstream during water in the reservoir decides how much potential
digestion. energy water possesses.
73. (c) Methylated spirit contains mostly ethanol. Ethanol is 87. (c) In people with emphysema, the lung tissue involved
slightly soluble in water (mixing 50 ml of ethanol and in exchange of gases (oxygen and carbon dioxide) is
water will make 80 – 90 ml of mixture, not 100ml). impaired or destroyed. Emphysema is included in a
74. (c) A sporozoite is the cell form that infects new hosts. In group of diseases called chronic obstructive pulmonary
plasmodium, for instance, the sporozoites are cells that disease of COPD.
develop in the mosquito’s salivary glands, leave the 88. (c) Potassium Nitrate occurs as a mineral niter and is a
mosquito during a blood meal, and enter liver cells natural solid source of nitrogen. Potassium nitrate is
(hepatocytes) where they multiply. one of several nitrogen-containing compounds
75. (c) Ex-situ conservation is the process of protecting an collectively referred to as saltpeter.
endangered species of plant or animal outside of its 89. (a) 90. (b) 91. (b) 92. (a) 93. (b)
natural habitat. Zoos and botanical gardens are the 94. (c) 95. (d) 96. (a) 97. (b) 98. (b)
most conventional methods of ex-situ conservation. 99. (c) 100. (d) 101. (d) 102. (a) 103. (a)
Endangered plants may also be preserved in part 104. (b) 105. (d)
76.
ww
through seed banks or germplasm banks.
(b) Diatom ooze (formed from microscopic unicellular algae
having cell walls consisting of or resembling silica) is
106. (c) Two pollutants emitted by motor vehicles react to form
ground-level ozone or smog which can cause
respiratory problems and reduce visibility.

77.
latitudes.
w.E
the most widespread deposit in the high southern

(b) Photosynthetic Chromatophores vesicles found in


107.
109.
(b) 108. (b)
(c) Cardiac muscle is an involuntary striated muscle tissue
found only in the organ heart. Involuntary muscles are

78.
light-harvesting systems in nature.
asy
some purple bacteria constitute one of the simplest

(a) Convex mirrors reflect light outwards; therefore they


smooth muscles that are not directly controllable at
will. For example You don't have to remind yourself to

En
are not used to focus light. So, the convex mirror has
a wide field of view and hence is used as rear view
make your heart beat, so it is involuntary. Voluntary
muscles are controllable like those found in your arms
legs, hands, etc.
,

79.
mirror as it gives a clear diminished and an erect image
of the traffic that is behind.
(b) The principal components of pyroligneous acid are gin
110.
111.
(b)
(c) Ringworm is common disease, especially among
acetic acid, acetone and methanol. It was once used as
a commercial source for acetic acid. eer
children. It is caused by a fungus, not a worm like the
name suggests. It is a common and highly infectious
80. (b) UCIL produced batteries, carbon products, welding
equipment, plastics, industrial chemicals, pesticides,
and marine products. 112.
skin.
(a) 113. (c) 114. (d) ing
skin infection that causes a ring-like red rash on the

81. (a) Most drying oils owe their drying properties to the
presence of a large percentage of linolenic acid (which
115.
.ne
(c) Centrifugal force is an example of a pseudo-force, that
is, an apparent force to someone whose frame of

82.
derives its name from "linseed"), which is highly
unsaturated.
(a) Carotenoids are tetraterpenoid organic pigments that
are naturally occurring in the chloroplasts and 116.
reference is rotating.
(d) 117. (c)
t
reference is not at rest or moving with a constant
velocity. In the case of centrifugal force, the frame of

chromoplasts of plants and some other photosynthetic 118. (c) An antacid is a substance which neutralizes stomach
organism like algae, some bacteria, and some types of acidity.
fungus. 119. (a) Annealing is the process by which both metal and
83. (a) The variations in sunlight affect different latitudes glass are treated with heat in order to change their
differently, as we might expect from the different angles properties.
at which sunlight falls on the earth's surface at different 120. (b) 121. (d) 122. (d) 123. (d)
latitudes. 124. (c) This is because the scattering in red light is less than
84. (a) Polar bears, which pile on fat to survive hibernation that of yellow colour. The longest visible wavelength
and yet do not become diabetic, hold clues for treating is red and the shortest is violet. The wavelength of red
Type II diabetes, a disease associated with obesity light is more than yellow light.
that afflicts more than 190 million people worlwide, 125. (a) Transboundary pollution is the pollution that
reaching epidemic proportions in many countries. originates in one country but is able to cause damage
85. (d) All electromagnetic waves regardless of their in another country's environment, by crossing borders
wavelengths, including all colors of light, have the through pathways like water or air. Acid rain is a classic
same identical speed in a vacuum. example of a transboundary pollution because it can

Downloded From : www.EasyEngineering.net


Downloded From : www.EasyEngineering.net

General Science 77
be blown anywhere by the wind. Sulphur dioxide and body temperature is around 98.6 F (37 C). Hypothermia
nitrous oxides are the two main chemicals that react occurs as your body temperature passes below 95 F
with water to make acid rain. The chemicals are (35 C).
commonly released from power stations, factories and 145. (c) Semiconductors are insulators at low temperatures and
transport. reasonably good conductors at higher temperatures.
126. (a) The Nicobar pigeon is a pigeon found on small islands As temperature increases, the semi–conductor material
and in coastal regions from the Nicobar Islands, India, becomes a better and better conductor.
east through the Malay Archipelago, to the Solomons 146. (c) Mass of proton is 1.672621777 (74) × 10–27 kg. An
and Palau. It is the only living member of the genus electron has a mass (9.1093829140) × 10–31 kg) that is
Caloenas and the closest living relative of the extinct approximately 1/1836 that of the proton. The mass of
dodo.. neutron is slightly larger than that of a proton. The mass
127. (a) A limnic eruption, also referred to as a lake overturn, of the hydrogen nucleus is 1.7 × 10–27 kg. The heaviest
is a rare type of natural disaster in which dissolved of these particles is the neutron.
carbon dioxide (CO2) suddenly erupts from deep lake 147. (a) A television channel is a physical or virtual channel
waters, forming a gas cloud that can suffocate wildlife, over which a television station or television network is
livestock and humans. distributed. Channel numbers represent actual
128. (c) The inventor of blood group is Karl Landsteiner, was frequencies used to broadcast the television signal.

ww
born in Vienna, June 14, 1868.
He is a scientist in the field of bio-treatment. He was
awarded Nobel Prize for Medicine and Fisiologi field in
For example, in North America. "Channel 2" refers to
the broadcast or cable band of 54 to 60 MHz, with
carrier frequencies of 55.25 MHz for NTSC analog video

1909.
w.E
1930. This is because the classification of all types of
blood into four namely; A, B, AB, and O in the year
148. (b)
(VSB) and 59.75 MHz for analog audio (FM), or 55.31
MHz for digital ATSC (8VSB).
Water never has an absolute density because its
129. (a)
131. (b)
130. (b)
asy
DPT refers to a class of combination vaccines against
density varies with temperature. Water has its maximum
density of lg/cm3 at 4 degrees Celsius. When the
three infectious diseases in humans: diphtheria,

En
pertussis (whooping cough), and tetanus. The vaccine
components include diphtheria and tetanus toxoidsand
temperature changes from either greater or less than 4
degrees, the density will become less than 1 g/cm3.
kills whole cells of the organism that cause pertussis
(wP). gin
149. (c) Photosynthesis is the process by which light energy
is converted into chemical energy by organisms.
Carbon dioxide and water are the raw materials of this
132. (a)
137. (b)
133. (c) 134. (b) 135. (b) 136. (d)
The oxalic acid is an ideal chemical for cleaning
eer
process. The light energy comes from the sun and its
end products are oxygen and glucose.
purposes. Its bleach-like qualities make it perfect for
sterilizing household items. It is also efficient in
removing rust on various different surfaces. Stains on
150. (b)
ing
An emulsion is a mixture of two or more liquids that are
normally immiscible (nonmixable or unblendable).
counters, bathtubs and kitchen sinks can be removed
through careful application of this chemical. .ne
Emulsions are part of a more general class of two–
phase systems of matter called colloids. In an emulsion,
138. (b)
139. (d) Rectified spirit, also known as neutral spirits,
rectified alcohol, or ethyl alcohol of agricultural
origin is highly concentrated ethanol which has been
151. (b)
other (the continuous.
t
one liquid (the dispersed phase) is dispersed in the

The active ingredient in Dettol that confers its antiseptic


property is chloroxylenol (C8H9CIO), an aromatic
purified by means of repeated distillation, a process chemical compound. Chloroxylenol comprises 4.8% of
that is called rectification. It is 95.5% alcohol and 4.5% Dettol's total mixture, with the rest composed of pine
water. It is treated with CaO to form lime of alcohol, oil, isopropanol, castor oil soap caramel and water.
which undergoes fractional distillation to give ethanol. 152. (c) Researchers in Mangolia and China completed studies
140. (c) to unravel the genomic Peculiaritces behind the
141. (d) The Intergovernmental Panel on Climate Change physiological tricks that camels use to survive in the
(IPCC) is a scientific and intergovernmental body harshest of conditions. They described the genomes
under the auspices of the United Nations, set up at the of wild and domesticated Bactrian camels.
request of member governments, dedicated to the task 153. (d) Fine particles known as PM10 and PM25can penetrate
of providing the world with an objective, scientific view deep into the lungs, creating health problems. People
of climate change and its political and economic impacts. with heart or lung diseases, older adults, and children
142. (c) 143. (b) are most likely to have problems because of contact
144. (c) Hypothermia is a medical emergency that occurs when with particle pollution. Short–term exposure to PM
your body loses heat faster than it can produce heat, among pregnant women has been associated with
causing a dangerously low body temperature. Normal prematurity and growth retardation.

Downloded From : www.EasyEngineering.net


Downloded From : www.EasyEngineering.net

78 General Science
154. (d) PNG is a mixture consisting mainly methane CH4 with a among other functions, it influences radio propagation
small percentage of other higher hydrocarbons. The to distant places on the Earth.
ratio of carbon to hydrogen is least in methane and 165. (a) Coral reefs are underwater structures made from
hence it burns almost completely making it the cleanest calcium carbonate secreted by corals. Coral reefs are
fuel. Domestic PNG customers, also known as colonies of tiny animals found in marine waters that
Residential customers, use gas for cooking purpose contain few nutrients.
and also for heating water through gas geysers. 166. (a) Phytochrome is a photoreceptor, a pigment that plants
155. (d) Green Park Stadium is a 60,000 capacity floodlit multi– use to detect light. It is a protein with a bilin
purpose stadium located in Kanpur, India, and the chromophore. It detects mainly red and fared region
home of the Uttar Pradesh cricket team. of the visible spectrum and regulates germination of
156. (c) According to the Red list of 2012. Ganges River seeds.
Dolphin is one of critically endangered species in lndia 167. (c) Quenching is a process in which absorbed light energy
Endangered species in India comprise large varieties is dissipated as heat and does not take part in
of rare species of wild animals, aquatic animals and photochemistry. The phenomenon involves
insects. quenching of chlorophylla (Chla) fluroescence, which
157. (c) Arson is the crime of intentionally and maliciously is induced under steady-state illumination.
setting fire to buildings, wild land areas, vehicles or 168. (c) AIDS virus destroys the T-cells inside of the immune

ww
other property with the intent to cause damage, Distinct
from spontaneous combustion and natural wildfires, it
is considered to be a man–made disaster which is
system. T cells or T lymphocytes belong to a group
of white blood cells known as lymphocytes, and play
a central role in cell – mediated immunity.
socially induced.
w.E
158. (b) The thyroid gland, is one of the largest endocrine gland
in the throat, and consists of two connected lobes. It
169. (a) Bragg Spectrometer is an instrument used to analyze
crystal structure by using X– rays. In it, a beam of
collimated X–rays strikes the crystal, and a detector

asy
is found at the front of the neck, below the Adam’s
apple. The thyroid gland secretes thyroid hormones,
measures the angles and intensities of the reflected
beam.

En
which influence the metabolic rate, protein synthesis,
and have a wide range of other effects, including on
development.
170. (a) Alpha particles consist of two protons and two
neutrons bound together intoa particle identical to a
helium nucleus, which is generally produced in the
159. (b) The cerebrum is the seat of intelligence and provides
us with the ability to read, write and speak: make gin
171. (c)
process of alpha decay.
The human body requires more calcium than any other
calculations and compose music; remember the past
and plan for the future; and create works.
eer
mineral. At least 99% of the calcium is found in the
bones and teeth, giving them strength and rigidity.
160. (c) The average adult has a blood volume of roughly 5
liters, which is composed of plasma and several kinds
of cells. By volume, the red blood cells constitute about
172. (a)

ing
173. (a) The sigmoid colon is the part of the large intestine. It
forms a loop that averages about 40 cm in length.
45% of whole blood, the plasma about 54.3%, and white
cells about 0.7%. .ne
174. (d) Electrons carry current in a good conductor of
electricity and they are negatively charged. This makes
161. (c) The normal range of, conentration of fasting blood
sugar is 80 to 120 mg/ml; in the testing of true blood
sugar, the normal range of concentration is 70 to 100
mg/ml. so normal glucose levels fall between 70 and
t
a good electrical conductor negatively charged.
175. (c) Containment of toxic or hazardous constituents in
industrial solid wastes is curr ently being
accomplished using: (i) coating grains of waste material
150 mg. Higher levels may indicate diseases such as with an inert and non-reactive impervious material
diabetes mellitus. (microencapsulation), and (ii) coating blocks of waste
162. (b) Entomology is the scientific study of insects, a branch material with an inert, non-reactive impervious material
of arthropodology, which in turn is a branch of biology. (microencapsulation).
It is derived from the Greek word 'entomos' which 176. (b) UV radiation can be an effective viricide and
means "that which is cut in pieces or engraved/ bactericide. Disinfection using UV radiation is
segmented", hence "insect". commonly used in wastewater treatment applications
163. (a) and is finding an increased usage in drinking water
164. (c) The ionosphere is a region of Earth's upper atmosphere, treatment.
from about 60 km (37 mi) to 1,000 km (620 mi) altitude 177. (b) When cells are exposed to sunlight, radiant energy
and includes the thermosphere and parts of the can damage the DNA. For example, ultraviolet
mesosphere and exosphere, it is ionized by solar irradiation cause covalent bond formation between
radiation, plays an important part in atmospheric adjacent thymines on the same strand of DNA.
electricity and forms the inner edge of the Ultraviolet light is absorbed by a double bond in
magnetosphere. It has practical importance because, thymine and cytosine bases in DNA.

Downloded From : www.EasyEngineering.net


Downloded From : www.EasyEngineering.net

General Science 79
178. (a) In order to manufacture silicones, alkyl-substituted 188. (c) Parathyroid hormone (PTH), parathormone or
chlorosilanes are used as starting material. Since parathyrin, is secreted by the chief cells of the
dimethylsilane contains – OH group at the end of the parathyroid glands as a polypeptide containing 84
chain, polymerization and chain length increases. amino acids. It acts to increase the concentration of
However, the hydrolysis of alkyl trichlorosilane gives calcium (Ca2+) in the blood, whereas calcitonin (a
very complex cross linked polymer. hormone produced by the parafollicular cells (C cells)
179. (b) Examples of natural coloids can be found in our body of the thyroid gland) acts to decrease calcium
itself : blood consists of colloidal sized red blood concentration.
corpuscles (RBC) which provide the vital oxygen to 189. (c) by tracheal system
all the body tissues. 190. (c) In nuclear physics and nuclear chemistry, a nuclear
180. (a) Nickel silver, also known as German silver, is a copper reaction is semantically considered to be the process
alloy with nickel and often zinc. The usual formulations in which two nuclei, or else a nucleus of an atom and a
is 60% copper, 20% nickel and 20% zinc. Nickel silver subatomic particle (such as a proton, neutron, or high
is named for its silvery appearance, but it contains no energy electron) from outside the atom, collide to
elemental silver unless plated. produce one or more nuclides that are different from
181. (d) Atomic absorption spectroscopy is an analytical the nuclide(s) that began the process.
chemistry technique used for determining 191. (b) annihilate each other

ww
concentration of particular metal element and is widely
used in phar maceutics. It was used for the
determination of cobalt in Vitamin B12.
192. (a) In the photoelectric effect, electrons are emitted from
solids, liquids or gases when they absorb energy from
light. Electrons emitted in this manner may be called

w.E
182. (b) Nitrosomonas is a genus comprising rod shaped
chemoautotrophic bacteria. This rare bacteria oxidizes
ammonia into nitrite as a metabolic process.
photoelectrons.
193. (d) a circle
194. (b) Cryolite (Na3AlF6, sodium hexafluoroaluminate) is an

asy
183. (a) Devarda's alloy , is an alloy of aluminium (44% - 46%),
copper (49% - 51%) and zinc (4% - 6%). Devarda's
uncommon mineral identified with the once large
deposit at Ivigtût on the west coast of Greenland,
depleted by 1987.

En
alloy is used as reducing agent in analytical chemistry
for the determination of nitrates after their reduction to
ammonia under alkaline conditions. It owes its name to
195. (c) Glucose (C6H12O6, also known as D-glucose, dextrose,
or grape sugar) is a simple monosaccharide found in
the Italian chemist Arturo Devarda (1859-1944), who
synthezised it at the end of the 19th century to develop gin plants. It is one of the three dietary monosaccharides,
along with fructose and galactose, that are absorbed
directly into the bloodstream during digestion.
a new method to analyze nitrate in Chile saltpeter.
184. (a) Grit chambers are long narrow tanks that are designed
eer
196. (b) Any of the homologous segments, lying in a
longitudinal series, that compose the body of certain
to slow down the flow so that solids such as sand,
coffee grounds, and eggshells will settle out of the
water. Grit causes excessive wear and tear on pumps
somite. ing
animals, such as earthworms and lobsters. Also called

and other plant equipment. Its removal is particularly


important in cities with combined sewer systems, which .ne
197. (a) 198. (d) 199. (d) 200. (c) 201. (b)
202. (a) 203. (c) 204. (c) 205. (a) 206. (c)
207. (d) 208. (c) 209. (d) 210. (c) 211. (a)
carry a good deal of silt, sand, and gravel that wash off
streets or land.
185. (a) Cysteine (abbreviated as Cys or C) is an a-amino acid
with the chemical formula HO2CCH(NH2)CH2SH. It is
212. (a) 213. (c) 214. (b) 215. (b)
t
216. (c) Succulent plants store water in their stems or leaves.
They include the Cactaceae family, which has round
stems and can store a lot of water. The leaves are often
a semi-essential amino acid, which means that it can be vestigial, as in the case of cacti, wherein the leaves are
biosynthesized in humans. The thiol side chain in reduced to spines, or they do not have leaves at all.
cysteine often participates in enzymatic reactions, Water is stored in the bulbs of some plants, at or below
serving as a nucleophile. ground level. They may be dormant during drought
186. (d) Neon is a chemical element with symbol Ne and atomic conditions and are, therefore, known as drought
number 10. It is in group 18 (noble gases) of the periodic evaders.
table. Neon is a colorless, odorless, inert monatomic 217. (a) Thorium is an element which are used in radioactive
gas under standard conditions, with about two-thirds chemicals where all other three options are power
the density of air. generating systems which are regenerated.
187. (b) The aorta is the largest artery in the human body, 218. (a) chegonium, the female reproductive organ in ferns
originating from the left ventricle of the heart and and mosses. An archegonium also occurs in some
extending down to the abdomen, where it bifurcates gymnosperms, e.g., cycads and conifers. A flask-
into two smaller arteries (the common iliac arteries). shaped structure, it consists of a neck, with one or
The aorta distributes oxygenated blood to all parts of more layers of cells, and a swollen base-the venter-
the body through the systemic circulation. which contains the egg.

Downloded From : www.EasyEngineering.net


Downloded From : www.EasyEngineering.net

80 General Science
219. (b) Trochodendron is a genus of flowering plants with 230. (b) Galvanization, or galvanisation, is the process of
one living species, Trochodendron aralioides, and six applying a protective zinc coating to steel or iron, to
extinct species known from the fossil record. prevent rusting. The most common method is hot-dip
220. (a) A Caesarean section (often C-section, also other galvanization, in which parts are submerged in a bath
spellings) is a surgical procedure in which one or more of molten zinc.
incisions are made through a mother's abdomen 231. (b) A homogeneous mixture is a type of mixture in which
(laparotomy) and uterus (hysterotomy) to deliver one the composition is uniform and every part of the
or more babies. The first modern Caesarean section solution has the same properties. A homogeneous
was performed by German gynecologist Ferdinand mixture in which there is both a solute and solvent
Adolf Kehrer in 1881. But in ancient medical history present is also a solution.
Julius Caesar was the first person to be borne by this 232. (c) The plant epidermis consists of three main cell types:
method and thus the operation named after him. pavement cells, guard cells and their subsidiary cells
that surround the stomata.
221. (b) Study of ants is called Myrmecology.
233. (a) As sugar concentration increases in the guard cells,
222. (a) A Reverse transcriptase (RT) is an enzyme used to
as a result water enters the guard cells. The guard cells
generate complementary DNA (cDNA) from an RNA
become turgid (swollen with water). The thin outer walls
template, a process termed reverse transcription. RT is
bulge out and force the inner wall into a crescent shape.
needed for the replication of retroviruses (e.g., HIV),

ww
and RT inhibitors are widely used as antiretroviral
drugs.
In this way a stoma or pore is formed between each
pair of guard cell.
234. (c) Mahyco, an Indian seed company based in Jalna,

w.E
223. (c) Fleming's right-hand rule (for generators) shows the
direction of induced current when a conductor moves
in a magnetic field. The right hand is held with the
Maharashtra, has developed the Bt brinjal. The
genetically modified brinjal event is termed Event EE 1
and Mahyco have also applied for approval of two

asy
thumb, first finger and second finger mutually
perpendicular to each other (at right angles).
224. (b) The watt (symbol: W) is a derived unit of power in the
brinjal hybrids.
235. (d) Electric current is measured using a device called an

En
International System of Units (SI), named after the
Scottish engineer James Watt (1736-1819).
ammeter.
236. (d) Photoperiodism is the physiological reaction of

225. (d) The NIOSH states "Under dry conditions, the


resistance offered by the human body may be as high gin organisms to the length of day or night. It occurs ni
plants and animals. It affects Flowering,Vegetative
growth and fruiting in plants.
as 100,000 Ohms. Wet or broken skin may drop the
body's resistance to 1,000 Ohms," adding that "high- eer
237. (d) The correct match is as follows:
Ascorbic acid- Vitamin C
voltage electrical energy quickly breaks down human
skin, reducing the human body's resistance to 500
Ohms."
Carotenoid- Quenchering
Chlorophyll- Photosynthetic pigment

226. (a) The critical temperature for superconductors is the


temperature at which the electrical resistivity of a metal
Superoxide dismutase- Enzyme

.ne
238. (b) Allantois is a part of a developingamniote's conceptus

drops to zero. The transition is so sudden and complete


that it appears to be a transition to a different phase of
matter; this superconducting phase is described by
the BCS theory.
is webbed withblood vessels.
239. (d) Haliotis belongs to Mollusca.
240. (a) Although most DNA is packaged in chromosomes
t
which primarily involved in nutrition and excretion, and

within the nucleus, mitochondria also have a small


227. (c) A central processing unit (CPU) is the electronic amount of their own DNA. This genetic material is
circuitry within a computer that carries out the known as mitochondrial DNA or mtDNA
instructions of a computer program by performing the 241. (c) First class proteins contain all the essential amino acids
basic arithmetic, logical, control and input/output (I/ in sufficient amounts.Animal proteins are obtained from
O) operations specified by the instructions. milk, egg, fish, meat etc. are first classproteins. These
228. (a) Brass is an alloy made of copper and zinc; the are also called adequate proteins.
proportions of zinc and copper can be varied to create 242. (b) Needle exerts more pressure than nail on the balloon.
a range of brasses with varying properties. 243. (c) The speed of sound is greater in moist air than in dry air.
229. (d) Wrought iron is an iron alloy with a very low carbon 244. (b) X rays are frequently used to check the defects in
content, in comparison to steel, and has fibrous Diamonds and other precious stones.
inclusions, known as slag. This is what gives it a "grain" 245. (a) When ice is kept on saw dust then it does not melt
resembling wood, which is visible when it is etched or quickly as it's an insulator of heat and air does not
bent to the point of failure. Wrought iron is tough, circulate in good insulators. So, it prevents ice from
malleable, ductile and easily welded. melting quickly .

Downloded From : www.EasyEngineering.net


Downloded From : www.EasyEngineering.net

General Science 81
246. (a) When a drop of Glycerol is added to crushed KMnO4 260. (b) 'Dentrification' is the biological conversion of nitrate
spread on a paper there is a violent explosion. to nitrogen gas, nitric oxide or nitrous oxide. These
247. (c) A bleaching agent is a material that lightens or whitens compounds are gaseous compounds and are not
a substrate through chemical reaction. The most readily available for microbial growth; therefore they
common bleaching agents generally fall into two are typically released to the atmosphere.
categories: chlorine and its related compounds (such 261. (b) Holstein Friesians are a breed of cattle known today as
as sodium hypochlorite) and the peroxygen bleaching the world's highest-production dairy animals. They are
agents, such as hydrogen peroxide and sodium found in Europe and America.
perborate. 262. (c) Coagulation removes dirt and other particles suspended
248. (a) Alpha particles are the least penetrating as they are in water. Alum and other chemicals are added to water
the most densely ionizing. The penetrating power of to form tiny sticky particles called "floc" which attract
nuclear radiation depends upon the ionizing power of the dirt particles. The combined weight of the dirt and
the radiation. The more localised the ionization the less the alum (floc) become heavy enough to sink to the
penetrating power it will possess. bottom during sedimentation.
249. (a) Hydrogen- peroxide is an effective sterilizing agent. 263. (c) Retroviral integrase (IN) is an enzyme produced by a
Water results when it readily loses active oxygen. retrovirus (such as HIV) that enables its genetic material

ww
250. (c) The maximum fixation of solar energy is done by green
plants.The energy is stored in the plants as
carbohydrates for their metabolic activities as
to be integrated into the DNA of the infected cell.
264. (b) 265. (d) 266. (c) 267. (a) 268. (d)
269. (a) High grade (HG) and ultra high grade (UHG) anthracite

w.E
growth,respiration etc.
251. (a) The term 'brown air' is used for photochemical smog.
The brown color is cause by the presence of a various
270. (a)
are the highest grades of anthracite coal.

Nitrous Oxides.
asy
252. (c) Peroxyacetyl nitrate is a secondary pollutant present
271. (b) Anemometer is an instrument for measuring the speed
of the wind, or of any current of gas.
272. (b) 273. (a) 274. (a) 275. (b) 276. (d)

En
in photochemical smog. It is thermally unstable and
decomposes into peroxyethanoyl radicals and nitrogen
277. (c) 278. (b) 279. (c) 280. (d)
dioxide gas.
253. (b) Wave particle duality is a quantum mechanics effect. It gin
281. (b) A parsec (symbol: pc) is a unit of length used t o
measure large distances to objects outside our Solar
System.
means the electron sometimes acts like a particle and
sometimes it acts like a wave. It depends on the
situations.
282. (a)
eer
283. (a) Phenol, also known as carbolic acid, is an aromatic
254. (b) Vitamin B12 consists of a class of chemically related
compounds (vitamers). It contains the biochemically ing
organic compound with the molecular formula
C6H5OH. It is a white crystalline solid that is volatile.
rare element cobalt.
255. (d) In 1904 the German agronomist and plant physiologist
bonded to a hydroxyl group (OH).
.ne
The molecule consists of a phenyl group (–C6 H5 )

Lorenz Hiltner first coined the term "rhizosphere" to


describe the plant-root interface, a word originating in
part from the Greek word "rhiza", meaning root.
Hiltner described the rhizosphere as the area around a
284. (b)

t
285. (b) It grows larger until puberty and then begins to shrink.
The gland produces thymosins, which are hormones
that stimulate the development of antibodies.
plant root that is inhabited by a unique population of 286. (b) 287. (d) 288. (d) 289. (c)
microorganisms influenced, he postulated, by the 290. (b) 291. (c) 292. (a)
chemicals released from plant roots. 293. (d) Lichens can be used asair pollution indicators,
256. (c) Reverse transcription were discovered by Howard especially of the concentration of sulfur dioxide in the
Temin and independently isolated by David Baltimore. atmosphere. Lichens are plants that grow in exposed
257. (d) Steam will produce more severe burns than boiling places such as rocks or tree bark. They need to be
water because steam has more heat energy than water very good at absorbing water and nutrients to grow
due to its latent heat of vaporisation. there.
258. (c) Fructose is the sweetest of all natural sugar types. 294. (c) Soil salinity is the salt content in the soil; the process
259. (c) Zone melting, any of a group of techniques used to of increasing the salt content is known as salinization.
purify an element or a compound or control its Salts occur naturally within soils and water. Salinization
composition by melting a short region (i.e., zone) and can be caused by natural processes such as mineral
causing this liquid zone to travel slowly through a weathering or by the gradual withdrawal of an ocean.
relatively long ingot, or charge, of the solid. 295. (b) 296. (b)

Downloded From : www.EasyEngineering.net


Downloded From : www.EasyEngineering.net

82 General Science
297. (c) Saffron, a spice derived from the dried stigmas of the 320. (b) The most electronegative element among the following
saffron crocus. (crocus sativus) a small plant about a is fluorine. Fluorine is a chemical element with symbol
foot tall. Each flower has three female parts. (stigmas) F and atomic number 9. It is the lightest halogen and
two male parts. (stamens) each stigmas is red or dark exists as a highly toxic pale yellow diatomic gas at
red in color towards the top and yellow towards the standard conditions. As the most electronegative
bottom of the stigma, where it is attached to the flower. element, it is extremely reactive: almost all other
298. (d) 299. (a) 300. (a) 301. (b) 302. (c) elements, in cluding some noble gases, form
303. (d) 304. (d) 305. (a) 306. (c) 307. (c) compounds with fluorine.
308. (c) 309. (c) 310. (b) 311. (a) 312. (b) 321. (b) NH4Cl is the compound which possesses ionic,
covalent and coordinate bonds. Ammonium chloride,
313. (a) The basic unit of nervous system is known as neuron.
an inorganic compound with the formula NH?Cl, is a
The nervous system consists of nerves, brain and
white crystalline salt, highly soluble in water. Solutions
spinal cord. They control the working of various
of ammonium chloride are mildly acidic. Sal ammoniac
organs of the body. The part of brain and spinal cord
is a name of the natural, mineralogical form of
is called the Autonomic Nervous System.
ammonium chloride
314. (a) A cellulosic wall is found in the cells of plants.Cellulose
322. (a) The depletion of Ozone layer is mainly due to
is an important structural component of the primary

ww
cell wall of green plants, many forms of algae and the
oomycetes. Some species of bacteria secrete it to form
biofilms. Cellulose is the most abundant organic
chlorofluorocarbons. A chlorofluorocarbon is an
organic compound that contains only carbon, chlorine,
and fluorine, produced as a volatile derivative of

polymer on Earth.
w.E
315. (c) The filtration unit of kidney is known as nephron.
methane, ethan e, and propane. They are also
commonly known by the DuPont brand name Freon.
323. (c) Bacterium is associated with tuberculosis, Fungus

asy
Kidneys filter the nitrogenous waste products of the
body through nephron and throw them out in the form
of urine. Kidneys and skin are the chief organs of
with ringworm, Protozoan with kaala-azaar and virus
with influenza.

excretion.
En
316. (d) The nutritive tissue in the seeds of higher plants is
324. (b) Matter waves are de Broglie waves. In quantum
mechanics, the concept of matter waves or de Broglie
waves reflects the wave-particle duality of matter. Th e
known as endosperm. Endosperm is the tissue
produced inside the seeds of most flowering plants gin theory was proposed by Louis de Broglie in 1924 in
his PhD thesis. The de Broglie relations show that the
around the time of fertilization. It surrounds the embryo
and provides nutrition in the form of starch, though it
eer
wavelength is inversely proportional to the momentum
of a particle and is also called de Broglie wavelength.
can also contain oils and protein.
317. (b) Yeast is an important source of vitamin B. Yeasts is
eukaryotic microorganisms classified in the kingdom ing
325 (a) When the milk is churned vigorously, the cream is
separated out due to centrifugal force. Centrifuga l

Fungi, with 1,500 species (estimated to be 1% of all


fungal species). Yeasts are unicellular, although some .ne
force is the apparent force that draws a rotating body
away from the center of rotation. It is caused by the
inertia of the body as the body's path is continually
species with yeast forms may become multicellular
through the formation of strings of connected budding
cells known as pseudohyphae, or false hyphae, as
seen in most molds.
redirected.
t
326. (b) Gas thermometers are more sensitive than liquid
thermometers because the gases have large coefficient
of expansion. Coefficient of Thermal expansion is the
318 (d) Enzymes are proteins. Enzymes are large biological tendency of matter to change in volume in response
molecules responsible for the thousands of metabolic to a change in temperature, through heat transfer.
processes that sustain life. They are highly selective When a substance is heated, its particles begin moving
catalysts, greatly accelerating both the rate and more and thus usually maintain a greater average
specificity of metabolic reactions, from the digestion separation.
of food to the synthesis of DNA. Most enzymes are 327. (b) Sun appearing red at sunset is not caused by
proteins, although some catalytic RNA molecules have atmospheric refraction of light. Refraction is
been identified. essentially a surface phenomenon. The phenomenon
319. (c) Heating of ore below its melting point in the absence is mainly in governance to the law of conservation of
of air is known as calcinations. Calcinations is a thermal energy and momentum. Due to change of medium, the
treatment process in presence of air or oxygen applied phase velocity of the wave is changed but its
to ores and other solid materials to bring about a frequency remains constant. This is most commonly
thermal decomposition, phase transition, or removal observed when a wave passes from one medium to
of a volatile fraction another at any angle other than 90° or 0°.

Downloded From : www.EasyEngineering.net


Downloded From : www.EasyEngineering.net

General Science 83
328. (d) Morphine is the most abundant opiate found in opium, 341. (b) Boiling point of water is lower at higher altitudes due
the dried latex from unripe seedpods of Papaver to the decreased air pressure. Boiling point of water
somniferum (the opium poppy). changes with altitude because atmospheric pressure
329. (a) Efforts by the U.S. Soil Conservation Service to changes with altitude.
promote contouring in the 1930s as an essential part 342. (a) Sodium thiosulphate, also called sodium hyposulphite
of erosion control eventually led to its widespread or "hypo" is used as a photographic fixer in photography.
adoption. The practice has been proved to reduce 343. (d)
fertilizer loss, power and time consumption, and wear 344. (d) Stalactites and Stalagmites are formed due to the
on machines, as well as to increase crop yields and
precipitation of Calcium carbonate (CaCO3).
reduce erosion. Contour farming is most effective
345. (d) Hermaphroditism is a form of sexual reproduction in
when used in conjunction with such practices as strip
cropping, terracing, and water diversion. which an organism can self-fertilize or mate with another
individual of the same species.
330. (b) Glucose has the molecular formula C6H12O6 and is
thus a hexose sugar. 346. (b) Lyman series of hydrogen atom spectral lines in the
Ultraviolet.
331. (d) Bacteria are prokaryotes, which, by definition, are cells
that don't possess membrane-bound organelles. 347. (b) An electric battery is a devices consisting of two or
Mitochondria are membrane-bound organelles. more electrochemical cells that convert stored chemical

ww
332. (d) The rainbow comes from the reflection and refraction
of the sunlight in the falling drops; its colors are mainly
due to dispersion, which means that the refractive index 348. (b)
energy into electrical energy.

Plane

w.E
of water depends on the wavelength of light.
333. (c) When a ray of light falls on sea shell, then its small
amount gets refracted (slightly polarised) and rest
Incident
speed X
Mirror

asy
almost gets reflected back (fully polarised).
334. (d) An object covers distance which is directly Reflected
X
is constant.
En
proportional to the square of the time. Its acceleration

Therefore, relative speed= x + x = 2x


335. (b) If the horizontal range of a projectile is four times its
maximum height, the angle of projection is 45 degrees.
Range is the total horizontal distance covered during gin
349. (d) The area reserved for the welfare of wildlife is calle d
National Park.

the time of flight. It is calculated as


u 2 sin 2 q
g
, where
350. (a)
352. (a) eer
351. (c)
Turner Syndrome, represented by 44A + XO. This occu r

336. (c)
q is the angle of projection.
Mercury has the least melting point of the following
missing. ing
in females in which one of the X-chromosome i s

metals (–38.83°C). Gold, silver and copper have 1064°C,


961.8°C and 1085°C respectively.
353. (d)
.ne
Myopia also known as short sightedness is a condition
of the eye where one looking at a distant object seems
337. (d) Marsh Gas or methane gas is produced when
vegetation decomposes in water. Methane, also called
marsh gas, colourless, odourless gas that occurs
abundantly in nature as the chief constituent of natural
354. (d)
a little out of focus.

t
In the process of photosynthesis, phytoplankton
release oxygen into the water. Half of the world’s
oxygen is produced from phytoplankton
gas, as a component of firedamp in coal mines, and as photosynthesis. The other half is produced via
a product of the anaerobic bacterial decomposition of photosynthesis on land by trees, shrubs, grasses and
vegetable matter under water. other plants.
338. (c) In cactus, spines are modified leaves, and thorns are 355. (a) 356. (a)
modified branches. 357. (d) Rate of Transpiration increases in hot, dry and windy
339. (b) Mycoplasma are the smallest bacterial cells yet condition.
discovered, can survive without oxygen and are 358. (d) Collateral Bundle is a type of vascular bundle in which
typically about 0.1 µm in diameter. the phloem and xylem lie on the same radius, with the
340. (d) Biodiversity can be thought of as the genetic library phloem located towards the periphery of the stem and
that keeps life going on Earth. Biodiversity, short for xylem towards the centre.
biological diversity, is the term used to describe the
359. (b) The Planck constant has dimensions of physical action,
variety of life found on Earth and all of the natural
these are the same as those of angular momentum. In
processes. This includes ecosystem, genetic and
SI units, the Planck constant is expressed in joule
cultural diversity, and the connections between these
and all species. seconds (J.S)

Downloded From : www.EasyEngineering.net


Downloded From : www.EasyEngineering.net

84 General Science
360. (a) Cnidaria (e.g., jellyfish, corals, sea anemones). Several
361. (b) Polarization is a property of waves that can oscillate such capsules occur on the body surface. Each is
with more than one orientation. Electromagnetic waves produced by a special cell called a cnidoblast and
such as light exhibit polarization, as do some other contains a coiled, hollow, usually barbed thread, which
types of wave, such as gravitational waves. quickly turns outward (i.e., is everted) from the capsule
362. (c) Ramanathapuran district of Tamil Nadu has vast upon proper stimulation. The purpose of the thread,
stretches of saline and alkaline soils in the wastal which often contains poison, is to ward off enemies or
region. The permeability rate of the sandy soil is upto to capture prey.
5 cm/hour and therefore, water availibility for the 376. (a) Lactobacillus, also called Döderlein's bacillus, is a
cultivation of crops is very less. genus of Gram-positive facultative anaerobic or
363. (b) microaerophilic rod-shaped bacteria.
364. (c) Minamata disease is a neurological syndrome caused 377. (c) Chlorophyll is a green pigment found in cyanobacteria
by severe mercury poisoning. and the chloroplasts of algae and plants. Its name is
365. (d) Gold is often referred as king of metals. derived from the Greek words chloros (“green”) and
phyllon (“leaf ”). Chlorophyll is an extremely important
366. (b)
biomolecule, critical in photosynthesis, which allows
367. (a) Fermi is the unit of length used to measure nuclear
plants to absorb energy from light.

368. (b) ww
distances. The unit is named after the Italian physicist
Enrico Fermi.
378. (b) In physics, the kinetic energy of an object is the energy
which it possesses due to its motion. It is defined as
the work needed to accelerate a body of a given mass

w.E
369. (d) Christmas factor is one of the proteases of the
coagulation system.
370. (a) “Romancing with life” is a memoir written by leading
from rest to its stated velocity. Having gained this
energy during its acceleration, the body maintains this
kinetic energy unless its speed changes.
Bollywood star, Dev Anand.
asy
371. (b) Highly concentrated sources of oxygen promote rapid
379. (c) The ozone layer is a layer in Earth's atmosphere that
absorbs most of the Sun's UV radiation. It contains

En
combustion. Fire and explosion hazards exist when
concentrated oxidants and fuels are brought into close
relatively high concentrations of ozone (O3), although
it is still very small with regard to ordinary oxygen, and
proximity; however, an ignition event, such as heat or
a spark, is needed to trigger combustion. Oxygen itself
is not the fuel, but the oxidant. gin is less than ten parts per million, the average ozone
concentration in Earth's atmosphere being only about
0.6 parts per million. The ozone layer is mainly found in
372. (a) The Dumas method in analytical chemistry is a method
for the quantitative determination of nitrogen in eer
the lower portion of the stratosphere from
approximately 20 to 30 kilometres (12 to 19 mi) above
chemical substances based on a method first described
by Jean-Baptiste Dumas in 1826.
ing
Earth, though the thickness varies seasonally and
geographically.
373. (a) Deoxyribonucleic acid (DNA) is a molecule that
encodes the genetic instructions used in the
development and functioning of all known living
380. (a)

.ne
Airlift pumps are often used in deep dirty wells where
sand would quickly abrade mechanical parts. (The
compressor is on the surface and no mechanical parts
organisms and many viruses. DNA is a nucleic acid;
alongside proteins and carbohydrates, nucleic acids
compose the three major macromolecules essential for
all known forms of life.
much deeper than the water table to allow for t
are needed in the well). However airlift wells must be

submergence. Air is generally pumped at least as deep


under the water as the water is to be lifted. (If the water
374. (a) A steroid hormone (abbreviated as sterone) is a steroid table is 50 ft below, the air should be pumped 100 feet
that acts as a hormone. Steroid hormones can be deep).
grouped into five groups by the receptors to which 381. (a) The passenger-side mirror on a car is typically a convex
they bind: glucocorticoids, mineralocorticoids, mirror. In some countries, these are labeled with the
androgens, estrogens, and progestogens. Vitamin D safety warning “Objects in mirror are closer than they
derivatives are a sixth closely related hormone system appear”, to warn the driver of the convex mirror's
with homologous receptors, though they are distorting effects on distance perception. Convex
technically sterols rather than steroids. Steroid mirrors are preferred in vehicles because they give an
hormones help control metabolism, inflammation, upright, though diminished, image. Also they provide
immune functions, salt and water balance, development a wider field of view as they are curved outwards.
of sexual characteristics, and the ability to withstand 382. (c) non-luminous zone
illness and injury. 383. (d) Gamma radiation, also known as gamma rays, and
375. (a) nematocyst, minute, elongated, or spherical capsule denoted by the Greek letter, refers to electromagnetic
produced exclusively by members of the phylum radiation of extremely high frequency and therefore

Downloded From : www.EasyEngineering.net


Downloded From : www.EasyEngineering.net

General Science 85
high energy per photon. Gamma rays are ionizing 394. (c) Aerenchyma refers to spaces or air channels in the
radiation, and are thus biologically hazardous. leaves, stems and roots of some plants, which allows
384. (a) Gives us atmospheric pressure - Ozone layer shields exchange of gases between the shoot and the root.
earth from UV radiation - Ionosphere layer allows radio Aerenchyma is widespread in aquatic and wetland
waves to be bounced off and radios to work - Prevents plants which must grow in hypoxic soils.
water from vaporising off the earth thanks to 395. (b) Night blindness is the first sign of vitamin A deficiency.
atmospheric pressure. Night blindness and its worsened condition,
385. (d) Bacillus Calmette-Guérin is a vaccine against xerophthalmia, are markers of vitamin A deficiency, as
tuberculosis that is prepared from a strain of the it can also lead to impaired immune function, cancer,
attenuated live bovine tuberculosis bacillus, and birth defects.
Mycobacterium bovis, that has lost its virulence in
396. (a) Cloves are the aromatic flower buds of a tree in the
humans.
family Myrtaceae, Syzygium aromaticum. They are
386. (b) A greenhouse gas (sometimes abbreviated GHG) is a native to the Maluku Islands in Indonesia, and are
gas in an atmosphere that absorbs and emits radiation
commonly used as a spice.
within the thermal infrared range. This process is the
397. (c) Heating gypsum partially dehydrates the mineral by
fundamental cause of the greenhouse effect. The
primary greenhouse gases in the Earth's atmosphere driving off approximately 75% of water contained in its

ww
are water vapour, carbon dioxide, methane, nitrous
oxide, and ozone.
chemical structure. The partially dehydrated mineral is
called calcium sulfate hemihydrate or calcined gypsum
commonly known to be Plaster of Paris.
387. (d)

w.E
Hydrogen bomb is based on the principle of nuclear
fusion. Hydrogen bomb or H-bomb, weapon deriving
a large portion of its energy from the nuclear fusion of
398. (c) Neutron was discovered by a British Physicist named
Sir James Chadwick. In1932, Chadwick showed that
the radiation from the element beryllium, caused by

asy
hydrogen isotopes. The hydrogen bomb functions by
the fusion, or joining together, of lighter elements into
heavier elements. The end product again weighs less
the bombardment of alpha particles is actually a stream
of electrically neutral particles. He called these particles
than its components, the difference once more
appearing as energy. Because extremely high
En neutrons. He also studied some other properties of
these particles. Neutrons directly emitted from atomi c
temperatures are required in order to initiate fusion
reactions, the hydrogen bomb is also known as a
thermonuclear bomb.
gin nuclei are termed as fast neutron.
399. (c) Carbonated beverages like soda get their name from

388. (b) The commonly used safety fuse wire is made up of an


alloy of tin and lead. Fuse is a safety device used in eer
the carbon dioxide dissolved in the beverage. Most
gases will dissolve in water (which is the main
component of soda) but carbon dioxide is particularly
any electrical installation, which forms the weakest link
between the supply and the load. It is a short length of ing
good at dissolving and it adds a nice slightly sour
taste to the beverage. It may also be added to make ti
wire made of lead / tin /alloy of lead and tin/ zinc having
a low melting point and low ohmic losses. A fuse wire
should always have a high resistance and low melting
beverages like champagne. .ne
reminiscent of the carbonation found in fermented

point. High resistance to heat up soon and low melting


point to melt away due to the heat produced by high
resistance so that the circuit is not damaged
t
400. (c) Bio gas is a clean unpolluted and cheap source of
energy in rural areas. It consists of 55-70% methane
which is inflammable. Bio gas is produced from cattle
dung in a bio gas plant commonly known as gobar gas
389. (b) When cooled from room temperature liquid water
plant through a process called digestion.
becomes increasingly dense, as with other substances,
but at approximately 4 °C (39 °F), pure water reaches 401. (c) Carbon credit is a permit which allows a country or
its maximum density. As it is cooled further, it expands organization to produce a certain amount of carbon
to become less dense. emissions and which can be traded if the full allowance
390. (b) The linear expansion of a solid rod is independent of is not used. It is associated with protection of
the time of heat flow. It is directly dependent on the environment.
temperature, length of the rod and material of the rod. 402. (b) A green leaf is green because of the presence of a
391. (b) X-rays are produced as a result of obstruction of pigment known as chlorophyll, which is inside an
cathode rays with metal. organelle called a chloroplast. When they are abundant
392. (c) Aristotle is the father of Biology and Zoology. in the leaf's cells, as they are during the growing
393. (a) the smallest unit of classification is species. The season, the chlorophylls' green color dominates and
hierarchy of biological classification's major taxonomic masks out the colors of any other pigments that may
ranks is as follows: species< genus< family < order< be present in the leaf. Thus the leaves of summer are
class< phylum < kingdom < domain<life. characteristically green

Downloded From : www.EasyEngineering.net


Downloded From : www.EasyEngineering.net

86 General Science
403. (c) Temperature is measured by a thermometer. One of the material which absorbs it. and then ejects incident
most common devices for measuring temperature is electrons. The reason that the photoelectric effect is
the glass thermometer. This consists of a glass tube evidence for the particle nature of light has to do with
filled with mercury or some other liquid, which acts as how materials absorb that light energy and then eject
the working fluid. Temperature increase causes the fluid it in the form of electrons.
to expand, so the temperature can be determined by 427. (d) The heaviest element in terms of heaviest per a given
measuring the volume of the fluid. Such thermometers number of atoms is the element with the highest atomic
are usually calibrated so that one can read the weight. This is the element with the largest number of
temperature simply by observing the level of the fluid protons, which is presently element 118, ununoctium.
in the thermometer. Unnoctium is the heaviest element, but it is man-made.
404. (a) At 25° C the pH of pure water is very close to 7. Acids The heaviest naturally-occurring element is uranium
have a pH less than 7 while bases have a pH greater (atomic number 92, atomic weight 238.0289).
than 7. Because it has a pH of 7, water is considered to 428. (b) Haptens are small molecules that react with a specific
be neutral. It is neither an acid nor a base, but is the antibody but cannot induce the formation of antibodies
reference point for acids and bases. unless bound to a carrier protein or other large antigenic
405. (c) Distillation is a process of separating the component molecule. They are incomplete or partial antigens.
substances (impurities) from a liquid mixture by 429. (a) Sulphur dioxide gas exhibits bleaching properties in

ww
selective evaporation and condensation. The end
result is pure water.
406. (b) H2SO4 is the sulphuric acid which is a highly corrosive
presence of moisture. It dissolves in water liberating
nascent hydrogen. Nascent hydrogen removes oxygen
atoms from the coloring matter (reduces coloring matter)

as a preservative. w.E
strong mineral acid . It cannot be used in food industry
430. (c)
and it loses its color.
Zirconium is a lustrous, grey-white. strong transition

asy
407. (d) The deep red colour that is present in tomatoes, pink
grapefruit, guava and watermelon is caused by
lycopene, a carotenoid.
metal that resembles titanium. Zirconium is mainly used
as a refractory and opacifier, although it is used in
small amounts as an alloying agent for its strong
408. (d) 409. (a)
En
410. (b) When a person cries there is a watery discharge from 431. (d)
resistance to corrosion.
Plant wilting occurs after excessive loss of water yb
the nose due to activation of lachrymal fluid through
nasal cavity. gin transpiration and lesser absorption. On a hot day, plants
transpire heavily and cannot absorb water speedily ot
keep pace with transpiration loss, even when there si
411. (c) The functions of root hairs is to collect water and
mineral nutrients present in the soil and take this
solution up through the roots to the rest of the plant. 432. (b) eer
enough water.
Gypsum is calcium sulphate which is essential to the
As root hair cells do not carry out photosynthesis
they do not contain chloroplasts. ing
sustainability of most irrigated soils. Irrigated land
eventually leads to sodicity and salinity unless extrem e
412. (d) In agriculture, a catch crop is a fast-growing crop that
is grown simultaneously with, or between successive .ne
care is taken. It improves sodic (saline) soils by
removing sodium from the soil and replacing it with
plantings of a main crop.
413. (d) Carotenoids protect the plant from damage from UV
light and this protection is attributed to their antioxidant
properties: they are able to absorb the harmful oxidative
433. (d)
calcium.

t
The Electromagnetic spectrum in order of decreasing
energy, thus increasing wavelength : Gamma Rays-
wavelength: 0.01 nm : X-rays -1 nm ; Ultra-violet Rays-
chemicals produced during photosynthesis, the O.l micrometres ; Visible Light-Red light : 0.7
process whereby plants make energy from sunlight. micrometres. Violet Light: 0.4 micrometres ; Infrared
414. (d) Radiation-0.01 mm ; Microwaves -less than 10 cm.
usually 1 cm : Radiowaves- Long, Medium and Short
415. (d) The substance that causes the worst air pollution is
Waves : 2 km-l0 m ; and Very High frequency (VHF)
carbon monoxide (CO).
and Ultra High Frequency (UHF): 10 m-l 0 cm.
416. (a) 417. (b) 418. (b) 419. (a) 420. (a)
434. (a) Eutrophication is the eco-system response to the addi-
421. (a) 422. (a) 423. (d) 424. (c)
tion of artificial or natural substances, such as nitrates
425. (b) Pyrolysis is a thermo-chemical decomposition of and phosphates. through fertilizers or sewage to an
organic material at elevated temperatures in the absence aquatic system. One example is the "bloom" or great
of oxygen (or any halogen). Pyrolysis generally takes increase of phytoplankton in a water body as a
place well above atmospheric pressure at temperatures response to increased levels of nutrients.
exceeding above 430 °C.
435. (d) The wings are modified forelimbs of birds which are
426. (c) Light is made up of photons. so it has some particle the key to bird flight. Each wing composed of three
properties. In the photoelectric effect. light hits some limb bones, the humerus, ulna and radius.

Downloded From : www.EasyEngineering.net


Downloded From : www.EasyEngineering.net

General Science 87
436. (b) Entropy and enthalpy together make a new quantity, "glues" the beak to the bones, and the beak, which is
the Free Energy which, under conditions of constant the outer sheath covering the jaw bones. The beak is
temperature and pressure, determines the direction of composed of keratin - the same tough, insoluble protein
spontaneous change. All spontaneous processes have found in fingernails, hoofs, antlers and horns.
a negative change in free energy. 445. (c) External ear is present in Mammals. Many mammals
437. (c) Sandstone can be turned into a metamorphic rock by can move the pinna (with the auriculares muscles) in
being either heated up to a point where it undergoes a order to focus their hearing in a certain direction in
significant change or subjected to high pressures, or much the same way that they can turn their eyes. Most
subjected to chemically active fluids, or some humans, unlike most other mammals, do not have this
combination thereof. A common result of the ability.
metamorphosis of sandstone is the creation of 446. (c) Archimedes used the principle of buoyancy to find
quartzite. the purity of gold. According to which a body immersed
438. (d) A Sonometer is a device for demonstrating the in fluid experiences a buoyant force equal to the weight
relationship between the frequency of the sound of the fluid it displaces. With the help of this principle,
produced by a plucked string, and the tension, length the density of the golden crown and solid gold was
and mass per unit length of the string. These compared by balancing the crown on a scale with a
relationships are usually called Mersenne's laws after reference piece, this set up is then immersed in water. If

ww
Marin Mersenne (1588-1648), who investigated and
codified them.
439. (b) Polio is a highly infectious disease caused by a virus.
the crown is less dense than gold, it will displace more
water, and thereby will experience a greater buoyant
force than the reference piece.

w.E
Its causative agent, poliovirus, was identified in 1908
by Karl Landsteiner. The polio virus invades the
nervous system, and can cause total paralysis in a
447. (c) The kinetic energy of a non-rotating object of mass m
1 2
traveling at a speed v is mv . If m and v are increased
matter of hours.
asy
440. (d) It is generally considered that the Panda belongs to
2
to twice its magnitude, then K.E= ½ × 2m × 2v × 2v =

En
the bear family, though differences persist. Some DNA
studies have shown that the giant panda is closer to
1 2
2
mv (8) = 8 times kinetic energy..
the bear family while the red panda is indeed closer to
the raccoon family.
441. (a) The pancreas is a glandular organ in the upper
gin
448. (d) The gravitational force is inversely proportional to the
square of the distance: If you double the distance

abdomen, but really it serves as two glands in one: a


digestive exocrine gland and a hormone-producing eer
between the two bodies, the force of gravity is reduce d
to one-fourth its original value.

endocrine gland. Functioning as an exocrine gland,


the pancreas excretes enzymes to break down the ing
449. (d) Stationary wave can occur because the medium is
moving in the opposite direction to the wave, or it can
arise in a stationary medium as a result of interference
proteins, lipids, carbohydrates, and nucleic acids in
food. Functioning as an endocrine gland, the pancreas
secretes the hormones insulin and glucagon to control .ne
between two waves travelling in opposite directions
450. (b) Two oxygen atoms can both achieve stable structures
blood sugar levels throughout the day.
442. (b) Due to the action of motor cells at the base of the
petiole and leaflets, the leaves of touch me not plant
electrons.
t
by sharing two pairs of electrons joined in a double
bond. O=O. Each line represents one pair of shared

451. (d) Helium is used by deep sea divers in their diving tanks
closes. The motor cells are activated by touch of the
as a substitute of nitrogen. The trouble with nitrogen
sensitive hairs present on the surface of touch-me-not
in this situation is that nitrogen is a fairly heavy gas,
plant. Like a number of other plant species, it undergoes
and is soluble in blood at high pressure. Long term use
changes in leaf orientation termed "sleep" or nyctinastic
of nitrogen can cause a strange sense of euphoria, or
movement. The foliage closes during darkness and
well being called nitrogen narcosis. This is a bit like
reopens in light.
being drunk, and makes the diver unable to assess
443. (c) The theoretical basis for plant tissue culture was
dangers. Divers who work at depth or for long periods
proposed by Gottlieb Haberlandt, German Academy of
use a mixture of 20% Oxygen and 80% Helium. Helium
science in1902 on his experiments on the culture of
is used for a number of reasons - It is light, cheap, and
single cell.
does not dissolve in blood the same way that nitrogen
444. (d) Beak is technically only the external surface of a bird's does. Being inert it cannot be toxic to the diver or
mouth. The entire mouth structure of a bird is called corrosive to equipment.
the bill. The bill (or rostrum) consists of a bony
452. (c) The atomic number of uranium is 92, and the mass
framework, a vascular layer containing the blood
number of the isotope is given as 238. Therefore, it has
vessels and nerves, a layer of connective tissue, which
92 protons, 92 electrons, and 238 – 92 = 146 neutrons.

Downloded From : www.EasyEngineering.net


Downloded From : www.EasyEngineering.net

88 General Science
453. (d) Auxanometer measure root pressure which is of water in and out of the cell but restricts the flow of
developed in the xylem of the roots . The auxanometer solute under many circumstances. As a result, when a
measures plant growth as well as the pressure cell is placed in a hypotonic solution, water rushes
developed within the xylem cells of roots. into the membrane, increasing the cell's volume.
454. (c) Cell becomes turgid because of endosmosis. Plant cells 457. (d)
need to be turgid (i.e. rigid) to support plant tissues. 458. (c) Chemotherapy is the use of medicines or drugs to treat
Plant cells become turgid when the concentration inside cancer.
the cell is higher than outside. Therefore water moves 459. (d) Oxygen is the most abundant element, making up about
into the cell by osmosis, and the vacuole swells and 47% of the earth's mass.
pushes against the cell wall. Endosmosis is the 460. (c) 461. (a) 462. (c)
movement of water molecules from outside to inside of
463. (d) Edward Jenner is often called "the father of
a cell through osmosis process.
immunology".
455. (c) Imbibition is a process by which a photographic print
464. (c) 465. (a)
is produced by absorption of a water-soluble dye by a
466. (d) Potash Alum is used to reduce bleeding in minor cuts
relief image or a differentially absorbing image in gelatin
and abrasions, nosebleeds, and haemorrhoids.
or a similar medium or in which a previously formed
dye image is transferred by absorption from one layer 467. (a)

ww
into another layer
456. (b) A physical phenomenon known as osmosis causes
water to flow from an area of low solute, high water
468. (c) Interferon is produced by the body's cells as a
defensive response to viruses.
469. (b) The first law of thermodynamics is a version of the law

w.E
concentration to an area of high solute, low water
concentration, until the two areas have an equal ratio
of solute to water. Normally, the solute diffuses toward
of conservation of energy.
470. (a) 471. (a) 472. (c) 473. (d) 474. (d)
475. (b) 476. (b) 477. (a) 478. (c) 479. (a)

asy
equilibrium as well; however, all cells are surrounded
by a lipid bilayer cell membrane which permits the flow
480. (c) 481. (c) 482. (c)

En
gin
eer
ing
.ne
t

Downloded From : www.EasyEngineering.net


Downloded From : www.EasyEngineering.net

6
CHAPTER
COMPUTERS
1. The messenger satellite launched by NASA is to study (c) North–East
(SSC CGL 1st Sit. 2010) (d) Indo–Pak Border in Punjab and Rajasthan
(a) Mercury (b) Venus 10. The Headquarters of MCF (Master Control Facility) – the
(c) Satrun (d) Jupiter nerve centre of the entire space craft operations in India is at
2. An Intelligent terminal (SSC CGL 1st Sit. 2010) (SSC CGL 2nd Sit. 2011)
(a) has a microprocessor, but can-not be programmed by (a) Hyderabad – Andhra Pradesh
the user (b) Thumba – Kerala
(b) can process small data processing jobs, with the use of

ww
a large CPU
(c) interacts with the user in English
11.
(c) Sriharikota
(d) Hassan
– Andhra Pradesh
– Karnataka
In which of the following areas, a spreadsheet software is

3. w.E
(d) cannot take data from the user
Which of the following countries has recently become the
third largest market for Twitter ? (SSC CGL 2nd Sit. 2010)
more useful?
(a) Psychology
(SSC CGL 2nd Sit. 2011)
(b) Publishing

(a) China
(c) Brazil
(b) India
asy
(d) Indonesia
12.
(c) Statistics
A Group Ware is a
(a) Hardware
(d) Message sending
(SSC CGL 2nd Sit. 2011)
(b) Network
4. Processor's speed of a computer is measured in
En
(SSC CGL 2nd Sit. 2010)
13.
(c) Software (d) Firmware
The transfer of date from a CPU to peripheral devices o f
(a) BPS
(c) Baud
(b) MIPS
(d) Hertz gin computer is achieved through
(a) interfaces
(SSC CGL 1st Sit. 2012)
(b) buffer memory
5. 'C' language is a
(a) Low level language (b)
(SSC CGL 2nd Sit. 2010)

14.
(c) modems
eer (d) computer ports
Which of the following items is not used in Local Area
High level language (c)
Machine level language
Networks (LANs)?
(a) Interface Card ing(b) Cable
(SSC CGL 1st Sit. 2012)

6.
(d) Assembly level language
Window 7, the latest operating system from Microsoft
Corporation has .............. Indian languages fonts. 15.
(c) Computer
What is m-commerce?
(d) Modem
.ne
(SSC CGL 2nd Sit. 2012)

(a) 14
(c) 37
(b) 26
(d) 49
(SSC CGL 2nd Sit. 2010)

16.
(a) machine commerce (b) mobile commerce
(c) money commerce (d) marketing commerce
Who invented the modem ?
t
(SSC CGL 1st Sit. 2012)
7. The first computer made available for commercial use was: (a) Apple Computers Inc.
(SSC CGL 1st Sit. 2011) (b) Digital Equipment Corporation
(a) MANIAC (b) ENIAC (c) Wang Laboratories Ltd.
(c) UNIVAC (d) EDSAC (d) AT&T Information Systems
8. A communication network which is used by large 17. What is the name given to the smallest unit of data space
organizations over regional, national or global area is called: available in a spreadsheet software application ?
(SSC CGL 1st Sit. 2011) (SSC CGL 1st Sit. 2012)
(a) LAN (b) WAN (a) Row (b) Block
(c) MAN (d) VAN (c) Cell (d) Pixel
9. Indian Army's Operation 'Saiyam' was related to : 18. An alternate name for the completely interconnected network
(SSC CGL 1st Sit. 2011) topology is (SSC CGL 2nd Sit. 2012)
(a) Kashmir (a) Mesh (b) Star
(b) Indo–China Border in the Central Region (c) Tree (d) Ring

Downloded From : www.EasyEngineering.net


Downloded From : www.EasyEngineering.net

90 Computer
19. Communication satellites are used to : 29. What is used to identify whether a data word has an odd or
(SSC CGL 1st Sit. 2013) even number of 1's ? (SSC CGL 1st Sit. 2015)
(a) transmit communication signal only (a) Sign bit (b) Zero bit
(b) receive communication signal only (c) Parity bit (d) Carry bit
(c) receive and redirect communication signal 30. Rearranging and allocating space in memory to provide for
(d) provide information of natural resources only multiple computing tasks is called
20. What type of information system would be recognised by (SSC CGL 1st Sit. 2015)
digital circuits ? (SSC CGL 1st Sit. 2013) (a) Multiprogramming
(a) Hexadecimal system (b) Multitasking
(b) Binary system (c) Memory Management
(c) Both hexadecimal and binary system (d) Networking
(d) Only Roman system 31. A computer executes programs in the sequence of :
21. Which of the following is an impact printe ? (SSC CGL 1st Sit. 2015)
(SSC CGL 1st Sit. 2013) (a) Decode, Fetch, Execute
(a) Daisy wheel printer (b) Execute, Fetch, Decode
(c) Fetch, Decode, Execute

ww
(b) Ink jet printer
(c) Bubble jet printer
(d) Laser printer
(d) Store, Fetch, Execute
32. Which of the following is in the ascending order of Data

w.E
22. A television channel is characterised by
(SSC CGL 2nd Sit. 2013)
(a) frequency of transmitted signal
hierarchy ? (SSC CGL 1st Sit. 2015)
(a) Bit–Byte – Record – Field – Database – File
(b) Byte – Bit – File – Record – Database – Field
(b) velocity of transmitted signal
asy
(c) physical dimension of television screen
(c) Bit– Byte – Field – Record – File – Database
(d) Field – Byte – Bit – Record – File– Database
(d) size of picture tube
En
23. Who is the founder of “Facebook” which is currently the
33. Data in database at a particular point of time is called as?
(SSC CGL 1st Sit. 2016)
No. 1 social networking website in India?
(SSC CGL 1st Sit. 2013) gin(a) Intension
(c) Back up
(b) Extension
(d) Application
(a) Orkut Buyukkokten (b) Mark Zuckerberg
(c) Bill Gates (d) Martin Cooper eer
34. CPU Scheduler is also known as _____ .

(a) Job Scheduler


(SSC CGL 1st Sit. 2016)
24. Which computer was the first to use the magnetic drum for
memory ? (SSC CGL 2nd Sit. 2013) (b) Resource Scheduler
ing
(c) Short-term Scheduler
(a) IBM - 650
(c) IBM - 701
(b) IBM - 7090
(d) IBM - 360
25. Identify the FIFO (First In First Out) structure among the
(d) Process Scheduler
.ne
35. Which of the following is not an operating system?
following :
(a) Stack
(c) De-queue
(b) Queue
(d) Array
(SSC CGL 2nd Sit. 2013)
(a) Android
(c) iOS
(b) Vista
(d) Opera
t
(SSC CGL 1st Sit. 2016)

26. Which was the first electronic computer constructed at the 36. In IT, the technique of delaying outgoing acknowledgements
Moore School of Engineering? (SSC CGL 1st Sit. 2013) temporarily is known as (SSC CGL 1st Sit. 2016)
(a) EOVAC (b) ONIVAC (a) AR Acknowledgement
(c) ENIAC (d) EDSAC (b) AR request
27. Which among the following standard protocols is the most (c) Piggybacking
widely used by the Internet? (SSC CGL 1st Sit. 2013) (d) Piggyframing
(a) HTTP (b) TCP/IP 37. CPU performance is often measured in:
(c) SMTP (d) SLIP (SSC CHSL 2012)
28. Who among the following introduced the world's first laptop (a) MHz (b) MIPS
computer in the market? (SSC CGL 2014) (c) Band rate (d) G B
(a) Hewlett-Packard 38. The indentations on CDs and DVDs are called:
(b) Epson (SSC CHSL 2012)
(c) Laplink travelling software Inc (a) pits (b) clusters
(d) Microsoft (c) tracks (d) lands

Downloded From : www.EasyEngineering.net


Downloded From : www.EasyEngineering.net

Computer 91
39. Programs which protect a disk from an infection are called 50. Image files can be sent along with the e-mail documents
as : (SSC CHSL 2012) using (SSC Multitasking 2014)
(a) Vaccines (b) Antidotes (a) Attachments (b) Subject
(c) Library routines (d) Interpreters (c) Signature (d) CC & BCC
40. A nibble is equal to _____________ bits. 51. The invention of _____________ led to the third generation
(SSC CHSL 2013) of computers. (SSC Multitasking 2014)
(a) 16 (b) 32 (a) Vacuum tubes
(c) 4 (d) 8 (b) Very Large Scale Integration (VLSI)
41. In HTML, tags consists of keywords enclosed within (c) Transistors
(SSC CHSL 2013) (d) Integrated chips
(a) flower brackets { } 52. In a _____________ network, all devices are connected to
(b) angular brackets < > a device called a hub and they communicate through it.
(c) parentheses ( ) (SSC Multitasking 2014)
(d) square brackets [ ] (a) Ming (b) Mesh
42. printer cannot print more than one character at a time. (c) Bus (d) Star

ww
(a) Line
(SSC CHSL 2013)
(b) Daisy-wheel
53. Which one of the following acts as a mediator between the
user’s program and the hardware?

43.
(c) Laser
w.E (d) Dot-matrix
The waste generated by end life personal computers is
known as (SSC CHSL 2013)
(a) Compiler
(c) Operating System
(SSC Sub. Ins. 2012)
(b) Editor
(d) Browser
(a) E-waste
(c) Physical waste asy
(b) PC-waste
(d) Computer waste
54. If you use a font that is not supported by a browser, then
the text (SSC Sub. Ins. 2012)

44. Telnet stands for


(a) Telephone Network En
(SSC CHSL 2014)
(b) Television Network
(a) will be displayed in the default font
(b) will not be displayed

45.
(c) Teletype Network (d) Telefax Network
In operating system, Round Robin Scheduling means: gin (c) will be displayed using ‘ARIAL’ font only
(d) willl be displayed with a particular background

(a) A kind of scheduling


(SSC CHSL 2015)
55.

eer
The telephone line is connected through the______on
the computer. (SSC Sub. Ins. 2013)

(b) A process allocation policy


(c) A memory allocation policy 56.
(a) USB
(c) Ethernet
ing (b) Modem
(d) PS2
In the nuclear reactors, moderators are used to :

46.
(d) Repetition policy
FORTRAN is called : (SSC CHSL 2015) (a) Accelerate neutrons .ne
(SSC Sub. Ins. 2013)

47.
(a) Floppy Translator
(c) File Translator
(b) Formula Translator
(d) Format Translator
All forms of ROM are also known as __________.
(b) Absorb the neutrons
(c) Slow down the neutrons
(d) Generate neutrons
t
(SSC CHSL 2015) 57. Which is not an extension of a picture file on a computer ?
(a) Middleware (b) Firmware (SSC Sub. Ins. 2014)
(c) Shareware (d) Freeware (a) .jpeg (b) .png
48. The Simplest CUP scheduling algorithm is (c) .gif (d) .mdb
(SSC CHSL 2015) 58. Which measure of memory is the largest ?
(a) Round-robin scheduling algorithm (SSC Sub. Ins. 2014)
(b) Multilevel scheduling algorithm (a) MB Megabyte (b) GB Gigabyte
(c) FCFS scheduling algorithm (c) TB Terabyte (d) KB Kilobyte
(d) SJF scheduling algorithm 59. Which of the following is the maximum number of electrons
49. In MS-Excel, _________ provide a visual representation of that can be present in M-shell?
the values in a worksheet. (SSC Multitasking 2013) (SSC Sub. Ins. 2014)
(a) Views (b) Charts (a) 2 (b) 8
(c) Formulae (d) Templates (c) 18 (d) 32

Downloded From : www.EasyEngineering.net


Downloded From : www.EasyEngineering.net

92 Computer
60. RAM is used as a short memory in a computer because it. 64. . ........helps in remote login. (SSC Stenographer 2013)
(SSC Sub. Ins. 2015) (a) Telnet (b) Usenet
(a) is very expensive (b) is programmable (c) Milnet (d) Arpanet
(c) is volatile (d) has less capacity 65. What kind of data can you send by e-mail?
61. In a computer system, which device is functionally opposite (SSC Stenographer 2014)
of a keyboard ? (SSC Sub. Ins. 2015) (a) Audio (b) Video
(a) Joystick (b) Trackball (c) Pictures (d) All of these
(c) Mouse (d) Printer 62. One 66. Which of the following units used to measure the speed of
strategy of allocating kernel memory is known as a computer ? (SSC Stenographer 2016)
(SSC Sub. Ins. 2016) (a) BAUD (b) Byte
(a) frames allocation (c) SYPS (d) MIPS
(b) register allocation 67. DHCP is mainly used to (SSC Stenographer 2016)
(c) slab allocation (d) (a) Converting IP address to domain name
resource allocation (b) Multicasting
63. In MS-Excel, . .........is a collection of rows and columns that (c) Provide IP address automatically to the devices

ww
holds text and numbers.
(a) Workbook
(c) File
(SSC Stenographer 2013)
(b) Worksheet
(d) Record
(d) Routing

w.E
asy
En
gin
eer
ing
.ne
t

Downloded From : www.EasyEngineering.net


Downloded From : www.EasyEngineering.net

Computer 93

HINTS & SOLUTIONS


1. (a) 2. (a) 3. (c) 28. (b) The Epson HX-20 (also known as the HC-20) is
4. (b) Acronym for million instructions per second. A old generally regarded as the first laptop computer,
measure of acomputer's speed and power, MIPS announced in November 1981, although first sold
measures roughly the number of machine instructions widely in 1983. Hailed by BusinessWeek magazine as
that a computer can execute in one second. the "fourth revolution in personal computing", it is
5. (b) 6. (d) 7. (c) 8. (b) generally considered both the first notebook and
9. (d) 10. (d) 11. (c) 12. (c) handheld computer.
29. (c) A parity bit, or check bit is a bit added to the end of a
13. (a) In computing, an interface is a shared boundary across
string of binary code that indicates whether the number
which two separate components of a computer system
of bits in the string with the value one is even or odd.
exchange information. The exchange can be between
Parity bits are used as the simplest form of error
software, computer hardware,peripheral devices,
detecting code.
humans and combinations of these.
14.
ww
(c) Interface Card, Ethernet Cable and Routers are used in
setting up a LAN. The router can be plugged directly
into the modem via an Ethernet cable, and all other
30. (c) Memory management is the process of controlling and
coordinating computer memory, assigning portions
called blocks to various running programs to optimize

w.E
computers are eventually connected in some fashion
to the route. In terms of LAN, a computer cannot be
overall system performance.It involves components
that physically store data, such as RAM (random
access memory) chips, memory caches, and flash-

15. (b) Mobile Commerce,


asy
said to be an item which constitutes this network.
also known
M-Commerce or mCommerce, is the ability to conduct
as 31.
based SSDs (solid-state drives).
(c) The first step the CPU carries out is to fetch some data
instructions from main memory then store them in its
commerce using a mobile device, such as a mobile
phone, a Personal Digital Assistant (PDA), a En own internal temporary memory areas. The next step is
for the CPU to make sense of the instruction it has jus t

16.
smartphone, or other emerging mobile equipment.
(d) 17. (c) 18. (a) 19. (c) 80. (b)
gin fetched this process is called Decode. Execute is th e
part of the cycle when data processing actually take s
place.
21. (a) Daisy wheel printing is an impact printing technology
invented in 1969 by David S. Lee at Diablo Data
Systems. It uses interchangeable pre-formed type
32. (c) ·
eer
Bit (Binary digit) = a bit can only have one of two
possible values.
elements, each with typically 96 glyphs, to generate
high-quality output comparable to premium typewriters
·
· ing
Byte = 1 byte = 8 bits
Field = one or more bits/bytes holding a single

22.
such as the IBM Selectric, but two to three times faster.
(a) A television channel is a physical or virtual channel ·
.ne
piece of information about a single entity
Record (Logical) = all the fields which apply to a
single entity (except for "Inverted" files)
over which a television station or television network is
distributed. Channel numbers represent actual
frequencies used to broadcast the television signal.
For example, in North America. "Channel 2" refers to
33.
·

·
t
File = all the records which apply to a collection
of entities
Database = a related collection of Files.
(b) The data in the database at a particular point of time is
the broadcast or cable band of 54 to 60 MHz, with
known as database instance or database sate or
carrier frequencies of 55.25 MHz for NTSC analog video
snapshot. The database state is also called an
(VSB) and 59.75 MHz for analog audio (FM), or 55.31 extension of the schema.
MHz for digital ATSC (8VSB). 34. (c) The short-term scheduler (also known as the CPU
23. (b) Mark Elliot Zuckerburg is an American computer scheduler) decides which of the ready, in-memory
programmer and internet entrepreneur. He is best processes is to be executed (allocated a CPU) after a
known as one of five co-founders of the social clock interrupt, an I/O interrupt, an operating system
networking website Facebook. call or another form of signal.
24. (a) The IBM 650 Magnetic Drum Data-Processing 35. (d) 36. (c) 37. (b) 38. (c) 39. (b)
Machine was one of IBM's early computers, and the 40. (c) A nibble is equal to 4 bits. In computing, a nibble (often
world's fir st mass-produced computer. It was nybble or even nyble to match the vowels of byte) is a
announced in 1953 and almost 2000 systems were four-bit aggregation, or half an octet. As a nibble
produced, the last in 1962. Support for the 650 and its contains 4 bits, there are sixteen possible values, so a
component units was withdrawn in 1969. nibble corresponds to a single hexadecimal digit (thus,
25. (a) stack 26. (c) 27. (b) it is often referred to as a "hex digit" or "hexit").

Downloded From : www.EasyEngineering.net


Downloded From : www.EasyEngineering.net

94 Computer
41. (b) In html, tags consist of keywords enclosed within of a central node, to which all other nodes are
angular brackets. In the HTML syntax, most elements connected; this central node provides a common
are written with a start tag and an end tag, with the connection point for all nodes through a hub.
content in between. An HTML tag is composed of the 53. (c) 54. (a)
name of the element, surrounded by angle brackets. 55. (b) A modem turns the digital data of a personal computer
An end tag also has a slash after the opening angle into modulated electrical signals in the voice frequency
bracket, to distinguish it from the start tag. range of a telephone channel. These Signals can be
42. (b) Daisy wheel printer cannot print more than one transmitted over telephone lines and demodulated by
character at a time. The daisy - wheel is named after its another modem at the receiver side to recover the digital
shape. The individual characters are arranged on data.
flexible petals (or spokes) 'radiating' from the centre of 56. (c) A moderator is a medium that reduces the speed of fast
the wheel. Each petal has a raised character at its end. neutrons, thereby turning them into thermal neutrons
The spoke hits the ribbon when it is struck by the capable of sustaining a nuclear chain reaction involving
hammer. uranium-235. Commonly used moderators include
43 (a) The waste generated by end life personal computers regular (light) water (roughly 75% of the world's
are known as e-waste. E-waste or "Electronic waste" reactors), solid graphite (20% of reactors) and heavy
may be defined as discarded computers, office water (5% of reactors).

44.
ww
electronic equipment, entertainment device electronics,
mobile phones, television sets, and refrigerators.
(a) The acronym TELNET stands for Telephone Network.
57. (d) .mdb is not a picture file. It is a database file used by
Microsoft Access. Though Access is tied into the

45.
46.
47.
(b)
w.E
(b) FORTRAN is derived from Formula Translator.
(b) Firmware is the permanent software programmed into
Microsoft Office, MDB is Access's own format, which
is based on the Access Jet Database Engine..jpeg; .gif;
and .png files are graphic files. The PNG, JPEG, and GIF

48. (c)
a read-only memory.
asy formats are most often used to display images on the
Internet. JPEG stands for Joint Photographic Experts
Group. PNG stands for Portable Network Graphics. GIF
49.
50.
(b) charts
En
(a) Image files can be sent along with the e-mail using 58. (c)
stands for Graphics Interchange Format.
Terrabyte (TB) is the largest measure of memory. 1 Bi t
attachments.An e-mail attachment is a file that is
attached to an e-mail message. For example, you may gin = Binary Digit; 8 Bits = 1 Byte;
1024 Bytes = 1 KB (Kilo Byte);1024 KB = 1 MB (Mega

51.
attach a graphic, a spreadsheet, or a word processing
document.
(d) The invention of integrated chips led to the third eer
Byte); 1024 MB = 1 GB(Giga Byte)
1024 GB = 1 TB(Terra Byte). Thus TB > GB> MB > KB
> Byte > Bit
generation of computers. During the period of 1964 to
1971 third generation computers were developed. The
59. (c)
ing
Maximum number of electrons present in M-shell are
18. The maximum number of electrons that can occup y
third generation computers emerged with the
development of IC (Integrated Circuits). IC is a single formula.
.ne
a specific energy level can be found using the following

52.
component containing a number of transistors. It made
the computers more fast and reliable.
(d) Star networks are one of the most common computer
network topologies. In its simplest form, a star network 60. (c) 61. (d) 62. (a) 63. (b)
t
Electron Capacity = 2n2, the variable n represents the
Principal Quantum Number. Shell M has principle
quantum number 3.

consists of one central switch, hub or computer, which 64. (a) 65. (d) 66. (d) 67. (c)
acts as a conduit to transmit messages. This consists

Downloded From : www.EasyEngineering.net


Downloded From : www.EasyEngineering.net

7
CHAPTER
GENERAL KNOWLEDGE
1 The book titled 'The Indian War of Independence' was 11. The 'Project Snow Leopard' to conserve the endangered
written by (SSC CGL 1st Sit. 2010) species, launched by the Union Ministry of Environment
(a) Krishna Verma (b) Madame Cama and Forests covers the states of: (SSC CGL 1st Sit. 2011)
(c) B.G. Tilak (d) V.D. Savarkar (a) Jammu & Kashmir and Himachal Pradesh only
2. Who among the following captured his third consecutive (b) Jammu & Kashmir, Himachal Pradesh and Uttarakhand
National Billiards title in the year 2009 ? only
(SSC CGL 1st Sit. 2010) (c) Jammu & Kashmir, Himachal Pradesh, Uttarakhand and
(a) Pankaj Advani (b) Devendra Joshi Arunachal Pradesh only

3. ww
(c) Geet Sethi (d) Dhruv Sitawala
What was the name of the ship that sank near the Paradip
(d) Jammu & Kashmir, Himachal Pradesh, Uttarakhand,
Arunachal Pradesh and Sikkim

(a) Red Rose


w.E
Port in September, 2009 causing an oil spill ?
(SSC CGL 1st Sit. 2010)
(b) Black Rose
12. The 2010 FIFA World Cup Final was held at:

(a) Paris
(SSC CGL 1st Sit. 2011)
(b) Berlin

4.
(c) White Rose
asy
(d) Green Rose
Who among the following has been honoured with the 13.
(c) Johannesburg (d) London
Who of the following received– Sangeet Natak Akademi's
prestigious Dadasaheb Phalke Award for 2007 ?

En
(SSC CGL 1st Sit. 2010)
Ustad Bismillah Khan Yuva Puruskar for 2009 in 'Hindustani
Vocal Music'? (SSC CGL 1st Sit. 2011)

5.
(a) Yash Chopra
(c) Manna Dey
(b) Ustad Amjad Ali Khan
(d) A. Nageshwara Rao
Who among the following won the 'ICC Cricketer of the
gin (a) Ornkar Shrikant Dadarkar
(b) Ragini Chander Shekar
(c) Abanti Chakravorty and Sukracharya Rabha
Year Award' for the year 2009 ?
(a) M.S. Dhoni
(SSC CGL 1st Sit. 2010)
(b) Gautam Gambhir 14. eer
(d) K. Nellai Manikandan
Which of the following folk/tribal dances is associated wit
h

6.
(c) Mitchell Johnson
NIS stands for
(d) Tillakaratne Dilshan
(SSC CGL 2nd Sit. 2010)
Uttar Pradesh?
(a) Veedhi ing (b) Thora
st
(SSC CGL 1 Sit. 2011)

(a) National Infectious diseases seminar


(b) National Irrigation Schedule 15.
(c) Tamasha
.ne
(d) Rauf
Which of the following books has been written by Atiq

7.
(c) National Immunisation Schedule
(d) National Information Sector
Who discovered cement ? (SSC CGL 2nd Sit. 2010)
Rahimi?
(a) Earth and Ashes
(c) The Red Devil
t
(SSC CGL 1st Sit. 2011)
(b) This Savage Rite
(d) Witness the Night
(a) Agassit (b) Albertus Magnus 16. Who is the recepient of the Sahitya Akademi Award 2010 in
(c) Joseph Aspdin (d) Janseen Hindi Literature category? (SSC CGL 1st Sit. 2011)
8. Red data book gives information about species which are: (a) Uday Prakash (b) Laxman Dubey
(SSC CGL 1st Sit. 2011) (c) Nanjil Nandan (d) Mangat Badal
(a) extinct (b) endangered 17. First Indian Prime Minister to visit Siachen has been
(c) dangerous (d) rare (SSC CGL 2nd Sit. 2011)
9. Which of the following is the smallest bird? (a) Rajiv Gandhi (b) Inder Kumar Gujaral
(SSC CGL 1st Sit. 2011) (c) Man Mohan Singh (d) None of the above
(a) Pigeon (b) Parrot 18. Which of the following books has been written by Kishwar
(c) Humming bird (d) House sparrow Desai? (SSC CGL 2nd Sit. 2011)
10. The world's only floating national park is situated in: (a) The Red Devil
(SSC CGL 1st Sit. 2011) (b) Witness the Night
(a) Manipur (b) Kuala Lumpur (c) Tonight This Savage Rite
(c) Bilaspur (d) Dispur (d) Earth and Ashes

Downloded From : www.EasyEngineering.net


Downloded From : www.EasyEngineering.net

96 GENERAL KNOWLEDGE
19. Which of the following folk/tribal dances is associated with 33. The most endangered Asiatic top predator on the edge of
Karnataka? (SSC CGL 2nd Sit. 2011) extinction is (SSC CGL 1st Sit. 2012)
(a) Yakshagana (b) Veedhi (a) Black Bear (b) Asiatic Lion
(c) Jatra (d) Jhora (c) Siberian Tiger (d) Dhole
20. The Headquarters of International Atomic Energy Agency 34. Analects is the sacred book of (SSC CGL 1st Sit. 2012)
is in (SSC CGL 2nd Sit. 2011) (a) Confucianism (b) Judaism
(a) Geneva (b) Paris (c) Shintoism (d) Taoism
(c) Vienna (d) Washington 35. Ram Sharan Sharma, who died in 2011 was an eminent
21. Vasundhara Summit was held in (SSC CGL 2nd Sit. 2011) (SSC CGL 2nd Sit. 2012)
(a) USA (b) UK (a) Chemist (b) Economist
(c) Brazil (d) Australia (c) Indologist (d) Archaeologist
22. The first Indian who was chosen as the Secretary General of 36. The 34th National Games were held in 2011 in:
Commonwealth is (SSC CGL 1st Sit. 2012) (SSC CGL 2nd Sit. 2012)
(a) Rakesh Verma (b) Gopalaswami (a) Uttarakhand (b) Kerala
(c) Krishna Murthy (d) Kamalesh Sharma (c) Karnataka (d) Jharkhand
23. Kuldip Nayer, a journalist, was appointed as a High 37. Torah is the scared book of: (SSC CGL 2nd Sit. 2012)
(a) Zoroastrianism (b) Confucianism

ww
Commissioner in
(a) Sri Lanka
(c) UK
(SSC CGL 1st Sit. 2012)
(b) Australia
(d) Pakistan 38.
(c) Taoism (d) Judaism
The 98th Indian Science Congress was held in 2011 at:
24.
adopted in the year
(a) 2001
w.E
The National Policy for Empowerment of Women was
(SSC CGL 1st Sit. 2012)
(b) 2005
(a) Bengaluru
(c) Chennai
(SSC CGL 2nd Sit. 2012)
(b) Bhopal
(d) Bhubaneshwar

25.
(c) 1991
Ballots were first used in
(d) 1995
asy
(SSC CGL 1st Sit. 2012)
39. Which bank was the first to introduce ATMs to the world?

(a) Hong Kong Bank


(SSC CGL 2nd Sit. 2012)
(a) Australia
(c) Ancient Greece
(b) USA
(d) England
En (b) Standard Chartered Bank
(c) Bank of America
26. Which of the following criteria is not used for the
classification of human races?
(a) Nose (b) Hair
(SSC CGL 1st Sit. 2012)
gin
40.
(d) Citi Bank
Mamta Sharma was appointed in 2011 as the chairperso n

27.
(c) Eyes
The study of lake is called
(d) Ear
(SSC CGL 1st Sit. 2012)
of:
eer (SSC CGL 2nd Sit. 2012)
(a) National Commission for Minorities
(a) Topology
(c) Limnology
(b) Hydrology
(d) Potomology ing
(b) National Commission for Protection of Child Right
(c) National Commission for women
(d) National Commission for BCs
28. The Central Drug Research Institute of India is located at
(SSC CGL 1st Sit. 2012)
41.
.ne
Human Development index was formulated by:
(SSC CGL 2nd Sit. 2012)

29.
(a) Madras
(c) Delhi
(b) Lucknow
(d) Bangalore
How many spokes are there in the Dharmachakra of the
National Flag? (SSC CGL 1st Sit. 2012)
42.
(a) ASEAN
(c) UNDP
(b) IBRD
(d) UNCTAD t
Who is the author of the book ‘Pakistan: Beyond the Crisis
State’? (SSC CGL 2nd Sit. 2012)
(a) 14 (b) 18 (a) Khuram Iqbal (b) Maleeha Lodhi
(c) 22 (d) 24 (c) Amir Mir (d) M. J. Akbar
30. The latest official language of the U.N. is 43. What is the name of the currency of Bangladesh ?
(SSC CGL 1st Sit. 2012) (SSC CGL 1st Sit. 2012)
(a) Russian (b) Arabic (a) Taka (b) Lek
(c) Chinese (d) Spanish (c) Dinar (d) Peso
31. The latest book 'Kurukshetra to Kargil' is written by 44. The famous car (chariot) festival is held at
(SSC CGL 1st Sit. 2012) (SSC CGL 1st Sit. 2012)
(a) Suryanath Singh (b) Kunal Bhardwaj (a) Puri (b) Gwalior
(c) Karan Singh (d) Kuldip Singh (c) Bhopal (d) Konark
32. The organisation involved primarily with environmental 45. The most literate Union Territory in India is
planning is (SSC CGL 1st Sit. 2012) (SSC CGL 1st Sit. 2012)
(a) CIFRI (b) ICAR (a) Chandigarh (b) Lakshadweep
(c) CSIR (d) NEERI (c) Delhi (d) Pondicherry

Downloded From : www.EasyEngineering.net


Downloded From : www.EasyEngineering.net

GENERAL KNOWLEDGE 97
46. The first Secretary-General of the United Nations was 56. The 'One Straw Revolution" was written by:
(SSC CGL 1st Sit. 2012) (SSC CGL 1st Sit. 2013)
(a) U. Thant (a) Masanobu Fukuoka
(b) Dr. Kurt Waldheim (b) Richael Carlson
(c) Dag Hammarskjold (c) M.S. Swaminathan
(d) Trygve Lie (d) Norman Borlaug
47. The book “Tinderbox : The Past and Future of Pakistan” is 57. Aung San Suu Kyi is a native to : (SSC CGL 1st Sit. 2013)
written by (SSC CGL 1st Sit. 2012) (a) Tibet
(a) Jamil Ahmad (b) Khatija Akbar (b) China
(c) Khurram Iqbal (d) M.J. Akbar (c) Myanmar
48. Which of the following cities in India is considered the (d) Arunachal Pradesh
greenest ? (SSC CGL 1st Sit. 2012) 58. Non Residential Indians (NRI) Day is marked on:
(a) Delhi (SSC CGL 1st Sit. 2013)
(b) Thiruvananthapuram (a) January 7 (b) January 9
(c) Bangalore (c) January 17 (d) January 19
(d) Chandigarh 59. "Whether I earned your vote or not, I have listened to you,
49.
ww
Synagogue is the place of worship of

(a) Zoroastrianism
(SSC CGL 1st Sit. 2012)
(b) Taoism
I have learned from you. You have made me a better
President", was said by :
(a) Pranab Mukherjee
(SSC CGL 1st Sit. 2013)
(b) Barack Obama

50.
(c) Judaism
w.E (d) Shintoism
One of the following Chief Ministers received the Highest
Civilian Award “Bharat Ratna” (SSC CGL 2nd Sit. 2012)
60.
(c) George Bush (d) APJ Abdul Kalam
First Hindu American to enter the US House of
Representative as senator is : (SSC CGL 1st Sit. 2013)
(a) Lalu Prasad Yadav of Bihar
asy
(b) Late M. G. Ramachandran of Tamilnadu
(a) Tulsi Gabbard
(c) Jyoti Sengupta
(b) Sunita Williams
(d) Ami bera
(c) Jyothi Basu of West Bengal
(d) Late N. T. Rama Rao of Andhra Pradesh En 61. Who was not a politician ?
(a) H.N. Bahuguna
(SSC CGL 1st Sit. 2013)
(b) I.K. Gujral
51. Which one of the following film-actors has been conferred
the Honorary Doctorate in Arts and Culture by the Bedford
(SSC CGL 2nd Sit. 2012)
gin
62.
(c) S.L. Bahuguna (d) J.Jayalalitha
Which of the following crop cultivation is banned by the
University, London?
(a) Shahrukh Khan
(c) Saif Ali Khan
(b) Amir Khan
(d) Anil Kapoor eer
Hon’ble Supreme Court of India?

(a) Lathyrus (Khesari)


(SSC CGL 2nd Sit. 2013)

52. Who advocated the adoption of ‘PURA’ model to eradicate


rural poverty? (SSC CGL 2nd Sit. 2012) ing
(b) Genetically modified brinjal
(c) Bt cotton for export
(a) Dr. A. P. J. Abdul Kalam
(b) Sri Abhijit Sen 63.
(d) Bt cotton for local use
.ne
Who is known for establishing-the “Anand Van”?

53.
(c) Maulana Abdul Kalam Azad
(d) Prof. A. M. Patha
Which one of the following though called a garden is infact,
not a garden? (SSC CGL 2nd Sit. 2012) 64.
(a) Jubilant Buddha
(c) Baba Amte
(b) H. N. Bahuguna
(d) Motilal Nehru
The civilian Airport of highest altitude is in
t
(SSC CGL 2nd Sit. 2013)

(a) Vrindavan Garden of Mysore (SSC CGL 2nd Sit. 2013)


(b) Hanging Garden of Mumbai (a) Tibet (d) Nepal
(c) Eden Garden of Kolkata (c) India (d) China
(d) Shalimar Garden of Kashmir 65. Greenpark Stadium is in (SSC CGL 2nd Sit. 2013)
54. Who wrote the book "Why Socialism"? (a) Bengaluru (b) Dehradun
(SSC CGL 1st Sit. 2013) (c) Chandigarh (d) Kanpur
(a) Jayaprakash Narayan 66. Human Environment Conference-1972 was held at
(b) Mahatma Gandhi (SSC CGL 2nd Sit. 2013)
(c) Acharya Narendra Dev (a) Stockholm (b) Paris
(d) M.N. Roy (c) Geneva (d) Australia
55. Multi purpose river valley projects are the "New temples of 67. Who said “Truth is the ultimate reality and it is God”?
modern India ". (SSC CGL 1st Sit. 2013) (SSC CGL 2nd Sit. 2013)
(a) Jawaharlal Nehru (b) Motilal Nehru (a) Swamy Vivekananda (b) Rabindra Nath Tagore
(c) Mahatma Gandhi (d) Rajiv Gandhi (c) M. K. Gandhi (d) Radhakrishnan

Downloded From : www.EasyEngineering.net


Downloded From : www.EasyEngineering.net

98 GENERAL KNOWLEDGE
68. Who is the founder of “Facebook” which is currently the 78. The first non-Englishman elected as Chairman of the
No. 1 social networking website in India? International Cricket Council was
(SSC CGL 1st Sit. 2013) (SSC CGL 2nd Sit. 2013)
(a) Orkut Buyukkokten (b) Mark Zuckerberg (a) Cydle Walcott (b) Gary Sobers
(c) Bill Gates (d) Martin Cooper (c) Imran Khan (d) Sunil Gavaskar
69. Who is the author of the book “No Full Stops in India”? 79. For which language included in the Indian Constitution, the
(SSC CGL 1st Sit. 2013) Jnanpith Award has not been given upto 2011?
(a) Nirad C. Choudhuri (b) Mark Tully (SSC CGL 2nd Sit. 2013)
(a) Sanskrit (b) Sindhi
(c) R. K. Narayan (d) Ved Mehta
(c) Kashmiri (d) Konkani
70. Which one of the following pairs is wrongly matched?
80. Who is the first British Author to win the Man Booker Prize
(SSC CGL 1st Sit. 2013)
for fiction twice ? (SSC CGL 2nd Sit. 2013)
Place Location
(a) Peter Carey (b) J.M. Coetzee
(a) Tiananmen Square - Beijing
(c) Hilary Mantel (d) None of the above
(b) Tahrir Square - Abu Dhabi 81. Which one of the following is wrongly paired?
(c) Trafalgar Square - London (SSC CGL 2nd Sit. 2013)
(d) Red Square - Moscow
71.
ww
The term of a non-permanent member of the U.N. Security
Council is (SSC CGL 1st Sit. 2013)
Country
(a) Japan
(b) Iran
Currency
Yen
Rand

72.
(a) 1 year
(c) 3 years w.E (b) 2 years
(d) 6 months
Julia Gillard is the Prime Minister of 82.
(c) Bangladesh
(d) Bhutan
Taka
Ngultrum
The Red Data Books published by the International Union

(a) Canada asy


(SSC CGL 1st Sit. 2013)
(b) Australia
for Conservation of Nature and Natural Resources
enumerate (SSC CGL 2nd Sit. 2013)

73.
(c) New Zealand (d) Belgium
En
Which is the first state in India to pass the Food Security
(a) Biodiversity parks and wild life sanctuaries in different
countries.
Law?
(a) Chhattisgarh
(SSC CGL 1st Sit. 2013)
(b) Gujarat gin (b) Centres of origin of cultivated plants.
(c) Centres of origin of all economically important plants
(d) Threatened species of plants and animals.
.

74.
(c) Punjab (d) Kerala
Who among the following Presidents of MCC was a non-
professional cricketer? (SSC CGL 1st Sit. 2013)
83.
eer
The two South American countries, which are members o f
the Organisation of Petroleum Exporting Countries (OPEC )
(a) Peter May
(b) Christopher Martin Jenkins
are
(a) Ecuador and Braziling (SSC CGL 2nd Sit. 2013)

(c) Ted Dexter


(d) Colin Cowdrey
(b) Ecuador and Bolivia
(c) Ecuador and Venezuela
.ne
75. Who won the Nobel Prize for Peace in 2012 ?
(SSC CGL 1st Sit. 2013)
(a) International Atomic Energy Agency
84.
(d) Venezuela and Brazil

t
Who is the Chairperson of the National Green Tribunal ?

(a) Markandey Katju


SSC CGL 2nd Sit. 2013)
(b) Swatantra Kumar
(b) Liu Xiaobo (c) Satyananda Mishra (d) K.G. Balakrishnan
(c) European Union 85. Which is the capital of Mali ? (SSC CGL 2nd Sit. 2013)
(d) U. N. Intergovernmental Panel on Climate Change (a) Mopti (b) Barmako
76. Who won the Jnanpith Award for 2011? (c) Cairo (d) Nairobi
(SSC CGL 1st Sit. 2013) 86. Which was the first film of Jaspal Bhatti ?
(a) Sachidananda Routray (SSC CGL 2nd Sit. 2013)
(b) Pratibha Ray (a) Power Cut
(c) Gopinath Mohanty (b) Mahaul Theek Hai
(d) Sitakant Mahapatra (c) Thank You Jijaji
77. Environmental impact assessment was first formally (d) None of the three above
established in 1969 in which country ? 87. Who was the first recipient of the "Bharat Ratna" Award
(SSC CGL 2nd Sit. 2013) posthumously ? (SSC CGL 2nd Sit. 2013)
(a) United Kingdom (b) United States (a) K. Kamaraj (b) Lal Bahadur Shastri
(c) France (d) Netherlands (c) M.G. Ramachandran (d) B.R. Ambedkar

Downloded From : www.EasyEngineering.net


Downloded From : www.EasyEngineering.net

GENERAL KNOWLEDGE 99
88. Sarus crane is the state bird of (SSC CGL 1st Sit. 2013) 101. 'Agha Khan Cup' is related with which of the following sport
(a) Rajasthan (b) Uttar Pradesh event ? (SSC CGL 2015)
(c) Madhya Pradesh (d) West Bengal (a) Cricket (b) Hockey
89. The Daocheng Yading Airport is located in (c) Table Tennis (d) Football
(SSC CGL 1st Sit. 2013) 102. Where was the first conference of SAARC (South Asian
(a) Thailand (b) Philippines Association for Regional Cooperation) held?
(c) China (d) Tibet (SSC CGL 2015)
90. "Martyr's Day" is marked on (SSC CGL 1st Sit. 2013) (a) Dhaka (b) New Delhi
(a) January 1 (b) January 15 (c) Colombo (d) Kathmandu
(c) January 30 (d) January 9 103. First Nobel Prize to India was given for :
91. Which of the following is only domestic Airport? (SSC CGL 2015)
(SSC CGL 1st Sit. 2013) (a) Physics (b) Literature
(a) Dabolin Airport, Goa (c) Medicine (d) Chemistry
(b) Srinagar Airport 104. Who was the Indian women president of the United Nations
General Assembly ? (SSC CGL 2015)
(c) Devi Ahilyabai Holkar Airport
(a) Margret Thatcher (b) Golda Mayer

ww
(d) None of the above
92. Second India–Africa Forum Summit–2011 was held in
(SSC CGL 1st Sit. 2013)
(c) Sarojini Naidu
(d) Vijya Lakshmi Pandit

(a) Eretrea
(c) Sudan w.E (b) Ethiopia
(d) Nigeria
105. What is the currency of Saudi Arabia ? (SSC CGL 2015)
(a) Riyal
(c) Lira
(b) Pound
(d) Dinar

at
(a) Hyderabad asy
93. NIN (National Institute of Nutrition) Central Office is located

(b) Mumbai
(SSC CGL 2014)
106. Reserve Bank of India was nationalised in :
(SSC CGL 2015)

(c) Bengaluru (d) Kolkata


94. "Bull's eye" is used in the game of En
(SSC CGL 2014)
(a) 1951
(c) 1935
(b) 1947
(d) 1949

(a) Boxing
(c) Polo
(b) Basketball
(d) Shooting gin
107. Which among the following is a folk dance of India.
(SSC CGL 2015)

95. Pablo Picasso, the famous painter was (SSC CGL 2014)
(a) French (b) Italian eer
(a) Kathakali
(c) Manipuri
(b) Mohiniattam
(d) Garba

(c) Flemish (d) Spanish


96. To whom the line 'A thing of beauty is a joy for ever' is
(a) Curie
(c) Soddy ing
108. Radio activity was discovered by :
(b) Beequeral
(d) Rutherford
(SSC CGL 2015)

attributed ?
(a) John Keats
(SSC CGL 2015)
(b) Dr. Charles Dickens
with? .ne
109. ’Mission Indradhanush’ Campaign in India is associated
(SSC CGL 2016)
(c) Dr. Jonathan Swift

as 'Teachers Day' in India?


(a) Dr. Rajendra Prasad
(d) William Wordsworth
97. The birthday of which of the following leaders is celebrated
(SSC CGL 2015)
(b) S. Radhakrishnan
(a) Nutrition to Pregnant Women
(b) Awareness of Diabetes
(c) Eradication of blindness
t
(d) Vaccination of children
(c) C. Rajgopalachari (d) Lala Lajpat Rai
110. Who is the recipient of Man Booker International Prize 2016
98. The award given for outstanding performance in sports is for the novel ‘The Vegetarian’? (SSC CGL 2016)
(SSC CGL 2015) (a) Orhan Pamuk (b) Han Kang
(a) Bharat Ratna (b) Padma Shri Award (c) Elena Ferrante (d) Tonkin
(c) Arjuna Award (d) Dronacharya Award 111. The novelist, poet and critic of which language has been
99. Which hill station's name means place of the thunderbolt'? conferred with Jnanpith Award for 2015?
(SSC CGL 2015) (SSC CGL 1st Sit. 2016)
(a) Shillong (b) Oottacamand (a) Bengali (b) Telugu
(c) Darjeeling (d) Gangtok (c) Odia (d) Gujarati
100. The ship building yard––Mazgaon Dock is located at – 112. Which one of the following countries is not a member of the
(SSC CGL 2015) "BRICS" group? (SSC CGL 1st Sit. 2016)
(a) Kochi (b) Kolkata (a) Brazil (b) Russia
(c) Mumbai (d) Vishakhapatnam (c) China (d) Indonesia

Downloded From : www.EasyEngineering.net


Downloded From : www.EasyEngineering.net

100 GENERAL KNOWLEDGE


113. Which pair of the following Indians has been selected for 124. Lakshmibai National Institute of Physical Education (LNIPE)
grant of Raman Magasaysay Award 2016? is in: (SSC CHSL 2012)
(SSC CGL 1st Sit. 2016) (a) Patiala (b) Gwalior
(a) Bezwada Wilson and T.M. Krishna (c) Indore (d) Jhansi
(b) T.M. Krishna and Arvind Adiga 125. The concept of 'Carbon credit' originated from :
(c) Arvind Adiga and Bezwada Wilson (SSC CHSL 2012)
(d) T.M. Krishna and Satish Gujral (a) Earth Summit, Rio-de-Janerio
114. At Rio Olympics, what was the final position of Dipa (b) Kyoto Protocol
Karmakar in her category? (SSC CGL 1st Sit. 2016) (c) Montreal Protocol
(a) 4 (b) 5 (d) None of the above
126. Minorities Rights Day is observed in India on
(c) 6 (d) 7
(SSC CHSL 2013)
115. The study of mountains is known as
(a) 18th December (B 23rd December
(SSC CGL 1st Sit. 2016)
(c) 5th September (d) 1st December
(a) Oncology (b) Lithology
127. Which one of the following institutes in List I is wrongly
(c) Orology (d) Ornithology matched with its location indicated in List II ?

ww
116. The famous Vishnu temple at Angkor Wat in Cambodia was
built by ?
(a) Shrutavarman
(SSC CGL 1st Sit. 2016)
(b) Suryavarman II
List-I List-II
(a) Central Institute of Mining Dhanbad
(SSC CHSL 2013)

(c) Indravarman
w.E (d) Aniruddha
117. The new symbol of Indian currency is designed by?
(SSC CGL 1st Sit. 2016)
and Fuel Research
(b) Central Building Research Roorkee
Institute
(a) Santosh Kumar
(c) Udayakumar asy
(b) Y.V. Reddy
(d) Dr. Rangarajan
(c) Central Arid Zone
Research Institute
Jodhpur

118. What is MUDRA?


(a) Development and Refinance Agency En
(SSC CGL 1st Sit. 2016) (d) Central Drug Research
Institute
Kanpur

(b) Scheme for Agricultural Insurance


(c) New Planet Discovered gin
128. Match the two lists given below:
List-I
(SSC CHSL 2013)
List-II
(d) Development and Regulatory Authority for Urban
Township
a. London

eer
b. Vatican City
c. Moscow
1. St. Peter's Square
2. Times Square
3. Trafalgar Square
119. Which day is observed as “International Day of Non-
Violence”
(a) 1 st May
(SSC CHSL 2012)
(b) 2 nd October
d. New York
(a) a-1, b-3, c-2, d-4 ing 4. Red Square
(b) a-2, b-4, c-3, d-1
(c) 24 th October (d) 30 th January
120. Which country is the largest producer of wool ?
(c) a-3, b-1, c-4, d-2
.ne
(d) a-4, b-2, c-1, d-3
129. Who among the following women tennis players has won

(a) U.S.A.
(a) Britain (d) Canada
(SSC CHSL 2012)
(b) Australia
(a) Serena Williams
(c) Steffi Graf
(b) Venus Williams
(d) Margaret Court
t
the Grand Slam titles for the maximum number of times ?
(SSC CHSL 2013)

121. Which one of the following African countries in not a


130. In which one of the following films did Pran act as a character
member of OPEC ? (SSC CHSL 2012)
artist and not in the role of a villain? (SSC CHSL 2013)
(a) Angola (b) Libya (a) Himalaya Ki Godh Mein
(c) South Africa (d) Algeria (b) Ram aur Shyam
122. Which one of the following National Park/Sanctuary is not (c) Zanjeer
in Rajasthan ? (SSC CHSL 2012) (d) Madhumati
(a) Sariska National Park 131. Who was the first posthumous recipient of Bharat Ratna?
(b) Sambar Wildlife Sanctuary (SSC CHSL 2013)
(c) Rajaji National Park (a) M.G. Ramachandran (b) B.R. Ambedkar
(d) Rhanthambore National Park (c) K. Kamraj (d) Lal Bahadur Shastri
123. Which one of following is called the primary pace maker of 132. Which country won the ICC Women’s World Cup held in
the heart ? (SSC CHSL 2012) February 2013 ? (SSC CHSL 2013)
(a) A. V. Node (b) Chordae tendinae (a) India (b) New Zealand
(c) A.V. Septum (d) S.A. Node (c) England (d) Australia

Downloded From : www.EasyEngineering.net


Downloded From : www.EasyEngineering.net

GENERAL KNOWLEDGE 101


133. Which country’s currency is Ngultrum ? 146. "Life Divine" is a book written by (SSC CHSL 2014)
(SSC CHSL 2013) (a) M. K. Gandhi (b) Rabindranath Tagore
(a) Bhutan (b) Laos (c) S. Radhakrishnan (d) Shri Aurobindo
(c) Bangladesh (d) Nepal 147. The Oscar Award was won 36 times by
134. "Sattriya Nritya” recognised as a classical dance form of (SSC CHSL 2014)
India by the Sangeet Natak Akademi only in 2000, originated (a) Charlie Chaplin (b) Alfred Hitchcock
from (SSC CHSL 2013) (c) Walt Disney (d) Akiro Kurosawa
(a) Tripura (b) Assam 148. Who among the following is not a Bharatanatyam dancer ?
(c) Karnataka (d) Gujarat (SSC CHSL 2015)
135. Grammy Award is given in the field of (SSC CHSL 2014) (a) Sitara Devi (b) Leela Samson
(a) Acting (b) Music (c) Geeta Ramachandran (d) Sonal Mansingh
(c) Singing (d) Boxing 149. Ryder Cup is a famous tournament of :
136. The first woman to get the Bharat Ratan Award is (SSC CHSL 2015)
(SSC CHSL 2014) (a) Badminton (b) Golf
(a) Mother Teresa (b) Indira Gandhi (c) Cricket (d) Lawn Tennis
(c) Lata Mangeshker (d) Sarojini Naidu

ww
137. Karl Marx wrote
(a) Asian Drama (b) Em ma
(SSC CHSL 2014)
150. Kanha National Park is located in :
(a) Tamil Nadu
(c) Andhra Pradesh
(b) Bihar
(SSC CHSL 2015)

(d) Madhya Pradesh


(c) Das Kapital
w.E (d) Good Earth
138. The religious text of the Jews is named as
(SSC CHSL 2014)
151. Who wrote 'Discovery of India'?
(a) Mahatma Gandhi
(SSC CHSL 2015)
(b) Jawahar Lal Nehru
(c) Bal Gangadhar Tilak (d) APJ Abdul Kalam
(a) The Analectus
(c) Tripitaka asy
(b) Torah
(d) Zend-Avesta
152. Who is the first woman IPS officer in India ?
(SSC CHSL 2015)
139. "Meghdoot" was written by
(a) Humayun Kabir (b) Khushwant Singh
En
(SSC CHSL 2014) (a) Sarojini Naidu
(c) Bachendri Pal
(b) Kiran Bedi
(d) Indira Gandhi
(c) Banabhatta (d) Kalidasa
140. Who among the following is a famous English writer ? gin
153. Which of the following is the right expansion of ILO?
(SSC CHSL 2015)

(a) Amrita Pritam


(SSC CHSL 2014)
(b) Mahadevi Verma
eer
(a) International Labour Organization
(b) Indian Legal Orientation
(c) Ashapurna Devi (d) Mulk Raj Anand
141. In which year were the States reorganized on a linguistic
basis ? (SSC CHSL 2014) ing
(c) Internatioanl Law and Order
(d) Inter–State Lawful Ordinance
154. In 2010 a newspaper published its 70,000th issue. Which
(a) 1951
(c) 1950
(b) 1947
(d) 1956
was the newspaper ?
(a) The Oxford Gazette .ne (SSC CHSL 2015)

142. The "Mein Kampf" was written by


(a) Hitler
(c) Bismarck
(SSC CHSL 2014)
(b) Mussolini
(d) Mazzini
(b) The Washington Post
(c) The Times of London
(d) The Hindustan Times
t
143. It was decided to observe Mahatma Gandhi's birthday 155. Project tiger programme was launched in :
October 2 as the International Nonviolence Day at (SSC CHSL 2015)
(SSC CHSL 2014) (a) 1994 (b) 1973
(a) International lndology Conference (c) 1975 (d) 1971
(b) Setyagraha Centenary Conference 156. Which Indian newspaper has the largest readership ?
(c) Congress Foundation Day Celebration (SSC CHSL 2015)
(d) None of these (a) The Malayala Manorama
144. ISRO's Master Control Facility is in (SSC CHSL 2014) (b) Indian Express
(a) Andhra Pradesh (b) Orissa (c) The Hindu
(c) Gujarat (d) Karantaka (d) The Dainik Jagran
145. Which place is called as "Silicon Valley" of India ? 157. Which day is celebrated as International Yoga Day ?
(SSC CHSL 2014) (SSC CHSL 2015)
(a) Delhi (b) Pune (a) April 23 (b) September 21
(c) Bengaluru (d) Hyderabad (c) July 21 (d) June 21

Downloded From : www.EasyEngineering.net


Downloded From : www.EasyEngineering.net

102 GENERAL KNOWLEDGE


158. December 1 is celebrated as : (SSC CHSL 2015) 172. Iron filings can be separated from a heterogenous mixture
(a) Indian Navy Day (b) UNICEF Day using the technique of (SSC CHSL 2015)
(c) Children's Day (d) World AIDS Day (a) Magnetization (b) Sedimentation
159. Who was the first Secretary General of UNO? (c) Evaporation (d) Sublimation
(SSC CHSL 2015) 173. In which year did Amartya Kumar Sen receive the Nobel
Prize in Economics? (SSC Multi Tasking 2013)
(a) Kurt Waldheim (b) Dag Hammarskjold
(a) 1990 (b) 1998
(c) Trygve Lie (d) U–Thant
(c) 1995 (d) 2000
160. Earth day is celebrated on : (SSC CHSL 2015)
174. The worldwide Great Depression took place in
(a) April 22 (b) September 17
(SSC Multi Tasking 2013)
(c) February 16 (d) April 4 (a) 1930 (b) 1936
161. What is the number of players on each side in Rugby (c) 1929 (d) 1928
Football? (SSC CHSL 2015) 175. Which one of the following is not an All India Service?
(a) 16 (b) 12 (SSC Multi Tasking 2013)
(c) 11 (d) 15 (a) Indian Police Service
162. Which was the first linguistic state to be created ? (b) Indian Foreign Service
(SSC CHSL 2015)

ww
(a) Tamil Nadu
(c) Maharashtra
(b) Andhra Pradesh
(d) Kerala
(c) Indian Forest Service
(d) Indian Administrative Service
176. The Sethusamudram Ship Canal Project (SSCP) is supposed

is located at :
(a) Dehradun
w.E
163. The headquarters of the Survey of India Dept. (department)
(SSC CHSL 2015)
(b) Hyderabad
to reduce the distance between Chennai and Tuticorin by
_________ nautical miles.
(a) 305
(SSC Multi Tasking 2013)
(b) 361
(c) New Delhi
asy
(d) Jaipur
164. The Lalit Kala Academy is devoted to the promotion of :
(c) 434 (d) 243
177. Cities with population from one to five million are called
(SSC Multi Tasking 2013)
(a) Literature (b) Music
En
(SSC CHSL 2015)
(a) Cosmopolitan
(c) Million City
(b) Conurbation
(d) Metropolitan
(c) Dance and Drama (d) Fine Arts
165. World wild life fund was founded in : (SSC CHSL 2015) gin
178. ‘Kyoto Protocol’, an agreement signed by various countries
is associated with (SSC Multi Tasking 2013)
,

(a) 1969
(c) 1961
(b) 1992
(d) 1965
eer
(a) Deep Sea Oil and Mineral Exploration
(b) Clean Environment and Climate Change
166. Arundhati Roy is the author of :
(a) Disgrace
(SSC CHSL 2015)
(b) The Tin Drum
(c) My Childhood Days (d) God of Small Things
ing
(c) Building common food stock to save human beings
from any natural disaster
(d) International Trade
167. The first Bio-sphere Reserve in India has been established
in : (SSC CHSL 2015) .ne
179. Sanjukta Panigrahi was famous for the dance
(SSC Multi Tasking 2013)
(a) Nilgiri
(c) Hazaribagh
(b) Nanda Devi
(d) Kanha
168. Kimono is a dress style of which Asian Country ?
(a) Odissi
(c) Kathak
(b) Bharatnatyam
(d) Manipuri
180. ‘White Revolution’ is related to
t
(SSC CHSL 2015) (SSC Multi Tasking 2013)
(a) Korea (b) Laos (a) Fish production (b) Wheat production
(c) China (d) Japan (c) Milk production (d) Flood control
169. Dr. P. Rama Rao Committee is related to which of the 181. Who proposed the adage “Survival of the fittest”?
following ? (SSC CHSL 2015) (SSC Multi Tasking 2013)
(a) Lamarck (b) Darwin
(a) Defence (b) Industry
(c) William (d) Huxley
(c) Agriculture (d) Taxes
182. What is the name given to Moon Mission in India?
170. Kathakali is a dance prevalent in which state ?
(SSC Multi Tasking 2014)
(SSC CHSL 2015)
(a) Vikram I (b) Chandrayaan I
(a) Andhra Pradesh (b) Tamil Nadu
(c) Kalpana II (d) Astrosat
(c) Orissa (d) Kerala 183. Lira was the currency of which country?
171. Rand is the currency of : (SSC CHSL 2015) (SSC Multi Tasking 2014)
(a) Iran (b) Romania (a) China (b) Australia
(c) Norway (d) Namibia (c) Japan (d) Italy

Downloded From : www.EasyEngineering.net


Downloded From : www.EasyEngineering.net

GENERAL KNOWLEDGE 103


184. Who is the Supreme Commander-in-Chief of the armed 196. Who was the first woman winner of Kaun Banega Crorepati
forces? (SSC Multi Tasking 2014) 6? (SSC Sub. Ins. 2014)
(a) Prime Minister (b) Defence Minister (a) Kiran Bedi (b) Lata Mangeshkar
(c) President (d) Vice-President (c) Sumeet Kaur Sawhney (d) Preity Zinta
185. Which is The second most populous State of India as per 197. Who among the following was named as “Haryana
Census 2011? (SSC Multi Tasking 2014) Hurricane” ? (SSC Sub. Ins. 2015)
(a) West Bengal (b) Rajasthan (a) Ajay Ratsa (b) Nawab Pataudi
(c) Maharashtra (d) Bihar (c) Kapil Dev (d) Ajay Jadeja
186. United Nations Conference on Environment and 198. What does the word ‘amphibian’ mean ?
Development is called (SSC Sub. Ins. 2012)
(SSC Sub. Ins. 2015)
(a) Earth Summit (b) Water Summit
(a) Two lives (b) Four lives
(c) Air Summit (d) Resource Summit
(c) Three lives (d) One life
187. The headquarters of International Atomic Energy Agency
is located at (SSC Sub. Ins. 2012) 199. The founder of the ‘Arya Samaj’ was:
(a) Geneva (b) Washington (SSC Sub. Ins. 2015)
(c) Vienna (d) Sydney (a) Dayananda Saraswati (b) Raja Ram Mohan Roy

ww
188. The first woman to conquer Mount Everest twice is

(a) Surja Lata Devi


(SSC Sub. Ins. 2012)
(b) Jyoti Randhawa
(c) Vivekananda

Justice" has been authored by


(d) Annie Besant
200. The book "Globalization, Democratization and Distributive
(SSC Steno 2013)
(c) Santosh Yadav
w.E (d) Suma Shirur
189. Which Indian hockey player has road named after him in
Germany? (SSC Sub. Ins. 2012)
(a) Mool Chand Sharma (b) Karan Bajaj
(c) Salman Rushdies (d) Anita Nair
201. Which of the following has become the first airport in the
(a) Dhyan Chand
(c) Roop Singh asy
(b) Zafar Iqbal
(d) Dhanraj Pillai
world to fully operation all on solar power?
(SSC Steno 2013)

by India in the Antarctica?


En
190. What is the name of the first research station established
(SSC Sub. Ins. 2012)
(a) Lokpriya Gopinath Bardoli International Airport.
(b) Amritsar International Airport.
(a) Dakshin Gangotri
(c) Agnihotri
(b) Maitri
(d) Aryabhatta
191. The five permanent members of the U.N. Security Council
gin (c) Netaji Subash Chandra Bose International Airport.
(d) Cochin International Airport Ltd. (Kerla).

are (SSC Sub. Ins. 2012)


(a) China, France, Russia, U.K. and U.S.A. eer
202. Which of the following countries has become the firs t
countryin the world to receivefunds from United Nations

(b) China, Canada, France, U.S.A and Germany


(c) China, Germany, Russia, U.K. and U.S.A.
(a) Afghanistan ing
for its fast growing Solar Home Systems?

(b) Burma
(SSC Steno 2013)

(d) China, Germany, U.S.A., U.K and Canada


192. Which of the following is not correctly paired?
(c) Bangladesh
.ne
(d) Switzerland
203. Who of the following is not one of the recipients of Arjuna
(a) Jwala Gutta
(b) Virat Kohli
(c) Harbhajan Singh
— Tennis
(SSC Sub. Ins. 2012)

— Cricket
— Kabaddi
Award 2015?
(a) Anoop Singh (Wrestling)
(b) Dipa Karmakar (Gymnastics)
t
(SSC Steno 2013)

(d) Saina Nehwal — Badminton (c) Jitu Rai (Shooting)


193. Which of the following States has the largest gap in male (d) P. R. Sreejesh (Hockey)
and female literacy as per 2011 Census (provisional data)? 204. Who wrote 'Communist Manifesto'? (SSC Steno 2013)
(SSC Sub. Ins. 2012) (a) Marx and Engels (b) Mao
(a) Uttar Pradesh (b) Madhya Pradesh (c) Lenin (d) Che - Cuevara
(c) Rajasthan (d) Kerala 205. Where is the permanent headquarters of SAARC?
194. Among the following states, which one has adopted me (SSC Steno 2013)
"Neem" tree as the state tree? (SSC Sub. Ins. 2013) (a) New Delhi (b) Kathmandu
(a) Maharashtra (b) Tamil Nadu (c) Thimphu (d) Dhaka
(c) Kerala (d) Andhra Pradesh 206. The only foreign national to receive the highest Bangladesh
195. When was the Geological Survey of India established ? Award ''Bangladesh Swadhinata Sammanona'' is
(SSC Sub. Ins. 2014) (SSC Steno 2013)
(a) 1841 (b) 1851 (a) P.V. Narasimha Rao (b) Indira Gandhi
(c) 1941 (d) 1951 (c) Sonia Gandhi (d) Rajiv Gandhi

Downloded From : www.EasyEngineering.net


Downloded From : www.EasyEngineering.net

104 GENERAL KNOWLEDGE


207. The headquarters of the Organization of petroleum 218. Where is the permanent secretarial of the SAARC?
Exporting Countries (OPEC) is in (SSC Steno 2013) (SSC Steno 2014)
(a) Baghdad (b) Abu Dhabi (a) New Delhi (b) Islamabad
(c) Teheran (d) Vienna (c) Colombo (d) Kathmandu
208. Who was the Captain of Kolkata Knight Riders in IPL - V? 219. 'Statue of Liberty' is the National Emblem of
(SSC Steno 2013) (SSC Steno 2014)
(a) M.S.Dhoni (b) Rahul Dravid (a) UK (b) USA
(c) Gautam Gambhir (d) Saurav Ganguly
(c) Germany (d) Russia
209. Which of the following pairs is correctly matched?
220. Ozone Day is celebrated on (SSC Steno 2014)
(SSC Steno 2013)
(a) April 21 (b) September 16
(a) Kuchipudi - Andhra Pradesh
(c) September 25 (d) June 5
(b) Kathakali - Tamil Nadu
221. The book 'Gokhale, My Political Guru' was written by
(c) Kathak - Manipur
(SSC Steno 2014)
(d) Bharatnatyam - Kerala
210. The term of a non-permanent member og the U. N. Security (a) Shaukat Ali (b) C.R. Das
Council is (SSC Steno 2013) (c) M.A. Jinnah (d) M.K. Gandhi

ww
(a) 6 months
(c) 2 years
(b) 1 year
(d) 3 years
211. Free and compulsory education for all children is provided
222. Who amongst the following is the author of the book 'Name
Sake'?
(a) Kiran Desai
(SSC Steno 2014)
(b) Shobha De

(a) 16 years
(c) 21 years
w.E
until they attain the age of
(b) 18 years
(d) 14 years
(SSC Steno 2014) (c) Vikram Seth

(a) 14th March


(d) Jhumpa Lahiri
223. National Science Day is observed on (SSC Steno 2014)
(b) 2nd June

asy
212. The Indian recipient of Noble Peace Prize is
(SSC Steno 2014)
(c) 5th January (d) 28th February
224. Which was the first National news Agency of free India ?
(a) Hargovind Khurana (b) Rabindra Nath Tagore
(c) Mother Teresa (d) Amartya Sen En (a) The Associated Press of India
(SSC Steno 2016)

213. London is situated on the banks of river


(SSC Steno 2014) gin (b) None of these
(c) The Indian Review
(a) Tiber
(c) Tigris
(b) Danube
(d) Thames
214. Identify the odd sportman among the following eer
(d) The Free press of India
225. The book 'Problem of Human Geography' was written by

(a) Narain karthikeyan


(SSC Steno 2014)
(b) Vijay Hazare
(a) Jean Brunches ing (SSC Steno 2016)
(b) None of these

(c) Sachin Tendulkar (d) Sunil Gavaskar


215. First Indian to make a speech in Hindi before the UN General
(c) Albert Demangeon
226. Philadelphia is famous for
.ne
(d) De Mortonne
(SSC Steno 2016)
Assembly was
(a) AB Vajpayee
(c) Lal Krishna Advani
(SSC Steno 2014)
(b) Lal Bahadur Shastri
(d) Morarji Desai
216. Which of the following awards is given for excellence in
(a) Locomotives
(c) Ship building
t
(b) Dairy industry
(d) Silk textiles
227. Which one among the following book is centered around
'environment'? (SSC Sub. Ins. 2016)
sports coaching? (SSC Steno 2014) (a) Here I Stand
(a) Sahitya Academi Award (b) And Then One Day
(b) Dronacharya Award (c) The Late, Great Planet Earth
(c) Arjun Award (d) Silent Spring
(d) Dada Saheb Phalke Award 228. An international movement with its motto to save the world
217. Who is the winner of the prestigious Man Booker Prize by involving itself with environmental problems is
2013? (SSC Steno 2014) (SSC Sub. Ins. 2016)
(a) Haruki Murakami (b) Eleanor Catton (a) Clean En (b) Eco-friend
(c) Malala Yousufzai (d) Vikram Seth (c) Green-field (d) Green-peace

Downloded From : www.EasyEngineering.net


Downloded From : www.EasyEngineering.net

GENERAL KNOWLEDGE 105

HINTS & SOLUTIONS


1. (d) 2. (a) 3. (a) 32. (d) The National Environmental Engineering Research
4. (c) Prabodh Chandra Dey known by his stage name Manna Insititute (NEERI) is a research institute created and
Dey, was an Indian playback singer. He debuted in the funded by Government of India. It was established in
film Tamanna in 1942, by this song “Upar Gagan Bishal” Nagpur in 1958.
and went on to record more than 4,000 songs from 1942 33. (d) The Dhole is on the edge of extinction. Also called the
to 2013. The Government of India honoured him with Asiatic wild dog or Indian wild dog, it is a species of
the Padma Shri in 1971, the Padma Bhushan in 2005 canid native to South and Southeast Asia.
and the Dadasaheb Phalke Award in 2007. 34. (a) The Analects of Confucius is an anthology of brief
5. (b) 6. (d) passages that present the words of Confucius and his
7. (c) Joseph Aspdin was an English cement manufacturer disciples, describe Confucius as a man, and recount
who obtained the patent for Portland cement on 21 some of the events of his life. The Analects includes
October 1824. twenty books, each generally featuring a series of
8.
13.
18.
ww
(b) 9. (c) 10. (a) 11. (d) 12. (c)
(a) 14. (b) 15. (a) 16. (a) 17. (c)
(b) 19. (a) 20. (c) 21. (c) 35.
chapter that encompass quotes from Confucius, which
were compiled by his disciples after his death.
(c) Ram Sharan Sharma was an eminent historian of Ancient
22.
w.E
(d) Mr. Kamlesh Sharma, an Indian diplomat, became
Commonwealth Secretary-General on 1 April 2008. He
was appointed to the post by Commonwealth Heads
36.
and early Medieval India.
(d) The 2011 National Games, also known as the 34th
National Games of India, was held from 12 February

November 2007. asy


of Government at their meeting in Kampala, Uganda, in
37.
2011 to 26 February 2011 in Ranchi, Jharkhand.
(c) The Torah is Judaism's most important text. It is
23. (c) He was appointed High Commissioner to Great Britain
in 1990.
En composed of the Five Books of Moses. The term
"Torah" means instruction and offers a way of life fo r
24. (a) The National Policy for Empowerment of Women 2001
has as its goal bringing about advancement,
development and empowerment of women in all spheres gin
38.
those who follow if.
(c) Indian Science congress Association (ISCA) is a
premier scientific organisation of India with
of life through creation of a more responsive judicial
and legal system sensitive to women and eer
headquarters at Kolkata, West Bengal. The 98th sessio
was held at SRM University, near Chennai in Januar
n
y

25.
mainstreaming a gender perspective in the development
process.
(a) First used in the Australian state of Victoria in 1857, the
39.
2011.
ing
(d) The Citibank of New York was the first such bank.
paper ballot listing all the candidates was first known
as "the Australian ballot. "In 1889, New York became
40.

.ne
(c) Former Rajasthan Mahila Congress President chief
Mamta Shrma was appointed as National Commission
for Women (NCW) chairperson. The appointment was
26.
the first American state to use these ballots.
(d) The term race or racial group usually refers to the
concept of dividing humans into populations or groups
on the basis of various sets of characteristics. The
41.
cleared by the Prime Minister's Office.
t
(c) The origins of the HDI are found in the annual Human
Development Reports of the United Nations
most widely used human racial categories are based Development Programme (UNDP). These were devised
on visible traits (especially skin colour, cranial or facial and launched by Pakistani economist Mehboob ul Haq
features and hair texture), and self-identification. in 1990.
27. (c) Limnology is the story of lakes and other freshwater 42. (b) Maleeha Lodhi is a journalist, academic and diplomat
basins. from Pakistan. She was the High Commissioner of
28. (b) CDRI, is located at Lucknow was inugurated in 1951 Pakistan to the United Kingdom and is a former
by the then Prime Minister of India, Jawahar lal Nehru. Pakistani Ambassador to the United States.
29. (d) The spokes in the Ashok Chakra represent 24 states as 43. (a) 44. (a) 45. (b) 46. (d) 47. (d)
24 were the number of states at that time. 48. (d) 49. (c) 50. (b) 51. (a) 52. (a)
30. (b) In 1980, the General Assembly made Arabic an official 53. (c) 54. (a) 55. (a) 56. (a)
and working language of all its committees and 57. (c) Aung San Suu Kyi, also called Daw Aung San Suu Kyi
subcommittees. (born June 19, 1945, Rangoon, Burma [now Yangon,
31. (d) The author is Brigadier (Retired) Kuldip Singh. This Myanmar]) politician and opposition leader of
book is an effort to understand now the future will Myanmar, daughter of Aung San (a martyred national
unfold in Asia 2030. hero of independent Burma) and Khin Kyi (a prominent

Downloded From : www.EasyEngineering.net


Downloded From : www.EasyEngineering.net

106 GENERAL KNOWLEDGE


Burmese diplomat), and winner of the Nobel Prize for 72. (b) Julia Eileen Gillard is the 27th and current Prime
Peace in 1991. She has held multiple governmental Minister of Australia and the Leader of the Australian
posts since 2016. Labor Party since 24 June 2010. She is the first women
58. (b) to hold the office.
59. (b) Barack Hussein Obama II who is the 44th and current 73. (a) Chhattisgarh was the first state in the country to
President of the United States. He is the first African introduce the Food Security Act, a unique law so that
American to hold the office and the first president born it could become a right of the people to get sufficient
outside the continental United States. quantity of food at subsidized rates.
60. (a) 74. (b) Christopher Martin - Jen - kins was a British cricket
61. (c) Sunderlal Bahuguna (born 9 January 1927) is a noted journalist and a past President of the MCC. He was
Garhwali environmentalist, Chipko movement leader also a commentator for Test Match Special (TMS) on
and a follower of Mahatma Gandhi's philosophy of BBC Radio from 1973 until diagnosed with terminal
Non-violence and Satyagraha. cancer in January 2012.
He was awarded the Padma Vibhushan, India's second 75. (c) The Nobel Peace Prize 2012 was awarded to European
highest civilian honour, on 26 January 2009. Union (EU) “for over six decades contributed to the
62. (a) Grass pea also known as khesari dal was banned for advancement of peace and reconciliation, democracy
consumption in 1961 as it was believed that Beta–N– and human rights in Europe”.

ww
Oxalylaminoalanine, a neuro–toxic amino–acid in the
legume, caused neurolathyrim or a paralysis of the
76. (b) Pratibha Ray is an Indian academic and writer who
won the Jnanpith Award in 2011. She was the first

63.
lower limbs,

w.E
(c) Founded in 1948 by noted social activist, Baba Amte,
Anandwan, located near Arora in Chandrapur district
77.
woman to win the Moortidevi Awards in 1991.
(b) Environmental impact assessments commenced in the
1960s, as part of increasing environmental awareness.

asy
in the state of Maharashtra. India, is an ashram, and a
community rehabilitation centre for leprosy patients
and the disabled from downtrodden sections of
EIAs involved a technical evaluation intended to
contribute to more objective decision making. In the
United States, environmental impact assessments

64.
society.
(d) Daocheng Yading Airport in A southwest China's En obtained formal status in 1969, with enactment of the
National Environmental Policy Act.
Sichuan Province is the world's highest–altitude
civilian airport. It is located at a height of 4,411 meters gin
78. (a) Sir Clyde Leopold Walcott, KA, GCM (17 January 1926
- 26 August 2006) was a West Indian cricketer. Walcott

65.
above sea level
(d) Green Park Stadium is a 60,000 capacity floodlit multi–
purpose stadium located in Kanpur, India, and the eer
was a member of the "three W's", the other two being
Everton Weekes and Frank Worrell: all were ver y
successful batsmen from Barbados, born within a short

66.
home of the Uttar Pradesh cricket team.
(a) The conference was held in Stockholm. the capital of ing
distance of each other in Bridgetown, Barbados in a
period of 18 months from August 1924 to January 1926;
Sweden. in 1972. The conference agreed upon a
declaration containing 26 principles concerning the
.ne
all made their Test cricket debut against England in
1948. In the mid-1950s, Walcott was arguably the best

67.
environment and development.
(c) Gandhi was known to have considered truth something
similar to that. Baruch Spinoza considered ultimate truth
as the ultimate reality of a rationally ordered system
t
batsman in the world. In later life, he had an active
career as a cricket administrator, and was the first non-
English and non-white chairman of the International
Cricket Council.
that is God. 79. (b) The Jnanpith Award is a literary award in India. Along
68. (b) Mark Elliot Zuckerburg is an American computer with the Sahitya Akademi Fellowship. It is one of the
programmer and internet entrepreneur. He is best two most prestigious literary honours in the country.
known as one of five co-founders of the social The award was instituted in 1961. Any Indian citizen
networking website Facebook. who writes in any of the official languages of India is
69. (b) Mark Tully is the author of No Full Stops in India eligible for the honour. It is presented by the Bharatiya
(1998). Its collection of journalistic essays, was Jnanpith, a trust founded by the Sahu Jain family, the
published in the US as The Defeat of a Congress-man. publishers of the The Times of India newspaper.
70. (b) Tahrir Square, also known as “ Martyr Square”, is a 80. (c) Hilary Mary Mantel is an English writer whose work
major public town square in Down town Cairo. Tahrir ranges in subject from personal memoir and short story
Square was the focal point of the 2011 Egyptain to historical fiction and essay. She has twice been
Revolution against former president Hosni Mubarak. awarded the Booker Prize. She won her first Booker
71. (b) There are 10 non-permanent members in the United Prize for the 2009 novel, Wolf Hall, a fictional account
Nations Security Council, with five elected each year of Thomas Cromwell's rise to power in the court of
Henry VIII. She won her second Booker Prize for the
to serve two-year terms.

Downloded From : www.EasyEngineering.net


Downloded From : www.EasyEngineering.net

GENERAL KNOWLEDGE 107


2012 novel, Bring Up the Bodies, the second instalment 97. (b) The birthday of Sarvepalli Radhakrishnan is celebrated
of the Thomas Cromwell trilogy. Mantel was the first as Teacher's Day. The day commemorates the birthday
woman to receive the award twice. of Dr Sarvepalli Radhakhrishnan, a philosopher and a
81. (b) The currency of Iran is Rial. teacher par excellence, and his contribution towards
82. (d) The Red Data Book is the state document established Indian education system. As a tribute to this great
for documenting rare and endangered species of teacher, his birthday is observed as Teacher's Day
animals, plants and fungi as well as some local sub- across India on 5th September.
species that exist within the territory of the state or 98. (c) The Arjuna Awards are given by the Ministry of Youth
country. This book provides central information for Affairs and Sports, government of India to recognize
studies and monitoring programmes on rare and outstanding achievement in National sports.
endangered species and their habits. 99. (c) "Place of the Thunderbolt" is associated with
83. (c) OPEC is an intergovernmental organization that was Darjeeling(west Bengal). The word Darjeeling is a
created at the Baghdad Conference on September 10- combination of two words 'dorje,' which means
14, 1960, by Iraq, Kuwait, Iran, Saudi Arabia and 'thunderbolt' and 'ling', which means 'place'. Hence the
Venezuela. Later it was joined by nine more word Darjeeling means 'the Land of Thunderbolt'.
governments: Libya, United Arab Emirates, Qatar, 100. (c) The Ship building yard Mazagon Dock Limited (MDL)
Indonesia, Algeria, Nigeria, Ecuador, Angola, and is located in Mumbai(Maharashtra).It is India's prime

ww
Gabon. OPEC was headquartered in Geneva, Switzerland
before moving to Vienna, Austria, on September 1, 1965.
shipyard. It manufactures warships and submarines
for the Indian Navy and offshore platforms and
associated support vessels for offshore oil drilling.
84.

85.
(b)

(b) w.E
Swatantra Kumar is the Chairperson of National Green
Tribunal.
Barmako is the capital of Mali.
101 (d) Aga Khan Cup is related to Football. When Prince
Aga Khan IV of Iran visit in 1958, expressed his interest
86. (b)

asy
Jaspal Singh Bhatti (3 March 1955 - 25 October 2012)
was an Indian television personality famous for his
satirical take on the problems of the common man. He
to start a major international football club tournament
in the region. The football authorities of East Pakistan
in collaboration with Asian Football Confederation

En
is most well known for his television series Flop Show
and mini capsules Ulta Pulta which ran on Doordarshan,
(AFC) decided to start this event.
102. (a) The first summit was held in Dhaka, Bangladesh on 7-
India's national television network, in the late 1980s
and early 1990s. In 2013, he was honoured with the gin 8 December 1985 and was attended by president of
Bangladesh, Maldives, Pakistan and Sri Lanka, the

87. (b)
Padma Bhushan (posthumously), India's third highest
civilian award
Lal Bahadur Shastri was the second Prime Minister of
India.
eer
kings of Bhutan and Nepal, and the prime minister of

the Republic of India and a leader of the Indian National


Congress party. Shastri joined the Indian independence ing
103. (b) Rabindranath Tagore was the first Indian ever to
receive a Nobel Prize. He was awarded the Nobel Priz e
for Literature in recognition of his work Geetanjali,a

88. (b)
movement in the 1920s.
89. (d) 90. (c) 91. (d) 92. (b)
collection of poems, in 1913.
.ne
104. (d) Vijaya Lakshmi Pandit became the first woman to be
93. (a) The National Institute of Nutrition (NIN) is an Indian
Public health, Biotechnology and Translational
research center located in Hyderabad, India. The
institute is one of the oldest research centers in India,
t
elected president of the UN General Assembly in 1953.
She was an Indian diplomat, politician, and a sister of
India's first prime-minister, Jawaharlal Nehru. She was
active in the Indian freedom movement and held high
and the largest center, under the Indian Council of national and international positions.
Medical Research, located in the vicinity of Osmania 105. (a) The currency of Saudi Arabia is Saudi Riyal.
University. 106. (d) After independence, the government passed Reserve
94. (d) Bullseye, also known as conventional pistol, is a Bank (Transfer to Public Ownership) Act, 1948 and
shooting sport in which participants shoot handguns took over RBI from private shareholders after paying
at paper targets at fixed distances and time limits. appropriate compensation. Thus, nationalisation of RBI
95. (d) Pablo Ruiz y Picasso, also known as Pablo Picasso, took place in 1949 and from January 1, 1949, RBI started
was a Spanish painter, sculptor, printmaker, ceramicist, working as a government owned central bank of India.
stage designer, poet and playwright who spent most 107. (d) Garba is a folk dance of state of Gujarat.
of his adult life in France. 108. (b) Antoine-Henri Becquerel (1852-1908) is known for his
96. (a) Endymion is a poem,written by John Keats. It begins discovery of radioactivity, for which he received the
with the line "A thing of beauty is a joy forever". The Nobel Prize for Physics jointly with Marie Curie) and
poem tells about how nature and its wonder mesmerize Pierre in 1903 and the contributions he made to that
us and take away all the sorrow that surrounds us from field. Henri Becquerel discovered radioactivity, and
time to time. Mari Curie coined the term.

Downloded From : www.EasyEngineering.net


Downloded From : www.EasyEngineering.net

108 GENERAL KNOWLEDGE


109. (d) Mission Indradhanush was launched by Union Health title in singles category. She is a retired World No. 1
Minister J.P Nadda on 25 December 2014. It aims to professional tennis player and Christian minister from
immunize all children under the age of 2 years and Australia.
pregnant women against seven vaccine preventable 130. (c) Pran acted as a character artist in the film Zanjeer. It
diseases namely diphtheria, whooping cough, tetanus, made him stand up in defiance as Sher Khan in Zanjeer
poliomyelitis, tuberculosis, measles and Hepatitis B (1973) against the six-feet tall, long-legged Amitabh
by 2020. In addition to this, vaccines for Japanese Bachchan.Pran had a successful career spanning over
Encephalitis (JE), rotavac and Haemophilus influenzae six decades, and his contribution to Hindi cinema
type B (HIB) are also being provided in selected states. extends beyond his role as an actor.
110. (b) 111. (d) 131. (d) Lal Bahadur Shastri was the first posthumous recipient
112. (d) BRICS is the acronym for an association of five major of Bharat Ratna in 1966. Lal Bahadur Shastri was the
emerging national economies: Brazil, Russia, India, third Prime Minister of the Republic of India and a
China and South Africa. Originally the first four were leader of the Indian National Congress party. Shastri
grouped as “BRIC” (or “the BRICs”), before the joined the Indian independence movement in the 1920s.
induction of South Africa in 2010. 132. (d) Australia won the ICC Women's World Cup held in
113. (a) Bezwada Wilson and TM Krishna are included in the February 2013. The 2013 Women's Cricket World Cup
list of the prestigious Ramon Magsaysay Award was the tenth Women's Cricket World Cup, which was

ww
winners for the year 2016.
114. (a) Dipa Karmakar returned empty handed from the Vaults
hosted by India for the third time. India previously
hosted the World Cup in 1978 and 1997. Australia won
the tournament for the sixth time, beating West Indies

w.E
final at the Gymnastics event of Rio 2016 Olympics
after she finished the event at fourth rank.
115. (c) Orology is the study of mountains.An example of
133. (a)
by 114 runs in the final.
Bhutan's currency is known as Ngultrum. n 1974, the
ngultrum was introduced, replacing the rupee at par.
to be formed.
asy
orology is research on how a particular mountain came

116. (b) It was built by the Khmer King Suryavarman II in the


The ngultrum is equal in value to the Indian rupee and
it does not exchange independently with other nations'
currencies but is interchangeable with the Indian rupee.
early 12th century.
En
117. (d) The Indian rupee sign is the currency sign for the Indian
134. (b) "Sattriya Nritya" is a classical dance form of India an
has originated in Assam. Sattriya or Sattriya Nritya ,si
d

rupee, the official currency of India. Designed by D.


Udaya Kumar, it was presented to the public by the gin one among the eight principal classical Indian dance
traditions. In the year 2000, the Sattriya dances of
Government of India on 15 July 2010, following its
selection through an "open" competition among Indian
residents. eer
Assam received recognition as one of the eight
classical dance forms of India.

118. (a) Micro Units Development and Refinance Agency Bank


(or MUDRA Bank) is a public sector financial
135. (b)

ing
A Grammy Award is an accolade by the National
Academy of Recording Arts and Sciences (NARAS )
of the United States to recognize outstanding
institution in India.
119. (b) 120. (b) 121. (c) 122. (c) 136. (b) .ne
achievement in the music industry.
Indira Gandhi was the first woman of India who received
123. (d) 124. (b) 125. (b)
126. (a) Minorities Rights Day is observed in India in 18th
December. National Commission for Minorities
celebrated Minorities Rights Day on 18 December 2012.
137. (c)
Republic of India, in 1971.
t
the Bharat Ratna, the highest civilian award of the

Das Kapital by Karl Marx, is a critical analysis of


political economy, intended to reveal the economic laws
Minorities Rights Day is celebrated on 18th December of the capitalist mode of production.
every year. The day is celebrated to protect rights of 138. (b) The Torah is written on scrolls and kept in a special
the minorities communities as well as bringing the better cabinet called the aron hakodish, the holy ark, in
understanding among religious minorities in India. synagogues. The Torah is read with a pointer called a
127. (d) Central Drug Research Institute is wrongly matched yad (hand) to keep it from being spoiled. Each week,
with Kanpur. The Central Drug Research Institute is a one section is read until the entire Torah is completed
and the reading begins again.
multidisciplinary research laboratory in Lucknow, India,
employing scientific personnel from various areas of 139. (d) Meghdoot is a lyric poem written by Kalidas,
biomedical sciences. considered to be one of the greatest Sanskrit poets.
128. (c) Trafalgar Square is associated with London, St.Peter's 140. (d) Mulk Raj Anand was an Indian writer in English, notable
for his depiction of the lives of the poorer castes in
Square is associated with Vatican City, Red Square is
traditional Indian society. One of the pioneers of Indo-
associated with Moscow and Times Square is
Anglian fiction, he together with R. K. Narayan, Ahmad
associated with New York.
Aliand Raja Rao, was one of the first India-based writers
129. (d) Margaret Court has won the Grand Slams title for the in English to gain an international readership.
maximum number of times. She has won 24 Grand Slams

Downloded From : www.EasyEngineering.net


Downloded From : www.EasyEngineering.net

GENERAL KNOWLEDGE 109


141. (d) The States Reorganisation Act, 1956 was a major reform 160. (a) Earth Day is celebrated of 22nd April all over the world
of the boundaries of India's states and territories, to demostrate support for environmental protection.
organising them along linguistic lines. 161. (d) In Rugby football each team starts the match with 15
142. (a) Mein Kampf is an autobiographical manifesto by Nazi players on the field and seven or eight substitutes.
leader Adolf Hitler, in which he outlines his political 162. (b) The first state to be formed on linguistic basis in India
ideology and future plans for Germany. was Andhra Pradesh, which was divided into two
143. (d) On 15 June 2007, the United Nations General Assembly Telugu-speaking states.
voted to establish 2 October as the International Day 163. (a) The Survey of India Dept. is headquartered at
of Non-Violence. The resolution by the General Dehradun. It has 18 civil engineering divisions ranging
Assembly asks all members of the UN system to from the prediction of tides to aerial survey.
commemorate 2 October in "an appropriate manner and 164. (d) The Lalit Kala Academi or National Academy of Art is
disseminate the message of non-violence, including India’s national academy of fine arts. Its headquarters
through education and public awareness are at Ravindra Bhawan, Ferozshah Road, New Delhi.
144. (d) The Master Control Facility (MCF) is a facility set up 165. (c) The World Wildlife Fund is an international non-
by the Indian Space Research Organisation (ISRO) in governmental organization founded on 29th April, 1961.
the city of Hassan in the Indian state of Karnataka. 166. (d) The God of Small Things (1997) is the debut novel of
145. (c) The Silicon Valley of India is a nickname of the Indian Indian writer Arundhati Roy.

ww
city of Bangalore. The name signifies Bangalore's status
as a hub for information technology (IT) companies in
167. (a) Nilgiri Biosphere Reserve is India’s first and foremost
biosphere reserves with a heritage rich in flora and

w.E
India and is a comparative reference to the original
Silicon Valley, based around Santa Clara Valley,
California, a major hub for IT companies in the United
fauna.
168. (d) The Kimono is a traditional Japanese garment.
169. (a) Dr. P. Rama Rao committee is related to defence.
States.

asy
146. (d) Shri Aurbindo has written 'Life Divine' which deals with
theoretical aspects of Integral Yoga.
170. (d) Kathakali is a traditional dance from the south Indian
state of Kerala.

147. (c) Walt Disney has won the most Oscar awards.
148. (b) Leela Samson is a Bharatanatyam dancer, En 171. (d)
172. (a) Magnetization technique can be used to seperate th e
iron fillings from the heterogenous mixture assuming
choreographer instructor and writer from India.
149. (b) Ryder Cup is a biennial men’s gold competition gin the other element of the mixture are non-magnetic.
173. (b) Amartya Kumar Sen, (born 3 November 1933) is an
between teams from Europe and the United States.
150. (d) Kanha Tiger Reserve, also called “Kanha National
eer
Indian economist and a Nobel laureate. He has made
contributions to welfare economics, social choic e
Park” is one of the tiger reserve of India and the largest
national Park of Madhya Pradesh.
151. (b) The Discovery of India was written by India’s first ing
theory, economic and social justice, economic theories
of famines, and indexes of the measure of well-being fo
citizens of developing countries. He was awarded the
Prime Minister Jawaharlal Nehru during his imprisoment
in 1942-46 at Ahmednagar fort in Maharashtra. .ne
Nobel Memorial Prize in Economic Sciences in 1998 for
his work in welfare economics.
152. (b) Kiran Bedi was the first woman IPS officer in India.
Bedi joined the Indian Police Service (IPS) in 1972.
153. (a) ILO stands for International Labour Organisation which
is a United Nations agency dealing with Labour issues.
174. (a) The Great Depression was a severe worldwide
t
economic depression in the decade preceding World
War II. The timing of the Great Depression varied
across nations, but in most countries it started in 1930
154. (c) The Times of London in 2010 Published its 70,000th and lasted until the late 1930s or middle 1940s. It was
issue. the longest, most widespread, and deepest depression
155. (b) Project Tiger was launched in 1973 by the Government of the 20th century.
of India during Prime Minister Indira Gandhi’s tenure. 175. (b) The Indian Foreign Service is the foreign service under
156. (d) According to the figures compiled by Media Research Group A and Group B of the Central Civil Services of
Users Council (MRUC) in the Indian Readership the executive branch of the Government of India. It is
Survey (IRS) 2014, the Indian newspaper with the the body of career diplomats of India. The Indian
largest readership is the Dainik Jagran. Foreign Service is part of the Central Civil Services of
157. (d) International Yoga day is celebrated annually on June the Government of India. The Foreign Secretary of India
21. is the administrative head of the Indian Foreign Service.
158. (d) World AIDS Day is observed on 1st December, to raise 176. (c) Sethusamudram Shipping Canal Project is a proposed
awareness of the AIDS pandemic caused by the spread project that would link Palk Bay and the Gulf of Mannar
of HIV infection. between India and Sri Lanka by creating a shipping
159. (c) Trygve Lie was the first secretary General of UNO. channel through the shallow sea called Sethusamudram
and through a chain of islands collectively called

Downloded From : www.EasyEngineering.net


Downloded From : www.EasyEngineering.net

110 GENERAL KNOWLEDGE


Adam's Bridge, Ramar Palam , Ram Sethu and similar 185. (c) Maharashtra is the second most populous state as per
names. This would provide a continuously navigable census 2011 after Uttar Pradesh.
sea route in and around the Indian Peninsula. 186. (a) 187. (c)
177. (d) A metropolitan area, sometimes referred to as a metro 188. (c) Santosh Yadav is an Indian mountaineer. She is the
area or metro, is a region consisting of a densely first woman in the world to climb Mount Everest twice
populated urban core and its less-populated in less than a year. She first climbed the peak in May
surrounding territories, sharing industry, infrastructure, 1992 and then did it again in May 1993.
and housing. A metropolitan area usually comprises 189. (c) Roop Singh Bais was an Indian hockey player. He was
multiple jurisdictions and municipalities: part of the celebrated Indian field hockey team which
neighborhoods, townships, cities, exurbs, countries, won gold medals for India at 1932 and 1936 Olympic
and even states. Games. After the final of the 1936 Summer Olympics,
178. (b) The Kyoto Protocol to the United Nations Framework Germans were so impressed with him that they named
Convention on Climate Change (UNFCCC) is an a street after him.
international treaty that sets binding obligations on 190. (a) In 1981 with the first Indian expedition to Antarctica.]
industrialised countries to reduce emissions of The program gained global acceptance with India's
greenhouse gases. The UNFCCC is an environmental signing of the Antarctic Treaty and subsequent
treaty with the goal of preventing "dangerous" construction of the Dakshin Gangotri Antarctic

179. (a)
ww
anthropogenic (i.e., human-induced) interference of the
climate system.
Sanjukta Panigrahi (24 August 1944 - 24 June 1997) [1]
research base in 1983 superseded by the Maitri base
from 1990.

w.E
was a dancer of India, who was the foremost exponent
of Indian classical dance Odissi. Sanjukta was the first
Oriya girl to embrace this ancient classical dance at an
191. (a) 192. (c)
193. (c) Rajasthan has the dubious distinction of having the
highest difference in male-female literacy in the country

180. (c) asy


early age and ensure its grand revival.
The father of the White Revolution in India was
at 28%, against the national average of 16.7%.
194. (d) Azadirachta indica, also known as Neem, is the state
tree of Andhra Pradesh. Products made from Neem trees
Verghese Kurien. The white mentioned is milk
production. He was able to implement programs that
En have been used in India for over two millennia for their
medicinal properties: Neem products are believed ot

181. (b)
took India from a country with very little dairy
production to the world's largest producer.
Charles Robert Darwin, FRS (12 February 1809 - 19
gin be anthelmintic, antifungal, antidiabetic. antibacterial
antiviral, contraceptive and sedative.
,

April 1882) was an English naturalist and geologist,


best known for his contributions to evolutionary theory. eer
195. (b) The Geological Survey of India (GSI), established ni
1851, is a government organization in India which isna
office attached to the Ministry of Mines of Unio n
He established that all species of life have descended
over time from common ancestors, and in a joint ing
Government of India for conducting geological surveys
and studies.
publication with Alfred Russel Wallace introduced his
scientific theory that this branching pattern of evolution
resulted from a process that he called natural selection, .ne
196. (c) Sunmeet Kaur Sawhney, a Punjabi woman, is the winner
of Rs 5 crore on TV game show Kaun Banega

182. (b)
in which the struggle for existence has a similar effect
to the artificial selection involved in selective breeding.
Chandrayaan-1 was India's first lunar probe. It was
198. (a)
Crorepati 6.

'Haryana Hurricane'.
t
197. (c) Kapil Dev's fast pace earned him the nickname,

launched by the Indian Space Research Organisation


in October 2008, and operated until August 2009. 199. (a) Arya Samaj is an Indian religious reform movement
founded by Swami Dayananda on 7th April 1875.
183. (d) Lira was the currency of Italy between 1861 and 2002.
Between 1999 and 2002, the Italian lira was officially a 200. (a) 201. (d) 202. (c) 203. (a) 204. (a)
national subunit of the euro. 205. (b) 206. (b) 207. (d) 208. (c) 209. (a)
184. (c) The President of India is the Supreme Commander of 210. (c) 211. (d) 212. (c) 213. (d) 214. (a)
the Indian Armed Forces. The Indian Armed Forces 215. (a) 216. (b) 217. (b) 218. (d) 219. (b)
are under the management of the Ministry of Defence 220. (b) 221. (d) 222. (d) 223. (d) 224. (a)
(MoD), which is led by the Union Cabinet Minister of 225. (c) 226. (d) 227. (d) 228. (d)
Defence.

Downloded From : www.EasyEngineering.net


Downloded From : www.EasyEngineering.net

PRACTICE SET - 1
1. Which of the following symbiotic associations forms a 9. Which of the following is not a laid down principle of the
lichen? Panchsheel ?
(a) An algae and a fungus (a) Mutual respect for each other's territorial
(b) An algae and a bryophyte integrity
(c) A bacterium and a fungus (b) Mutual non-aggression
(d) A bacterium and a gymnosperm (c) Mutual support for each other in world
2. The headquaters of which one of the following organizations forum
(d) Mutual non-interference in each other's
is not in Geneva?
internal affairs

ww
(a) Food and Agricultural Organisation
(b) World Meteorological Organisation
(c) World Health Organisation
10. Denatured alcohol
(a) is a form of alcohol

3. w.E
(d) World Trade Organisation
The opportunity cost of a factor of production is
(a) what it earns in its present use.
(b) is unfit for drinking as it contains poisonous
substances
(c) contains coloured impurities

asy
(b) what it can earn in the long period.
(c) what it can earn in some other use.
11.
(d) is sweet to taste
The city of Prayag was named Allahabad - the city of Allah
by

4.
(d) the cost of production.
En
Which Amendment Act is referred as mini constitution?
(a) Aurangzeb
(b) Akbar
(a) 7th Constitutional Amendment Act, 1956
(b) 24th Constitutional Amendment Act, 1971
(c) 42nd Constitutional Amendment Act, 1976
gin
12.
(c) Shahjahan
(d) Bahadur Shah Zafar
Arrange the following historical events chronologically

5.
(d) 44th Consitutional Amendment Act, 1978
Inflation is caused by eer
choosing the correct response:
I. French Revolution
(a) decrease in production
(b) increase in money supply and decrease in ing
II. Glorious Revolution
III. American War of Independence
IV. Russian Revolution
production
(c) increase in money supply
(a) I II III IV
(b) II III I IV .ne
6.
(d) increase in production
The equilibrium of a firm under perfect competition will be
determined when
(a) Marginal Cost > Average Cost
13.
(c) II I
(d) III II I
IV III
IV
Chromosomes are made up of
(a) DNA (b) Protein
t
(b) Marginal Revenue > Average Cost (c) DNA and Protein (d) RNA
(c) Marginal Revenue > Average Revenue 14. While the computer executes a program, the program is held
(d) Marginal Revenue = Marginal Cost in
7. Which one of the following cities and the personalities (a) RAM (b) ROM
associated with their establishment is wrongly matched? (c) Hard Disk (d) Floppy Disk
(a) Calcutta – Robert Clive 15. Presidential form of government consists of the following?
(b) Pondicherry – Francis Martin (a) Popular election of the President
(c) Ahmedabad – Ahmad Shah I (b) No overlap in membership between the executive and
(d) Madras – Francis Day the legislature
8. Arihant is a (c) Fixed term of office
(a) Multi barrel rocket launcher (d) All of the above
(b) Airborne Early Warning and Control 16. Which of the following places of Sikh religious heritage is
System not in India?
(c) Unmarmed Combat Aerial Vehicle (a) Nankana Sahib (b) Nanded
(d) Nuclear-powered ballistic missile submarine (c) Paonta Sahib (d) Keshgarh Sahib

Downloded From : www.EasyEngineering.net


Downloded From : www.EasyEngineering.net

PS-2 Practice Set-1


17. The total population divided by available arable land area is 22. In human body, vitamin A is stored in the –
referred to as (a) liver (b) skin
(a) Population density (c) lung (d) kidney
(b) Nutritional density 23. Ondometer is a –
(c) Agricultural density (a) Measuring instrument for distance covered by motor
(d) Industrial density wheels
18. The danger signals are red while the eye is more sensitive to (b) Measuring instrument for frequency of
yellow because electromagnetic waves
(a) absorption in red is less than yellow and hence red is (c) Device for measuring sound intensity
visible from a distance (d) Measuring instrument for electric power
(b) scattering in yellow light is less than red 24. Which acid is used in rubber, textile, leather and
(c) the wavelength of red light is more than yellow light electroplating industries ?
(d) none of the above reasons (a) Ethanoic acid
19. Who was the author of "India of My Dreams" ? (b) Methanoic acid
(a) J.B. Kripalani (b) M.K. Gandhi (c) Malanic acid
(c) G.K. Gokhale (d) Jawaharlal Nehru (d) Butairic acid
20. How many players are there in a Polo team ?
(a) 4
(c) 8 ww (b) 7
(d) 6
25. What is the theme of 2016 National Statistics Day?
(a) Social Development
(b) Trees and their calcualtive lives
21. Hemophilia is –

w.E
(a) caused by bacteria (b) caused by virus
(c) caused by pollutants (d) a hereditary defect
(c) Human empowerment
(d) Agriculture and Farmers’ welfare

asy
En
gin
eer
ing
.ne
t

Downloded From : www.EasyEngineering.net


Downloded From : www.EasyEngineering.net

Practice Set-1 PS-3

HINTS & EXPLANATIONS


1. (a) 2. (a) 3. (c) 4. (c) 5. (b) 12. (b) 13. (c) 14. (a) 15. (d) 16. (a)
6. (d) 7. (a) 17. (a)
8. (d) Arihant is a Nuclear powered ballistic missile submarine. 18. (c) This is because the scattering in red light is less than
9. (c) Option (c) is not in the list of principles of panchsheel. that of yellow colour. The longest visible wavelength
This agreement stated the five principles as: is red and the shortest is violet. The wavelength of red
1. Mutual respect for each other's territorial integrity
light is more than yellow light.
and sovereignty.
19. (b)
2. Mutual non-aggression.
20. (a) There are 4 players in a polo team.
3. Mutual non-interference in each other's internal
affairs. 21. (d) Hemophilia is a hereditary defect.
4. Equality and cooperation for mutual benefit. 22. (a) In human body, vitamin A is stored in the liver.
23. (b) Ondometer is a measuring instrument for frequency
10.
11.
(b)
ww
5. Peaceful co-existence.

(b) Emperor Akbar named Prayag as Allahabad - City of 24.


of electromagnetic waves.
(b) Methanoic acid is a colorless, pungent smelling liquid

w.E
God- also called Allahabad in 1575 AD. The city of
Allahabad is situated at the confluence of three rivers
- Ganga, Yamuna and the invisible Saraswati. Every
with a boiling point 373.5 K. Due to the presence of
aldehyde-like hydrogen, it is powerful reducing
agent.It reduces Tollen's reagent and Fehling's solution.

asy
12th year when the waters are felt to be especially
purifying, Allahabad holds a much greater festival called
Kumbh Mela. Built by Emperor Akbar in 1583 AD, the
It is used in rubber, textile, dyeing, leather and
electroplating industries.

En
Allahbad fort stands on the banks of the river Yamuna
near the confluence site i.e SANGAM.
25. (d)

gin
eer
ing
.ne
t

Downloded From : www.EasyEngineering.net


Downloded From : www.EasyEngineering.net

PS-4 Practice Set-2

PRACTICE SET - 2
1. Which Article of the Indian Constitution guarantees rights 12. Tattvabodhini Sabha was founded by ………....... In 1839
to arrested persons ? (a) Swami Vivekanand
(a) Article 22 (b) Article 35 (b) Keshav Chandra Sen
(c) Article 20 (d) Article 42
(c) Dabendranath Tagore
2. For a person having hypermetropia, the near point is
(d) Swami Sahajanamd
…………………..
13. After the revolt of 1857, British pursued the policy of
(a) Greater than 20 cm (b) Lesser than 25cm
(c) Greater than 25cm (d) Lesser than 30cm ……………
3.
ww
Cryogenic is a science deals with
(a) High Temperatures(b) Low Pressure
(a) Divide and Policies
(b) Rules and Regulation

4.
(c) High Pressure
w.E (d) Low Temperature
..................... is an active factor of production
(a) Product (b) Labour 14.
(c) Divide and Rule
(d) Unity and Poliy
Prithvi-I missile was inducted into the . ................in 1994

5.
(c) Wages
asy
(d) Price
When total utility becomes maximum, then marginal utility
(a) Indian Army
(c) Indian Navy
(b) Indian Air Force
(d) All of these
will be
(a) Maximum En 15. 2018 FIFA World Cup to be held in …………….
(a) China (b) Russia
(b) Minimum
(c) Either maximum or minimum gin
16.
(c) India (d) Brazil
.…………………is issued by the court in case of illegal
6.
(d) Zero
Revealed Preference Theory was propounded by
eer
detention of a person
(a) Quo Warranto (b) Habeas Corpus
………………..
(a) Robbins
(c) Samuelson
(b) Smith
(d) Schumpter 17.
(c) Mandamus
ing
(d) Certiorari
At the time of Emergency, the Indian State become unitary
7. One Carat of diamond is equal to ..……………
(a) 200m (b) 100m
from …………………….
(a) Semi Federal (b) Federal .ne
8.
(c) 150m (d) 300m
Wood Spirit is which of the following ?
(a) Ethyl Alcohol (b) Propanol
18.
(c) Unitary (d) Quasi-federal
t
The book titled 'The Life and Death of Adolf Hitler' is penned
by
(c) Methyl Alcohol (d) Butanol (a) Z.A. Bhutto
9. Which of the following is chief source of Napthalene ? (b) James Cross Giblin
(a) Moth balls (b) Mothflakes
(c) J.M. Barrie
(c) Tar Camphor (d) Coal tar
(d) Gunnar Myrdal
10. Study of crop production is ………………
19. Bos Taurus is a scientific name of ………………..
(a) Entology (b) Ecology
(c) Botany (d) Agronomy (a) Buffalo (b) Horse
11. Who was the last guru of the Sikhs ? (c) Cow (d) Cat
(a) Guru Granth Sahib 20. …………….. includes all prokaryotic organism likes bacteria,
(b) Guru Gobind Singh cynobacterioa and archiobacteria
(c) Guru Angad (a) Animalia (b) Protista
(d) Guru Amar Das (c) Monera (d) Planatae

Downloded From : www.EasyEngineering.net


Downloded From : www.EasyEngineering.net

Practice Set-2 PS-5

21. …………………… is the Kuchipudi dancer 24. World's largest Charkha (spinning wheels) that was unveiled
(a) Anupama Mohan at Terminal 3 of the Indira Gandhi International Airport (IGI),
(b) Bimbavati Devi New Delhi is made of the teak wood of which country?
(c) Arush Mudgal (a) Burma (b) Sri Lanka
(d) Swapnasundari (c) Nepal (d) Ukraine
22. Which of the following is not a chief organ of the United 25. The Union Cabinet recently approved The High Courts
Nations Organisations? (Alteration of Names) Bill, 2016 to be introduced in the
(a) International Labour Organisation Parliament. The bill will facilitate the changing of the names
(b) Security Council of which two high courts?
(c) International Court of Justice (a) Bombay High Court and Calcutta High Court
(d) General Assembly (b) Bombay High Court and Madras High Court
23. The treaty of Mangalore was signed between (c) Calcutta High Court and Madras High Court
(a) the English East India Company and Haidar Ali (d) Bombay High Court and Gauhati High Court
(b) the English East India Company and Tipu Sultan
(c) Haidar Ali and the Zamorin of Calicut

ww
(d) the French East India Company and Tipu Sultan

w.E
asy
En
gin
eer
ing
.ne
t

Downloded From : www.EasyEngineering.net


Downloded From : www.EasyEngineering.net

PS-6 Practice Set-2

HINTS & EXPLANATIONS


1. (a) Article 22 proceeds to guarantee certain fundamental 13. (c) After the revolt, the British pursued the policy of divide
rights to every arrested person. and rule, towards the general populace.
2. (c) Hypermetropia Myopia is corrected by spectacles 14. (a) Prithvi (Sanskrit: prthvi "Earth") is a tactical surface-
having concave lens.Near point of aperson suffering to-surface short-range ballistic missile .This class of
from hypermetropia is more than 25cm. Prithvi missile was inducted into the Indian Army in
3. (d) Cryogenics is the study of the production and 1994
behaviour of materials at very low temperatures. 15. (b) FIFA's decision to award Russia the right to host the
2018 World Cup surprised many - including some of
4. (b) Some of the important factors of production are: (i) Land
the country's leaders
(ii) Labour (iii) Capital (iv) Enterprnuer. Land is a passive
factor whereas labour is an active factor of production 16. (b) Habeas corpus ("You may have the body") is a recourse
in law whereby a person can report an unlawful
5. (d) When total utility is maximum at the 5th unit, marginal

6. ww
utility is zero
(c) Revealed preference theory, pioneered by American
economist Paul Samuelson, is a method of analyzing
17.
detention or imprisonment before a court, usually
through a prison official
(d) Professor K.C. Wheare, who regards the American

7. w.E
choices made by individuals
(a) One carat is equal to 0.2 grams
constitution as the model of a true federation has
described the Indian constitution as 'quasi federal', that
is 'a unitary state with subsidiary federal features rather
8.

asy
(c) Wood spirit is a poisonous colorless liquid used as a
solvent and fuel; ingestion may cause blindness or
death. Called also methyl or wood alcohol.
18.
than a federal state with subsidiary unitary features
(b) Many people believe Hitler was the personification of

9. (d) Naphthalene is an organic compound with formula C


En
10H 8. It is the simplest polycyclic. Naphthalene is the
evil. In this Sibert Medal-winning biography, James
Cross Giblin penetrates this façade and presents a pictur
of a complex person-at once a brilliant, influential
e

10.
most abundant single component of coal tar.
(d) Agronomy is the science and technology of producing gin politician and a deeply disturbed man. Giblin explores
the forces that shaped the man as well as the socia l
and using plants for food, fuel, fiber, and land
reclamation 19. eer
conditions that furthered his rapid rise to power.
(c) Cows are raised in many different countries around
11.

12.
(b) Guru Gobind Singh was The Tenth Nanak or the last of
the Sikhpreachers to live.
(c) The Tattwabodhinl Sabha ("Truth Propagating/
as milk, meat
ing
the world, mainly for the cowsnatural resources such

Searching Society") was a group started in Calcutta on 6


October 1839 as a splinter group of the Brahmo Samaj,
20.
are prokaryotes
.ne
(c) Monera Kingdom- All the organisms of this kingdom

reformers of Hinduism and Indian Society. The founding


member was Debendranath Tagore
21.

22.
of Kuchipudi.
(a) 23. (b) 24. (a) 25. (b) t
(a) Anupama Mohan is one of the best-known disciples

Downloded From : www.EasyEngineering.net


Downloded From : www.EasyEngineering.net

Practice Set-3 PS-7

PRACTICE SET - 3
1. Which one of the following is not a computer language? 11. Which one of the following Schedules of the Constitution
(a) Cobol (b) Visual Basic of India includes the disqualification of a Legislator on
(c) HTML (d) Netscape grounds of defection?
2. Who among the following was the first Governor General of (a) 8th Schedule (b) 7th Schedule
India? (c) 9th Schedule (d) 10th Schedule
(a) Lord Amherst 12. Which one of the following causes the chikungunia
(b) Lord William Bentinck disease?
(c) Sir Charles Metcalfe (a) Bacteria (b) Helminthic worm

3. ww
(d) Robert Clive
Which one of the following is not a constituent of biogas? 13.
(c) Protozoan (d) Virus
Who among the following recommends to the Parliament

4.
(a) Methane
(c) Hydrogen
w.E (b) Carbon dioxide
(d) Nitrogen dioxide
In which one of the following sessions was the Indian
for the abolition of the Legislative Council in a State?
(a) The President of India
(b) The Governor of the concerned State

(c) Surat
asy
National Congress split into moderates and extremists?
(a) Nagpur (b) Allahabad
(d) Calcutta 14.
(c) The Legislative Council of the concerned State
(d) The Legislative Assembly of the concerned State
Which one of the following vitamins helps in clotting of
5. Bar is a unit of which one of the following?
En blood?
(a) Vitamin-A (b) Vitamin-B6

6.
(a) Force
(c) Pressure
(b) Energy
(d) Frequency
Which of the following metals are present in gin
15.
(c) Vitamin-D (d) Vitamin-K
The ‘Thomas Cup is associated with
haemoglobin and chlorophyll, respectively?
(a) Fe and Mg (b) Fe and Zn eer
(a) Table Tennis
(c) Badminton
(b) Lawn Tennis
(d) Billiards

7.
(c) Mg and Zn (d) Zn and Mg
A mother of blood group O has a group O child. What could
16.
ing
Which one among the following pairs is correctly matched
(a) The Second Battle : Defeat of Jaichand
?

be the blood group of father of the child?


(a) Only O (b) A or B or O
of Tarain
.ne
of Kannauj
by Muhammad of Ghori

8.
(c) A or B

League?
(a) Muhammad Ali Jinnah
(d) Only AB
Who among the following was the founder of the Muslim
(b) The First Battle of : Defeat of Sikander
Panipat

(c) The Battle of


Lodhi by
Babur
: Defeat of Humayun
t
(b) Shaukat Ali Chausa by Sher Shah
(c) Nawab Salimullah (d) The Battle of : Defeat of Rana
(d) Aga Khan Khanwa Pratap by Akbar
9. Which one among the following is not a source of tax 17. What is the purpose of adding baking soda to dough?
revenue for the Central Government in India ? (a) To generate moisture
(a) Income tax (b) Customs duuties (b) To give a good flavour
(c) Service tax (d) Motor Vehicle tax (c) To give good colour
10. Which of the following does not form part of current account (d) To generate carbon dioxide
of Balance of Payments? 18. The ‘Arthasastra’ is a treatise on which one of the following?
(a) Export and import of goods (a) Economics
(b) Export and import of services (b) Environment
(c) Income receipts and payments (c) Political Philosophy
(d) Capital receipts and payments (d) Religion in Administration

Downloded From : www.EasyEngineering.net


Downloded From : www.EasyEngineering.net

PS-8 Practice Set-3


19. Which one of the following glands in the human body stores 23. What is the recently extended deadline for states to join
iodine? Union Government's Ujwal Discom Assurance Yojana
(a) Parathyroid (b) Thyroid (UDAY) scheme?
(c) Pituitary (d) Adrenal
(a) March 2017
20. India's first integrated Defense Communication Network
(DCN) has been launched in which of the following cities? (b) January 2018
(a) New Delhi (b) Lucknow (c) March 2018
(c) Ahmedabad (d) Kochi (d) January 2017
21. The book "The mind of the terrorist: the Psychology of 24. As of now, how many countries are members of Nuclear
terrorism from the IRA to al-Qaeda" has been authored by Suppliers Group (NSG)?
whom? (a) 48 (b) 56
(a) Amitav Ghosh (c) 64 (d) 96
(b) Ashwin Sanghi 25. India's first AYUSH university will be set up in which state
(c) Kunal Basu of India?
(d) Jerrold M. Post (a) Sikkim
22. The Government of India (GoI) has extended the ban on the
(b) Haryana

ww
import of milk and milk products of China till which year?
(a) June 2017
(c) December 2017
(b) March 2017
(d) January 2017
(c) Karnataka
(d) West Bengal

w.E
asy
En
gin
eer
ing
.ne
t

Downloded From : www.EasyEngineering.net


Downloded From : www.EasyEngineering.net

Practice Set-3 PS-9

HINTS & EXPLANATIONS


1. (d) Netscape is an Internet browser that was popular 14. (d) Vitamin-K adds in blood clotting. Vitamin-K acts as an
during the early 1990's. essential cofactor for factor-II, VII, IX, X and also for
2. (a) proteins Z, C and S.
3. (d) Nitrogen dioxide (NO2) is not a component of biogas. 15. (c) Thomas Cup is associated with Badminton.
4. (c) The 23rd Session (1907) of the Congress was held at 16. (b) First Battle of Panipat (1526) was fough between two
Surat.In the session, there was an open clash between mega-powers- Babur, then ruler of Kabul and Ibrahim
the Moderates and the Extremists and ultimately it led Lodhi, king of Delhi Sultanate. It was fought near
to a split in the Congress. Panipat (present day Haryana). Babur won the battle
5. (c) 1 Bar = 105 Pa. Both bar and Pa are the unit of pressure. and established the Mughal Empire.Second Battle of
6. (a) Fe and Mg metals are present in haemoglobin and Panipat (1556) was fought between Akbar (Ruler of
chlorophyll respectively. Mughal Dynasty) and Muhammad Adil Shah (ruler of
7. (b) The blood group of father of the child could be A or B Pashtan Suri Dynasty), along with his Prime Minister

8. ww
or O.
(c) The All India Muslim League, a political organization
Hemu. Third Battle of Panipat (1761) was fought
between the Afghans and the Marathas. The battle
lasted for two months which ultimately resulted in the

w.E
was founded in 1906 by Aga Khan under the Nawab of
Dhaka Salimullah. Its main purpose was to safeguard
the political rights of Muslims in India.
17.
defeat of Marathas and end of their dominance in
India.
(d) Baking soda has sodium bicarbonate as the chief
9.

10.
central government in India.
asy
(d) Motor Vehicle tax is not a source of tax revenue for the

(d) Capital receipts and payments do not form part of


constituent. It decomposes on heating giving carbon
dioxide. This causes dough, cakes, biscuits etc. to

11.
current account of Balance of Payment.
En
(d) The 10th Schedule to the Indian Constitution is known 18.
expand and become light.
(c) The Arthasastra is a treatise on Political philosophy.
as Anti-Defection Law. It was inserted by the 52nd
Amendment Act 1985 to the Constitution. It sets the gin The book, written in Sanskrit, discusses theories and
principles of governing a state. The meaning
provisions for disqualification of elected members on
the grounds of defection to another political party.
19. eer
ofArthashastrais 'Science of Polity'. It is written by
Kautilya.
(b) Thyroid gland in human body contains iodine.
12. (d) Chikungunia is caused by chikenguniya virus which
is an insect borne virus of genus Alphavirus.
Symptoms show high fever, maculopapular rash, ing
Deficiency of iodine creates goitre disease. Which is
observed by the enlargement of larynx.

13. (d)
headache, etc. 20.
24.
(a) 21. (d) 22.
(a) 25. (b)
(b) 23.
.ne
(a)

Downloded From : www.EasyEngineering.net


Downloded From : www.EasyEngineering.net

PS-10 Practice Set-4

PRACTICE SET - 4
1. Which one of the following is a programme that converts 10. In the human body, Cowper's glands form a part of which
high level language to machine language? one of the following system?
(a) Linker (b) Assembler (a) Digestive system
(c) Interpreter (d) Compiler (b) Endocrine system
2. Classification of an enterprise into public or private sector (c) Reproductive system
is based on (d) Nervous system
(a) number of employees in the enterprise 11. Mist is a result of which one of the following
(b) ownership of assets of the enterprise (a) Condensation

ww
(c) employment conditions for workers in the enterprise
(d) nature of products manufactured by the enterprise
(b) Evaporation
(c) Sublimation
3.
w.E
Which one of the following glands produces the growth
hormone (somatotrophin)?
(a) Adrenal (b) Pancreas
12.
(d) Saturation
‘Dyarchy’ was first introduced in India under
(a) Morley-Minto reforms

4.
(c) Pituitary
asy
(d) Thyroid
Who among the following was elected as the President of
(b) Montford reforms
(c) Simon Commision plan
All India Khilafat Conference met at Delhi in 1919?
(a) Motilal Nehru (b) Mahatma Gandhi
En 13.
(d) Government of India Act, 1935
When Lord Mountbatten became the first Governor-Genera l

5.
(c) M A Jinnah (d) Shaukat Ali
Which one of the following Indian states does not have a
common international border with Bangladesh?
gin of India, who among the following became the Governor -
General for Pakistan?
(a) Lord Mountbatten
(a) Manipur
(c) Tripura
(b) Paschim Banga
(d) Asom eer
(b) Muhammad Ali Jinnah
(c) Liaquat Ali Khan
6. Who among the following is the author of the book. ‘The
Namesake’? 14.
(d) Shaukat Ali
ing
Fiscal Policy in India is formulated by
(a) Arundhati Roy
(c) Jhumpa Lahiri
(b) Amitava Ghosh
(d) Kiran Desai
(a) the Reserve Bank of India
(b) the Planning Commission .ne
7. Who among the following was not a member of the
Constituent Assembly?
(a) Sardar Vallabhbhai Patel 15.
(c) the Finance Ministry
(d) the Securities and Exchange Board of India t
Fat can be separated from milk in a cream separation because
of
(b) Acharya JB Kriplani
(c) Lok Nayak Jayprakash (a) cohesive force
(d) K M Munshi (b) gravitational force
8. Carbon dioxide is called a greenhouse gas because (c) centrifugal force
(a) its concentration remains always higher than other (d) centripetal force
gases 16. The average fixed cost curve will always be
(b) it is used in photosynthesis (a) a rectangular hyperbola
(c) it absorbs infrared radiation . (b) a downward sloping convex to the origin curve
(d) it emits visible radiation (c) a downward sloping straight line
9. Laser is a device to produce (d) a U-shaped curve
(a) a beam of white light
17. Malaria in the human body is caused by which one of the
(b) coherent light following organisms?
(c) microwaves (a) Bacteria (b) Virus
(d) X-rays (c) Mosquito (d) Protozoan

Downloded From : www.EasyEngineering.net


Downloded From : www.EasyEngineering.net

Practice Set-4 PS-11

18. The focal length of convex lens is 22. Mohammad Shahid is associated with which sports?
(a) the same for all colours (a) Hockey (b) Badminton
(b) shorter for blue light than for red (c) Wrestling (d) Boxing
(c) shorter for red light than for blue 23. KG Subramanyan, who passes away recently, was a famous
(d) maximum for yellow light personality of which field?
19. The Name of Ram Prasad Bismil is associated with (a) Art (b) Journalism
(a) Kanpur Conspiracy Case (c) Sports (d) Politics
(b) Alipore Conspiracy Case 24. Who has won the 2016 wins Iceland's presidential election?
(c) Kakori Conspiracy Case (a) Davíd Oddsson
(d) Meerut Conspiracy Case
(b) Olafur Ragnar Grimsson
20. The Indian Research Station 'Himadri' is located at
(c) Gudni Johannesson
(a) Siachen (b) Darjeeling
(d) Andri Snaer Magnason
(c) Arctic Region (d) Antarctica
25. Which of the following countries has become the newest
21. What is the theme of the first-ever "National Yoga Olympiad",
member of the Missile Technology Control Regime (MTCR)?
which has been organized by the NCERT?
(a) Canada (b) India
(a) Yoga for Peace and Dhyana
(c) Brazil (d) Poland

ww
(b) Yoga for Health and Harmony
(c) Yoga fo r Kriya and Harmony
(d) Yoga for Emotional and Mental development

w.E
asy
En
gin
eer
ing
.ne
t

Downloded From : www.EasyEngineering.net


Downloded From : www.EasyEngineering.net

PS-12 Practice Set-4

HINTS & EXPLANATIONS


1. (d) A compiler is a special programme that processes 13. (b) Mohammed Ali Jinnah was Indian Muslim politician,
statements written in a particular programming founder and first governor-general (1947-48) of
language and turns them into machine language or Pakistan. As the first Governor-General of Pakistan,
"code" that a computer's processor uses. Jinnah worked to establish the new nation's
2. (b) A company organized for commercial purposes is government and policies, and to help settle the millions
called an enterprise. Classification of an enterprise into of Muslim migrants who had emigrated from the new
public or private sector is based on ownership of nation of India to Pakistan after the partition. He is
assets of the enterprise. revered in Pakistan as Quaid-i-Azam.
3. (c) Somatotrophin is produced by the anterior pituitary. 14. (c) The Department of Economic Affairs (DEA) under
It is a peptide hormone that induces growth, cell Ministry of Finance is the nodal agency of the Union
reproduction and regeneration. Government to formulate and monitor country's
4. (b) Gandhiji was elected President of the All-India Khilafat
economic policies and programmes having a bearing

ww
Conference which met at Delhi on November 23, 1919.
They decided to withdraw all cooperation from the
government if their demands were not met.
15. (c)
on domestic and international aspects of economic
management.
Centrifugal force separates fat from milk.
5.
6.
w.E
(a) Manipur does not have boundary with Bangladesh.
(c) The Namesake (2004) is the first novel by Jhumpa
Lahiri.
16. (a) Total fixed costs are constant, so the average fixed
cost curve diminishes with the output. Thus, the
average fixed cost curve is a rectangular hyperbola.
7.

asy
(c) The Constitution of India was drafted by the
constituent assembly and it was set up under the
cabinet Mission plan on 16 May 1946. The members
17. (d) Malaria is a mosquito borne disease of humans and
other animals caused by Plasmodium protozoan.
of the constituent assembly were elected by the
En
Provincial assemblies by method of single transferable
Severe disease is largely caused by Plasmodium
falciparum whereas mild forms are due to P vivax, P
vote system of proportional representations.Members
of the committee: Sardar Vallabhbhai Patel, K. M.
Munshi,Acharya J.B kriplani . Lok Nayak Jai Prakash gin
18. (b)
oval and P malariae.
The focal length of a convex lens is shorter for blue
light than for red.

8.
was not the member of the constituent assembly.
(c) Greenhouse gases catch the sun's radiation on its way
19. (c)
eer
Ram Prasad Bismil was the famous freedom fighter
who was involved in the historic Kakori train robbery .
back into space and reflect some of that warmth back
to Earth, increasing temperatures. Carbon dioxide is
known as greenhouse gas because of their ability to ing
He was born in 1897 at Shahjahanpur,Uttar Pradesh.
On 9th August, 1925, Ram Prasad Bismil along with
his fellow followers looted the money of the British
9.
trap and reflect the sun's radiation back to Earth.
(b) A laser is a device that emits coherent light through a .ne
government from the train while it was passing through
Kakori, Lucknow. Except Chandrashekhar Azad, all
10.
process called stimulated emission.
(c) Cowper’s gland is related to reproductive system.
Cowper’s gland is the bulbourethal gland found in
human males. They are found in pair and secrete
t
other members of the group were arrested. Ram Prasad
Bismil along with others was given capital punishment.
This great freedom fighter of India was executed on
19th December, 1927.
viscous secretion called pre ejaculate that helps in
coitus. 20. (c) Himadri Station is India's first Arctic research station
11. (a) Mist is a thin fog resulting from condensation in the located at Spitsbergen, Svalbard, Norway. It was
air near to the earth's surface. inaugurated on the 1st of July, 2008 by the Minister of
12. (a) Dyarchy was a system of double government Earth Sciences.
introduced by British India. 21. (b) 22. (a) 23. (a) 24. (c) 25. (a)

Downloded From : www.EasyEngineering.net


Downloded From : www.EasyEngineering.net

Practice Set-5 PS-13

PRACTICE SET - 5
1. When had Muslim league passed the resolution "Divide 10. According to the Constitution of India, the Right to
and Quit" movement ? Property is a –
(a) 1945 (b) 1943 (a) Fundamental Right
(c) 1944 (d) None of these (b) Directive Principle
2. What is the ratio of money held by the public in currency to (c) Legal Right
that they held in deposit ? (d) Social Right
(a) The currency deposit ratio 11. Babar declared himself as an emperor first at –
(b) The reserve deposit ratio (a) Samarqand
(c) Cash reserve ratio

3. ww
(d) Cash deposit ratio
The chemical behavior of an atom depends upon –
(a) the number of Neutrons in the nucleus
(b) Farghana
(c) Kabul
(d) Panipat

w.E
(b) the number of Nucleons in the nucleus
(c) the number of Protons in its nucleus
(d) the number of Electrons orbiting around the nucleus
12. How many times has financial emergency been declared in
India, so far?
(a) Five times (b) Four times
4.
asy
88th amendement of the Indian Constitution is related to –
(a) The demarcation of new boundaries between states
(b) The Constitution of the National Judicial Commission
(c) Once (d) Never
13. Economy is in the "Liquidity Trap" when –
(a) Rate of interest on bonds is minimum

En
(c) Empowering the Centre to levy and appropriate Service
tax
(b) Rate of interest on bonds is maximum
(c) Transaction demand for money is maximum
(d) Readjustment of electroal constituencies on the basis
of the population census 2001
gin(d) None of the above
14. Who is the author of "The Unseen Indira Gandhi"?
(a) K.P. Mathur
5. The joint sitting of both Houses of Indian Parliament is held
in connection with –
(a) Constitutional amendment bill eer
(b) Bilal Siddique
(c) Anurag Mathur
(b) Ordinary bill
(c) Money bill ing
(d) N.R. Narayana Murthy
15. What is 'biomagnification'?

6.
(d) Election of the Vice – President of India
Many Fungi belonging to the genera Microporum .ne
(a) blowing up of environmental issues by man
(b) growth of organisms due to food consumption
(c) reduction of dissolved O2 caused by microbial

7.
Trichophyton and Epidermophyton are responsible for –
(a) Filarial
(c) Ringworms
(b) Cancer
(d) AIDS
A boat will submerge when it displaces water equal to its
organisms
t
(d) increase in the concentration of nondegradable
pollutants as they pass through food chain
16. Subhash Chandra Bose formed the government for
own – independent India in Singapore, on –
(a) volume (b) weight (a) 22nd September, 1943
(c) surface area (d) density (b) 20th October, 1943
8. Which organ of Human body is affected by Alzheimer (c) 21st October, 1943
(d) 22nd October, 1943
disease ?
17. 'Laffer Curve" shows the relationship between –
(a) Brain
(a) Government Revenue and Government Expenditure.
(b) Bone Marrow (b) Tax Rates and Tax Revenue.
(c) Lung (c) Direct Taxes and GDP.
(d) Intestine (d) None of the above
9. What is the chemical name of vitamin E ? 18. 'Cerebral palsy' is a brain disorder found generally in –
(a) Calciferol (a) Old people
(b) Tocopherol (b) Drug addicts
(c) Riboflavin (c) Small children
(d) Phylloquinone (d) Only in ladies

Downloded From : www.EasyEngineering.net


Downloded From : www.EasyEngineering.net

PS-14 Practice Set-5


19. India is not a member of which of the following constituent 23. Which of the following statements are NOT correct?
organizations of the World Bank Group? (a) Prithvi-II is a Surface-to-Surface Missile
(a) International Centre for Settlement of Investment (b) Prithvi-IIcan carry a 500 kg nuclear warhead
Disputes (ICSID)
(c) Prithvi-IIhas a range of 350 KM
(b) International Development Association (IDA)
(c) International Finance Corporation (IFC) (d) Akash is the naval variant of the Prithvi missile
(d) Multilateral Investment Guarantee Agency (MIGA) 24. The 2016 Copa America Football tournament has been won
20. The mirror used in search light is – by which of the following countries ?
(a) Concave Mirror (b) Convex Mirror (a) Argentina
(c) Plane Mirror (d) None of these (b) Colombia
21. A.T.F. is related to – (c) Chile
(a) Civil Aviation (b) Railways (d) Peru
(c) Road transport (d) None of these
25. Which historical site has been declared as the SARRC
22. A hybrid computer is the one having the combined properties
cultural capital for 2016-17?
of
(a) Super and micro computers (a) Bamiyan
(b) Mini and micro computers (b) Maynamati

ww
(c) Analog and digital computers
(d) Super and mini computers
(c) Shilaidah
(d) Mahashangarh

w.E
asy
En
gin
eer
ing
.ne
t

Downloded From : www.EasyEngineering.net


Downloded From : www.EasyEngineering.net

Practice Set-5 PS-15

HINTS & EXPLANATIONS


1. (b) The communal question had become a baffling one as 14. (a) The book “The Unseen Indira Gandhi” has been
the Muslim League tightened its demand for Pakistan. authored by Dr. KP Mathur, who was the personal
Against the congress demand of “quit India”, the physician of the former Prime Minister Indira Gandhi
Muslim League’s new slogan was “Divide and quit”. for nearly 20 years till her assassination in 1984. The
On March 21, 1943, Muslim League observed as foreword of the book was written by her granddaughter
Pakistan Day. Priyanka Gandhi Vadra. The book provides some
2. (a) The currency deposit ratio shows the amount of interesting peeps into the responses of Mrs. Gandhi’s
currency that people hold as a proportion of aggregate to challenges both personal and political.
deposits. An increase in cash deposit ratio leads to a 15. (d) Biomagnification, also known as bioamplification or
decrease in money multiplier. An increase in deposit biological magnification, is the increasing concentration
rates will induce depositors to deposit more, thereby of a substance, such as a toxic chemical, in the tissues

ww
leading to a decrease in cash to Aggregate Deposit
ratio.
This will in turn lead to a rise in Money Multiplier. 16.
of organisms at successively higher levels in a food
chain.
(c) On 21st October, 1943, Subhas Chandra Bose
3.

4.
w.E
(d) The chemical behaviour of an atom depends upon the
number of Electrons orbiting around the nucleus.
(c) 5. (b) 6. (c)
proclaimed the formation of the Provisional
Government of Free India at the Cathay Cinema Hall.

7.
asy
(b) A boat will float when the weight of the water displaces
equals the weight of the boat and anything will float if
Two days later, he declared war on Britain and the
United States. With help from the Japanese, he
reorganised and rejuvenated the Azad Hind Fauj (also

En
it is shaped to displace its own weight of water before
it reaches the point where it will submerge. Floating of
called the Indian National Army). He lobbied
aggressively for funds in Malaya and other parts of
the boat works on the principle of buoyancy force which
is an upward force exerted by a liquid, gas or other
gin Southeast Asia and launched a recruitment drive for
the Azad Hind Fauj.

8.
fluid, that opposes the weight of an immersed object.
(a) Alzheimer’s disease affects the brain. The disease
causes degeneration of brain tissues and nerve cells.
17.
eer
(b) The Laffer curve, invented by Arthur Laffer, show s
the relationship between tax rates and tax revenue

9.
10.
(b) Chemical namee of Vitamin E is Tocopherols.
(c) The Indian Constitution does not recognize the
the Laffer Curve:
ing
collected by governments. The chart below show s

property right as a fundamental right. In the year 1977,


the 44th amendment eliminated the right to acquire, hold .ne
and dispose of property as a fundamental right.
However, in another part of the Constitution, Article
300 (a) was inserted to affirm that no person shall be
deprived of his property by the authority of law.
t
11. (d) Babur declared himself as the emperor at Panipat.
12. (d) Financial emergency in India has never been declared
so far.
13. (a) A liquidity trap is a situation, described in the 18. (c) Cerebral palsy (CP) is a group of permanent movement
Keynesian Economics, in which injections of cash into disorders that appear in early childhood. Signs and
the private banking system by a central bank fail to symptoms vary between people. Often, symptoms
dec rease interest rates and hence make monetary include poor coordination, stif f muscles, weak
policy ineffective. A liquidity trap is caused when muscles, and tremors. There may be problems with
people hoard cash because they expect an adverse sensation, vision, hearing, swallowing, and speaking.
event s uch as defl ati on, insufficient aggregate Often babies with cerebral palsy do not roll over, sit,
demand, or war. Common characteristics of a liquidity crawl, or walk as early as other children their age.
trap are interest rates that are close to zero and Difficulty with the a bi lity to think or reason and
fluctuations in the money supply that fail to translate seizures each occurs in about one third of people with
into fluctuations in price levels. CP.

Downloded From : www.EasyEngineering.net


Downloded From : www.EasyEngineering.net

PS-16 Practice Set-5


19. (a) The World Bank Group consists of – India is a member of four of the five constituents of
- Internation al Bank for Reconstr uction and the World Bank Group viz., International Bank for
Development (IBRD), established in 1945, which Reconstruction and Developmen t (IBRD),
provides debt financing on the basis of sovereign International Development Association (IDA),
guarantees; - International Finance Corporation (IFC), International Finance Corporation (IFC) and
established in 1956, provides various forms of Multilateral Investment Guarantee Agency (MIGA).
financing without sovereign guarantees, primarily to India is not a member of ICSID (International Centre
the private sector; for Settlement of Investment Disputes).
- International Development Association (IDA), 20. (d) A search light produces an intense parallel beam of
established in 1960, provides concessional financing light. This requires a reflector of large aperture. When
(interest-free loans or grants), usually with sovereign a source is placed at the focus of a large concave
guarantees; mirror only the paraxial rays (not the marginal rays)are
- International Centre for Settlement of Investment reflected as parallel beam, but when a source is placed
Disputes (CSID), established in 1965, which works at the focus of parabolic mirror all the rays are reflected
with governments to reduce investment risk; as an intense parallel beam.
- Multilateral Investment Guarantee Agency (MIGA), 21. (a) ATF is Aviation Turbine Fuel related to Civil Aviation.

ww
established in 1988, which provides insurance against
certain types of risk, including political risk, primarily
to the private sector.
22.
23.
24.
(c)
(d) Dhanush is the naval variant of the Prithvi missile.
(c) 25. (d)

w.E
asy
En
gin
eer
ing
.ne
t

Downloded From : www.EasyEngineering.net


Downloded From : www.EasyEngineering.net

Current Affairs & GK Update

ww
w.E
asy
En
gin
eer
ing
.ne
t

Downloded From : www.EasyEngineering.net


Downloded From : www.EasyEngineering.net

GK-2 Current Affairs & GK Update

ww
w.E
asy
En
gin
eer
ing
.ne
t

Downloded From : www.EasyEngineering.net


Downloded From : www.EasyEngineering.net

Current Affairs & GK Update 3


GK-

ww
w.E
asy
En
gin
eer
ing
.ne
t

Downloded From : www.EasyEngineering.net


Downloded From : www.EasyEngineering.net

GK-4 Current Affairs & GK Update

ww
w.E
asy
En
gin
eer
ing
.ne
t

Downloded From : www.EasyEngineering.net


Downloded From : www.EasyEngineering.net

Current Affairs & GK Update GK-5


BOOKS & AUTHORS
The Legend of Lakshmi Prasad– Twinkle Khanna The Unseen Indira Gandhi–Dr KP Mathur
Driven: The Virat Kohli Story–Vijay Lokapally A Life in Diplomacy–Maharajakrishna Rasgotra
Democrats and Dissenters–Ramachandra Guha Blood on my Hands: Confessions of Staged Encounters–
One Indian girl–Chetan Bhagat Kishalay Bhattacharjee
Six Machine (I Don’t Like Cricket I Love It)–Chris Gayle The Making of India: The Untold Story of British Enterprises–
His Bloody Project–Graeme Macrae Brunet Kartar Lalvani
AB The autobiography–AB De Villiers A State in Denial–BG Verghese
Citizen and Society–Hamid Ansari The Kiss of Life–Emraan Hashmi
Anything But Khamosh: The Shatrughan Sinha Biography–
The Ocean of Churn: How the Indian Ocean Shaped Human
Bharathi S Pradhan
History–Sanjeev Sanyal
Gandhi: An Illustrated Biography–Pramod Kapoor
R D Burmania: Panchamemoirs–Chaitanya Padukone
Fixed! Cash and Corruption in Cricket–Journalist Shantanu
India Rising: Fresh Hope, New Fears–Ravi Velloor Guha Ray
The Great Derangement: Climate Change and the “Who was Shivaji?”–Govind Pansare
Unthinkable–Amitav Ghosh A Kingdom for His Love–Vani Mahesh, Shinie Antony
Courage & Commitment–Margaret Alva Nathuram Godse: The Story of an Assassin–Anup Ashok

Haqqani ww
India vs Pakistan: Why Can’t we just be Friends?–Husain

Chaos and Caliphate: Jihadis and the West in the Struggle for
Sardesai
The Turbulent Years, Volume II–President Pranab Mukherjee
The Z Factor–Subhash Chandra

w.E
the Middle East–Patrick Cockburn
A Call to Mercy: Hearts to Love, Hands to Serve–Mother
Teresa
Jawaharlal Nehru and The Indian Polity in Perspective–
Hamid Ansari
Maru Bharat Saru Bharat–Jain Acharya Maharaj

asy
IMPORTANT APPOINTMENTS 2016

Jan 1,
JANUARY
En
Amitabh Kant appointed CEO of NITI (National
Mar 15,
2016
Htin Kyaw elected as first civilian President of
Myanmar.
2016
Jan 4,
Institution for Transforming India) Aayog.
RK Mathur sworn in as 8th Chief Information gin
Mar 23,
2016
Justice Permod Kohli appointed as Chairman of
Central Administrative Tribunal.
2016
Jan 25,
Commissioner (CIC).
K Durga Prasad appointed as DG of CRPF April 7, eer APRIL
CP Gurnani appointed as Chairman of
2016 (Central Reserve Police Force).
FEBRUARY
2016
ing
NASSCOM (National Association of Software
and Services Companies).
MAY
Feb 2,
2016
Archana Ramasundram becomes 1st woman DG
of Sashastra Seema Bal. May 3,
2016 .ne
Bhupendra Kainthola takes charge as FTII (Film
and Television Institute of India) Director.
Feb 3,
2016
Feb 13,
2016
DJ Pandian appointed AIIB (Asian Infrastructure
Investment Bank) Vice-President.
Ashok Chawla appointed as new Chairman of
TERI (The Energy and Resources Institute).
May 31,
2016
JUNE
t
Admiral Sunil Lanba takes charge as Navy chief.

June 3, Navin Agarwal appointed as DG of NADA


Feb 23, KN Vyas appointed as Director of Bhabha Atomic
2016 (National Anti Doping Agency).
2016 Research Centre.
June 15, Peter Thomson elected as President of 71st session
Feb 25, 2016 of United Nations General Assembly.
Rajendra Singh appointed as DG of Coast Guard.
2016
June 15, Hollywood actress Anne Hathaway appointed as
Feb 29, Former CJI HL Dattu takes over as Chairman of 2016 UN Women Goodwill Ambassador.
2016 NHRC (National Human Rights Commission).
June 22, Ken Miyauchi appointed as President and COO of
MARCH 2016 SoftBank Group.
Mar 2, Vice Admiral Atul Kumar Jain appointed as Chief June 23, BCCI appoints Anil Kumble as the head coach of
2016 of Staff of Eastern Naval Command. 2016 Indian Cricket Team.
Mar 11, Justice Balbir Singh Chauhan appointed as June 27, Sujoy Bose appointed as first CEO of NIIF
2016 Chairman of 21st Law Commission of India. 2016 (National Investment and Infrastructure Fund).
Mar 12, Lt Gen N P S Hira appointed Deputy Chief of June 28, NS Vishwanathan appointed as deputy governor
2016 Army. 2016 of RBI.

Downloded From : www.EasyEngineering.net


Downloded From : www.EasyEngineering.net

GK-6 Current Affairs & GK Update


JULY OCTOBER
July 4, Sudarshan Sen appointed Executive Director of Oct 3, S Sivakumar appointed as full-time member of 21st
2016 RBI. 2016 Law Commission of India.
July 11, D Rajkumar appointed as CMD of BPCL (Bharat Oct 6, Sushil Chandra appointed CBDT Chairman.
2016 Petroleum Corporation Ltd). 2016
July 13, Theresa May appointed as PM of the United Oct 14, Portugal’s Antonio Guterres appointed UN
2016 Kingdom. 2016 Secretary-General.
July 14, Rakesh Kumar Chaturvedi appointed as Chairman Amandeep Singh Gill appointed as India’s
2016 of CBSE. Oct 18,
Ambassador to UN Conference on Disarmament,
2016
July 19, Guruprasad Mohapatra appointed as Chairman of Geneva.
2016 Airports Authority of India. Oct 21, Vice Admiral SV Bhokare assumes office as the
July 21, Ajay Bhushan Pandey appointed as CEO of 2016 Commandant of Indian Naval Academy.
2016 UIDAI (Unique Identification Authority of India). Oct 27, Karnal Singh appointed as Director of Enforcement
AUGUST 2016 Directorate.
Aug 1, Rani Singh Nair appointed as Chairman of CBDT NOVEMBER
2016 (Central Board of Direct Taxes). Nov. 7, M Rajeshwar Rao new RBI Executive Director.

Sep 4,
2016
ww SEPTEMBER
Urjit Patel assumes charge as 24th Governor of
Reserve Bank of India.
2016
Nov. 24,
2016
Yudhvir Singh Malik appointed as new Chairman
of NHA.
Sep 19,
2016
Sep 28,
w.E
Alka Sirohi appointed UPSC chairman.

World Bank reappoints Jim Yong Kim for second


Dec 5,
2016
DECEMBER
Senior AIADMK leader O Panneerselvam
appointed as the new AIADMK chief and the new
2016 term as President.
asy OBITUARY IN 2016
Tamil Nadu Chief Minister.

Jan 7, 2016
Mufti Mohammad Sayeed Former-Jammu En Jun 22, 2016 Amjad Sabri–a famed Pakistani Qawwal.

Jan 10, 2016


Jan 19, 2016
and Kashmir Chief Minister.
David Bowie–Legendary artist.
Ravindra Kalia–Noted Hindi writer.
gin
Jun 29, 2016
Jul 1, 2016
KG Subramanyan–Renowned painter.
Ramchandra Chintaman Dhere–Eminent
Marathi literary scholar.
Jan 21, 2016 Mrinalini Sarabhai–Legendary dancer.
Kalpana Ranjani–Noted Malayalam film
Jul 20, 2016
Jul 28, 2016 eerMohammed Shahid – Indian Hockey legend.
Mahasweta Devi–Writer and social activist.
Jan 25, 2016
Feb 2, 2016
actress.
S S Tarapore–Former RBI Deputy Governor.
Jul 28, 2016
maestro. ing
Pandit Lacchu Maharaj–Veteran tabla

Feb 2, 2016
Feb 9, 2016
Intizar Hussain–Veteran Urdu writer.
Sushil Koirala–Former Nepal Prime
Minister.
Aug 14, 2016
Aug 8, 2016
.ne
Na Muthukumar–Tamil Lyricist.
Mahim Bora–Eminent Assamese litterateur.
Kalikho Pul–Former Chief Minister of

Feb 18, 2016


Feb 19, 2016
Ustad Abdul Rashid Khan–Veteran singer of
Gwalior Gharana.
Bhubaneswari Mishra–Renowned singer.
Aug 9, 2016

Aug 22, 2016


Sep 2, 2016
Arunachal Pradesh.

serving president.
t
Sellapan Ramanathan–Singapore’s longest

Islam Karimov–President of Uzbekistan.


Purno Agitok Sangma–Former Lok Sabha
Mar 4, 2016 Sep 5, 2016 Lindsay Tuckett–World’s oldest cricketer.
Speaker.
Sep 24, 2016 Reoti Saran Sharma–Hindi and Urdu writer.
Mar 5, 2016 Ray Tomlinson–inventor of E-mail.
Syed Shamsul Haq–Renowned Bangla
Ustad Ali Ahmad Hussain–Famous Shehnai Sep 27, 2016
Mar 16, 2016 writer.
exponent. Shimon Peres–Former Israeli Prime
Mar 26, 2016 Jim Harrison–Renowned American author. Sep 28, 2016
Minister.
Barbara Turner–Hollywood actress and Oct 13, 2016 Bhumibol Adulyadej–King of Thailand.
Apr 5, 2016
screenwriter. M. Balamuralikrishna–Indian Carnatic
Apr 14, 2016 Satyanand Munjal–Hero Group co-founder. Nov 22, 2016
musician and composer
Apr 21, 2016 Prince Rogers Nelson–Music legend. Fidel Castro - Cuban Politician, Prime
Apr 27, 2016 V T Thomas–Kerala cartoonist. Nov 25, 2016
Minister.
May 11, 2016 Tony Cozier–Renowned commentator. Dilip Padgaonkar–Journalist, Editor in
Nov 25, 2016
May 27, 2016 RG Jadhav – Noted Marathi writer. Chief.
Jun 3, 2016 Muhammad Ali–American Boxer. Jayalalithaa Jayaraman- Indian actor and
Dec 5, 2016
Jun 12, 2016 Achyut Lahkar–Noted Playwright and Actor. Chief Minister of Tamil Nadu.

Downloded From : www.EasyEngineering.net


Downloded From : www.EasyEngineering.net

Current Affairs & GK Update GK-7


BILLS AND ACTS 2016
Maternity Benefits (Amendment) Bill, 2016 • The fine for driving without license goes up ten times to `
• Rajya Sabha has passed this bill on August 11, 2016 which 5,000 while penalty for drunken driving increases fivefold to
aims to raise maternity leave for women from 12 to 26 weeks. ` 10,000.
• The Maternity Benefit Act, 1961, protects the employment • The significant provisions include increase in compensation
of women during the time of maternity and entitles them for Hit & Run cases from ` 25000 to ` 2 lakhs. It also has
of a full paid absence from work to take care for the child. provision for payment of compensation up to ` 10 lakh in road
The amendment bill seeks to increase maternity leave to 26 accidents fatalities.
weeks in all establishments, including private sector. • For offences by juveniles, the guardian/owner shall be deemed
• The act is applicable to all establishments employing to be responsible. They will have to pay a fine of ` 25,000
10 or more persons. The bill also provides 12 weeks leave apart from three-year imprisonment. The juvenile will be
for commissioning and adopting mothers and makes it tried under the Juvenile Justice Act and registration of his or
compulsory to provide crèche facility for establishment where her motor vehicle will be cancelled.
the number of workers is 50 and above. Adopting mother is • In the area of road safety, bill proposes to raise penalties to
the woman who legally adopts a child below 3 months of age. act as restraint against traffic violations. Stricter provisions
A commissioning mother is a biological mother who uses her are being proposed in respect of offences like juvenile
egg to create an embryo implanted in another woman. driving, drunken driving, driving without license, dangerous

ww
• There is already a provision of 26-week or 6-month
maternity leave for the government employees, most private
driving, over-speeding, overloading, etc. Stricter provisions
for helmets have been introduced along with provisions for
electronic detection of violations.

w.E
sector firms offer maximum three months of such leave. The
bill may also have an enabling provisions to allow working
mothers to exercise work-from-home option.
• To bring agreement of the registration and licensing process,
it is proposed to create National Register for Driving Licence
and National Register for Vehicle registration through

asy
The Constitution (122nd Amendment) (GST) Bill, 2014
• Rajya Sabha gave its approval on August 3, 2016 and Lok
“Vahan” & “Sarathi” platforms. This will assist uniformity
of the process across the country.

Amendment to turn the bill into law. The Lok Sabha had
En
Sabha on August 8, 2016 for the crucial 122nd Constitutional

already passed the key provisions of the bill on May 06,


Benami Transaction Bill 2016
• The Benami Transactions (Prohibition) Amendment Bill,
2015.
• The Bill amends the Constitution to introduce the goods and gin
2016 was passed by the Rajya Sabha. It was already passed
by Lok Sabha in the last week of July 2016.
services tax (GST).
• Parliament and state legislatures will have concurrent
powers to make laws on GST. Only the Centre may levy an
assets.
eer
• The bill has provision for confiscating ‘benami’ (proxy)

• The bill seeks to establish adjudicating authorities and an


integrated GST (IGST) on the interstate supply of goods and
services, and imports. ing
appellate tribunal to deal with benami transactions as also
specify penalty for entering into benami transactions.
• Alcohol for human consumption has been exempted from the
purview of GST. GST will apply to five petroleum products
at a later date.
tackle the problem of black money.
.ne
• The bill is part of measures planned by the Government to

• The GST Council will recommend rates of tax, period of levy


of additional tax, principles of supply, special provisions to
certain states, etc. The GST Council will consist of the Union
Finance Minister, Union Minister of State for Revenue, and
t
• A transaction is considered as benami (meaning nameless)
where a property is transferred to or is held by a person and
the consideration for such property has been provided or paid
by another person.
• People keeping ‘benami’ properties to evade taxes will get
state Finance Ministers.
up to seven years rigorous imprisonment and fine, as per a
• The Bill empowers the Centre to impose an additional tax of
new law aimed at checking black money in real estate sector.
up to 1%, on the inter-state supply of goods for two years or
more. This tax will accrue to states from where the supply Lokpal and Lokayukta (Amendment) Bill, 2016
originates. • The Lokpal and Lokayukta (Amendment) Bill, 2016 was
• Parliament may, by law, provide compensation to states for passed by the Lok Sabha on July 27, 2016.
any loss of revenue from the introduction of GST, up to a five • The bill seeks to amend the Lokpal and Lokayukta Act, 2013
year period. which provides Lokpal (at Centre) and Lokayukta (at States)
• India’s 29 states currently levy their own sales taxes, statutory backing to inquire into allegations of corruption
dividing the Indian market. GST aims to provide a uniform against certain public functionaries and for related matters.
tax structure across the country.
Key Provisions
Motor Vehicle (Amendment) Bill, 2016 • The bill enables the leader of the single largest opposition
• The Union Cabinet chaired by the Prime Minister Narendra party in the Lok Sabha in the absence of a recognised Leader
Modi has given its approval on August 03, 2016 for Motor of Opposition to be a member of the selection committee that
Vehicle (Amendment) Bill 2016. would select the ombudsman.

Downloded From : www.EasyEngineering.net


Downloded From : www.EasyEngineering.net

GK- 8 Current Affairs & GK Update


• It amends section 44 of parent Act that deals with provision • It will lead to greater inclusiveness and will make the
of furnishing of details of assets and liabilities of public transgender persons productive members of the society.
servants within 30 days of joining the government service. • The Bill will make all the stakeholders responsive and
• The amendment has removed the period of 30 days. Now accountable for upholding the principles underlying the Bill.
the public servants will make declaration of their assets • It will bring greater accountability on the part of the Union
and liabilities in the form and manner as prescribed by Government and State Governments/UT administrations for
government. issues concerning Transgender persons.
• It gives extension of the time given to public servants • Background Transgender community is among one of the
and trustees and board members of Non-Governmental most marginalized communities in the country because they
Organisations (NGOs) to declare their assets and those of don’t fit into the stereotypical categories of gender of ‘men’
their spouses. or ‘women’.
• In this case public servants and trustees and board members • They face problems ranging from social exclusion, ostracism
of NGOs must be receiving government funds of more than and discrimination, as well as verbal, physical and sexual
` 1 crore or foreign funding of more than ` 10 lakh.
abuse.
• They also lack of access to education and employment
Child Labour (Prohibition and Regulation) opportunities, ending up into organised groups, forced
Amendment Bill, 2016 begging or demanding money.
• Lok Sabha has passed the Child Labour (Prohibition and

ww
Regulation) Amendment Bill, 2016, despite opposition by
several MPs.
Insolvency and Bankruptcy Code Bill, 2016
• Rajya Sabha passed the major economic reform Bill moved
by the Government i.e. ‘Insolvency and Bankruptcy Code,

w.E
• The Bill, passed by the Rajya Sabha on July 19, 2016.
• The Bill seeks to improve the Child Labour (Prohibition and
Regulation) Act, 1986, which prohibits the employment of
2016’ on 11th May 2016.
• The Lok Sabha had earlier passed the Bill on 5th May, 2016.
• The objective of the new law is to promote entrepreneurship,

asy
children below 14 years in 83 hazardous occupations and
processes. The amendment extends this ban on employment
of children under 14 across all sectors, prohibits the
availability of credit, and balance the interests of all
stakeholders by consolidating and amending the laws
relating to reorganization and insolvency resolution of
employment of adolescents aged 14-18 years in risky
En
occupations and introduces more severe jail term and fines
corporate persons, partnership firms and individuals in a time
bound manner and for maximization of value of assets of
for offenders: a jail term of six months to two years and a fine
of ` 20,000 to ` 50,000.
• The opposition by members was mainly to the fact that
gin
such persons and matters connected therewith or incidental
thereto.

children under 14 years will be allowed to work in family


businesses, outside of school hours and during holidays, and eer
• The essential idea of the new law is that when a firm defaults
on its debt, control shifts from the shareholders/promoters
to a Committee of Creditors, who have 180 days in which to
in entertainment and sports. MPs objected to the definition of
family which includes not only parents and siblings but also ing
evaluate proposals from various players about resuscitating
the company or taking it into liquidation.
the siblings of either parent.
• The Bill brings down the list of hazardous
occupations from the earlier 83 to just three: mining, .ne
• The Insolvency and Bankruptcy Code is a comprehensive
and systemic reform, which will give a quantum leap to the
functioning of the credit market.
inflammable substances, and hazardous processes under the
Factories Act. Read together, with the family enterprises
clause, critics say, children can be made to work in carpet,
zari and bidi units, mica or diamond cutting, scavenging,
Mines and Minerals Amendment Bill, 2016
t
• Parliament passed the Mines and Minerals (Development
and Regulation) Amendment Bill 2016. The Bill amends
the Mines and Minerals (Development and Regulation) Act,
brick kilns, slaughterhouses, handling e-waste or as domestic
1957.
help. Several MPs objected to the clause that also gives the
• The Bill was passed in the Lok Sabha on 16th March 2016
Centre the right to add to the list of non-hazardous work.
and in the Rajya Sabha on 2nd May 2016.
Transgender Persons (Protection of Rights) Bill, 2016 • The Bill allows for the transfer of mining leases which have
• The Transgender Persons (Protection of Rights) Bill, 2016 been granted through procedures other than auction, and
was approved by the Union Cabinet on July 21 to protect where the minerals are used for captive purpose.
transgenders from discrimination. Through this Bill the • The Bill adds a definition of leased area, as the area within
Union Government has evolved a holistic mechanism for which mining operations can be undertaken. This will also
the social, economic and educational empowerment of include the non-mineralised area required for the activities
marginalized transgenders community. defined under mine in the Mines Act, 1952.
Key Facts • The Mines and Minerals (Development and Regulation) Act,
• The Bill will benefit a large number of transgender persons, 1957 regulates the mining sector in India and specifies the
mitigate the stigma, discrimination and abuse against them requirement for obtaining and granting leases for mining
and also bring them into the mainstream of society. operations.

Downloded From : www.EasyEngineering.net


Downloded From : www.EasyEngineering.net

Current Affairs & GK Update 9 GK-

Real Estate (Regulation and Development) Act, 2016 • It provides to give 10% reservation to these castes for class
III and class IV posts and 6% in class 1 and class 2 posts.
• Rajya Sabha passed the Real Estate (Regulation and
Development) Bill, 2016 which aims to guard the interests • These castes will be given 10% reservation for admission
of numerous aspiring house buyers alongside enhancing in educational institutions. State Legislative Assembly also
the credibility of construction industry by promoting passed Haryana Backward Classes Commission Bill, 2016
transparency, accountability and efficiency in execution of for institution of a permanent Backward Classes Commission.
projects. Aadhaar Act, 2016
• The Bill seeks to set up an effective regulatory mechanism
for orderly growth of the sector which is the second largest The Aadhaar (Targeted Delivery of Financial and other
seasonal employer after agriculture. Subsidies, benefits and services) Act, 2016 has been notified
• It provides that 70% of sale proceeds will have to be kept by the Union Government on March 28. The Act will provide
aside by the developer in an escrow account which is a statutory backing for transfer of subsidies and benefits to eligible
financial instrument held by a third party on behalf of the people having Aadhaar (UID) number.
other two parties in a transaction. Key facts
• The Bill will set up state-level real estate regulatory authorities • The act seeks to provide efficient, transparent and targeted
and appellate tribunals to monitor transactions relating to delivery of subsidies, benefits and services to individuals
both residential and commercial projects and their timely residing in India by assigning them unique identity numbers

ww
completion and handover. It will also look after complaints
at both appellate tribunals and regulatory authorities within
60 days ensuring timely resolution of disputes.
(UID) or Aadhaar number.
• It will be used for all benefit that will be linked to consolidated
fund of India or the expenditure incurred from it.

w.E
• The Real Estate Bill enables the people meet their genuine
aspirations of owning a house including those of urban poor
by giving a fillip to affordable housing initiative under which
• Both Central and state governments can use Aadhaar for
disbursal for benefits and subsidies. However the Aadhaar
number cannot be used as a proof of citizenship or domicile.

asy
the Government intends to enable construction of 2 crore by
the year 2022 under Prime Minister’s Awas Yojana (Urban).
Industries (Development and Regulation)
• The Act gives statutory for establishment of the Unique
Identification Authority of India consisting of a Chairperson
(part time or full time) and two Members (part time).
Amendment Bill, 2015
En • Those individuals not having Aadhaar number shall be
offered alternate and viable means of identification for
• The Industries (Development and Regulation) Amendment
Bill, 2015 was passed by the Parliament on April 29,
2016. The Bill amends the Industries (Development and gin
delivery of the subsidies, benefits or service.
Scheduled Tribes (Prevention of Atrocities)
Regulation) Act, 1951. It was first was passed in Lok Sabha
in December 2015 and later in Rajya Sabha in April 2016. eer
Amendment Act, 2015
• The amended law for prevention of atrocities against the
Now it will go for Presidential assent as per Article 111 of the
Indian Constitution.
Key facts ing
Scheduled Castes and the Scheduled Tribes was enforced
from January 26, 2016 after the Central Government’s
• The bill seeks to bring industries engaged in the manufacture
of potable alcohol under the exclusive control of States in all
approval.
.ne
• The SC/ST Bill was unanimously passed by the Rajya Sabha
respects.
• The bill amends 1st schedule of the parent Act to bring
required changes for regulating industries manufacturing
potable alcohol. However, Union Government will continue
on December 21, 2015 without a debate, and had been

the bill on 4thAugust, 2015.


t
awaiting a formal notification. The Lok Sabha had passed

• After the enforcement, there will be stricter prosecution


to be responsible for formulating policy and regulating for new offences of atrocities, such as tonsuring of head,
foreign collaboration for all products of fermentation moustache, or similar acts, which are derogatory to the
industries, including industrial and potable alcohol. dignity of members of the SC/ST community, garlanding
Jat Reservation Bill with chappals, denying access to irrigation facilities or
forest rights, dispose or carry human or animal carcasses,
The Haryana Backward Classes (Reservation in services
or to dig graves, using or permitting manual scavenging,
and admission in educational institutions) Bill, 2016 was
dedicating an SC/ST woman as devadasi, abusing in caste
unanimously passed by the Haryana Legislative Assembly in
name, perpetrating witchcraft atrocities, imposing social or
May 2016.
economic boycott, preventing SC/ST candidates from filing
Key Provisions of the bill
of nomination to contest elections, hurting an SC/ST woman
• It seeks to provide reservation in services and admission in
by removing her garments, forcing a member of these
educational institutions to persons belonging to backward
classes including Jats and five other castes. communities to leave house, village or residence, defiling
• The backward classes have been divided into three blocks as objects sacred to members of SC/ST, touching or using
Block A, Block B and Block C. Jats, Jat Sikhs, Ror, Bishnoi, words, acts or gestures of a sexual nature against members of
Tyagi and Mulla/Muslim Jat have been included in the Block C. the community.

Downloded From : www.EasyEngineering.net


Downloded From : www.EasyEngineering.net

GK-10 Current Affairs & GK Update


The Juvenile Justice (Care and Protection of • The 228th report of the Law Commission of India has also
recommended for prohibiting commercial surrogacy and
Children) Act, 2015 allowing ethical altruistic surrogacy to the needy Indian
• The Juvenile Justice (Care and Protection of Children) Act, citizens by enacting a suitable legislation.
2015 has come into force from January 15, 2016 and repeals the
Juvenile Justice (Care and Protection of Children) Act, 2000. HIV and AIDS (Prevention and Control) Bill, 2014
• The Bill was passed by Lok Sabha on 7th May, 2015 The Union Cabinet has given its approval to introduce
and Rajya Sabha on 22nd December, 2015 and received amendments to the HIV and AIDS (Prevention and Control)
Presidential assent on 31st December, 2015. Bill, 2014.
• Under Section 15, special provisions have been made to • Prevent and control the spread of HIV and AIDS. It prohibits
tackle child offenders committing heinous offences in the discrimination against persons with HIV and AIDS. No
age group of 16-18 years. person will be compelled to disclose his HIV status except
President assent to the Sugar Cess (Amendment) with his informed consent, and if required by a court order
establishments keeping records of information of PLHIV
Bill, 2015
must adopt data protection measures.
• President Pranab Mukherjee gave his assent to the Sugar • Obligations on establishments to safeguard rights of persons
Cess (Amendment) Bill, 2015 on 8th January, 2016. living with HIV arid create mechanisms for redressing
• This will raise the ceiling of the impost from ` 25 to ` 200 per complaints. Lists various grounds on which discrimination

ww
quintal, a measure that will help sugarcane growers and cash
starved mills.
• The Sugar Cess (Amendment) Bill, 2015 was introduced
against HIV-positive persons and those living with them is
prohibited.
• These include the denial, discontinuation, termination or

w.E
in the Lok Sabha by Mr. Ram Vilas Paswan, Minister
of Consumer Affairs, Food and Public Distribution on
December 11, 2015. The Bill proposes to amend the Sugar
Cess Act, 1982.
unfair treatment with regard to employment, educational
establishments, health care services, renting property etc.
Prohibits, requirement for HIV testing as a pre-requisite for

Surrogacy (Regulation) Bill, 2016


asy obtaining employment or accessing health care or education.
• Prohibits any individual from publishing information or
advocating feelings of hatred against HIV positive persons
• Union Cabinet chaired by the Prime Minister Narendra
Modi gave its approval for introduction of the “Surrogacy
(Regulation) Bill, 2016” on 24th August. En and PLHIV.
National Water Framework Bill, 2016
• The Bill will regulate surrogacy in India by establishing
National Surrogacy Board at the Central level and State gin
The Central Government has brought final draft of the Nationa l
Water Framework Bill, 2016 to provide uniform national legal
Surrogacy Boards and Appropriate Authorities in the State
and Union Territories. The legislation will ensure effective
regulation of surrogacy, prohibit commercial surrogacy and eer
framework to manage water in a better and efficient way. The
comprehensive draft Bill proposes model law for all states.
allow ethical surrogacy to the needy infertile couples.
• All infertile Indian married couple who want to avail ethical ing
However, water being a State subject under VII Schedule of
Constitution the law will be not binding on States for adoption.
Key Features of Bill
surrogacy will be benefited. Further the rights of surrogate
mother and children born out of surrogacy will be protected.
The Bill shall apply to whole of India, except the state of .ne
• Every person has a right to sufficient quantity of safe water
for life within easy reach of the household regardless of his/
Jammu and Kashmir.
• The major benefits of the Act would be that it will regulate the
surrogacy services in the country. While commercial surrogacy
will be prohibited including sale and purchase of human
her socio-economic factors.
t
• All basin states have equitable rights over the use of river
water provided such use does not violate the right to water
for life of any person in the river basin. States must recognise
embryo and gametes, ethical surrogacy to the needy infertile the principle that the rivers are public trustees and not owned
couples will be allowed on fulfillment of certain conditions by the basin-States.
and for specific purposes. As such, it will control the unethical • All the basin States are equal in rights and status, and there
practices in surrogacy, prevent commercialization of surrogacy is no hierarchy of rights among them. Here equality of
and will prohibit potential exploitation of surrogate mothers rights means not equal but equitable shares in river waters.
and children born through surrogacy. Managing water at river basin-level and right measurement
• No permanent structure is proposed to be created in the Draft of State’s contribution to river system in order to resolve
Bill. Neither there are proposals for creating new posts. The conflicts.
proposed legislation, while covering an important area is • Establishing River Basin Authority (RBA) for each inter-
framed in such a manner that it ensures effective regulation State basin to ensure optimum and sustainable development
but does not add much vertically to the current regulatory of rivers and valleys. Establishing institutional arrangements
structure already in place at the Central as well as states. to deal with inter-state water disputes in order to “obviate”
Accordingly, there will not be any financial implications disputes through negotiations, mediation or conciliation.
except for the meetings of the National and State surrogacy • Proposes other mechanisms such as National water quality
Boards and Appropriate Authorities which will be met out of and footprint standards, Integrated river basin development
the regular budget of Central and State governments. and management plan and graded pricing system.

Downloded From : www.EasyEngineering.net


Downloded From : www.EasyEngineering.net

Current Affairs & GK Update 11GK-

Merchant Shipping Bill, 2016 • It also provides for survey, inspection and certification of
vessels to enable simplified regime for convenience of Indian
Union Cabinet approved the introduction of Merchant Shipping
shipping industry.
Bill, 2016 in the Parliament. Simultaneously, the Cabinet also
approved repealing of the Merchant Shipping Act, 1958 and the Taxation Laws (Second Amendment) Bill, 2016
Coasting Vessels Act, 1838. The Union Government has introduced the Taxation Laws
The Merchant Shipping Act, 1958 has been amended 17 times (Second Amendment) Bill, 2016 in the Parliament. The bill
between 1966 and 2014 resulting in an increase in the number of amends existing tax laws to impose a higher rate of tax and
sections to more than 560 sections. However, these provisions penalty in respect of undisclosed incomes. It proposes, Taxation
have been shortened to 280 sections in the Bill. and Investment Regime for Pradhan Mantri Garib Kalyan
Salient Features Yojana, 2016 (PMGKY), an anti-poverty scheme. It seeks to
• It allows substantially-owned vessels and vessels on Bare attract more people to disclose their unaccounted cash and also
Boat-cum-Demise (BBCD) to be registered as Indian flag to put in a framework in place to use that for the welfare of
vessels. the people especially in the rural areas. It also ensures that tax
defaulting assessees and black money holders are subjected to
• It recognises Indian controlled tonnage as a separate category.
tax at a higher rate and stringent penalty provision
• It provides for issuing the licences to Indian flag vessels
Key features of Bill
for coastal operation and for port clearance by the Customs
• Mandatory for black money declarants to deposit 25% of
authorities.

ww
• It calls for making separate rules for coastal vessels to
develop and promote coastal shipping. •
amount disclosed in Pradhan Mantri Garibi Kalyan Yojana
(PMGKY) 2016 for a 4 year lock-in period without interest.
Those who choose to declare their ill-gotten wealth stashed

w.E
• It introduces welfare measures for seafarers by providing
them with the wages till they are released from hostage
captivity of pirates and reach back home safely.
till now in banned `1,000 and Rs. 500 currency notes under
the PMGKY scheme will have to pay a tax at the rate of
30% of the undisclosed income.

asy
• It directs owners of vessels to take insurance of crew engaged
on vessels including fishing, sailing, etc. This applies for the
vessels whose net tonnage is less than 15 tons.
• Additionally, 10% penalty will be levied on the undisclosed
income and PMGK Cess (Surcharge) at the rate of 33% of
tax (33% of 30%).

En
• It says that the requirement of signing of articles of agreement
by the crew will no longer be necessary.
• Further, the declarants will have to deposit 25% of the
undisclosed income in a scheme to be notified by the
• It calls for registration of few residuary category of vessels
not covered under any statute and makes provisions for

gin
government in consultation with the Reserve Bank of India
(RBI).
The money from PMGK will be used for projects in
security-related aspects.
• It adds provisions relating to seven different conventions-
Intervention Convention 1969, Search and Rescue
eer
irrigation, infrastructure, primary education, primary
health, housing, toilets and livelihood so that there is justice
Convention 1979, Protocol for Prevention of Pollution from
Ships Annex VI to Marine Pollution Convention, Convention

and equality.
ing
For those who continue to hold onto undisclosed cash and
are caught they will be levied with flat 60% tax plus a
for Control and Management of Ships Ballast Water and
Sediments 2004, Nairobi Wreck Removal Convention 2007, .ne
surcharge of 25% of tax (15%), which will amount a levy
of 75%.
Salvage Convention 1989 and International Convention for
Bunker Oil Pollution Damage, 2001.

POLICIES AND SCHEMES 2016



t
Besides, if the assessing officer can charge a 10% penalty in
addition to the 75% tax.

Pashudhan Bima Yojana • Under it, the insurance companies would give compensation
Haryana Government launched the Pashudhan Bima Yojana in case of death of animal (cattle). The scheme is free of cost
in Jhajjar on 29th July. The Yojna was launched by the state’s for cattle breeders from the scheduled castes.
• The state Animal Husbandry and Dairying Department has
Animal Husbandry and Dairying Minister, O P Dhankar. The
set a target to cover one lakh cattle under Pashudhan Bima
Yojana was started in accordance to the recommendations
Yojana.
made in Swaminathan Report. Animal Husbandry and Dairying
Department has set a target to cover one lakh cattle under Sewage & Waste Water Policy in Rajasthan
Pashudhan Bima Yojana. Rajasthan has become the first state to give go-ahead to the
Key Facts sewage and waste water policy, in the country. The state cabinet
• Pashudhan Bima Yojana will provide insurance cover to meeting chaired by the Chief Minister Vasundhara Raje in Jaipur
cattle breeders in the state at different premium rates for approved the first-of-its-kind policy.
different animals. Key Facts
• Insurance cover will be provided at a premium of ` 100 for • Under this policy, all district headquarters, heritage cities, and
cows, buffaloes, bulls, camels and at ` 25 for sheep, goat and cities in the state which have more than one lakh population
pig for a period of three years. will be included.

Downloded From : www.EasyEngineering.net


Downloded From : www.EasyEngineering.net

12
GK- Current Affairs & GK Update
• Waste water treated under it will be used in irrigation and National Apprenticeship Promotion Scheme
agricultural work. The requirement of next 30 years will be
The Union Cabinet chaired by the Prime Minister Narendra
covered in this poilcy.
Modi approved a National Apprenticeship Promotion Scheme
• It will be compulsory for every house in the state to connect
on 5th July. The Scheme has an outlay of ` 10,000 crore with a
with sewage system in those cities which have 100% sewage
target of 50 lakh apprentices to be trained by 2019-20.
system in next five years.
Features of the scheme
KVK Portal • The Scheme would be implemented by Director General of
The Union Minister of Agriculture and Farmers Welfare, Radha Training (DGT) under Ministry of Skill Development and
Mohan Singh formally launched Krishi Vigyan Kendra (KVK) Entrepreneurship (MSDE). It provides for incentivizing
portal (http://kvk.icar.gov.in) on 8th July in New Delhi. employers to engage apprentices. 25% of the total stipend
Features of the Portal payable to an apprentice would be shared with employers
• The portal provides provisions for online monitoring of directly by Government of India. It is for the first time a
KVKs which will include reporting of major events on scheme has been designed to offer financial incentives
regular basis and submission of monthly reports online. to employers to engage apprentices. In addition, it also
• The portal will also provide information on different services supports basic training, which is an essential component
being provided by different KVKs. of apprenticeship training. 50% of the total expenditure
• Weather and Market related informations can also be incurred on providing basic training would be supported by

ww
accessed by the farmers and others.
• The forthcoming programmes will also be available on the
website which will benefit farmers and youth in joining
Government of India.
• The Scheme will catalyze the entire apprenticeship
ecosystem in the country and it will offer a win-win situation

w.E
different training programmes being organized by KVKs.
• Question and answer facility will also be available for the
farmers.
for all stakeholders. It is expected to become one of the most
powerful skill-delivery vehicle in the country.
• The National Policy of Skill Development and

available on the portal. asy


• Agriculture related information of the districts will also be Entrepreneurship, 2015 launched by the Prime Minister
focuses on apprenticeship as one of the key components
• The farmers and the Agricultural Officers may register
themselves and seek different informations related to KVKs.
En for creating skilled manpower in India. The present scheme
also aims to achieve the objective as stated in the National
‘Namami Gange’ programme
Union Minister Uma Bharti on 7th July launched 43 projects worth gin
Policy, 2015. The policy proposes to work pro-actively with
the industry including MSME to facilitate ten fold increase
opportunities in the country by 2020.
` 250 crore under Namami Gange programme in Uttarakhand.
The Government has lined up 1,000 projects in different states
including Delhi, Uttarakhand, Uttar Pradesh, Bihar, Jharkhand eer
National Green Highways Mission

and West Bengal as part of the National Mission for Clean


Ganga. Bharati said the second phase of the project will begin ing
Union Minister of Road Transport & Highways and Shipping
Nitin Gadkari launched the initial plantation drive on 1,500 km
of National Highways at a cost of about ` 300 crore under the
by October 2018.
Key Facts
• The Namami Gange Programme aims at cleaning the holy
Key features .ne
National Green Highways Mission in New Delhi on 1st July.

river and ensuring its unfettered flow. Union Government


has allocated ` 20,000 crore budget for it.
• Under it, projects such as modernization and redevelopment
of ghats, crematoriums and other development activities will
• NGHM was launched under the Green Highways Policy,

and green National Highways.


t
2015 to provide a holistic vision of developing eco-friendly

• The mission aims to provide a green canopy along 100,000


be undertaken. km of highways and create jobs for 1 million youth.
• Besides it will also undertake project for development • It has been initiated to engage corporates, Public Sector
of sewage infrastructure and treatment, tree plantation, units, Government organizations and other institutions for
afforestation, pilot drain project, interceptor drain project, trash developing green corridor along National Highways through
skimmers and conservation of biodiversity. plantation and allied activity on median, avenue and other
• The projects were launched simultaneously in five basin states available nearby land patches .
of the river Ganga viz. Uttarakhand, Uttar Pradesh, Bihar, • The funds for the mission will be met by Green Fund corpus
Jharkhand and West Bengal. Besides, projects were also which is 1% of the total project cost set aside for plantation
launched in Delhi and Haryana through which Ganga’s purposes. The afforestation across National Highways is
largest tributary Yamuna passes. expected to help in sequestering approximately 12 lakh
• In Uttar Pradesh alone total 112 projects were launched metric ton carbon annually.
through this ambitious scheme. 47 projects were inaugurated • It will also generate jobs and can be a game-changer for
at various locations in Uttrakhand. 20 projects were agriculture and rural economy. In future, the mission will be
inaugurated at various locations in West Bengal. 26 projects linked with the Mahatma Gandhi National Rural Employment
were inaugurated in Bihar. Guarantee (MNREG) scheme.

Downloded From : www.EasyEngineering.net


Downloded From : www.EasyEngineering.net

Current Affairs & GK Update 13


GK-

Chandranna Bima Yojana • The benefits will also be extended to family pensioners
including war widows and disabled pensioners.
Andhra Pradesh Chief Minister N Chandrababu Naidu has launched
• All arrears will be paid in four half-yearly instalments.
Chandranna Bima Yojana in Vijayawada on 1st May. The Yojana
However all widows, family pensioners including those
launched on the occasion of May Day is an insurance scheme
in receipt of Gallantry award will be paid arrears in one
for the working class people.
instalment.
Salient features of the scheme
• Pension will be re-fixed every five years and future
• All unorganised workers in the State in the age group of
enhancements in rates of pension would be automatically
18 to 70 years are eligible to be registered as unorganised
passed to the past pensioners.
workers and enrolled as beneficiaries of the Chandranna
• Armed forces personnel who retire voluntarily would be not
Bima Scheme. covered under OROP scheme. However, it will be applicable
• All unorganised workers will be registered under the to personnel who have already retired prematurely.
Unorganised Workers Social Security Act, 2008 and enrolled
as beneficiaries under Chandranna Bima Scheme. Stand up India scheme for SC/ST & Women
• SERP shall facilitate registration of unorganised workers and The Stand-up India scheme was launched on April 6 by
their enrollment in the Chandranna Bima Scheme, data entry, Prime Minister Narendra Modi to provide credit to Scheduled
maintenance of data base, hard copies of signed applications Caste (SC), Scheduled Tribe (ST), and women borrowers
and other connected matters. in the non-farm sector. The scheme is intended to promote

ww
• Assistant Labour Officer of the Labour Department shall be
the Registering Authority.
• The registered unorganised workers will be enrolled as
entrepreneurship among SC/ST and women and further gives a
push to government’s financial inclusion programme.
Main features of the scheme

w.E
members under State Accident Death and Disability Scheme
and under Aam Admi Bima Yojana (AABY) and will also
be covered under the Pradhan Mantri Suraksha Bima Yojana
• Composite loan between 10 lakh rupees and up to 1 crore
rupees will be provided to SC/ST and women and also
inclusive of working capital component for setting up any
(PMSBY).
UJALA Scheme asy new enterprise.
• Debit Card (RuPay) for withdrawal of working capital:
Credit history of borrower to be developed of these under-
The National LED programme – Unnat Jyoti by Affordable
LEDs for All (UJALA) was launched by the Union Government
on May 1. The scheme was launched by Union Minister for En banked sections of society.
• Refinance window through Small Industries Development

State (IC) for Power, Coal and Renewable Energy Piyush Goyal
in Bhopal, Madhya Pradesh. gin
Bank of India (SIDBI) with an initial amount of 10,000 crore
rupees.
• Creation of a corpus of 5,000 crore rupees for credit guarante
e
Key facts
• The UJALA scheme is being implemented by Energy
eer
through National Credit Guarantee Trustee Company
(NCGTC).
Efficiency Services Limited (EESL), a joint venture of PSUs
under the Union Ministry of Power.
• It is LED based Domestic Efficient Lighting Programme ing
• Handholding support for borrowers with comprehensive
support for pre loan training needs, facilitating loan,
factoring, marketing, etc.
(DELP).
• Under the scheme, 3 crore LED Bulbs will be distributed in .ne
• Web Portal for online registration and support services will
be provided.
Madhya Pradesh in the next 6 months.
• People of Madhya Pradesh will get subsidised 9W energy
efficient LED bulbs by paying just 85 rupees per LED bulb.
• The scheme will help reduce electricity bills of consumers,
t
• Under the scheme, 1.25 lakh bank branches will provide loans
up to 1 crore rupees to SC/ST and women entrepreneurs. Thus
it will help in creating 2.5 lakh entrepreneurs throughout the
country.
contribute to the energy security of India and also help in Pradhan Mantri Awaas Yojana – Gramin
environment protection.
The implementation of the rural housing scheme of Pradhan
• It will help in saving energy around 24 crore units every year. Mantri Awaas Yojana- Gramin was given approval by the Union
One Rank One Pension Scheme Cabinet on March 25. Under the scheme, financial assistance
will be provided for construction of pucca houses to all houseless
One Rank One Pension (OROP) scheme was given ex-post facto
and households living in dilapidated houses.
approval on 7 April by the Union Cabinet for implementation of
Key facts
retrospectively with effect from 1st July, 2014. Decision in this • The expenditure involved in implementing the project is `
regard was taken by Union Cabinet meeting chaired by Prime 81975 crore in a span of 3 years from 2016-17 to 2018-19.
Minister Narendra Modi in New Delhi. OROP will provide ex- • Under the scheme, it is proposed to provide assistance to one
servicemen of the same rank and same length of service uniform crore households for construction of pucca house.
pension regardless of date of retirement. • It will be implemented in rural areas throughout the country
Key facts except in Delhi and Chandigarh.
• The benefits of OROP will be given with effect from 1st • The beneficiaries will be identified using Socio Economic and
of July 2014 and it will benefit ex-servicemen of all three Caste Census (SECC)-2011 data ensuring total transparency
services- Army, Navy and Air Force. and objectivity.

Downloded From : www.EasyEngineering.net


Downloded From : www.EasyEngineering.net

GK- 14 Current Affairs & GK Update


• The cost of houses would be shared between Centre and • To improve technology depth in Indian capital goods from
States Governments in the ratio 60:40 in plain areas and the current basic and intermediate levels to advanced levels.
90:10 for North Eastern and hilly states. • Objectives: Increase total production to achieve total
• Under this scheme, annual list of beneficiaries will be production in excess of 5 lakh crore rupees by 2025 from the
identified from the total list through participatory process by current 2.2 lakh crore rupees.
the Gram Sabha.
• To increase domestic employment from the current 15
• Funds will be transferred electronically directly to the
account of the beneficiary. lakhs to at least 50 lakhs by 2025 thus providing additional
• He will be able to track the progress of his payments through employment to over 35 lakhs people.
the app. Inspection will be done through a mobile app. • To increase the share of domestic production in India’s
• Locally appropriate house designs, will be mainly practised capital goods demand from 56% to 80% by 2025 and in the
to address the natural calamities. process improve domestic capacity utilization to 80-90%.
• The beneficiary is entitled to 90 days of unskilled labour • To improve skill availability by training 50 lakhs people by
from MGNREGA. This will be ensured through a server 2025.
linkage between PMAY and MGNREGA. • To improve ‘technology depth’ in capital goods sub-sectors
• The additional requirement of building materials, bricks
by increasing research intensity in India from 0.9% to at least
using cement stabilised earth or fly ash will be taken up
2.8% of GDP.
under MGNREGA.

ww
National Technical Support Agency will be established
to provide technical support to the Centre and
States to ensure their quality of construction of the houses.
Pradhan Mantri Ujjwala Yojana
On May 1, the Pradhan Mantri Ujjwala Yojana (PMUY) was
launched. The scheme will benefit the poor, especially the
Udyam Aakansha scheme
w.E
Chhattisgarh Government has launched ‘Udyam Aakansha’
Scheme for promoting micro, small and medium enterprises in
women. The scheme aims to provide cooking gas connections
to five crore below-poverty-line beneficiaries over the next three
years.
the state on February 9.
Highlights of the scheme asy Salient features of the scheme
• Cabinet Committee on Economic Affairs (CCEA) approved
8000 crore rupees for the next 3 years starting from financial
• The scheme would facilitate free online registration and
En
self-certification to the investors interested in investment on
year 2016-17.
• PM Ujjwala Yojana will provide 5 crore LPG connections to
micro, small and medium enterprises and service businesses.
• The investors can enrol themselves online through website gin
BPL Families.
• A budgetary provision of the scheme is 2000 crore rupees.
of the Industries department which will be based on self-
certification process.
• For the online registration, investors need not submit any eer
• 5 crore LPG connections will be provided to BPL families
with a support of 1600 rupees.
• Connections will be given in the name of women beneficiaries .
documents nor have to pay any fee.
• Chhattisgarh is among some leading states where this scheme ing
• Identification of BPL families will be made in consultation
with State Governments and Union territories.
has been initiated.
• Keeping in view the objective of promoting the ease of doing
business, the process of registration and self-certification for Pradhan Mantri Yuva Yojana .ne
• EMI Facility for meeting the cost of Stove and Refill cost.

entrepreneurs has been simplified.


National Capital Goods Policy 2016
National Capital Goods Policy 2016 was unveiled by Union
t
The Union Ministry of Skill Development and Entrepreneurship
(MSDE) launched the Pradhan Mantri YUVA Yojana to boost up
an ecosystem of entrepreneurship for young people.

Ministry of Heavy Industries on 15 February 2016. The policy, Highlights of the Yojana
with the theme Building India of Tomorrow, seeks to achieve • The scheme was launched for the time period of over five
the vision of Make in India initiative and establishes India as a years (2016-17 to 2020-21) with a project cost of 499.94
global manufacturing hub by unlocking the potential for capital crore rupees.
goods sector. • It will offer entrepreneurship education and training to
Main Features around 7 lakh students in 5 years through 3050 Institutes.
• Vision: To increase the share of capital goods contribution • It will also offer easy access to information and mentor
from present 12 to 20% of total manufacturing activity by network, credit and advocacy to build a way-out for the
2025. youngsters.
• Mission: To become one of the top capital goods producing • The institutes under the Yojana include 2200 Institutes
nations of the world by raising the total production to over of Higher Learning (colleges, universities, and premier
twice the current level. institutes), 300 schools, 500 ITIs and 50 Entrepreneurship
• To raise exports to a significant level of at least 40% of total Development Centres.
production and thus gain 2.5% share in global exports of • These institutes will carry out the training through Massive
capital goods. Open Online Courses (MOOCs).

Downloded From : www.EasyEngineering.net


Downloded From : www.EasyEngineering.net

Current Affairs & GK Update 15 GK-

Other Launches The scheme had also found its mention in Prime Minister
• The Ministry also unveiled the guidelines for State Narendra Modi’s monthly radio address Mann Ki Baat in June
Engagement under Pradhan Mantri Kaushal Vikas Yojana 2016.
2.0 (2016-2020). It allocated around 3000 crore rupees for Salient Features
the States to achieve its target of training 10 million people • To provide healthy life to pregnant women
in over 4 years. • To lower the maternity mortality rate
• To make pregnant women aware of their health issues and
• It also launched the Lab Guidelines towards standardisation
diseases
of lab equipment across skill development training centres in
• To ensure safe delivery and healthy life of the baby
India. • The Pradhan Mantri Surakshit Matritva Abhiyan is applicable
• MSDE announced the institutionalisation of National for all pregnant women. It will provide medical checkups
Entrepreneurship awards for first generation achievers below completely free to pregnant women.
30 years for the very first time. The Entrepreneurship Awards • The checkups will take place at the medical centres,
will be given on 16 January 2017. government and private hospitals and private clinics across
the country.
Pradhan Mantri Surakshit Matritva Abhiyan • These free of cost tests will include blood pressure, sugar
level, weight, haemoglobin test, blood test and screening.
The scheme aims at providing pregnant women ante-natal
• Women will be marked differently using coloured stickers

ww
services and required treatment for free at government health
centres and hospitals on 9th of every month.
based on their health problems so that doctors can easily
detect the problem.

w.E
INDIAN STATES AND UNION TERRITORIES

India/State Capital
(Sq.Km)
asy
INDIAN STATES & UNION TERRITORIES
Area Language Establishment Sex Literacy
Year
Pop.
Ratio Average Density
Festivals Dance Tribes

INDIA New Delhi 3.3 En


/1000
No National 15-08-1947 940
% (Sq. Km)
73% 382 G.Jayanti, – –
Million Language
gin I.Day. &
R. Day
1. Andhra
Pradesh
2. Arunachal
Hyderabad

Itanagar
160205 Telugu,

83,743
Urdu
English
01-10-1953

20-02-1987
992

920/
67.7%

66.95% 17eer
308 Sankranti,
Ugadi
Kuchipudi

Losar” or The Bardo Chham


Andh, Bagata,
Bhil, Konda
Abor, Aka,
Pradesh
3. Assam Dispur 78,550 Assamese,
1000
1st April 1912 – – 397 Bihu ing
New Year
Ankia Naat
Apatani
Mikirs, Khasis,
(Assom)
4. Bihar Patna 99,200
Bengali,
Bhojpuri, 1st April 1936 916 63.4% 1,102 Chhath .ne
(Onkeeya)
Bidesia Kajari
Nagas, Boro
Gonda,

5. Chhattisgarh Raipur
Maithili
135,194 Chattisgarhi, 1-11-2000
Hindi
991 71.04% 189 Bastar
Dussere,
Bhoramdeo
Panthi, Rawat
Nacha
t
Mundas, Gaur
Agariya, Andh,
Baiga, Bhaina,

6. Goa Panaji 3,702 Konkani 30-05- 1987 968 88.70% 394 per Ganesh Dekhnni, Fugdi Dhodia, Dubla
sq km. Chaturthi (Halpati),
7. Gujarat Gandhinagar 196,204 Gujarati 01-05-1960 918 79.31% 310 Makar Rass-garba Bhils, Barda,
Sankranti Bavacha
8. Haryana Chandigarh 44,212 Punjabi, 01-11-1966 877 76.64% 573 Haryali Teej, Saang, Dhamal Meo, Ror
Haryanvi Lohri,
9. Himachal Shimla 55,673 Pahari, 25-01-1971 974 83.78% 123 Kullu, Losar Shona Bhot, Bodh,
Pradesh Kangri Shoolini Chuksam Gaddi, Gujjar
10. J & K Srinagar 222,236 Kashmiri, 26-10-1947 883 66.7% 56 Hemis, Urs Dumhal, Rouff Balti, Beda,
Urdu Bot, Boto
11. Jharkhand Ranchi 79,714 Santhali, 15-11-2000 947 67.6% 414 Jhumar, Paika, Karam, Vat Asur, Agaria,
Mundari, Ho Chau, Agni savitri Baiga, Banjara
12. Karnataka Bengaluru 191,791 Kannada 01-11-1956 968 75.60% 320 Mysore Bharatanatyam, Adiyan, Barda,
Dasara, Ugadi Bolak-aat Bavacha, Bhil

Downloded From : www.EasyEngineering.net


Downloded From : www.EasyEngineering.net

GK-16 Current Affairs & GK Update


13. Kerala Thiruvanantha- 38,863 Malayalam, 01-07-1949 1,084 93.91% 860 Onam Kathakali Adiyan,
puram English Arandan
14. Madhya Bhopal 308,245 Hindi 01-11-1956 930 70.60% 236 Shivratri, Badhai, Rai, Bhil, Bhunjia,
Pradesh Bahgoriya Saira Biar, Binjhwar
15. Maharashtra Mumbai 307,713 Marathi 01-05-1960 929 82.9% 370 Vijayadashami Lavani, Koli Andh, Baiga,
or Dasara Barda
16. Manipur Imphal 22,327 Meeteilon 21-01-1972 987 79.21% 120 Lui-ngai- Manipuri Aimol, Anal,
ni Ningol Angami
Chakouba,
Yaoshang
17. Meghalaya Shillong 22,429 Khasi,Garo 21-01-1972 986 75.84% 130 Nongkrem, Nongkrem` Chakma,
Wangala Dimasa, Garo
18. Mizoram Aizawl 21,087 Mizo 20-02-1987 975 91.58% 52 Chapchar Kut, Cheraw, Chakma,
Thalfavang Khuallam Dimasa, Garo
Kut,
19. Nagaland Kohima 16,579 English 01-12-1963 931 80.11% 119 Hornbill, Zeliang Naga, Kuki,

20. Odisha ww Bhubaneshwar 155,820 Odia, Eng-


lish
01-04-1936 978 73.45% 270
Sekrenyi
Ganesh
Chaturthi
Odissi
Mikir, Garo
Agata, Bathudi,
Birhor
21. Punjab Chandigarh
w.E 50,362 Punjabi 15-08-1947 893 76.68% 550 Bandi Chhor, Bhangra,
Vaisakhi,
Lohri
Giddha

22. Rajasthan Jaipur 342,239 Hindi,


asy
Rajasthani
01-11-1956 926 67.68% 201 Gangaur, Teej, Ghoomar
Gogaji

23. Sikkim Gangtok 7,096 Nepali,


Bhutia
En
16-5-1975 889 82.2% 86 Maghe, Losar Singhi Chham Bhutia, Lepcha,
Limboo
24. Tamil Nadu Chennai 130,058 Tamil 26-01-1950 995
gin
80.33 % 550 Pongal Bharata-natyam Adiyan,
Aranadan,
25. Telangana

26. Tripura
Hyderabad

Agartala
114,840 Telugu,
Urdu
10,491.69 Bengali ,
02-06-2014

21-01-1972

961
66.50%

94.65% eer
310

350
Ugadi


Kuchipudi

Goria, Jhum
Andh, Konda

Bhil, Bhutia,

27. Uttarakhand Dehradun 53,483


Kokborok
Garhwali, 9-11-2000 963 79.63% 189 Kandali,ing Chaimal
Langvir Nritya Bhotia, Buksa,

28. Uttar Pradesh Lucknow


Kumaoni
243,286 Hindi, Urdu 01-04-1937 908 69.7% 820
Ramman

.ne
Makar Sankranti, Kathak
Jaunsari, Raji
Bhotia, Buksa,

29. West Bengal Kolkata

Union
88,752 Bengali and 15-08-1947
English
947 77.08% 1,000
Chhath
Durga Puja,
Kali Puja
Chau dance
t
Tharu, Baiga
Asur, Baiga,
Bedia, Chero

Territory
1. Andaman and Port Blair 8,073 English, 01-11-1956 878 86.27% 46 – – Andamanese,
Nicobar Islands Hindi Chariar, Chari
2. Chandigarh Chandigarh 114 Punjabi 01-11-1966 818 81.9% 9,300/ Lohri Bhangra –
3. Dadra and Silvassa 102 English, 11-08-1961 775 77.65% 698 Pongal Tarpa, Bhavada Warlis, dublas
Nagar Haveli Gujarati
4. Daman and Daman 102 Gujarati, 30-05-1987 618 87.07% 2169 Garba Mando, Vira Dhodia, Dubla
Diu Marathi (Halpati)
5. Lakshadweep Kavaratti 32 English, 01-11-1956 946 92.28% 2013 Eid-Ul-Fitr, Lava, Kolkali Koya, Malmi
Malayalam Muharram
6. NCT of Delhi New Delhi 1,484.0 Hindi 01-02-1992 866 86.34% 11,297 Diwali, Eid – –
ul-Fitr
7. Puducherry Pondicherry 492 Malayalam, 07-01-1963 1,038 86.34% 2,500 Pongal Garadi Grulas, Villi
Tamil

Downloded From : www.EasyEngineering.net


Downloded From : www.EasyEngineering.net

Current Affairs & GK Update GK- 17


STATE ANIMALS AND BIRDS
State Animal Birds
Andhra Pradesh Blackback (Antilope cervicapra) Indian Roller (Coracias benghalensis)
Arunachal Pradesh Gayal (Bos frontalis) Great Hornbill (Buceros bicornis)
Asom (Assam) One-horned rhino (Rhinoceros unicornis) White-winged wood duck (Cairina scutulata)
Bihar Gaur (Bos gaurue) House Sparrow (Passer domesticus)
Chhattisgarh Wild buffalo (Bubalis arnee) Bastar Hill myna (Gracula religiosa)
Goa Gaur (Bos gaurus) Black-crested bulbul (Pycnonotus gularis)
Gujarat Asiatic lison (Panthera leo persica) Greater Flamingo (Phoenicopterus roseus)
Haryana Blackbuck (Antilope cervicapra) Black Francolin (Francolinus francolinus)
Himachal Pradesh Snow Leopard (Uncia uncia or Panthna uncia) Jujurana Western Irogapa (Trogopan melanocephalus)
Jammu and Kashmir Kashmir Stag (Cervus elaphus hanglin) Black-necked crane (Grus nigricollis)
Jharkhand Indian Elephant (Elephas maximus indicus) Asian koel (Eudynamys scolopacea)
Karnataka Indian Elephant (Elephas maximus indicus) Indian Roller (Coracias benghalensis)
Kerala Indian Elephant (Elephas maximus indicus) Great Hornbill (Buceros bicornis)
Madhya Pradesh Barasingha (Rucervus duvaucelii) Asian Paradise Flycatcher (Trepsiphone paradisi)
Maharashtra
Manipur
Meghalaya
ww Indian Giant Squirrel (Ratufa indica)
Sangai (Cervus eldi)
Clouded Leopard (Neofelis nebulosa)
Yellow footed green pigeon (Treron phoenicoptera)
Mrs. Humes Pheasant (Syrmaticus humiae)
Hill Myna (Gracula religiosa)
Mizoram
Nagaland
Odisha
w.E
Hoolock gibbon (Hoolock hoolock)
Gaur (Bos gaurus)
Sambar (Rusa unicolor)
Mrs. Humes pheasant (Syrmaticus humaie)
Blyth’s Tragopan (Tragopan blythii)
Indian Roller (Coracias benghalensis)
Punjab
Rajasthan asy
Blackbuck (Antilope cervicapra)
Chinkara (Gazella bennettii)
Northern Goshawk (Accipiter gentilis)
Great Indian Bustard (Ardeotis nigps)
Sikkim
Tamil Nadu
Red panda (Ailurus fulgens)

En
Nilgiri Tahr (Nilgiritragus hylocrius)
Blood pheasant (Ithaginis cruentus)
Emerald Dove (Chalcophaps indica)
Telangana
Tripura
Chital (Zinka)
Phayre’s Langur (Trachypithecus phayrel)
gin Pala Pitta (Coracias benghalensis)
Green Imperial Pigeon (Dacula genea)
Uttarakhand
Uttar Pradesh
Musk deer (Moschus cupreus)
Swamp Deer (Rucervus duvaucelii)
eer
Himalayan Monal (Lophophorus impejanus)
Sarus Crane (Grus antigone)
West Bengal
Lakshadweep
Fishing cat (Prionailurus viverrinus)

ing
White-breasted Kingfisher (Halcyon smyrnensis)
House Sparrow (Passer Domesticus)
National Capital
Territory of Delhi
Puducherry Squirrel Sciuridae ratufinae
.ne
House Sparrow (Passer domesticus)

Asian Koel (Eudynamys scolopaceus)

Four Ends of India


Easternmost point of India is known as Kibithu; situated on right
Foundation day of States
1st January Nagaland Day
t
bank of river Lohit separating India from China-Tibet region. 21st January Manipur, Meghalaya and Tripura Day
It is a small village at the altitude of 3,350 metre in Arunachal 6th February Jammu and Kashmir Day
Pradesh. Westernmost point is situated in Kuch area of Gujarat
20th February Mizoram and Arunachal Pradesh day
called as Ghuar Mota. The region is famous for its harsh climate
11th March Andman and Nicobar Islands Day
with 45°C in summer and 20°C in winter. During monsoon
22nd March Bihar Day (Bihar Diwas)
season this region looks like a tortoise surrounded by sea water.
30th March Rajasthan Day
Northernmost point of India has been in controversies ever since
India’s independence. The Siachen Glacier in the state of J&K is 1st April Utkal (Odisha) Day
the northern boundary of India according to the official division 14th April Tamil Nadu Day
of India during the time of independence. The Southernmost 15th April Himachal Pradesh Day
point of the mainland of India is Kanyakumari District in the state 1st May Gujarat and Maharashtra Day
of Tamil Nadu. Kannyakumari, formerly was known as Cape 16th May Sikkim Day
Comorin. It is the second largest and urbanized of Tamil Nadu. 1st November Chattisgarh
Indira Point is a village in the Nicobar district of Andaman and 9th November Uttaranchal (Now Uttarakhand) Day
Nicobar Islands, India. It is located in the Great Nicobar tehsil. 15th November Jharkhand Day (Jharkhand Diwas)
It is the location of the southernmost point of India’s territory. 2nd June (2014) Telangana Day

Downloded From : www.EasyEngineering.net


Downloded From : www.EasyEngineering.net

GK-18 Current Affairs & GK Update


AWARDS AND HONOURS
Gallantry Awards Gandhi Peace Prize
Param Vir Chakra: The highest Gallantry Award • Established in 2 October, 1994. It carries a cash prize of ` 1
Mahavir Chakra: The second highest Gallantry Award crore.
Vir Chakra: The third highest Gallantry Award
Indira Gandhi Prize for Peace, Disarmament and
Ashok Chakra: The highest peacetime Gallantry award
Development
Kirti Chakra: For conspicuous Gallantry
Shaurya Chakra: For an act of Gallantry • Instituted in 1985, this prestigious award is regarded as
Bharat Ratna ‘Nobel’ and over the years it has been awarded to those
The highest civilian award of India. persons who have done outstanding work for international
• The first three recipients of Bharat Ratna: C. Rajagopalchari, peace, disarmament and development.
Dr. S. Radhakrishnan and DR. C.V.Raman (1954) Dadasaheb Phalke Award
Padma Awards
• It is India’s highest award in cinema.
• Padma Vibhushan : The second highest civilian award given • It is presented annually at the National Film Awards
for exceptional and distinguished service.
ceremony by the Directorate of film festival.
• Padma Bhushan : The third highest civilian award given for

ww
distinguished service of a high order.
• Padma Shri : The fourth highest civilian award given for
distinguished service.
Borlaug Award
• Instituted in 1973, carries a cash prize of ` 1 lakh.
• Instituted to honour outstanding agricultural scientists.
Bharatiya Jnanpith Award
w.E
• Instituted in 22 May, 1961, carries a cash prize of ` 5 lakh, a
citation and a bronze replica of Vagdevi (Saraswati).
Sahitya Akademi Award
• Awarded for outstanding literary work and carries a cash

asy
• This award is given for the best literary writing by an Indian
citizen in a language listed in eight schedule of the Indian
Constitution.
prize of ` 1 lakh.
• Sahitya Academi gives 22 awards for literary works in the
languages which has recognized works.

En
INDIA’S POLITICAL JOURNEY 70 YEARS
Date/ Period
1946
Events
gin
The Constituent assembly elected in 1946 served as the Parliament
15 Aug. 1947 • India became independent
• Jawaharlal Nehru becomes the 1st P. M. of Independent India eer
30 Jan. 1948 •

Mahatma Gandhi was shot dead by Nathuram Godse
India sends troops and took over the State of Hyderabad ing


Sheikh Abdulla became the Prime Minister of Jammu and Kashmir.
Pakistan adopted Guerrilla War in J & K but India recaptured Dras, Kargil and Poonch. .ne
26 Nov. 1949
26 Jan. 1950
Constitution of India was adopted with a Preamble, 395 Articles & 8 Schedules.
• Constitution of India was enforced & India was declared a Republic State
• Rajendra Prasad, 1st President of India
• Home Minister Vallabhbhai Patel died on 15 Dec. 1950.
t
1951 • India’s first general/Lok Sabha election ( 25 Oct. 1951 to 21 Feb. 1952) & Indian National Congress won
with full majority (75%)
1953 • Sheikh Abdulla was dismissed, arrested and Ghulam Mohammed became the Prime Minister of Jammu and
Kashmir.
1954 • French finally gave up its territory Pondicherry to India.
1956 Death of B. R. Ambedkar on 6 Dec. 1956
1957 • Second general elections held in India & Congress won 371 seats
1959 • Mrs. Indira Gandhi was elected the president of Indian National Congress.
1960 • The states of Gujarat and Maharashtra came into being on May 1, 1960 & Bombay got allocated to
Maharashtra.
1961 • Indian troops liberated Goa from Portuguese colony by Operation Vijay
1962 • Congress retained its majority in the general elections (361 seats out of 494).

Downloded From : www.EasyEngineering.net


Downloded From : www.EasyEngineering.net

Current Affairs & GK Update 19


GK-

1964 • Jawahar Lal Nehru passed away. Gulzarilal Nanda was sworn in as acting prime minister.
• Congress Working Committee finalized on Lal Bahadur Shastri as Nehru’s successor. (Other contenders were
Tamilian K. Kamraj & Morarji Desai)
1965 • Pakistan invaded India over a salt marsh in Kachchh desert forcing Indians to withdraw some 40 miles.
1966 • L.B. Shastri & Pakistani President Ayub Khan met in Taskent along with the Soviet Union P.M. Kosygin, and
signed “Taskent Agrement”.
• Lal Bahadur Shashtri dies in sleep, of cardiac arrest.
• Gulzarilal Nanda was (once again) sworn in as acting prime minister.
• Indira Gandhi became PM on Congress president Kamraj endorsement for which Morarji Desai was interested.
• Punjab and Haryana divided into separate states.
1967 • 4th general elections held in February & INC got 283 seats.
1969 14 banks were nationalized which was opposed by then Finance Minister Morarji Desai
1971 • The Indo-Pakistan War of 1971 over Bangladesh liberation. Pakistani troops surrendered & Provisional
Government of the People’s Republic of Bangladesh came into being.

ww • 5th general elections of India was held 14 months before scheduled time and Congress returned with 362
seats out of 520
1975
w.E
• On 26th June 1975 national emergency was declared.
• Opposition leaders, including, Jayprakash Narayan, Morarji Desai, were sent to jail. Sanjay Gandhi became
leading political executive overshading Indira Gandhi
1977
asy
• Mrs. Gandhi dissolved Lok Sabha and general elections were held.
• The Congress lost around 200 seats. Both Indira Gandhi and Sanjay Gandhi lost.

1979 En
• Janata Party came with majority & Morarji Desai became PM of India.
• Morarji Desai resigned after Janata Party splits into three parts: headed by Morarji Desai, Jagjivan Ram and
Charan Singh.
gin
• Charan Singh was sworn as Prime Minster, but a month later Indira’s Congress (I) withdrew support, which

1980
forced a mid-term election.
eer
General elections of India was held & INC won 353 seats as a result Mrs.I. Gandhi became PM. Sanjay Gandhi

1984
died in a plane accident.
• Indira Gandhi shot dead by two of her Sikh personal bodyguards. ing
• Rajiv Gandhi became Prime Minister of India.
• 8th general elections of India was held & INC won 404 seats. .ne
1989

1991
BJP 85 & Left Parties 45 seats. V.P. Singh became PM with support of BJP & Left parties.
• Rajiv Gandhi assassinated by suicide bomber sympathetic to Sri Lanka's Tamil Tigers(LTTE).
t
General elections of India was held & INC won 197 seats. The Janata Dal led by V P Singh secured 143 seats,

• 10th General elections were held. INC+ got 244, BJP+ 120 & NF got 69 seats. P.V. Narasimha Rao became
PM with the support of Left parties.
1992 Babri Mosque in Ayodhya was demolished, triggering widespread Hindu-Muslim violence.
1996 • General elections were held. UF 192(Janata Dal+) won 192, BJP 187, & INC 140 seats.
• AB Vajpayee became PM only for 13 days followed by H.D. Deve Gowda & IK Gujral (Janata Dal). The 11th
Lok Sabha produced three Prime Ministers in two years.
1999 General elections were held. NDA(BJP+) won 254(182+), INC+ 144 & UF 64 seats. AB Vajpayee became PM.
1999 Kargil war between India & Pakistan. India came as victorious.
2004 General elections were held. UPA got 218, NDA 181 seats & Manmohan Singh became PM of India.
2009 General elections were held. UPA got 262, NDA 159 seats & Manmohan Singh became PM of India.
2014 General elections were held. BJP got 282, INC 44. Narendra Modi became PM of India.
2016 India got entry into Missile Technology Control Regime (MTCR) group.

Downloded From : www.EasyEngineering.net


Downloded From : www.EasyEngineering.net

GK-20 Current Affairs & GK Update

INDIA'S ECONOMIC JOURNEY 70 YEARS

ww
w.E
asy
En
gin
eer
ing
.ne
t

Downloded From : www.EasyEngineering.net


Downloded From : www.EasyEngineering.net

Current Affairs & GK Update GK-21

IMPORTANT DAYS OF THE YEAR


9th January: NRI Day 28th July: World Hepatitis Day
10th January: World Laughter Day 29th July: International Tiger Day
12th January: National Youth Day 30th July: International Day of Friendship
15th January: Indian Army Day 6th August: Hiroshima Day
23rd January: Netaji Subhash Chandra Bose’s birth anniversary 12th August: International Youth Day
24th January: National Girl Child Day 15th August: India’s Independence Day
25th January: National Tourism Day 19th August: World Humanitarian Day
26th January: Republic Day 29th August: National Sports Day
29th January: National Newspaper Day 5th September: Teacher’s Day
30th January: Martyr’s Day 8th September: International Literacy Day
2nd February: World Wetlands Day 14th September: Hindi Day
4th February: World Cancer Day 15th September: World Engineer’s Day
13th February: World Radio Day 16th September: World Ozone Day
14th February: St. Valentine’s Day 18th September: International Day of Peace

ww
21st February: International Mother Language Day
28th February: National Science Day
8th March: International Women’s Day and Mother’s day
21st September: Biosphere Day
24th September: Girl Child Day
25th September: Social Justice Day, World Maritime Day

w.E
13th March: No Smoking Day
15th March: World Consumer Rights Day
22nd March: World Water Day
27th September: World Tourism Day
1st October: International Music Day

23rd March: World Meteorological Day


24th March: World Tuberculosis Day asy 2nd October: Gandhi Jayanti, International Non-Violence Day
3rd October: World Habitat Day
8th October: National Air force Day
27th March: World Theatre Day
2nd April: World Autism Awareness Day En 9th October: World Postal Day National Postal Week (9th
October to 14th October)
7th April: World Health Day
18th April: World Heritage Day gin
12th October: World Sight Day
16th October: World Food Day
22nd April: World Earth Day
25th April: World Malaria Day eer
31st October: National Integration Day
7th November: World Cancer Awareness Day
30th April: World Jazz Day
1st May: International Labour Day
3rd May: World Asthma Day
ing
11th November: National Education Day
14th November: Children’s Day

5th May: World Athletics Day


8th May: International Red Cross Day
16th November: National Press Day
.ne
21st November: World Fisheries Day, World Hello Day, World

11th May: National Technology Day


17th May: World Telecommunications Day
20th May: World Refugee Day
Television Day

Violence against Women


26th November: National Constitution day
t
25th November: International Day for the Elimination of

21st May: Anti-terrorism Day 26th November: National Milk day


24th May: Commonwealth Day 30th November: Flag Day
31st May: World No Tobacco Day 3rd December: World Conservation Day
1st June: World Milk Day 5th December: World Soil Day
5th June: World Environment Day 7th December: Armed Forces Flag Day
8th June: World Ocean Day 9th December: International Day against Corruption, National
20th June: International Refugee Day Immunization Day
21st June: International Yoga Day 10th December: World Human Rights Day, International
23rd June: International Olympic Day Broadcasting Day
27th June: World Diabetes Day 11th December: UNICEF Day
1st July: World Doctor’s Day Van Mahotsav Week (1st July to 14th December: National Energy Conservation Day
7th July) 16th December: Vijay Diwas
11th July: World Population Day 22nd December: National Mathematics Day
26th July: Kargil Victory Day 23rd December: Kisan Diwas (Farmers’ Day)

Downloded From : www.EasyEngineering.net


Downloded From : www.EasyEngineering.net

GK-22 Current Affairs & GK Update


SPACE SCIENCE
Space mission 1975-2016 EDUSAT 20-Oct-04 GSLV-F01
Satellite Launch Date Launch Vehicle HAMSAT 5-May-05 PSLV-C6
Aryabhata 19-Apr-75 u-11 Interkosmos CARTOSAT-1 5-May-05 PSLV-C6
Bhaskara-I 7-Jun-79 C-1 Interkosmos INSAT-4A 22-Dec-05 Ariane-5GS
Rohini Technology 10-Aug-79 SLV-3 INSAT-4C 10-Jul-06 GSLV-F02
Payload CARTOSAT-2 10-Jan-07 PSLV-C7
Rohini RS-1 18-Jul-80 SLV-3 Space Capsule Recovery 10-Jan-07 PSLV-C7
Rohini RS-D1 31-May-81 SLV-3
Experiment (SRE-1)
Ariane Passenger 19-Jun-81 Ariane-1 (V-3)
INSAT-4B 12-Mar-07 Ariane-5ECA
Payload Experiment
INSAT-4CR 2-Sep-07 GSLV-F04
Bhaskara -II 20-Nov-81 C-1 Intercosmos
INSAT-1A 10-Apr-82 Delta 3910 PAM-D CARTOSAT-2A 28-Apr-08 PSLV-C9
Rohini RS-D2 17-Apr-83 SLV-3 IMS-1 (Third World 28-Apr-08 PSLV-C9
INSAT-1B 30-Aug-83 Shuttle [PAM-D] Satellite – TWsat)
Stretched Rohini 24-Mar-87 ASLV Chandrayaan-1 22-Oct-08 PSLV-C11
Satellite Series RISAT-2 20-Apr-09 PSLV-C12
(SROSS-1)
IRS-1A ww 17-Mar-88 Vostok
ANUSAT
Oceansat-2 (IRS-P4)
20-Apr-09
23-Sep-09
PSLV-C12
PSLV-C14
Stretched Rohini
Satellite Series
(SROSS-2)
INSAT-1C
w.E
13-Jul-88

21-Jul-88
ASLV

Ariane-3
GSAT-4
CARTOSAT-2B
StudSat
15-Apr-10
12-Jul-10
12-Jul-10
GSLV-D3
PSLV-C15
PSLV-C15
INSAT-1D
IRS-1B
12-Jun-90
29-Aug-91 asy
Delta 4925
Vostok
GSAT-5P / INSAT-4D
RESOURCESAT-2
Youthsat
25-Dec-10
20-Apr-11
20-Apr-11
GSLV-F06
PSLV-C16
PSLV-C16
INSAT-2DT
Stretched Rohini
26-Feb-92
20-May-92 ASLV
En
Ariane-44L H10
GSAT-8 / INSAT-4G
GSAT-12
21-May-11
15-Jul-11
Ariane-5 VA-202
PSLV-C17
Satellite Series
(SROSS-C)
INSAT-2A 10-Jul-92 Ariane-44L H10 gin
Megha-Tropiques
Jugnu
12-Oct-11
12-Oct-11
PSLV-C18
PSLV-C18
INSAT-2B
IRS-1E
23-Jul-93
20-Sep-93
Ariane-44L H10+
PSLV-D1
RISAT-1
SRMSAT eer 26-Apr-12
26-Apr-12
PSLV-C19
PSLV-C18
Stretched Rohini
Satellite Series
(SROSS-C2)
4-May-94 ASLV GSAT-10
SARAL ing
29-Sep-12
25-Feb-13
Ariane-5 VA-209
PSLV-C20

IRS-P2
INSAT-2C
15-Oct-94
7-Dec-95
PSLV-D2
Ariane-44L H10-3
IRNSS-1A
INSAT-3D
GSAT-7
1-Jul-13
26-Jul-13
30-Aug-13 .ne
PSLV-C22
Ariane-5
Ariane-5
IRS-1C
IRS-P3
INSAT-2D
IRS-1D
29-Dec-95
21-Mar-96
4-Jun-97
29-Sep-97
Molniya
PSLV-D3
Ariane-44L H10-3
PSLV-C1
Mars Orbiter Mission
(MOM)
GSAT-14
5-Nov-13

5-Jan-14
PSLV-C25

GSLV-D5
t
IRNSS-1B 4-Apr-14 PSLV-C24
INSAT-2E 3-Apr-99 Ariane-42P H10-3
IRNSS-1C 10-Nov-14 PSLV-C26
Oceansat-1 (IRS-P4) 26-May-99 PSLV-C2
GSAT-16 7-Dec-14 Ariane-5
INSAT-3B 22-Mar-00 Ariane-5G
IRNSS-1D 28-Mar-15 PSLV-C27
GSAT-1 18-Apr-01 GSLV-D1
GSAT-6 27-Aug-15 GSLV-D6
Technology 22-Oct-01 PSLV-C3
Astrosat 28-Sep-15 PSLV-C30
Experiment Satellite
(TES) GSAT-15 11-Nov-15 Ariane 5 VA-227
INSAT-3C 24-Jan-02 Ariane-42L H10-3 IRNSS-1E 20-Jan-16 PSLV-C31
Kalpana-1 (METSAT) 12-Sep-02 PSLV-C4 IRNSS-1F 10-Mar-16 PSLV-C32
INSAT-3A 10-Apr-03 Ariane-5G IRNSS-1G 28-Apr-16 PSLV-C33
GSAT-2 8-May-03 GSLV-D2 Corbo Set-2C 22-Jun-16 PSLV-C34
INSAT-3E 28-Sep-03 Ariane-5G Insat 3DR 8-Sep-16 GSLV-F05
RESOURCESAT-1 17-Oct-03 PSLV-C5 SCATSAT-1 26-Sep-16 PSLV-C35
(IRS-P6) GSAT-18 5-Oct-16 Ariane 5 ECA VA-231

Downloded From : www.EasyEngineering.net


Downloded From : www.EasyEngineering.net

Current Affairs & GK Update GK- 23


FIRST IN MALE
First Governor of Bengal Lord Clive (1757-60) The first Prime Minister of India who Charan Singh
Last Governor of Bengal Warren Hastings did not face the Parliament
(1772-74) The first Field Marshal of India S.H.F. Manekshaw
The first British Governor General of Lord Warren Hasting The first Indian to get Nobel Prize in C.V.Raman
Bengal (1774-1885) Physics
The first British Governor General of Lord William The first Indian to receive Bharat Ratna Dr. Radhakrishnan
India Bentinck award
(1833-1835) The first Indian to cross English Mihir Sen
The first British Viceroy of India Lord Canning (1856- Channel
62) The first Person to receive Jnanpith Sri Shankar Kurup
The first Governor General of free Lord Mountbatten award
India (1947-1948) The first Speaker of the Lok Sabha Ganesh Vasudeva
Mavalankar
The first and the last Indian to be C. Rajgopalachari
Governor General of free India (1948-1950) The first Vice-President of India Dr. Radhakrishnan
The first President of Indian Republic Dr. Rajendra Prasad The first Education Minister Abdul Kalam Azad
The first Prime Minister of free India Pt. Jawahar Lal The first Home Minister of India Sardar Vallabh Bhai

ww
The first Indian to win Nobel Prize
Nehru
Rabindranath Tagore
The first Indian Air Chief Marshal
The first Indian Naval Chief
Patel
S. Mukherjee
Vice Admiral R.D.
Congress
w.E
The first President of Indian National

The first Muslim President of Indian


W.C. Banerjee

Badruddin Tayyabji
Katari
The first Judge of International Court Dr. Nagendra Singh
of Justice
National Congress
The first Muslim President of India
The first man who introduced printing asy
Dr. Zakir Hussain
James Hicky
The first person to reach Mt. Everest Sherpa Anga Dorjee
without oxygen
The first person to get Param Vir Major Somnath
press in India
The first Indian to join the I.C.S Satyendra Nath En Chakra Sharma
The first Chief Election Commissioner Sukumar Sen

India’s first man in Space


Tagore
Rakesh Sharma
The first Prime Minister of India who Morarji Desai
gin
The first person to receive Magsaysay Acharya Vinoba
Award Bhave

resigned without completing the full


term eer
The first person of Indian origin to Hargovind Khurana
receive Nobel Prize in Medicine
The first Chinese traveller to visit India Fa-hein
The first Indian Commander-in-Chief General Cariappa
of India
The first Chief of Army Staff Gen. Maharaj
ing
The first person to receive Stalin Prize Saifuddin Kitchlu
The first person to resign from the Shyama Prasad

Rajendra Singhji
The first Indian Member of the S.P.Sinha
Central Cabinet

.ne
Mukherjee
The first person to receive Nobel Prize Amartya Sen
in Economics
Viceroy’s Executive Council
The first President of India who died Dr. Zakir Hussain
while in office
The first Chief Justice of Supreme Justice Hirala J.
Court
The first Indian Pilot
Kania t
J.R.D. Tata (1929)
FIRST IN FEMALE
The first lady to become Miss World Rita Faria The first woman President of Indian Mrs Annie Besant
The first woman judge in Supreme Mrs. Meera Sahib National Congress
Court Fatima Bibi The first woman pilot in Indian Air Harita Kaur Dayal
The first woman Ambassador Miss C.B. Muthamma Force
The first woman Governor of a state in Mrs Sarojini Naidu The first woman Graduates Kadambini Ganguly
free India and Chandramukhi
The first woman Speaker of a State Shanno Devi Basu, 1883
Assembly The first woman Airline Pilot Durga Banerjee
The first woman Prime Minister Mrs Indira Gandhi The first woman Honours Graduate Kamini Roy, 1886
The first woman Minister in a Rajkumari Amrit The first woman Olympic medal Winner Karnam Malleswari,
Government Kaur 2000
The first woman to climb Mount Everest Bachhendri Pal The first woman Asian Games Gold Kamlijit Sandhu
The first woman to climb Mount Everest Santosh Yadav Medal Winner
twice The first woman Lawyer Cornelia Sorabjee

Downloded From : www.EasyEngineering.net


Downloded From : www.EasyEngineering.net

GK-24 Current Affairs & GK Update


The first woman President of United Mrs Vijaya Laxmi The first woman Lieutenant General Puneeta Arora
Nations General Assembly Pandit The first woman Air Vice Marshal P. Bandopadhyaya
The first woman Chief Minister of an Mrs Sucheta The first woman chairperson of Indian Sushma Chawla
Indian State Kripalani Airlines
The first woman Chairman of Union Roze Millian Bethew The first woman IPS officer Mrs. Kiran Bedi
Public Service Commission The first and last Muslim woman ruler Razia Sultan
The first woman Director General of Kanchan Chaudhary of India
Police Bhattacharya The first woman to receive Ashoka Chakra Neerja Bhanot
The first woman Judge Anna Chandy (She The first woman to receive Jnanpith Ashapurna Devi
became judge in a Award
district court in 1937) The first woman to cross English Aarti Saha
The first woman Chief Justice of High Mrs Leela Seth Channel
Court (Himachal Pradesh The first woman to receive Nobel Prize Mother Teresa
High Court) The first woman to receive Bharat Ratna Mrs Indira Gandhi
The first woman Judge in Supreme Kumari Justice M. The first woman to receive Jnanpith Ashapurna Devi
Court of India Fathima Beevi Award

FIRST IN OTHERS

ww
First Wax statue of a Living Indian
First Exclusive internet magazine
First Miss India to participate in Miss Universe
Mahatma Gandhi at Madame Tussaud’s in 1939
Bharat Samachar
Indrani Rehman

w.E
First Judge in International Court of Justice
First Graduate in Medicine
India’s First University
Dr. Nagender Singh
Soorjo Coomar Goodeve Chukerbutty
Nalanda University
India’s First Open University
asy
India’s First Lok Sabha Member to be elected with a record
Andhra Pradesh Open University
P.V.Narasimha Rao
maximum number of votes
First Indian to reach Antarctica
First Test tube baby of India En Lt. Ram Charan
Indira (Baby Harsha)
First Post Office opened in India

SUPERLATIVES
gin
Kolkata(1727)

Structures eer
• Longest Railway Tunnel- Pir Panjal Railway Tunnel (11 km)
• Longest Road Tunnel - 9.2 km long tunnel on Jammu-
• Highest Tower ( Minaret ) – Qutub Minar
• Higher Gateway – Buland Darwaza at Fatehpur Sikri near
Agra. Built by Akbar (53.5 m /175 ft High) ing
Srinagar National Highway
• Largest Public Sector Bank- State Bank of India
• Highest Dam – Bhakra Dam
• Highest Bridge – Chenab Bridge
Kolkata
.ne
• Largest Botanical Garden - National Botanical Garden in

• Largest Church- Se Cathedral at Old Goa, 10 km from Panaji.


• Highest Airport- Leh Air Port in Ladakh (3256 m/ 16080 ft
high)
• Highest Hydel Power Station- Rongtong Hydel Project in
Kinnaur district of Himachal Pradesh.
and Brahmaputra in West Bengal and Bangladesh
• Largest Stupa- Kesariya Stupa in Bihar
• Largest Library- National Library, Kolkata
t
• Largest Delta- Sunderbans (75,000 sq km) formed by the Ganga

• Highest Mountain Peak- Kanchenjunga • Largest Planetarium- Birla Planetarium, Kolkata.


• Highest Road- Road at Khardungla in the Leh-Manali Sector • Largest Prison- Tihar Jail, Delhi
• Highest Waterfall- Jog Waterfall, Karnataka • Largest Concentration of Scheduled Tribes- Madhya Pradesh
• Largest Residence – Antilia Bhawan built by Mukesh • Largest Scheduled Caste- Community Santhal
Ambani • Longest River Bridge – Bandra-Worli sea link which is 5.6
• Largest Cinema Hall – Prasad Max, Hyderabad km.
• Largest Museum – National Museum Delhi • Largest Corridor – Rameshwaram Temple Corridor
• Largest River Barrage – Farakka Barrage • Largest irrigation Canal-Indira Gandhi Canal or Rajasthan
• Biggest Auditorium (Mumbai) – Sri Shanmukhanand Hall Canal (959 km long)
• Largest zoo – Arignar Anna Zoological Park • Longest Dam-Hirakund Dam on Mahanadi river in Odisha
• Largest Cave Temple – Ellora (24.4 km long)
• Largest Gurudwara – Golden Temple, Amritsar • Longest Glacier-Siachen Glacier on the Indo-Pakistan
• Largest Mosque – Jama Masjid, Delhi (built by Shah Jahan border (75.6 km long and 2.8 km wide)
in 1644-58) • Longest Railway Bridge Nehru Setu Bridge (4.62 km) long
• Largest Man-made Lake – Govind Sagar (Bhakra) • Fastest Train-Gatiman Express between New Delhi and
• Largest Dome – Gol Gumbaz (Karnataka) Agra at a speed of 160 kmph
• Largest Cantilever Bridge – Howrah Bridge • Tallest Light House – Jakhau, light hour, Gujarat

Downloded From : www.EasyEngineering.net


Downloded From : www.EasyEngineering.net

Current Affairs & GK Update 25


GK-

• Tallest Statues – Statue of Jain Saint Gomateswara at Leader of Nations-Famous Father


Sravanabelagola in Karnataka
• America—George Washington
• Tallest Chimber – Hanuman Swami statue with 135ft. tall.
• Bangladesh—Mujibur Rehuman
• Oldest Church- St. Thomas Church at Palayar in Trichur
• China—Sunyatsen
district in Kerala built in 52 AD.
• India—Gandhiji
• Oldest Monastery- Buddhist Monastery, (situated at an
• Indonesia—Sukarno
altitude of 3,048 m /10,000 ft) at Tawang in Arunachal Pradesh.
• Mauritius—Ramgoolam
• Largest mall- Lulu Mall Kochi
• Namibia—Sam Nujoma
• Most Populous City- Mumbai
• Pakistan—Muhammad Ali Jinna
Natural • Sri Lanka—D.S.Senanayeke
• Longest River – Ganges • Tanzania—Julius Nerera
• Largest Desert – Thar ( Rajasthan) • Turkey—Musthafa Kamal
• Largest Lake – Wular Rajasthan
• Largest Fresh Water Lake-Kolleru in Andhra Pradesh
Founders of Towns in India
• Largest Cave- Amarnath (about 44 km from Pahalgam in • Agra- Sikandar Lodhi
Jammu and Kashmir) • Ahmedabad - Ahmed Shah
• Ajmer- Ajaypal Chauhan
Founders of Indian Institutions • Allahabad- Akbar


• ww
Arya Samaj-Swami Dayanand Saraswati
Athmiya Sabha-Raja Ram Mohan Roy
Brahma Samaj-Raja Ram Mohan Roy



Calcutta- Job Charnock
Delhi- Anankapalan
Fatehpur Sikri - Akbar



w.E
Deccan Education Society-G.G.Agarkar, M.G.Ranade,
V.G.Gibhongar
Dharma Sabha-Radhakanthadev
Indian Brahma Samaj-Keshav Chandra Sen




Hisar- Ferozshah Tuglaq
Hyderabad - Quli Qutabshah
Jodhpur- Rao Jodha
Mahabalipuram - Narasimhawarman



Manavadharma Sabha-Durgaram Manjaram
Prarthana Samaj-Athmaram Pandurang asy
Pune Sewa Sadan-Smt.Remabhai Ranade, G.K.Devdhar


Siri- Alauddin Khilji
Vijayanagaram - Hariharan 1



Ramakrishna Mission-Swami Vivekananda
Sadharan Brahma Samaj-Shivananda Sashtri, Anand MohanEn Agricultural Revolution
• Blue Revolution - Fisheries Development



Bose
Servants of India Society-Gopalakrishna Gokhale
Sewa Sadan-Bahuramji M.Malabari



gin
Brown Revolution - Leather Production
Grey Revolution - Housing Development
Green Revolution - Agriculture Production


Sewa Samithi-H.N.Kunsru
Social Service League-N.M.Joshi

• eer
Pink Revolution - Drugs & Pharmaceuticals
Silver Revolution - Egg Production


Thathwabodhini Sabha-Debendranatha Tagore
Theosophical Society-Madam H.P.Blavadski, Col.H.L.Olkott


ing
White Revolution - Dairy Development
Yellow Revolution - Oil Seed Production

SOBRIQUETS
A sobriquet is a nickname, occasionally assumed and often Manitas de Plate .ne
Flamenco guitarist Ricardo
given by another.
Person
Anna
Primary Names
C N Annadurai
Netaji
Nightingale of India
Baliardo
t
Subhash Chandra Bose
Sarojini Naidu
Badshah Khan/ Frontier Gandhi Abdul Ghaffar Khan Panditji Jawahar Lal Nehru
Buddha Siddhartha Gautama Punjab Kesari Lala Lajpat Rai
Chacha Jawahar Lal Nehru Rajaji C Rajagopalachari
Deenabandhu C F Andrews Saint of the Gutters Mother Teresa
Deshbandhu C. R. Das Father of the Nation Mohandas Karamchand
Father of the Nation Mohandas Karamchand Gandhi
Gandhi Haryana Hurricane Kapil Dev
Grand Old Man of India Dadabhai Naoroji
Prince of Kolkata Saurav Ganguly
Gurudev Rabindranath Tagore
Places Primary Names
Guruji M S Gohlwalkar
Bengal’s Sorrow Damodar River
Kaviguru Rabindranath Tagore
Blue Mountain Niligiri Hills
Lokmanya Bal Gangadhar Tilak
Loknayak Jayaprakash Narayan City of Golden Temple Amritsar
Mahatma Gandhi Mohandas K. Gandhi City of Palaces Kolkata
Man of Peace Lal Bahadur Shastri Diamond City in India Surat, Gujarat

Downloded From : www.EasyEngineering.net


Downloded From : www.EasyEngineering.net

GK-26 Current Affairs & GK Update


Garden City of India Bengaluru States & its Slogans
Garden of India Kashmir • Rice bowl of India, Egg bowl of Asia – Andhra Pradesh.
Gateway of India Mumbai • The Orchid state of India or the paradise of the Botanists-
Arunachal Pradesh.
God’s Own Country Kerala
• Rice bowl of Central India – Chhattisgarh.
Land of Five Rivers Punjab • Jewel of the Western part of India– Gujarat.
Pink City Jaipur • State of Apples, Dev Bhoomi (Adobe of Gods) – Himachal
Queen of Arabian Sea Kochi Pradesh
Spice Garden of India Kerala • Heaven on Earth – Jammu and Kashmir.
The City of Joy Kolkata • The Land of Jungles & Jharis (Bushes) – Jharkhand
• God’s own country – Kerala
The City of Palaces Kolkata
• Heart of India – Madhya Pradesh
Venice of East Alleppey • Gateway of the East – Manipur
Queen of Arabian Sea Kochi • The adobe of the clouds – Meghalaya
Garden City of India Bangalore • Land of the hill people – Mizoram
Blue Mountains Niligiri Hills • Falcon capital of the world-Nagaland
Silicon Valley of India Bengaluru • Granary of India, India’s bread-basket – Punjab
• Land of the Gods – Uttarakhand
Planned City of India Chandigarh

ww HISTORY
• Hindi Heartland of India – Uttar Pradesh

w.E LITERATURE OF MUGHAL PERIOD


Book
Tuzuk-i-Baburi
Author
Babur
Contents

asy
Describes military tactics and
administrative organisation
Nuriyya-i-
Sultaniyya
Waqt-i-
Abdul Haq

Nimat Khan
Theory of Kinship during
Mughal Period
Aurangzeb's Golconda conquest

Qanun-i- Khwand
during Babur’s reign
Describes En
Humayun’s
Hyderabad
Futuhat-i-
Ali
Ishwar Das Aurangzeb's history
Humayu Amair administration, festivities and
buildings of that period gin
Alamgiri
Nuskha-i- Bhimsen Analysis of Aurangzeb's rule
Humayun Nama Gulbadan
Begum
Biography of Humayun Dilkusha
Khulasat-ul-
Tawarikh
eer
Saxena
Sujan Raj
Khatri
and character
History of Aurangzeb's rule
Akbar Nama Abul Fazl Gives a history of Akbar’s
reign Padshah Namah Abdul Hamid
Lahori ing History of Shah Jahan's reign
Tobaqat-i-
Akbari
Khwajah
Nizamuddin
Ahmad Baksh
-do-
Padshah Namah Muhammad
Waris
-do-
.ne
Tuzuk-i-
Jahangiri
Iqbalnama-i-
Jahangir

Muhammad
Memoirs of his own reign

History of Jahangir's reign


Shahjahan
Namah
Shahjahan
Namah
Muhammad
Salih
Inyat Khan
-do-

-do-
t
Jahangiri Khan
Hamlai-Haidri Muhammad History of Aurangzeb's rule
Chahar Chaman Chandra History of Shah Jahan's rule Rafi Khan
Bhan
Brahman Namah-e- Aquil Khan -do-
Alamgiri Zafar
Alamgir-nama Munshi Mirza Gives an account of
Sirr-i-Akbar Dara Shikoh Urdu translation of
Muhamma Aurangzeb's first 10 years of
Upanishad
Kazin rule
Safinat-ul- -do- Biographies of Sufi Saints
Massir-i- Saqi Mustaid Official history of Auliya
Alamgiri Khan Aurangzeb’s reign written
after his death Majma-ul- -do- Philosophical ideas discussed
Bahrain
Ain-i-Akbari Abul Fazl History of Akbar's reign
Raqqat-e- Aurangzeb A compendium of his letters
Muntakhab-ul- Badauni History of Akbar's rule Alamgiri
Tawarikh
Hasmat-ul- Dara Shikoh Religious ideas discussed
Tawarikh-i-Alfi Mulla Daud -do- Arifin

Downloded From : www.EasyEngineering.net


Downloded From : www.EasyEngineering.net

Current Affairs & GK Update GK-27


THE FREEDOM STRUGGLE TIME LINE
1885 Pherozeshah Mehta, K.T. Telang, Badruddin Tyabji formed Bombay Presidency Association.
28 Dec. 1885 Indian National Congress was formed by Allan Octavian Hume.
28-31 Dec. 1885 First session of Indian National Congress was attended by 72 delegates under the presidency of W.C. Bonnerjee.
1896-97 Bal Gangadhar Tilak initiated a no-tax campaign in Maharashtra.
20 July, 1905 Partition of Bengal order was passed by Lord Curzon.
Dec. 1905 Gokhale then the president of Congress condemned the partition of Bengal and supported Swadeshi and
Boycott movement.
1906 Dadabhai Naoroji became the president of National Congress and clearly declared their goal to be self-
government or Swaraj like the other colonies.
30 Dec. 1906 All India Muslim League was formed by Aga Khan III and the founding meeting was hosted by Nawab Sir
Khwaja Salimullah.
1909 The Indian Councils Act or Morley-Minto Reform was announced.
1911 Government announced the withdrawal of Partition of Bengal.
1913 Ghadar Party founded by Punjabi Indians in the United States and Canada aiming at securing India’s
independence.
April, 1915
26 Dec. 1916
ww First session of Hindu Mahasabha was held under the presidentship of Maharaja of Kasim Bazar.
Lucknow Pact was signed dealing with the structure of the government of India and with relation to the Hindu
and Muslim communities.
1917
1918
w.E
Indigo Satyagraha started by M.K. Gandhi in Champaran, Bihar.
Edwin Montagu, then the Secretary of State and Lord Chelmsford, the Viceroy produced a scheme of
constitutional reform which was called as the Montague-Chelmsford reforms.
1919
March, 1919
13 April, 1919
asy
Enactment of the Government of India Act.
Rowlatt Act was passed which enabled government to imprison people without trial.
Unarmed crowd gathered at Jallianwala Bagh to protest against the arrest of Dr. Saifuddin Kitchlew and Dr.

31 August, 1920 En
Satyapal was attacked by the British army as commanded by General Dyer.
Khilafat Committee launched a non-cooperation Movement.
1 February, 1922
5 Feb. 1922 gin
M.K. Gandhi announced mass Civil Disobedience movement.
Protesters participating in the Non-cooperation Movement turned violent, leading to police opening fire in

1925 Communist Party came into existence. eer


Chauri Chaura. Congress as a result halted the non-cooperation Movement.

Nov. 1927

17 Nov. 1928
Government of India Act, 1919.
ing
Simon headed commission was set up to submit report on working of Indian constitution established by

Lala Lajpat Rai died due to the injuries by the beating of local police during a protest demonstration at Lahore.
Dec. 1928
26 Jan. 1930
Gandhi joined back the active politics at Calcutta session.
.ne
Was fixed as the First Independence Day and since then was celebrated every year up to 1947.
Feb. 1930
12 March, 1930 Dandi March lead by M.K. Gandhi took place. Together with 78 companions he walked 375 km from
Sabarmati Ashram to Dandi.
t
Chandra Shekhar Azad was shot dead in a park called Azad Park at Allahabad, in an encounter with British police.

6 April, 1930 Gandhi reached Dandi and broke the Salt law.
12 Nov. 1930 First round table conference was held in London, was chaired by British Prime Minister Ramsay MacDonald.
5 March, 1931 Gandhi-Irwin Pact was signed between Gandhi and then viceroy of India Lord Irwin. According to which
British agreed to withdraw all ordinances and end prosecutions and release all political prisoners.
24 August, 1932 Poona Pact was signed between Gandhi and Dr. B.R. Ambedkar at Yerwada Central Jail.
1935 Government of India Act was passed according to which All India Federation was established including
British India and Princely States (representative were appointed by the rulers) forming a bicameral federal
legislature.
October, 1940 Gandhi gave an order for limited satyagraha (for few individuals only).
8 August, 1942 Quit India Movement was launched by M.K.Gandhi.
1945 Congress working committee adopted a resolution to abolish landlordism.
2 Sept. 1946 Interim government of India formed the newly elected Constituent Assembly of India. This idea was rejected
by Muslim league.
9 Dec. 1946 The Constituent Assembly met for the first time.

Downloded From : www.EasyEngineering.net


GK-
28

Can be Suspended FRs available only to citizens

ww
Magna Carta Part III (Article Borrowed from during Emergency & Not to Foreigners- Art. 15,
of India 12 to 35) USA & France except Art. 20 & 21 16, 19, 29, 30

Fundamental Rights

w.E
Right to Equality Right to Freedom Right Against Exploitation Right to Freedom of Cultural and Right to Constitutional
Religion Educational Rights Remedies

asy
Articles Articles Articles Articles Articles Articles
14-18 19-22 23-24 25-28 29-30 32

En
(14) Equality before Law (19) 6 Rights: Freedom of (23) Prohibition of traffic in (25) Freedom of conscience & (29) Protection of Includes writs:
& Equal protection speech & expression, human beings & forced free profession, practice & interests of (1) Habeas Corpus
of law. Assembly, Association, labour. propagation of religion. minorities.
POLITY

(2) Mandamus
Movement, Residence & (3) Prohibition
Fundamental Rights

Profession. (4) Certiorari


(5) Quo–Warranto

gin
(15) Prohibition of (20) Protection in respect of (24) Prohibition of (26 ) Manage religious affairs. (30) Right of minorities
discrimination on conviction for offences. employment of to establish
Downloded From : www.EasyEngineering.net

grounds of religion, children in factories & administer


race, sex etc. etc. educational

eer
institutions.
(16) Equality of (21) Protection of life & (27) Freedom from payment of
opportunity in personal liberty. taxes for promotion of any
public employment religion.
(17) Abolition of
untouchability.
(22) Protection against
arrest & detention in
certain cases.
ing (28) Freedom from attending
religious instruction
or worship in certain
educational institutions.
(18) Abolition of titles. .ne
t
Current Affairs & GK Update

Downloded From : www.EasyEngineering.net


Downloded From : www.EasyEngineering.net

Current Affairs & GK Update 29


GK-

Indian Legislature

Union Leg
State Leg
(Parliament)

President Rajya Sabha Lok Sabha Bicameral Unicameral

Legislative Assembly
Upper House Lower House

Also known as House of People


council of states Maximum Leg. Council or Leg. Assembly or
Total members-250 Strength - 552 Vidhan Parishad Vidhan Sabha

238 12 530 20 2 (Nominated Lower House


Indirectly Nominated (States) (UTs) members from Upper House
Anglo-Indian Maximum
elected Maximum
Community) Strength - 500
Permanent Body Strength - 1/3 rd
Members are Minimum

ww
of total strength
directly elected of Assembly. Strength - 60
Term of individual
member - 6 years Term - 5 years Minimum Members are
Strength - 40 directly elected
Presiding Officers by people

w.E
1/3 rd members Members are
retires at expiration indirectly Term - 5 years
of every 2nd year. elected
Speaker Deputy
Speaker Permanent body
Presiding officers

Chairman
asy
Deputy
Chairman
Qualification

Citizen Not less


1/3rd of its members
retire every second
year.
Term - 6 years
Qualification

En
of India than 25
years.

Citizen of
India
Not less than
30 years
gin
GEOGRAPHY
eer
Name
MAJOR STRAIT OF THE WORLD
Joins Location
Taurus Strait Arafura Sea & Gulf of
Papua ing Papua New Guinea -
Australia

Malacca Strait Andaman Sea & South


China Sea
Indonesia - Malaysia
Bass strait Tasman Sea & South
Sea
Bering Strait Bering Sea & Chukchi
.ne
Australia

Alaska-Russia
Palk Strait Palk Bay & Bay of
Bengal
Sunda Strait Java Sea & Indian
Ocean
India-Sri Lanka

Indonesia
Sea
Bonne-Fasio Mediterranean Sea
Strait
t
Corsika-Sardinia

Bosphorus Black Sea and Marma- Turkey


Yucatan Strait Gulf of Mexico and Mexico-Cuba
Strait ra Sea
Caribbean Sea
Dardanelle Marmara Sea and Ae- Turkey
Messina Strait Mediterranean Sea Italy-Sicily
Strait gean Sea
Otranto Strait Adriatic Sea & Ionian Italy-Albania Davis strait Baffin Bay & Atlantic Greenland-Canada
Sea Ocean
Bab-el-Man- Red Sea & Gulf of Yemen-Djibouti Denmark North Atlantic and Greenland-Iceland
deb Strait Aden strait Arctic Ocean
Cook Strait South Pacific Ocean New Zealand (N & S Dover strait English Channel & England-France
islands) North Sea
Mozambique Indian Ocean Mozambique - Mala- Florida Strait Gulf of Mexico and USA-Cuba
Strait gasy Atlantic Ocean
North Chan- Irish Sea & Atlantic Ireland-England Hormuz strait Gulf of Persia & Gulf Oman-Iran
nel Ocean of Oman

Downloded From : www.EasyEngineering.net


Downloded From : www.EasyEngineering.net

GK-30 Current Affairs & GK Update


Hudson strait Gulf of Hudson & At- Canada Gulf of Mannar, between India and Sri Lanka
lantic Ocean
Gulf of Mexico, between Mexico, the United States, and Cuba
Gibraltar Mediterranean Sea & Spain-Morocco
Strait Atlantic Ocean Gulf of Morbihan, a natural harbour on the coast of the
Département of Morbihan in the south of Brittany
Magellan Pacific and South At- Chile
strait lantic Ocean Gulf of Nicoya, in Costa Rica, Central America.
Makassar Java Sea & Celebeze Indonesia Gulf of Oman, between the south eastern Arabian Peninsula,
Strait Sea Iran, Pakistan and Arabian Sea.
Tsugaru Strait Japan Sea and Pacific Japan (Hokkaido-Hon- Gulf of Oristano, near Oristano on the Western Sardinian
Ocean shu island) coast
Tatar Strait Japan Sea & Okhotsk Russia (E Russia-
Gulf of Panama in the Pacific Ocean south of Panama
Sea Sakhalin Island)
Persian Gulf between Iran and the Arabian Peninsula
MAJOR GULFS AND THEIR LOCATION Gulf of Roses, the most northeastern bay on the Catalan coast
Gulf of Aden of the southwestern corner of the Arabian Gulf of Saint Lawrence, the world’s largest estuary and the
Peninsula outlet of the Saint Lawrence River into the Atlantic Ocean
Gulf of Alaska in the Pacific Ocean south of the state of Alaska

ww
Amundsen Gulf in the Arctic Ocean northwest of Canada
Gulf of Aqaba in the northern end of the Red Sea, leading to
Gulf St Vincent, separated from Spencer Gulf by the Yorke
Peninsula
Gulf of Sidra, just north of Libya in the Mediterranean
Israel and Jordan
w.E
Gulf of Bahrain, part of the Persian Gulf
Spencer Gulf, near Port Lincoln, South Australia
Gulf of Suez, in the northern end of the Red Sea, leading to

Finland
asy
Gulf of Bothnia, part of the Baltic Sea between Sweden and

Gulf of Cádiz, part of the Atlantic Ocean off the southern


the Suez Canal
Gulf of Thailand, just south of Thailand in the Indian Ocean

border of Spain and Portugal


Gulf of California in the Pacific Ocean in northwestern En Gulf of Tonkin, just east of North Vietnam in the Pacific
Ocean

Mexico
Gulf of Carpentaria a large bay off northern Australia gin
Gulf of Tunis, in the Mediterranean off the coast of Tunisia

Tribal Groups of India


Gulf of Cazones a large gulf in southern Cuba
Gulf of Corinth, which extends into Greece from the
Tribal
Group eer Region Tribal
Group
Region

Mediterranean
Davao Gulf in the Philippines
Abhor Arunachal Pradesh
ing Kharia Jharkhand,
Odisha

Gulf of the Farallones, westward from the opening of the San


Francisco Bay and Drakes Bay to the Farallon Islands
Adivasi A.P, Bihar, Odisha, Kol
Jharkhand, Madhya
Pradesh, Maharashtra, .ne
Madhya Pradesh

Gulf of Finland, between the southern coast of Finland and


the northern coast of Estonia in the Baltic Sea.
Gulf of Genoa inside the Ligurian Sea on the northwestern
Rajasthan, Tamil Nadu,
Some Northeastern
States, West Bengal, An-
daman and Nicobar
t
coast of Italy
Ahgani Manipur Kolam Maharashtra,
Gulf of Guinea in the Atlantic Ocean off the coast of Equatorial Andhra Pradesh,
Africa Telengana, M.P
Gulf of Izmir in the Aegean Sea between Turkey and Greece. Apatani Arunachal Pradesh Kota Karnataka
It was formerly called the Gulf of Smyrna.
Baiga Madhya Pradesh Kuki Mizoram
Gulf of Khambhat in the Arabian Sea, formerly known as the Bakar- Jammu and Kashmir Lahaula Himachal
Gulf of Cambay wal Pradesh
Gulf of Kutch in the Arabian Sea Bhil M.P and Rajasthan Lepcha Sikkim
Lingayen Gulf of western Luzon, the Philippines, in the South Birhor M.P and Bihar Lushai Mizoram,
China Sea Manipur
Gulf of Lion, a bay on the Mediterranean coastline of Chang Nagaland Muria Chhattisgarh
Languedoc-Roussillon and Provence in France Chench- Telengana, Karnataka Miha Rajasthan
Gulf of Maine, off the State of Maine, New Brunswick, and uas
Nova Scotia in the Atlantic Ocean Sutiya Assam Moplah Malabar

Downloded From : www.EasyEngineering.net


Downloded From : www.EasyEngineering.net

Current Affairs & GK Update GK-31


Gaddis Himachal Pradesh Munda West Bengal, Ghana Bird Rajasthan Water Bird, Black-buck, Cheetal,
Jharkhand, Sanctuary Sambar
Odisha,
Panchmarhi Madhya Tiger, Panther, Sambhar, Nilgai,
Chhattishgarh
Pradesh Baskeng, Deer
Gallong Arunachal Pradesh Nishi Assam
Dandeli Karnataka Tiger, Panther, Elephant, Cheetal,
Garo Meghalaya Naga Nagaland Sanctuary Sanbhar, Wild Boar
Gond M.P and Bihar Oraon MP, Bihar and
Kutch Near Great Indian Bustard, Lesser Florican,
Odisha, Chhota
Bustard Naliya, Houbara bustard, Chinkara, Blue
Nagpur, W.B,
Sanctuary Kachchh bull, Herpetofauna
Gujjar Rajasthan Onges Andaman &
Nicobar BIOSPHERE RESERVES IN INDIA AREA-WISE
Irula Tamil Nadu Singpho Assam, Arunachal Name State Key Fauna
Pradesh
Nilgiri Tamil Nadu, Nilgiri tahr, lion-tailed macaque
Jaintia Meghalaya Santhal WB, Odisha,
Bihar, Jharkhand, Biosphere Kerala and
Assam Reserve Karnataka
Jarawa Andaman, Islands Sangtam Nagaland Nanda Devi Uttarakhand Himalayan musk deer, mainland

ww
Kanikar Tamil Nadu and Kerala Sema
Kalkari Maharashtra Sentine-
lese
Nagaland
Andaman &
Nicobar
National Park
& Biosphere
Reserve
serow, Himalayan tahr

Kharia Maharashtra
w.E Shompen Andaman &
Nicobar
Gulf of
Mannar
Nokrek
Tamil Nadu Dugong or sea cow

Meghalaya Red panda


Khond
Khasi
Jharkhand
Meghalaya
Toda
Uralis
Wancho
asy
Tamil Nadu
Kerala
Arunachal
Sundarbans
Manas
West Bengal Royal Bengal tiger
Assam Golden langur, red panda

Warli
Pradesh
En
Maharashtra,
Simlipal
Dihang-
Odisha
Arunachal
Gaur, Royal Bengal tiger, elephant
Mishmi takin, red goral, musk
Daman and Diu,
Bihar, Madhaya
Pradesh, West
gin
Dibang
Pachmarhi
Pradesh
Madhya
deer
Giant squirrel, flying squirrel

Bengal Dadra and


Nagar Haveli
Biosphere
Reserve
Achanakmar- eer
Pradesh

Madhya Four horned antelope (Tetracerus


WILDLIFE SANCTUARIES IN INDIA
Name of the Location Major Species
Amarkantak
Biosphere
Pradesh,
ing
quadricornis), Indian wild dog (Cuon
Chhattisgarh alpinus), Saras crane (Grus antigone),
Sanctuary
Gir Wildlife Sasan Gir, Lion, Leopard, Chausinga, Chital,
Reserve

.ne
Asian white-backed vulture (Gyps
bengalensis), Sacred grove bush frog
(Philautus sanctisilvaticus)
Sanctuary

Wild Ass
Sanctuary
Junagadh,
Amreli
Little Rann
of Kachchh
Hyena, Sambar, Chinkara,
Herpetofauna, Crocodiles and birds
Wild Ass, Chinkara, Blue
Great Rann
of Kutch
Gujarat

Cold Desert Himachal


Indian wild ass

Snow leopard
t
bull, Houbara bustard, Wolf, Pradesh
Waterfowls, Herpetofauna Khangchendzonga Sikkim Snow leopard, red panda
Hingolgadh Hingolgadh, Chinkara, Blue bull, Wolf, AgasthyamalI Kerala, Nilgiri tahr, elephants
Sanctuary Rajkot Hyena, Fox, Birds, Herpetofauna Biosphere Tamil Nadu
Marine Gulf of Sponges, Corals, Jellyfish, Reserve
Sanctuary Kachchh, Sea horse, Octopus,Oyster, Great Andaman Saltwater crocodile
Jamnagar Pearloyster, Starfish, Lobster, Nicobar and Nicobar
Dolphin, Dugong, waterfowls Biosphere Islands
Simlipal Odisha Elephant, Tiger, Leopard, Gaur, Reserve
Sactuary Cheetal Dibru- Assam Golden langur
Kutch Great Rann Chinkara, Hyena, Fox, Flamingo, Saikhowa
Desert of Kachchh Pelicans & other waterfowls, Seshachalam Andhra Yellow-throated bulbul
Sanctuary Herpetofauna Hills Pradesh
Rampara Rampara, Blue bull, Chinkara, Wolf, Fox, Panna Madhya Tiger, chital, chinkara, sambhar
Sanctuary Rajkot Jackal, Birds, Herpetofauna Pradesh and sloth bear

Downloded From : www.EasyEngineering.net


Downloded From : www.EasyEngineering.net

GK-32 Current Affairs & GK Update

BIOLOGY
DIGESTION OF FOOD
Name of the Name of the enzymes Substrate End product
Digestive juice
Saliva Ptyalin (Salivary amylase) Starch Maltose
Pancreatic Juice Amylopsin (pancreatic amylase) Starch, Glycogen Maltose and Glucose
Intestinal juice Sucrase (invertase), Maltase, Lactase Sucrose; Maltose, Lactose Glucose, fructose and galactose
Gastric Juice Pepsin, Rennin Proteins, Casein Proteoses and peptones, Calcium caseinate
Pancreatic Juice Trypsin, Chymotrypsin, Carboxyl Proteins, Proteins Peptides Proteoses and peptides Peptides Amino
peptidases acid.
Intestinal juice Amino peptidase, Dipeptidase Peptides Amino acids
Vitamin Required by the Body
Vitamin Chemical Name Function in Body Deficiency Disease Sources
B1 Thiamine Part of coenzyme for respirationBeri-beri: nerve and heart disorders Cereals, legumes, beans,

B2
ww
pyrophosphate
Riboflavin Part of coenzyme FAD needed for Ariboflavinosis: skin and eye
respiration disorders
nuts.
Milk, yogurt, meat, leafy
greens, whole grains.
B12

B5 Nicotinic
w.E
Cyanoco-balamin Coenzyme needed for making red Pernicious anaemia
blood cells, bone, blood and nerve
changes
Part of coenzymes NAD, NADP Pellagra: skin, gut and nerve
Meat, fish, poultry,
shellfish, eggs, cheese,
milk.
Widespread in foods.

C
acid (‘niacin’)
Ascorbic acid asy
used in respiration
Not precisely known
disorders
Scurvy: degeneration of skin teeth Citrus fruits, e.g. lemon.

A Retinol
visual pigment, rhodopsin En and blood vessels.
Not fully known but forms part of Xeropthalmia: ‘dry eyes’ Milk, eggs, meat, fish liver
oils, Green leafy vegetables.
D Cholecalciferol Stimulates calcium absorption by
small intestine, needed for proper
gin
Rickets: bone deformity Exposed to sunlight, Dairy
products, egg yolk, fish

E Tocopherol
bone growth
Not precisely known Infertility eer liver oils, oysters, yeast.
Plant oils, green, leafy

K Phylloquinone Involved in blood clotting Possible haemorrhage ing vegetables, egg yolk, nuts,
seeds, and liver.
Bacterial synthesis in the

.ne
digestive tract. Green
leafy vegetables, cabbage

Pratozoan Diseases
DISEASE AND DEFENCE MECHANISM
Bacteria Diseases
and milk

t
Disease Pathogen Disease Pathogen
Malaria Plasmodium
Amoebiasis Enta moeba histolylica Dysentery Shigella
Giardiasis Giardia lambia Plague Pasteurellapestis
Sleeping sickness Trypanosoma
Leshmanis Leishmania Diphtheria Corynebacterium diphtheriae
Trichomoniasis Trichomonas vaginalis Cholera Vibrio cholerae
Fungal diseases Tuberculosis Mycobacterium tuberculosis
Disease Pathogen Tetanus Clostridium tetani
Aspergillosis Aspergillus fumigatuo Whooping cough Bordetella pertussis
Candidiasis Candida albicens
Leprosy Mycobacterium leprae
Ringworm Trichoplyton
Blastomycosis Blasto myces dermatitidis Anthrax Bacillus anthrasis
Sporotnichosis Sporothrix Schenckii Weil’s disease Leptospira

Downloded From : www.EasyEngineering.net


Downloded From : www.EasyEngineering.net

Current Affairs & GK Update GK-33


Viral Diseases Important Vaccines
Disease Pathogen Vaccine Developed by Country Year
Rabies Rabies virus Small Pox Edward Jenner England 1796
Dengue Dengue virus Cholera Louis Pasteur France 1880
Influenza Influenza virus Diphtheria and Emil Adolf Von Behring Germany/ 1891
Tetanus and Shibasaburo Kitasato Japan
Measles Rubeola virus
TB Vaccine Albert Calmette and France 1922
German measles Rubella virus
Camille Guerin
Mumps Mumps virus
Polio Vaccine Jonas E. Salk US 1952
Chicken pox Varicella zoster
Oral Polio Albert Bruce Sabin US 1955
Small pox Variola virus Vaccine
Polio Polio virus Measles Vaccine John F. Enders, Thomas US 1953
Chikungunya Chikungunya (CHIK) Virus peeble
Avian flu H5N1 virus Rabies Vaccine Louis Pasteur France 1885
H1N1(Swine flu) H1N1 virus Typhus Vaccine Charles Nicolle France 1909

ww SPORTS

w.E TROPHIES ASSOCIATED WITH SPORTS


NATIONAL Merdeka Cup Football (Asia)
Name of the Trophy
Aga Khan Cup Hockey asy
Related game Thomas Cup
Uber Cup
US-Open
World Badminton (Men)
World Badminton (women)
Lawn Tennis
Barna Bellack Cup
Beighton Cup
Bombay Gold Cup
Table Tennis
Hockey
Hockey En French-Open
Australian Open
Lawn Tennis
Lawn Tennis
Burdwan Trophy
D.C.M. Trophy
Weight Lifting
Football gin
Wimbledon
Masters Champions Trophy
British Open
Lawn Tennis
Hockey
Golf
Dhyan chand Trophy
Dr. B.C. Roy Trophy
Hockey
Football Tata Open eer
Malaysian Open Badminton
Lawn Tennis
Duleep Trophy
Durand Cup
Ezra Cup
Cricket
Football
Polo Badminton ing
Number of Players on Each Side
1 or 2 Polo 4
I.F.A Shield
Lady Ratan Tata Trophy
Football
Hockey
Baseball
Basketball
9
5 .ne
Rugby Football 15
Tennis and Table tennis 1 or 2
Moin ud daula Gold Cup
Rangaswami Cup
Ranji Trophy
Santosh Trophy
Cricket
Hockey
Cricket
Football
Cricket
Football
Hockey
Chess
11
11
11
1
Water Polo
Volleyball
Kabaddi t 7
6
7

Scindia Gold Cup Hockey National Sports of Famous Countries


Subroto Mukherjee Cup Football (Inter-School) Name National game
Wellington Trophy Rowing Australia Cricket
INTERNATIONAL Brazil Football
Canada Ice Hockey
Name of the Trophy Related game China Table Tennis
Nehru Trophy Hockey England Cricket
American Cup Yatch Racing India No Game
Ashes Cup Cricket (Australia-England) Japan Judo or Ju Jitsu
Azlan Shah Hockey Malaysia Badminton
US Masters Golf Pakistan Hockey
Hopman Cup Lawn Tennis Russia Chess, Football
Colombo Cup Trophy Football Scotland Rugby, Football
Davis Cup Lawn Tennis Spain Bull Fighting
Kings Cup Race Air Races (England) United States of America Baseball

Downloded From : www.EasyEngineering.net


Downloded From : www.EasyEngineering.net

GK-34 Current Affairs & GK Update


TERMS USED IN SPORTS AND GAMES Hockey Bull, Carry, Centre Forward, Carried, Dribble,
Goal, Hat trick, Penalty corner, Scoop, Short corner,
Badminton Deuce, Double, Drop, Fault, Game, Let, Love, Sticks, Striking circle, Under cutting.
Smash. Horse Racing Jockey, Place, Protest, Punter, Win.
Baseball Bunt, Diamond, Home, Pitcher, Put out, Strike. Lawn Tennis Back-hand-drive, Service, Smash, Volley, Deuce,
Billiards Break, Cannons, Cue, In off, Jigger, Scratch, Game, Set, Love.
Boat Race Cox Polo Bunder, Chuckker, Mallet.
Boxing Hook, Jab, Knock-out, Punch, Upper cut. Rifle Shooting Bull’s eye.
Bridge Diamonds, Dummy, Grand slam, Little slam, Rugby Drop kick, Screen.
Revoke, Ruff, Tricks, Trump. Swimming Stroke.
Volley ball Booster, Deuce, Love, Service, Spikers.
Chess Check, Checkmate, Gambit, Stalemate
Wrestling Half Nelson, Heave.
Cricket Bowling, Bouncer, Crease, Cover point, Drive,
Duck, Follow on, Googly, Gulley, Hat Trick, Hit SPECIFIC NAMES OF PLAYING AREAS
wicket, L.B.W. (Leg Before Wicket), Leg Break, Sport Name of Sport Name of Playing
Leg spinner, Leg bye, Maiden over, No ball, Pitch, Playing Area Area
Run, Silly point, Stumped, Wicket keeper. Badminton Court Golf Link, Green**
Football Dribble, Drop Kick, Foul, Hattrick, Off-side, Penalty, Baseball Diamond Lawn Tennis Court

Golf ww
Throw in, Touch Down.
Bogey, Caddie, Hole, Links, Put, Putting the green,
Stymie, Tee.
Boxing
Cricket
Ring
Pitch*
Ice Skating Rink
Wrestling Ring, Arena
* Pitch in fact is the space between the wickets and not the entire cricket field.

w.E ** It is the area around the hole only.

asy
En
gin
eer
ing
.ne
t

Downloded From : www.EasyEngineering.net

You might also like